Quick viewing(Text Mode)

BA-101 English-Compulsory Guru Jambheshwar University of Science

BA-101 English-Compulsory Guru Jambheshwar University of Science

English (Compulsory) BA-101

Bachelor of Arts

(B.A. )

BA-101

English-Compulsory

ITY ERS OF IV SC N IE U N R C A E

W &

H

T

S E

E

C

B

H

M N

A O

J

L

O U

G

R

Y U G

Directorate of Distance Education Guru Jambheshwar University of Science & Technology HISAR-125001

DDE, GJUS&T, Hisar 1 | English (Compulsory) BA-101

CONTENTS

No. Title Author Editor Page/s Literature and Language-I

1 Speech Sound Dr.NutanYadav Dr.NutanYadav 6

2 a)Choosing Our Universe Dr.NutanYadav Dr.NutanYadav 28

b)Are Dams the temples of 46 Modern ?

Author-

3 The Generation Gap Dr.NutanYadav Dr.NutanYadav 69

4 Language and National Identity Dr.NutanYadav Dr.NutanYadav 85

5 a)Wounded Plants Dr.NutanYadav Dr.NutanYadav 106

Author- Jagadish Bose

b) Playing the English 125 Gentleman

6 Great Books Born out of Great Dr.NutanYadav Dr.NutanYadav 146 Minds

7 a)The Responsibility of Young Dr.NutanYadav Dr.NutanYadav 161 Men

b)Author-Lal Bahadur Shastri 173 Bharat Mata

Author- Jawaharlal Nehru

DDE, GJUS&T, Hisar 2 | English (Compulsory) BA-101

Literature and Language-II

1 Pigeons at Daybreak Dr. Pallavi Dr. Pallavi 189

By Anita Desai

2 With the Photographer Dr. Pallavi Dr. Pallavi 210

By – Stephen Leacock

3 The Journey Dr. Pallavi Dr. Pallavi 224

By Temsula Ao

4 The Refugee Dr. Pallavi Dr. Pallavi 241

By Khwaja Ahmed Abbas

5 Panchlight Dr. Narsingh Dr. Narsingh 256 Jangra Jangra

6 The Child Dr. Narsingh Dr. Narsingh 282 Jangra Jangra

7 The Blind Dog Dr.NutanYadav Dr.NutanYadav 313

R.K.Narayan

Comment [g1]: Grammar Grammer Section

No. Title Author Editor Page

1 Grammar: Noun and Pronoun Dr Astha Dr. Shakuntla 339

Gupta

2 Verbs and Adverbs Dr Astha Dr. Shakuntla 352 Gupta Devi

3 Adjectives and Prepositions Dr Astha Dr. Shakuntla 372

DDE, GJUS&T, Hisar 3 | English (Compulsory) BA-101

Gupta Devi

4 Modals and Conjunctions. Dr Astha Dr. Shakuntla 401 Gupta Devi

5 Grammar: Tenses and Voice Dr Astha Dr. Shakuntla 424 Gupta Devi

6 Narration (Direct and Indirect Dr Astha Dr. Shakuntla 460 Speech) and Phrasal Verbs. Gupta Devi

7 Comprehension: Candidates Dr Astha Dr. Shakuntla 493 will be required to answer the Gupta Devi questions that follow the passages taken from the prescribed textbooks. Candidate will be required to attempt both the passages.

8 Paragraph: Candidates will be Dr Astha Dr. Shakuntla 518 required to write a paragraph 1 Gupta Devi in about 200 words on any one out of the four.

DDE, GJUS&T, Hisar 4 | English (Compulsory) BA-101

LITERATURE AND

LANGUAGE-I

DDE, GJUS&T, Hisar 5 | English (Compulsory) BA-101

Subject: English ( Compulsory)

Course Code: BA101 Author: Dr.NutanYadav

Comment [g2]: done Lesson No:01 Editor: Dr.NutanYadav

Speech Sound

Lesson Structure

1.1 Learning Objectives

1.2 Introduction

1.3 Main Body of the Text

1.3.1 What is a speech sound?

1.3.2 What is a vowel?

1.3.3 What is a consonant?

1.4 Further Body of the text

1.4.1 Phonemes

1.4.2 Pure vowels

1.4.3 Vowel sound symbols

1.4.4 Consonant sound symbols

1.4.5 Pronunciations of the letters of the English alphabets

1.5 Check your Progress

1.6 Summary

1.7 Keywords

1.8 Self -Assessment Questions (SAQs)

DDE, GJUS&T, Hisar 6 | English (Compulsory) BA-101

1.9 Answers to Your Progress

1.10 References/ Suggested Readings

1.1 LEARNING OBJECTIVES

After going through this lesson you will be able to-

 To develop reading, writing, speaking, and listening skills of the English language.  To enrich the taste of literature.  To develop a reading habit  Know the difference between poetry and prose.  Trained in critically analyzing phonetic transcription of sound system

1.2 INTRODUCTION

Sounds play a very important role in a language. It is the way how we express ourselves. Students need to know the sound structure of a language.

1.3 MAIN BODY OF THE TEXT

1.3.1 What is a speech sound?

The English alphabet contains 26 letters which are divided into vowels and consonants. There is no one to one correspondence between spellings and sounds. It is that these 26 letters of the English alphabet along with some other symbols, represent 44 English speech sounds. Due to these 44 sounds, the letters of the alphabets in English don’t always correspond to the same speech sound. For example, ‘gh’ is pronounced differently in Laugh, ghost, through. The International Phonetic Association (IPA) has evolved to overcome this problem.

Phonetic transcription following the IPA makes it possible for us to understand the correct pronunciation of words through dictionaries.

1.3.2 Phonetic Transcription

DDE, GJUS&T, Hisar 7 | English (Compulsory) BA-101

It is the visual representation of speech sounds following the international Phonetic Alphabet (IPA), which is used extensively by phoneticians all over the world.

According to IPA

The 44 speech sounds of English are divided into 20 vowels and 24 consonants.

44 English speech sounds devided in-

20 vowels sounds(12 vowels +8 diphthongs)

24 consonants

1.3.3- What is a vowel?

A vowel is a speech of sound in the production of which the air comes out of the mouth without any obstruction at all.

1.3.4 What is a consonant?

A consonant is a speech sound in the production of which there is some hindrance to the airstream at some point in the mouth.

All these 44 sounds are phonemes.

1.4 FURTHER BODY OF THE TEXT

1.4.1 Phonemes

DDE, GJUS&T, Hisar 8 | English (Compulsory) BA-101

A phoneme is the smallest possible sound unit which is combined with other phonemes to form meaningful units such as words.

Two words can differ in meaning through the contrast of a single phoneme. For example-

Tail- Fail

Kiss- Kill

Rat- Cat

Hit- Heat

Speech sounds can be understood easily through this diagram

Speech Sounds (44)

Vowel Consonants Sounds(20) Sounds(24)

Pure Diphthong(8) Vowel(12)

1.4.2 Pure Vowel

A pure vowel is that vowel in which only one sound is made for example - a, e, I, o, u, etc.

Diphthongs- A diphthong is a vowel in which two vowel sounds are heard. (ai),

It is the mixture of two vowel sounds e.g.- night

DDE, GJUS&T, Hisar 9 | English (Compulsory) BA-101

Hence, a diphthong means double so

1.4.3 Vowel sound symbols

Examples IPA

ʌ CUP, LUCK

ɑ: ARM, FAT HER

a CAT , BLACK

e MET , BED

ə AWAY, CINEMA

ɜ:ʳ T URN, LEARN

ɪ HIT , SITTING

i: SEE, HEAT

ɒ HOT , ROCK

ɔ: CALL, FOUR

ʊ PUT , COULD

u: BLUE, FOOD

Diphthongs

Ai FIVE, EYE

aʊ NOW, OUT

eɪ SAY, EIGHT

DDE, GJUS&T, Hisar 10 | English (Compulsory) BA-101

Examples IPA

oʊ GO, HOME

ɔɪ BOY, JOIN

eəʳ WHERE, AIR

ɪəʳ NEAR, HERE

ʊəʳ PURE, T OURIST

1.4.4 Consonants Sound symbols

Consonants

IPA Examples listen

b BAD, LAB

d DID, LADY

f FIND, IF

g GIVE, FLAG

h HOW, HELLO

j YES, YELLOW

k CAT , BACK

l LEG, LITT LE

m MAN, LEMON

DDE, GJUS&T, Hisar 11 | English (Compulsory) BA-101

Consonants

IPA Examples listen

n NO, T EN

ŋ SING, FINGER

p PET , MAP

r RED, T RY

s SUN, MISS

ʃ SHE, CRASH

t T EA, GETTING

tʃ CHECK, CHURCH

θ THINK, BOTH

ð THIS, MOTHER

v VOICE, FIVE

w WET , WINDOW

z ZOO, LAZY

ʒ PLEASURE, VISION

dʒ JUST , LARGE

 Vowels- in phonetics, a vowel is a sound in spoken language that is pronounced with an open vocal tract.

DDE, GJUS&T, Hisar 12 | English (Compulsory) BA-101

 Diphthongs –A diphthong is a gliding vowel in the articulation of which there is a transition of sound from one vowel position to another.  Consonant- a consonant is a sound in spoken language that is articulated with the complete or partial closure of the vocal tract. For example, to pronounce/m/ one has to press one’s lips together; /t/ is pronounced using the front of the tongue;/k/ is pronounced using the back of the tongue.

1.4.5 Pronunciation of the letters of the English Alphabet

The first letter of the English alphabet ‘A’ the ‘I’, is pronounced as/e/. The IPA transcriptions of the pronunciations of all the letters of the English alphabet are listed below.

Syllable-syllables are the units into which a word is divided. Each unit consists of a vowel sound and usually one or more consonants. The syllabus may be ‘stressed’ strong form (when spoken with emphasis) and ‘unstressed’/weak form (when spoken without emphasis)

The number of a syllable in a word maybe one or more i.e, monosyllabic or polysyllabic.

Example of words containing one syllable:

 Late  Bat  Cheese  Cake  Door  Eat

Example of words containing two syllables:

 Walking  Chicken

DDE, GJUS&T, Hisar 13 | English (Compulsory) BA-101

 Father  Waited  Teacher  Enter

Examples of words containing three syllables:

 Editor  Reminder  Camera  Interrupt  Syllable

The letter ‘y’ can be counted as a vowel or a diphthong if it creates the sound of a vowel or a diphthong: e.g. ready, try, dry, etc.

1.5 CHECK YOUR PROGRESS

Transcribe these words-

Vowel sound phonetic diagram

DDE, GJUS&T, Hisar 14 | English (Compulsory) BA-101

Phonetic Symbols

Vowel Sounds cat, fat, rat bed, ten, pen teacher, feather, about the ship, sit, fit got, cot, on cup, bus, much good, full, foot far, car, father bird, curd, dirty sheep, reap, deep saw, flaw, call

DDE, GJUS&T, Hisar 15 | English (Compulsory) BA-101 shoot, pool, cool

Diphthong Sounds

Phonetic Symbols of Diphthongs

Diphthongs- my, fly, die cow, bow, now wait, straight, late the show, slow, go here, fear, near boy, joy, joy pure, sure, cure hair, bare, care

DDE, GJUS&T, Hisar 16 | English (Compulsory) BA-101

Phonetic symbols of Vowels, Diphthongs, and consonants in the English Language

Consonants Sounds-

DDE, GJUS&T, Hisar 17 | English (Compulsory) BA-101

Transcribe Words pea, pet, pole boat, bell, bat

DDE, GJUS&T, Hisar 18 | English (Compulsory) BA-101 ten, tell, tea dog, doctor, door chain, cheese

June, joker, jet car, kettle, character go, gold, get fly, feather, fun video, van, vine think, thunder, throw this, that, them see, sell, soap zoo, zebra, zone shall, shoe, shame vision, measure, television man, more, make now, never, new sing, song, thing hat, hello, hair love, labor, lost red, rose, rope wet, well, watch

1.6 SUMMARY

DDE, GJUS&T, Hisar 19 | English (Compulsory) BA-101

The English alphabet contains 26 letters which are divided into vowels and consonants. There is no one to one correspondence between spellings and sounds. It is that these 26 lette rs of the English alphabet along with some other symbols, represent 44 sounds or English speech sounds. Due to these 44 sounds, the letters of the alphabets in English don’t always correspond to the same speech sound. For example, gh is pronounced differently in Laugh, ghost, through. The International Phonetic Association (IPA) has evolved to overcome this problem.

 Phonetic transcription following the IPA makes it possible for us to understand the correct pronunciation of words through dictionaries. Vowels- in phonetics, a vowel is a sound in spoken language that is pronounced with an open vocal tract.  Diphthongs –A diphthong is a gliding vowel in the articulation of which there is a transition of sound from one vowel position to another.  Consonant- a consonant is a sound in spoken language that is articulated with a complete or partial closure of the vocal tract. For example, to pronounce/m/ one has to press one’s lips together;/t/ is pronounced using the front of the tongue;/k/ is pronounced using the back of the tongue.

1.7 KEYWORDS

 Phonemic  Syllabus  Transcription  Consonant  Vowel  Diphthong  IPA(International Phonetic Alphabet)

1.8 SELF ASSESSMENT QUESTIONS(SAQ’S)

Answer the following questions

DDE, GJUS&T, Hisar 20 | English (Compulsory) BA-101

 What is speech sound?  How many vowel sound we have in English?  What is a diphthong? How many diphthongs we have in English?  What is the phonetic Transcription?  What is a phoneme?  What is a syllable?

1.9 ANSWER TO YOUR PROGRESS

Answer 1-

The English alphabet contains 26 letters which are divided into vowels and consonants. There is no one to one correspondence between spellings and sounds. It is that these 26 letters of the English alphabet along with some other symbols, represent 44 sounds or English speech sounds. Due to these 44 sounds, the letters of the alphabets in English don’t always correspond to the same speech sound. For example, gh is pronounced differently in Laugh, ghost, through.

Answer 2

There are 12 vowel sounds we have in the English language.

Answer 3

A diphthong is a gliding vowel in the articulation of which this is a transition of sound from one vowel position to another. There are 8 diphthong sounds in English.

Answer-4

It is the visual representation of speech sounds following the International Phonetic Alphabet (IPA), which is used extensively by phoneticians all over the world. According to IPA, the 44 speech sounds of English are divided into 20 vowels and 24 consonants.

Answer-5

DDE, GJUS&T, Hisar 21 | English (Compulsory) BA-101

A phoneme is the smallest possible sound unit which is combined with other phonemes to form meaningful units such as words

Answer-6

Syllables are the units into which a word is divided. Each unit consists of a vowel sound and usually one or more consonants. The syllabus may be ‘stressed’ strong form (when spoken with emphasis) and ‘unstressed’/weak form (when spoken without emphasis)

List of common words for phonetic transcription a, about, above, across, act, active, activity, add, afraid, after, again, age, ago, agree, air, all, along, already, always, am, amount, an, and, angry, another, answer, any, anyone, a nything, anytime, appear, apple, are, area, arm, army, around, arrive, art, as, ask, at, attack, aunt, autumn,

B baby, base, back, bad, bag, ball, bank, basket, bath, be , bean, bear, beautiful, beer, bed, bedroom, behave, before, begin, behind, bell, below, besides, best, better, betwe en, big, bird, birth, birthday, bit, bite, black, bleed, block, blood, blow, blue, board, b oat, body, boil, bone, book, border,

, borrow, both, bottle, bottom, bowl, box, boy, branch, brave, bread, bread, breakfast , breathe, bridge, bright, bring, brother, brown, brush, build, burn, business, busy, b usy, but, buy, by.

C cake, call, can, candle, cap, car, card, care, careful, careless, carry, case, cat, catch, central, century, certain, chair, chance, change, chase, cheap, cheese, chicken, child

DDE, GJUS&T, Hisar 22 | English (Compulsory) BA-101

, children, chocolate, choice, choose, circle, city, class, clever, clean, clear, climb, clo ck, cloth, clothes, cloud, cloudy, close, coffee, coat, coin, cold, collect, color, comb, co me, comfortable, common, compare, complete, computer, condition, continue, contro l, cook, cool, copper, corn, corner, correct, cost, contain, count, country, course, cove r, crash, cross, cry, cup, cupboard, cut.

D dance, dangerous, dark, daughter, day, dead, decide, decrease, deep, deer, depend, d esk, destroy, develop, die, different, difficult, dinner, direction, dirty, discover, dish, d o, dog, door, double, down, draw, dream, dress, drink, drive, drop, dry, duck, dust, d uty.

E each, ear, early, earn, earth, east, easy, eat, education, effect, egg, eight, either, elect ric, elephant, else, empty, end, enemy, enjoy, enough, enter, equal, entrance, escape, even, evening, event, ever, every, everyone, exact, everybody, examination, example, except, excited, exercise, expect.

F face, fact, fail, fall, false, family, famous, far, farm, father, fast, fast, fault, fear, feed, f eel, female, fever, few, fight, fill, film, fine, fine, finger, finish, fire, first, fit, five, fix, fla t, flat, float, floor, flour, flower, fly, fold, fold, fool, foot, football, for, force, foreign, for est, forget, forgive, fork, form, fox, four, free, freedom, freeze, fresh, friend, friendly, fr om, front, fruit, full, fun, funny, furniture, further, future.

G

the game, garden, gate, general, gentleman, get, gift, give, glad, glass, go, goat, god, gold, good, goodbye, grandfather, grandmother, grass, grave, great, green, grey, ground, g roup, grow, gun.

DDE, GJUS&T, Hisar 23 | English (Compulsory) BA-101

H hair, half, hall, hammer, hand, happen, happy, hard, hate, hate, have, he, head, heal thy, hear, heavy, hello, help, heart, heaven, height, help, hen, her, here, hers, hide, h igh, hill, him, his, hit, hobby, hold, hole, , home, hope, horse, hospital, hot, h otel, house, how, hundred, hungry, hour, hurry, husband, hurt

I

I, ice, idea, if, important, in, increase, inside, into, introduce, invent, the iron, invite, is, island, it, its.

J

jelly, job, join juice, jump, just.

K

keep, key, kill, kind, king, kitchen, knee, knife, knock, know.

L ladder, lady, lamp, land, large, last, late, late, laugh, lazy, lead, leaf, learn, leave, leg, left, lend, length, less, lesson, let, letter, library, lie, life, light, like, lion, lip, list, liste n, little, live, lock, lonely, long, look, lose, lot, love, low, lower, luck.

M machine, main, make, male, man, many, map, mark, markets, marry, matter, may, me, meal, mean, measure, meat, medicine, meet, member, mention, method, middle, milk, million, mind, minute, miss, mistake, mix, model, modern, moment, money, m onkey, month, moon, more, morning, most, mother, mountain, mouth, move, much, music, must, my.

N name, narrow, nation, nature, near, nearly, neck, need, needle, neighbor, neither, ne

DDE, GJUS&T, Hisar 24 | English (Compulsory) BA-101 t, never, new, news, newspaper, next, nice, night, nine, no, noble, noise, none, nor, n orth, nose, not, nothing, notice, now, number.

O obey, object, ocean, of, off, offer, office, often, oil, old, on, one, only, open, opposite, o r, orange, order, other, our, out, outside, over, own.

P page, pain, paint, pair, pan, paper, parent, park, part, partner, party, pass, past, pat h, pay, peace, pen, pencil, people, pepper, per, perfect, period, person, petrol, photog raph, piano, pick, picture, piece, pig, pin, pink, place, plane, plant, plastic, plate, pla y, please, pleased, plenty, pocket, point, poison, police, polite, pool, poor, popular, po sition, possible, potato, pour, power, present, press, pretty, prevent, price, prince, pri son, private, prize, probably, problem, produce, promise, proper, protect, provide, pu blic, pull, punish, pupil, push, put.

Q

queen, question, quick, quiet, quiet.

R radio, rain, rainy, raise, reach, read, ready, real, receive, record, red, remember, remi nd, remove, rent, repair, repeat, reply, report, rest, restaurant, result, return, rice, ri ch, ride, right, ring, rise, road, rob, rock, room, round, rubber, rude, rule, ruler, run, rush.

S sad, safe, sail, salt, same, sand, save, say, school, science, scissors, search, seat, sec ond, see, seem, sell, send, sentence, serve, seven, several, sex, shade, shadow, shake , shape, share, sharp, she, sheep, sheet, shelf, shine, ship, shirt, shoe, shoot, shop, s hort, should, shoulder, shout, show, sick, side, signal, silence, silly, silver, similar, si

DDE, GJUS&T, Hisar 25 | English (Compulsory) BA-101 mple, single, since, sing, sink, sister, sit, six, size, skill, skin, skirt, sky, sleep, slip, sl ow, smoke, small, smell, smile, smoke, snow, so, soap, sock, soft, some, someone, so mething, sometimes, son, soon, sorry, sound, soup, south, space, speak, special, spe ed, spell, spend, spoon, sport, spread, spring, square, stamp, stand, star, start, stati on, stay, steal, steam, step, still, stomach, stone, stop, store, storm, story, strange, st reet, strong, structure, student, study, stupid, subject, substance, successful, such, sudden, sugar, suitable, summer, sun, sunny, support, sure, surprise, sweet, swim, sword.

T table, take, talk, tall, taste, taxi, tea, teach, team, tear, telephone, television, tell, ten, tennis, terrible, test, then, that, the, their, then, there, therefore, these, thick, thin, t hing, think, third, this, though, threat, three, tidy, tie, title, too, today, toe, together, tomorrow, tonight, too, tool, tooth, top, total, touch, town, train, tram, travel, tree, tr ouble, true, trust, twice, try, turn, type.

U

uncle, under, understand, unit, until, up, use, useful, usual, usually.

V

vegetable, very, village, voice, visit.

W wait, wake, walk, want, warm, wash, waste, watch, water, way, we , weak, wear, weat her, wedding, week, weight, welcome, well, west, wet, what, wheel, when, where, whi ch, while, white, who, why, wide, wide, wild, will, win, win, window, wine, winter, wir e, wise, wish, wish, without, woman, wonder, word, work, world, worry, worst, write, wrong

Y

DDE, GJUS&T, Hisar 26 | English (Compulsory) BA-101

year, yes, yesterday, yet, you, young, your.

Z

zero, zoo

1.10 REFERENCES /SUGGESTED READINGS

 Michael Rosen (2015). Alphabetical: How Every Letter Tells a Story. Counterpoint. ISBN 978-1619027022.  Upward, Christopher; Davidson, George (2011), The History of English Spelling, Oxford: Wiley-Blackwell, ISBN 978-1-4051-9024-4, LCCN 2011008794.

DDE, GJUS&T, Hisar 27 | English (Compulsory) BA-101

Course Code: BA101 Author: Dr.NutanYadav

Lesson No:02

Chapter-2(a) Choosing Our Universe

Author-Stephen Hawking and Leonard Mlodinow

Chapter -2(b) Are Dams the Temple of Modern India?

Author –Arundhati Roy

Lesson Structure

2.1 Learning Objectives

2.2 Introduction

2.3 Main Body of the Text

2.3.1 About the Author

2.3.2 About the Essay

2.4 Further Body of the text

2.5 Check your Progress

2.6 Summary

2.7 Keywords

2.8 Self -Assessment Questions (SAQs)

2.8.1 Language Activity 2.9 Answers to Your Progress 2.9.1 Extented Composition

2.10 References/ Suggested Readings

DDE, GJUS&T, Hisar 28 | English (Compulsory) BA-101

2.1 LEARNING OBJECTIVES

After going through this lesson you will be able to-

 To develop reading, writing, speaking, and listening skills of the English language.  To enrich the taste of literature.  To develop a reading habit  Know the difference between poetry and prose.  Trained in critically analyzing phonetic transcription of sound system

2.2 INTRODUCTION

The essay ‘Choosing Our Universe’ is an excerpt from The Grand Design, a collective book by Stephen Hawking and Leonard Mlodinow. It offers a thought-provoking and innovative approach towards understanding the universe. The laws of Physics have helped us get answers to a lot many puzzles related to the cosmos. The Theory of Relativity and Quantum Physics together have helped unravel many deep mysteries about the universe. However, the quest to arrive at a single unified theory that will explain everything still evades us

2.3 MAIN BODY OF THE TEXT

DDE, GJUS&T, Hisar 29 | English (Compulsory) BA-101

ACCORDING TO THE BOSHONGO PEOPLE of central Africa, in the beginning, there was only darkness, water, and the great god Bumba. One day Bumba, in pain from a stomachache, vomited up the sun. In time the sun dried up some of the water, leaving land. But Bumba was still in pain and vomited some more. Up came the moon, the stars, and then some animals: the leopard, the crocodile, the turtle, and finally man. The Mayans of Mexico and Central America tell of a similar time before creation when all that existed were , the sky, and the Maker. In the Mayan legend the Maker, unhappy because there was no one to praise him, created the earth, mountains, trees, and most animals. But the animals could not speak, and so he decided to create humans. First, he made them of mud and earth, but they only spoke nonsense. He let them dissolve away and tried again, this time fashioning people from wood. Those people were dull. He decided to destroy them, but they escaped into the forest, sustaining damage along the way that altered them slightly, creating what we today know as monkeys. After that fiasco, the Maker finally came upon a formula that worked and constructed the first humans from white and yellow corn. Today we make ethanol from corn, but so far haven’t matched the Maker’s feat of constructing the people who drink it.

Creation myths like these all attempt to answer the questions we address in this book: Why is there a universe, and why is the universe the way it is? Our ability to address such questions has grown steadily in the centuries since the ancient Greeks, most profoundly over the past century. Armed with the background of the previous chapters, we are now ready to offer a possible answer to these questions.

One thing that may have been apparent even in early times was that either the universe was a very recent creation or else human beings have existed for only a small fraction of cosmic history. That’s because the human race has been improving so rapidly in knowledge and technology that if people had been around for millions of years, the human race would be much further along in its mastery.

DDE, GJUS&T, Hisar 30 | English (Compulsory) BA-101

According to the Old Testament, God created Adam and Eve only six days into creation. Bishop Ussher, primate of all Ireland from 1625 to 1656, placed the origin of the world even more precisely, at nine in the morning on October 27, 4004 BC. We take a different view: that humans are a recent creation but that the universe itself began much earlier, about 13.7 billion years ago.

The first actual scientific evidence that the universe had a beginning came in the 1920s. As we said in Chapter 3, that was a time when most scientists believed in a static universe that had always existed. The evidence to the contrary was indirect, based upon the observations Edwin Hubble made with the 100-inch telescope on Mount Wilson, in the hills above Pasadena, California. By analyzing the spectrum of light they emit, Hubble determined that nearly all galaxies are moving away from us, and the farther away they are, the faster they are moving. In 1929 he published a law relating their rate of recession to their distance from us and concluded that the universe is expanding. If that is true, then the universe must have been smaller in the past. If we extrapolate to the distant past, all the matter and energy in the universe would have been concentrated in a very tiny region of unimaginable density and temperature, and if we go back far enough, there would be a time when it all began—the event we now call the big bang.

2.3.1 About the Author

Stephen William Hawking Leonard Mlodinow

DDE, GJUS&T, Hisar 31 | English (Compulsory) BA-101

Stephen William Hawking is an English theoretical physicist cosmologist author and Direct of the Research center at the University of Cambridge. He was the first to se t forth a cosmology explained by a union of the general theory of relativity and quantum mechanics. He has achieved success with popular science. He is the author of ‘A Brief History of Time’ a phenomenal successful science book that explains a range of subjects related to cosmology. He has worked extensively on Black Holes. He suffered from a motor disease that has paralyzed him.

Leonard Mlodinow is a famous American physicist and screenwriter and scientist. As a child, he was interested in both mathematics and chemistry. He got a doctoral degree from the University of California. He did pioneering work on the quantum theory of Dielectric Media.

Between 2008 and 2010 Mlodinow worked on a book with Stephen Hawking entitled ‘The Grand Design’ from which this essay has been taken.

2.3.2 About the Essay

This essay offers a new approach to understanding the universe. The laws of physics have helped us to get answers for a lot of many deep mysteries about the universe. However, the quest to arrive at a single unified theory that will explain everything still evades us; we still don’t know exactly how this universe was formed. We don’t know for certain how life originated on this earth.

(Stephen Hawking and Leonard Mlodinow)

2.4 FURTHER BODY OF THE TEXT

1. According to the Boshongo people, in the beginning, there was only darkness, water, and the great God Bumba. Bumba was the god in the mythology of Central Africa. Those people believe that Bumba is the god of universe creation. One day due to pain in the stomach vomited up the sun. The Sundried up some of the water, leaving land. But God was still in pain, and

DDE, GJUS&T, Hisar 32 | English (Compulsory) BA-101

vomited some more and resulted in the moon, the stars, and then some animals: the leopard, crocodile, the turtle, and finally man. 2. The Mayan tube of Mexico and Central America tells of a similar time before creation where all that existed were the sea, the sky, and the maker. The maker was unhappy because there was no one to praise him, created the earth, mountains, trees, and most animals. But the animals could not speak, so he decides to create humans. He first made human beings with mud and earth, but they only spoke nonsense. He let them dissolve and tried again. Then he made the people from the wood but those people were dull. He decided to destroy them but they escaped into the forest, creating what we today known as monkeys. Thus, in the beginning, God failed to make human beings. This is the fiasco (mistake) of the Maker. According to the people of Mexico, God made the first humans from white and yellow corn. Sarcastically authors say that scientists make ethanol from corn but they have not been able to make men from it. Now, these types of myths from various people and religion are helpful to answer the questions we address-  Why is there a universe  Why is the universe the way it is?

Our ability to address such questions has grown steadily in the centuries since the ancient Greeks most profoundly over the past century. We are now ready to offer a possible answer to these questions. It has been apparent even from the early time that either the universe was a very recent creation of the human beings have existed for only a small fraction of comic history. It is not very old.

According to the Old Testament God created Adam and Eve only six days into creation.

DDE, GJUS&T, Hisar 33 | English (Compulsory) BA-101

Bishop Usher believed that they originated at nine in the morning on October 27, 4004 BC. But the scientists have found out that that the universe itself began much earlier about 13.7 billion years ago. But as compare to it, humans are a recent creation.

The first actual scientific evidence about the beginning of the universe came in the 1920s. In the beginning, most scientists believed in a static universe that had always existed. But Edwin Hubby in 1929 published a law relating their rate of recession to their distance from us. It was based upon the observation with the 100-inch telescope which he mounted on Mount Wilson, in the hills above Pasadena, California. He concluded that the universe is expanding. The universe must have been smaller in the past. If we extrapolate to the distant past, all the matter and the energy in the universe would have been concentrated in a very small region of unimaginable destiny and temperative and if we go back far enough there would be a time when it all began. The name given to the origin of the universe is the Big Bang. The Big Bang Theory is an effort to explain what happened at the very beginning of our universe. In the beginning, there was nothing. The big bang theory is an effort to explain what happened during and after that moment.

2.5 CHECK YOUR PROGRESS

Answer the questions in 25-30 words

1. Who was Bumba? 2. What is the joke that the authors make regarding ethanol? 3. What do you understand by the term ‘Big Bang Theory’?

Answers

1. Bumba was the god in the mythology of Central Africa. Those people believed that Bumba was the god of creation. In the beginning, there was only darkness, water, and the great god Bumba. The Bumba created the universe.

DDE, GJUS&T, Hisar 34 | English (Compulsory) BA-101

2. According to the people of Mexico, God made the first humans with white and yellow corn. The authors say that scientists make ethanol from corn, but they have not been able to make men from it. 3. The Big Bang Theory is an effort to explain what happened at the very beginning of our universe. In the beginning, there was nothing. The big bang theory is an effort to explain what happened during and after that moment.

Answer the questions in 75-100 words

1. Discuss the relevance of this title. 2. What is the ‘fiasco’ which has been discussed in the chapter? 3. What do scriptures say about the origin of the universe?

Answers

1. This essay offers a new approach to understanding the universe. The laws of physics have helped us to get answers to a lot of many puzzles related to the cosmos. The Theory of Relativity and Quantum Physics together have helped unravel many deep mysteries about the universe. However, the quest to arrive at a single unified theory that will explain everything still evades us. We still do not know exactly how this universe was formed. We do not know for certain how life originated on this earth. That is why the title of this essay is relevant. 2. According to the Mayan tribe of Mexico, the Maker first made humans of mud and earth, but they only spoke nonsense. He let them dissolve away and tried again. This time, the Maker made people from wood. But he found that they were dull. He decided to destroy them. But they escaped in the forest, sustaining damage along the way that altered them slightly. They changed into monkeys. Thus, in the beginning, God failed to make human beings. This was the fiasco of the maker. 3. It has been apparent even from the early times that either the universe was a very recent creation of human beings have existed for only a small fraction of cosmic history. According to the Old Testament, God has created Adam and

DDE, GJUS&T, Hisar 35 | English (Compulsory) BA-101

Eve only 6 days into creation. Bishop Usher believes that the world originated at nine in the morning on October 27, 4004 BC. But the scientists have found out that the universe itself began much earlier, about 13.7 billion ye ars ago. But as compared to it, humans are a recent creation.

Comprehensive passages

Passage 1

In 1929 he published a law relating their rate of recession to their distance from us and concluded that the universe is expanding. If that is true, then the uni verse must have been smaller in the past. If we extrapolate to the distant past, all the matter and energy in the universe would have been concentrated in a very tiny region of unimaginable density and temperature, and if we go back far enough, there would be a time when it all began—the event we now call the big bang.

Questions-

1. Who published the law relating to the recession of the galaxies? 2. What did Hubble conclude? 3. What does the passage say about the universe? 4. What is the name given to the origin of the universe?

Answers-

1. In 1929, Hubble published a law relating their rate of recession to their distance from us. 2. He concluded that the universe is expanding. 3. The universe must have been smaller in the past. 4. The name given to the origin of the universe is the Big Bang.

Passage 2

The first actual scientific evidence that the universe had a beginning came in the 1920s. As we said in Chapter 3, that was a time when most scientists believed in a static universe that had always existed. The evidence to the

DDE, GJUS&T, Hisar 36 | English (Compulsory) BA-101

contrary was indirect, based upon the observations Edwin Hubble made with the 100-inch telescope on Mount Wilson, in the hills above Pasadena, California. By analyzing the spectrum of light they emit, Hubble determined that nearly all galaxies are moving away from us, and the farther away they are, the faster they are moving.

Questions- 1. When did the first actual scientific evidence come about at the beginning of the universe? 2. What did most scientists believe in the beginning? 3. What was the evidence to the contrary based on? 4. Where was Hubble’s telescope mounted?

Answers

1. The first actual scientific evidence about the beginning of the universe came in the 1920s 2. In the beginning, most scientists believed in a static universe that had always existed. 3. The evidence to the contrary was indirect, based upon the observations made by Edwin Hubble with the 100- inch telescope. 4. It was mounted by Mount Wilson, in the hills above Pasadena, California.

Passage 3

According to the Old Testament, God created Adam and Eve only six days into creation. Bishop Ussher, primate of all Ireland from 1625 to 1656, placed the origin of the world even more precisely, at nine in the morning on October 27, 4004 BC. We take a different view: that humans are a recent cre ation but that the universe itself began much earlier, about 13.7 billion years ago.

DDE, GJUS&T, Hisar 37 | English (Compulsory) BA-101

Questions- 1. What does the Old Testament say about Adam and Eve? 2. Who were Adam and Eve? 3. When did the world originate according to Bishop Usher? 4. When did the universe begin, according to the scientists?

Answers-

1. According to the Old Testament, God created Adam and Eve only six days into creation. 2. Adam and Eve were the first ancestors of mankind. 3. According to him, the world originated at nine in the morning on October 27, 4004 BC. 4. According to scientists, the universe began about 13.7 billion years ago. 2.6 SUMMARY

Word formation

Prefix and suffix A prefix is a letter or a group of letters added to the beginning of a word to form a new word For eg. Im-improve Prove is a root word and Im is added as a prefix. Here are some more words where the use of prefix changes the meaning of the root word- Prefix Root word New word Spell Misspell Mis

Dis Satisfied Dissatisfied

DDE, GJUS&T, Hisar 38 | English (Compulsory) BA-101

2.7 KEYWORDS  Form words with the help of the following prefixes Anti-,dis-,un-,re- pre-  Form words with the help of the following suffix. -ed,--iy,-less,-ing,-er  Write down ten examples where the use of a prefix changes the meaning of the root word to produce the opposite meaning.

2.8 SELF ASSESSMENT QUESTIONS (SAQ’S)

Language Activity

The noun- A noun is a word used to name a person, place, thing, quality, or action.

CommonNoun Proper Noun

Noun

Collective Noun Abstract Noun

Common A noun that donates any or all members of a class. Noun (common here means general, unspecified categories)

Proper A noun that donates a particular thing. It is the name of a

DDE, GJUS&T, Hisar 39 | English (Compulsory) BA-101

Noun particular place or person. It is always written with a capital letter at the beginning to distinguish them from common nouns.

collective It is a noun that is singular in form but is used for a group of Noun people or things. E.g. Team, Staff, Nation

Abstract It is a name used for things you cannot see or touch, such as Noun a quality, an action, or a state. E.g. Goodness, Honesty, Cowardice etc.

2.8.1 Language Activities Form abstract noun from the following adjectives- 1. Broad…… 2. Brave…… 3. Wide……… 4. Happy……… 5. Novel……. 6. Ignorant………

Form abstract noun from the following verbs- 1. Know….. 2. Please……. 3. Advise……. 4. Free…… 5. Believe……. 6. laugh…………. 7. Success………..

Categories the noun is given below-

DDE, GJUS&T, Hisar 40 | English (Compulsory) BA-101

 Heath  Team  Soldier  Verdict  Bunch  Rome  Truth  Chemistry  Slave

2.9 ANSWERS TO YOUR PROGRESS

Form abstract noun from the following adjectives-

1. Broad……broadly 2. Brave……bravery 3. Wide………widely 4. Happy………happiness 5. Novel…….novelty 6. Ignorant………ignorance

Form abstract noun from the following verbs-

7. Know…..knowledge 8. Please…….pleasure 9. Advise…….advice 10. Free……freedom 11. Believe…….belief 12. laugh…………laughter. 13. Success………..successful

Categorize the noun given below-

a. Heath common noun

DDE, GJUS&T, Hisar 41 | English (Compulsory) BA-101

b. Team collective noun c. Soldier common noun d. Verdict collective noun e. Bunch collective noun f. Rome proper noun g. Truth abstract noun h. Chemistry abstract noun

Subject-verb agreement

In a sentence, the subject and the verb must agree in number and person. Singular subjects take singular verbs and plural subjects take plural verbs. Person Helping Present past Present Past Present Past verb perfect perfect negative negative

1st person

Singular I am was Have had don’t didn’t

Plural We are were Have had don’t didn’t

2nd person You are were Have had Don’t Didn’t Singular/plural

3rd person He is was Has had doesn’t didn’t Singular

She is was Has had doesn’t Didn’t

It is was Has had doesn’t Didn’t They are were Have had don’t Didn’t

The following table gives the various forms of simple present tense -

Simple Present Negative Interrogative

DDE, GJUS&T, Hisar 42 | English (Compulsory) BA-101

I sing. I do not sing. Do I sing?

We sing. We do not sing. Do we sing? You do not sing. Do you sing? You sing.

He/she sings. He/she does not sing. Does he/she sing?

They sing. They do not sing. Do they sing?

Fill the blanks with the suitable forms of the verb ‘be’ (is, am, are)

1. My mother is a teacher. 2. What is the latest score? 3. Shyam is very tall. 4. I am now 20 years tall. How old are you? 5. I am sure that you will pass. 6. Today is a holiday. 7. There are 7 days in a week. 8. Flour is made from wheat. 9. We are sorry to hear that you have not been well.

Rewrite the following sentences into their negative and interrogative form-

S.No. Simple Sentence Negative Form Interrogative Form

1. Ravi plays football. Ravi does not play Does Ravi play football. football?

2. Kiran is a doctor. Kiran is not a Is Kiran a doctor? doctor.

3. Mohit studies every day. Mohit does not Does Mohit study study every day. every day?

4. Children enjoy their Children do not Do children enjoy holidays. enjoy the holidays. the holidays?

DDE, GJUS&T, Hisar 43 | English (Compulsory) BA-101

5. My sister lives in My sister does not Does my sister live Mumbai. live in Mumbai. in Mumbai?

2.9.1 Extended Composition

Write an essay on preparing to go to the playground in about 150 words-

Playgrounds are relaxing places to visit for people of all ages. They are important for children because their facilities allow children to develop their physical and mental abilities. Both newly constructed and renovated playgrounds today should incorporate theme-based options, such as a wet sand area, to encourage children to use their creativitiy. Playgrounds should also provide safety measure such as soft ground surfaces to reduce the risk of injury from accidents.

Through many years, playground has been a comfort place for people to visit. The playground in our communities are getting old, therefore we should renew our playground. I'm part of the playground planning community, and I have few ideas to improve our playground. There were many difference types of playground; they were adventure playgrounds, creative playground, tot lots, mini parks, and theme park. The ideas of the playground will be build base on the satisfaction of the potential users. The design of the playground must be exciting and provide the developmental needs to the children. Safety is another important issue to construct a playground. The playground is available to many difference people in difference ages. The condition of the playground should be provided to the whole community.

2.10 REFERENCES/ SUGGESTED READINGS

 Michael Moorcock (2010-09-05). "Book review: 'The Grand Design' by Stephen Hawking and Leonard Mlodinow". Los Angeles Times. Retrieved 2010-09-03.  Richard Allen Greene (2010-09-02). "Stephen Hawking: God didn't create the universe". CNN. Retrieved 2010-09-04.  Nick Watt (2010). "Stephen Hawking: 'Science Makes God Unnecessary'". ABC News. Retrieved 2015-05-01.

DDE, GJUS&T, Hisar 44 | English (Compulsory) BA-101

 Laura Roberts (2010-09-02). "Stephen Hawking: God was not needed to create the Universe". The Telegraph. Retrieved 2015-05-01.

********************

DDE, GJUS&T, Hisar 45 | English (Compulsory) BA-101

Course Code: BA101 Author: Dr.NutanYadav

Lesson No:02

b) Are Dams the temples of Modern India?

Author-Arundhati Roy

2.2 INTRODUCTION

Are Dams the temples of modern India?

Human-made dams are important in our modern life. The concept behind dam- building includes water supply, electricity generation, flood control, recreation, and irrigation. But another question that arises in mind is-Are Dams favorable? Are Dam Fail or pass? This essay tries to answer these questions.

2.3 Main Body of the text

Original Text

In the fifty years since Independence, after Nehru’s famous “Dams are the Temples of Modern India” speech (one that he grew to regret in his lifetime), his footsoldiers threw themselves into the business of building dams with unnatural fervor. Dam-building grew to be equated with Nation-building. Their enthusiasm alone should have been reason enough to make one suspicious. Not only did they build new dams and new irrigation systems, but they also took control of small, traditio nal systems that had been managed by village communities for thousands of years, and allowed them to atrophy. To compensate for the loss, the Government built more and more dams. Big ones, little ones, tall ones, short ones. The result of its exertions is that India now boasts of being the world’s third-largest dam builder. According to the Central Water Commission, we have three thousand six hundred dams that qualify as

DDE, GJUS&T, Hisar 46 | English (Compulsory) BA-101

Big Dams, three thousand three hundred of them built after Independence. One thousand more are under construction. Yet one-fifth of our population – 200 million people – does not have safe drinking water and two-thirds – 600 million – lack basic sanitation.

Big dams started well, but have ended badly. There was a time when everybody loved them; everybody had them- the Communists, Capitalists, Christians, Muslims, Hindus, and Buddhists. There was a time when Big Dams moved men to poverty. Not any longer. All over the world, there is a movement growing against big dams. In the First World, they’re being de-commissioned, blown up. The fact that they do more harm than good is no longer just conjecture. Big Dams are obsolete. They are uncool. They are undemocratic. They’re a government’s way of accumulating authority (deciding who will get how much water and who will grow what where). They are brazen means of taking water, land, and irrigation away from the poor and gifting it to the rich.

Their reservoirs displace huge populations of people leaving them homeless and destitute. Ecologically, they’re in the doghouse. They lay the earth to waste. They cause floods, water-logging, salinity, they spread disease. There is mounting evidence that links Big Dams to earthquakes.

Big Dams haven’t lived up to their role as the monuments of Modern Civilization, emblems of Man’s ascendancy over Nature. Monuments are supposed to be timeless, but dams have an all too finite lifetime. They last only as long as it takes Nature to fill them with silt. It is common knowledge now that Big Dams do the opposite of what their Publicity People say they do- the Local Pain for National Gain myth has been a wide pen.

For all these reasons, the dam-building industry in the First World was troubled and out of work. So it is exported to the Third World in the name of Development Aid, along with their other waste like old weapons, superannuated aircraft carriers, and band pesticides.

DDE, GJUS&T, Hisar 47 | English (Compulsory) BA-101

2.3.1 About the Author-

Arundhati Roy

Suzanna Arundhati Roy is an Indian author best known for her novel ’’ which won the Man for fiction in 1997 and became the bestselling book by a non-expatriate Indian author. She is also a political activist involved in human rights and environmental causes. She is also a relentless critic of government policies and actions that affect the environment, the poor, and the dispossessed. Her views on such issues have found expression in the Algebra of infinite Justice, for which she was awarded the Sahitya Akademi Award in 2006. However, Roy declined the award protesting against the governmental policies.

2.3.2 About the Essay-

This essay is an excerpt from Arundhati Roy’s (1961) ‘The Algebra of Infinite Justice’. The book discusses various perspectives of global and local concerns. In this essay, she says that big dams are harmful to the environment. They have become obsolete. The advanced countries have stopped constructing big dams. These dams displace huge populations and make people homeless. They cause floods and spread diseases.

2.4 FURTHER BODY OF THE TEXT

DDE, GJUS&T, Hisar 48 | English (Compulsory) BA-101

Referring to Nehru’s famous speech in which he said that the dams are the temples of modern India, Arundhati Roy advocates against the construction of big dams. She says that it is due to the love for construction of the dam at all times, big dams were constructed in India. Dam building was equated with nation-building. The followers of Nehru showed the same enthusiasm and sometimes it became suspicious. Not only they build new dams and new irrigation systems they took control of a small traditional system that village communities had managed for thousands of years and allowed them to weaken.

This resulted in a big loss, to compensate for the loss, the government built more and more dams and we had a large variety of dams- big, little, tall, short. The result of this love for dam building is that India now boasts of being the world’s third- largest dam builder. According to the report of the Central Water Commission, we have 3600 dams that qualify as the big dams, 3300 of them built after independence. More than 1000 are under construction. Despite that 1/4th of our population (near about 200 million people) don’t get safe drinking water and 2/3rd (600 million) lack sanitation.

The starting of dam building as well but it have ended badly there was a time when everybody loved them. They were for everyone- the communists, capitalists, Muslims, Christians, and Buddhists. Big dams moved men to poetry. But this ended very soon.

2.5 CHECK YOUR PROGRESS

Answer the questions in 25-30 words

1. Which speech of Jawaharlal Nehru is referred to in this essay? 2. Why did Nehru regret his statement about dams later in his life? 3. How did the government compensate for the loss caused by dam building? 4. Why have big dams become obsolete? 5. Why are big dams undemocratic? 6. How did big dams affect ecology?

DDE, GJUS&T, Hisar 49 | English (Compulsory) BA-101

7. How does the author present a contrast between monuments and big dams?

Answers-

1. At the beginning of the essay, the writer refers to a speech given by Nehru. In this speech, he said, “Dams are the temples of modern India”. 2. In his speech, Nehru said, “Dams are the temples of modern India”. But later it was found that dams were not as beneficial as they were thought to be. There were many bad effects of dams. So Nehru regretted his statement. 3. The dam-building had some side effects. They took control of small traditional irrigation systems of Indian villages. To compensate for that loss, the government built more dams. 4. There was a time when big dams were liked by people. But now it has been found that the big dams do more harm than good. So advanced countries have stopped building big dams. Thus big dams have become obsolete. 5. Big dams are undemocratic because they favor the rich more than the poor. They are the governmental way of accumulating authority. They are the means of taking water, land, and irrigation from the poor and giving it to the rich. 6. The big dams lay the earth to waste. They cause floods, waterlogging, and salinity. There is evidence that big dams are linked to earthquakes. In this way, the big dams affect the ecology. 7. Some people call big dams the monuments of modern civilization. But the author says that monuments are permanent. However, the big dams have a limited lifetime. In this way, the author compares monuments and big dams.

Answer the questions in 75-100 words

1. ‘There was a time when big dams moved men to poetry’. Explain this statement. 2. What are the dangers associated with the construction of big dams?

Answers-

DDE, GJUS&T, Hisar 50 | English (Compulsory) BA-101

1. When big dams were constructed for the first time, they looked very impressive. The water of a river was blocked and collected in a big reservoir. These reservoirs looked like big lakes. People were attracted towards them and responded to them as they reacted to the sea or the big lakes. The romantic people even wrote poems about them. That is why the author says that were was a time when the big dams moved men to poetry. But now the big dams have lost their charm. 2. The big dams have to create big reservoirs. These reservoirs displace people and make them homeless and destitute. The big dams affect the ecology. They lay the earth to waste. They cause floods, waterlogging, and salinity. There is evidence that big dams are linked to earthquakes. It has been found out that the big dams do more harm than good. So advanced countries have stopped building big dams. Big dams are undemocratic be cause they favor the rich more than the poor. They are the government's way of accumulating authority. They are the means of taking water, land, and irrigation from the poor and giving it to the rich.

The comprehensive passage from the text

Passage 1

“In the fifty years since Independence, after Nehru’s famous “Dams are the Temples of Modern India” speech (one that he grew to regret in his lifetime), his footsoldiers threw themselves into the business of building dams with unnatural fervor. Dam-building grew to be equated with Nation-building. Their enthusiasm alone should have been reason enough to make one suspicious. Not only did they build new dams and new irrigation systems, but they also took control of small, traditional systems that had been managed by village communities for thousands of years, and allowed them to atrophy

Questions-

DDE, GJUS&T, Hisar 51 | English (Compulsory) BA-101

1. What did Nehru say about dams in his speech? 2. With what was dam-building equated? 3. What was the reason to make one suspicious? 4. What did the big dams do?

Answers-

1. Nehru said that dams were the temples of modern India. 2. Dam building was equated with nation-building. 3. The enthusiasm of Nehru’s followers was the reason to make one suspicious. 4. The big dams took control of small, traditional systems that village communities had managed for thousands of years.

Passage 2

To compensate for the loss, the Government built more and more dams. Big ones, little ones, tall ones, short ones. The result of its exertions is that India now boasts of being the world’s third -largest dam builder. According to the Central Water Commission, we have three thousand six hundred dams that qualify as Big Dams, three thousand three hundred of them built after Independence. One thousand more are under construction. Yet one-fifth of our population – 200 million people – does not have safe drinking water and two-thirds – 600 million – lack basic sanitation.”

Questions-

1. What did the government do to compensate for the loss? 2. What is India’s position in the world with regards to dams? 3. How many big dams are in India? 4. How many people of India do not have safe drinking water?

Answers-

DDE, GJUS&T, Hisar 52 | English (Compulsory) BA-101

1. To compensate for the loss, the government built more and more dams. 2. India is the world’s third-largest dam- builder. 3. We have 3600 dams that qualify as Big Dams, 3300 of them built after Independence. 4. One- fifth of our population- 200 million people- does not have safe drinking water.

Passage 3

Big dams started well, but have ended badly. There was a time when everybody loved them; everybody had them- the Communists, Capitalists, Christians, Muslims, Hindus, and Buddhists. There was a time when Big Dams moved men to poverty. Not any longer. All over the world, there is a movement growing against big dams. In the First World, they’re being de -commissioned, blown up. The fact that they do more harm than good is no longer just conjecture. Big Dams are obsolete. They are uncool. They are undemocratic. They’re a government’s way of accumulating authority (deciding who will get how much water and who will grow what where). They are brazen means of taking water, land, and irrigation away from the poor and gifting it to the rich.

Questions-

1. How did the dam start and end? 2. What moved men to poverty? 3. What movement is growing in the world? 4. Dams are the brazen means of what?

Answers-

1. Big Dams started well, but have ended badly. 2. Big Dams moved men to poverty. 3. A movement is growing against Big Dams.

DDE, GJUS&T, Hisar 53 | English (Compulsory) BA-101

4. Dams are brazen means of taking water, land, and irrigation away from the poor and gifting it to the rich.

2.6 SUMMARY

This essay is an excerpt from Arundhati Roy’s (1961) ‘The Algebra of Infinite Justice’. The book discusses various perspectives of global and local concerns. In this essay, she says that big dams are harmful to the environment. They have become obsolete. The advanced countries have stopped constructing big dams. These dams displace huge populations and make people homeless. They cause floods and spread diseases. When big dams were constructed for the first time, they looked very impressive. The water of a river was blocked and collected in a big reservoir. These reservoirs looked like big lakes. People were attracted towards them and responded to them as they reacted to the sea or the big lakes. The romantic people even wrote poems about them. That is why the author says that were was a time when the big dams moved men to poetry. But now the big dams have lost their charm. The big dams have to create big reservoirs. These reservoirs displace people and make them homeless and destitute. The big dams affect the ecology. They lay the earth to waste. They cause floods, waterlogging, and salinity. There is evidence that big dams are linked to earthquakes. It has been found out that the big dams do more harm than good. So advanced countries have stopped building big dams. Big dams are undemocratic because they favor the rich more than the poor. They are the government's way of accumulating authority. They are the means of taking water, land, and irrigation from the poor and giving it to the rich.

2.7KEYWORDS

Give the phonetic transcriptions of the following words taken from the chapter-

DDE, GJUS&T, Hisar 54 | English (Compulsory) BA-101

1. Famous 2. Nation 3. Control 4. Flood 5. Ban 6. Work 7. People 8. Myth 9. Wide 10. Open 11. Badly 12. Home 13. Make 14. One 15. New 16. Year 17. And 18. More 19. Our

Vocabulary Exercise

Write synonymous of the words-

1. Begin ……start 2. Excited……curious 3. Verse…..poetry

Write the antonyms of these words-

1. Destruction …..Construction 2. Rare ……………common 3. Truth …………..conjecture

DDE, GJUS&T, Hisar 55 | English (Compulsory) BA-101

4. Denial ……….acceptance 5. Accuse ……….acquit 6. Staid ………….fresh 7. Enlarge …….reserved 8. Virtue …….vice

Explain the following words-

1. Geology- the science of the study of the earth. 2. Amnesty –general pardon to the wrongdoer. 3. Cardiologist- a doctor who specializes in heart dise ase. 4. e-mail- electronic mail

2.8 SELF ASSESSMENT QUESTIONS-(SAQ’S)

 What is a pronoun? Classify various forms of pronoun.  Describe the present continuous tense.  Describe phrasal verbs.

2.9 ANSWERS TO YOUR PROGRESS

A pronoun is the word that is used in place of the noun is called a pronoun. It is used to avoid the repetition of the same noun in speech or writing.

The pronoun can be classified as such-

Pronoun

Personal Demonstrative Interrogative Possessive Reflexive Indefinite Relative Pronoun pronoun pronoun pronoun Pronoun Pronoun Pronoun

DDE, GJUS&T, Hisar 56 | English (Compulsory) BA-101

1. Personal pronoun-

The pronouns stand for a person or a thing. Personal pronouns replace a noun that names people/things in the subject and object position. I, we, you, he, she, and they are the personal pronoun which can be used in subject positions, whereas me, you, him, her, it, and us are used in object position. for example -

 I came here with my friend.

(I here is a personal pronoun in the subject position.)

 Please give me a glass of water.

(Me here is a personal pronoun in object position)

2. Possessive Pronoun-

These pronouns are used to talk about things belonging to people. Mine, ours, hers, his, theirs, and its, are possessive pronoun.

 You can use your pen, I will use mine.  We have bought a new car. It is ours now.

3. Reflexive pronoun-

These pronouns are used when the subject and object of a verb are same. These are used for the sake of emphasis. These are myself, himself, herself, itself, ourselves, yourselves, and themselves.

 They themselves committed the crime.  She herself says so.

4. Demonstrative pronoun-

The pronoun which points out the person or objects it refers to is a demonstrative pronoun. it includes this, that, these, those,

DDE, GJUS&T, Hisar 57 | English (Compulsory) BA-101

 This is my pen.  These are books. 5. Interrogative pronoun-

Words like what, which, who, whom, whose are interrogative pronouns, for they are used for asking questions.

 What are you looking for?  Who are you?  Whose pen is it? 6. Indefinite pronoun

The pronouns which refer to a person or things in a general way, but not in a particular way, are called indefinite pronoun.They are someone, somebody, everyone, everybody, few, none, each, every.

 Everybody was present at the meeting.  Have you seen anybody entering the house? 7. Relative pronouns

The words like who, whom, whose, which, etc. are relative pronouns because they refer or relate to an earlier noun which is called it's antecedent.

 The girl who is dancing is my sister.  I found the book which I had lost.

Exercises

Identify the kind of pronoun used in each of the following sentences.

 I hate the man who is patronizing.  I know the person whose daughter was part of the winning team.  A chair is a piece of furniture which can be used to sit.  Ask Robin to come with us for a drive.  You should use your common sense.

DDE, GJUS&T, Hisar 58 | English (Compulsory) BA-101

 The children are cooking something for the party.  You look sad. Is everything all right?  Have you checked this?  What is your name?  I cannot see anyone in the house?

Join each of the following pair of sentences using pronoun-

 The boy stole my camera. The boy was punished.

The boy was punished who stole my camera.

 I know a person. The person has been to Australia to study.

I know a person who has been to Australia to study.

 I saw a girl. She was dancing.

I saw a girl who was dancing.

 This is the person. His son helped you book the ticket.

This is the person whose son helped you book the ticket.

 The teacher sat on a chair. It was printed black.

The teacher sat on a chair which was printed black.

 He is a well-known person. His honesty is the talk of the time.

He is a well known person whose honesty is the talk of the time.

 This is the bridge. It was built in a month.

This is the bridge which was built in a month.

2.9.1Grammar exercise

Present continuous tense

This tense is used to donate an action which is going on at the time of speaking.

DDE, GJUS&T, Hisar 59 | English (Compulsory) BA-101

I am waiting for the train.

She is talking to her daughter.

Affirmative Negative Interrogative

I am watching T.V. I am not watching T.V. Am I watching T.V.

She is coming by car. She is not coming Is she coming by car?

My son is learning the My son is not learning Is my son learning the piano. the piano piano?

We are working on a We are not working on a Ang on a research pre we research project. research project. wroject?orki

This tense is also used when we are in the middle of an action, but not necessarily actually at the moment of speaking.

I am reading a novel these days.

We are attending the party tomorrow.

Change the following sentence into the present continuous tense.

1. My father takes his bath. My father is taking his bath. 2. I read the Time of India I am reading the Time of India. 3. The children play in the garden. The children are playing in the garden. 4. The vendor sells fruit in our street. The vendor is selling fruit in our street. 5. The boys rehearse a play for college days. The boys are rehearsing a play for college days.

Fill the blanks in the following sentence using the present continuous tense

DDE, GJUS&T, Hisar 60 | English (Compulsory) BA-101

 Hurry up, we are getting late. (get)  The men are sitting at the table on our left (sit)  The train is leaving the platform. (Leave)  The clouds are moving across the sky (moving)  Are you drinking coffee now?

2.9.2 Phrasal verb

Phrasal verbs are that consist of two or some things, three words. The first word is a verb and it is followed by An adverb(He turned down my request.) A preposition(Inflation is eating into my savings) An adverb and a preposition both(I cannot put up with your demand.)

Exercise-

 Bring about- Cause to happen  Bring out- Bring to light  Bring up-To rear

Use the following phrasal verbs in sentences

Throw in to He has thrown himself into the work of writing a book.

Equated with Helping the poor is equated with worshipping God

Boast of He always boasts of his new car.

Move to Her story moved me to tears.

Blow up The government blew up a mountain to make a road.

Live up to Over government has not lived up to people’s hopes.

Blow wide open The investigators blew the criminal’s story wide open.

DDE, GJUS&T, Hisar 61 | English (Compulsory) BA-101

List of important Phrasal verbs

ask around ask many I asked around but nobody has people the seen my wallet. same question

add up to something equal Your purchases add up to $205.32.

back something up reverse You'll have to back up your car so that I can get out.

back somebody up support My wife backed me up over my decision to quit my job.

blow up explode The racing car blew up after it crashed into the fence.

blow something up add air We have to blow 50 balloons up for the party.

break down stop Our car broke down at the side of functioning the highway in the snowstorm. (vehicle, machine)

break down get upset The woman broke down when the police told her that her son had died.

DDE, GJUS&T, Hisar 62 | English (Compulsory) BA-101

break something down divide into Our teacher broke the final smaller parts project down into three separate parts.

break in force entry to a Somebody broke in last night and building stole our stereo.

break into something enter forcibly The firemen had to break into the room to rescue the children.

break something in wear I need to break these something a shoes in before we run next week. few times so that it doesn't look/feel new

break-in interrupt The TV station broke in to report the news of the president's death.

break up end a My boyfriend and I broke up before relationship I moved to America.

break up start laughing The kids just broke up as soon as (informal) the clown started talking.

break out escape The prisoners broke out of jail when the guards weren't looking.

break out develop a skin I broke out in a rash after our in something condition camping trip.

DDE, GJUS&T, Hisar 63 | English (Compulsory) BA-101

bring somebody down make unhappy This sad music is bringing me down.

bring somebody up raise a child My grandparents brought me up after my parents died.

bring something up start talking My mother walks out of the room about a when my father brings up sports. subject

bring something up vomit He drank so much that he brought his dinner up in the toilet.

call around phone many We called around but we weren't different able to find the car part we needed. places/people

call somebody back return a phone I called the company back but the call offices were closed for the weekend.

call something off cancel Jason called the wedding off because he wasn't in love with his fiancé.

call on somebody ask for an The professor called on me for answer or question 1. opinion

DDE, GJUS&T, Hisar 64 | English (Compulsory) BA-101

call on somebody visit somebody We called on you last night but you weren't home.

call somebody up phone Give me your phone number and I will call you up when we are in town.

calm down relax after You are still mad. You need being angry to calm down before you drive the car.

not care not like I don't care about his behavior. for somebody/ (formal) something

catch up get to the You'll have to run faster than that same point as if you want to catch up with somebody else Marty.

check-in arrive and We will get the hotel keys when register at a we check-in. hotel or airport

check out leave a hotel You have to check out of the hotel before 11:00 AM.

check somebody/ look at it The company checks out all new something out carefully, employees. investigate

DDE, GJUS&T, Hisar 65 | English (Compulsory) BA-101

check out somebody/ look at Check out the crazy hair on that something (informal) guy!

cheer up become She cheered up when she heard happier the good news.

cheer somebody up make happier I brought you some flowers to cheer you up.

chip in help If everyone chips in we can get the kitchen painted by noon.

clean something up tidy, clean Please clean up your bedroom before you go outside.

come find I came across these old photos across something unexpectedly when I was tidying the closet.

come apart separate The top and bottom come apart if you pull hard enough.

come down become sick My nephew came down with something with chickenpox this weekend.

come forward volunteer for a The woman came forward with her task or to give husband's fingerprints. evidence

come from someplace originate in The art of origami comes from Asia.

DDE, GJUS&T, Hisar 66 | English (Compulsory) BA-101

count on somebody/ rely on I am counting on you to make something dinner while I am out.

cross something out draw a line Please cross out your old address through and write your new one.

cut back consume less My doctor wants me to cut back on something on sweets and fatty foods.

cut something down make We had to cut the old tree something fall to the ground

2.9.3 Extended Composition

Write a paragraph on ‘The prize Distribution Function’

The prize distribution function of our college was held on 6th April this year. It started at 8 a.m. The whole school was decorated. The Education Minister arrived to preside over the function. He arrived at 8 a.m. The Principal received him. Teachers and students offered him bouquets. The function started with a prayer song by two girls. Then the girls staged a dance item. Then a drama was staged. All these items were liked very much. The singers and dancers were cheered. Then the Principal read out the Annual Report. It showed that the school had made good progress. The chief guest then distributed the prizes. The prize -winners received prizes and cheers. In the end, the chief guest made a short speech and congratulated the prize -winners. He advised all to work hard. The Principal thanked the chief guest. The chief guest departed. The function came to an end.

2.10 REFERENCES/SUGGESTED READINGS

Recommended works of Arundhati Roy

DDE, GJUS&T, Hisar 67 | English (Compulsory) BA-101

 The Hanging of Afzal Guru and the Strange Case of the Attack on the Indian Parliament. New Delhi: Penguin. 2013. ISBN 978-0143420750.  Capitalism: A Ghost Story. Chicago: Haymarket Books, 2014. ISBN 978-1- 60846-385-5[106]  Things that Can and Cannot Be Said: Essays and Conversations (with John Cusack). Chicago: Haymarket Books, 2016. ISBN 978-1-608-46717-4  The Doctor and the Saint: Caste, Race, and Annihilation of Caste, the Debate Between B.R. Ambedkar and M.K. Gandhi. Chicago: Haymarket Books, 2017. ISBN 978-1-608-46797-6  My Seditious Heart: Collected Non-Fiction. Chicago: Haymarket Books, 2019. ISBN 978-1-608-46676-4  Azadi: Freedom, Fascism, Fiction. Haymarket Books, 2020. ISBN 1642592609 ********************

DDE, GJUS&T, Hisar 68 | English (Compulsory) BA-101

Course Code: BA101 Author: Dr.NutanYadav

Lesson No:03 The Generation Gap

The Generation Gap

Lesson Structure

3.1 Learning Objectives

3.2 Introduction

3.3 Main Body of the Text

3.3.1 About the Author

3.3.2 About the Essay

3.4 Further Body of the text

3.5 Check your Progress

3.6 Summary

3.7 Keywords

3.8Self -Assessment Questions (SAQs)

3.9Answers to Your Progress

3.9.1Language Activity

3.9.2Extended Composition

3.10 References and Suggested Readings

3.1 LEARNING OBJECTIVES

After going through this lesson you will be able to-

DDE, GJUS&T, Hisar 69 | English (Compulsory) BA-101

 To develop reading, writing, speaking, and listening skills of the English language.  To enrich the taste of literature.  To develop a reading habit  Know the difference between poetry and prose.  Trained in critically analyzing phonetic transcription of the sound system.

3.2 INTRODUCTION

Two Generations

What Is a Generation Gap?

A generation gap refers to the chasm that separates the thoughts expressed by members of two different generations. More specifically, a generation gap can be used to describe the differences in actions, beliefs, and tastes exhibited by members of younger generations, versus older ones.

The subjects at hand may be vast and varied but can include politics, values, and pop culture. While generation gaps have been prevalent throughout all periods of history, the breadth of differences of these gaps has widened in the 20th and 21st centuries. Generation gaps are caused by increased life expectancy, rapid changes in society, and the mobility of society. Effects of the generation gap include conflict among family members of different generations and misunderstandings. In short, a

DDE, GJUS&T, Hisar 70 | English (Compulsory) BA-101 generation gap is commonly perceived to refer to differences between generations that cause conflict and complicate communication creating a "gap."

3.3 MAIN BODY OF THE TEXT

This chapter discusses the problems of self-identity and independence faced by adolescent boys and girls. A young person goes through various stages of identification rejects the earlier role models and forms a new identity following the needs of the times. This desire to establish a new identity may be misunderstood by the elders as insolence or rebellion. However, all rebellion is not negative. It can be channeled positively and be used in the transformation of a young individual into a responsible adult.

3.3.1 About the Author

Benjamin McLane Spock was a famous American pediatrician whose book ‘Body and Child Care’ (1946) are one of the best selling volumes in history. The book’s premise to mothers in that ‘you know more than you think you do’

Spock was the first pediatrician to study psychoanalysis to try to understand children’s needs and family dynamics. His ideas about childcare influenced several generations of parents to be more flexible and affectionate with their children and to treat them as an individual. However, his theories were also widely criticized by colleagues for relying too heavily on anecdotal evidence rather than serious academic research.

3.3.2 About the Essay

This chapter discusses the problems of self-identity and independence faced by adolescent boys and girls. A young person goes through various stages of identification rejects the earlier role models and forms a new identity following the needs of the times. This desire to establish a new identity may be misunderstood by the elders as insolence or rebellion. However, all rebellion is not negative. It can be

DDE, GJUS&T, Hisar 71 | English (Compulsory) BA-101 channeled positively and be used in the transformation of a young individual into a responsible adult.

3.4 FURTHER BODY OF THE TEXT

Rebelliousness against parents is a natural, inevitable aspect of adolescence. It assists them in giving up the comforts and security of home, achieving real independence, working for progress.

The most basic problem for a young person, though he doesn’t usually think of it in these terms, is to find his own identity, to find out what sort of person he desires to be, and to get started being a person. This just doesn’t mean just the specific job he will take or the hobbies he will enjoy. It means the kind of personality he’ll end up with, how he’ll be thought of by his friends, family, and himself. It is not so much a matter of coming to conscious decisions- it’s gaining a sense of being an independent person, with a job to do.

Three principal elements go to make up the identity an individual finally achieves. First and foremost is the character he was developing all through childhood. A boy, beginning at about the age of three, has been striving to be a man just like his father and the little girl to be like her mother. So they have had to cast themselves in the molds of their parents- they are made of their parents.

But to become an effective adult an individual must break off his dependence on his parents, not just free enough to leave home but to develop ideas and aims of his own so that he can help to solve the existing problems of the society in which he lives. His feeling of impatience and criticism towards his parents is what put him on the lookout for new ideas, new methods that may better solve problems of the day- and incidentally, also show up his parents’ generation as pretty well stuck-in-the-mud. How strong the rebelliousness in each individual is and what form it takes is a second element in determining his eventual identity.

DDE, GJUS&T, Hisar 72 | English (Compulsory) BA-101

The third element is the nature of the times and the need of the times. These call on the youth for different qualities in different historical periods. As Erik Erikson explains in Young Man Luther, the corruption of the church in Martin Luther’s time, the real need for major reform, inspired in Luther fearlessness, clarity, eloquence, and perseverance of heroic proportions. In a less critical age, Luther might remain an undistinguished cleric.

Many of the advances of civilization- technical inventions, scientific discoveries- have been conceived by young people just on the threshold of adulthood. Because they were impatient with the achievements of the past and because they did not need to defend the present, they were able to envision and bring on the future.

3.5 CHECK YOUR PROGRESS

Answer the questions in 25-30 words

1. Why is rebelliousness common in young people? 2. What is the first formative influence on the individual? 3. To become an effective adult what should a growing child do? 4. What is the second element that makes up the identity of an individual? 5. What brought about many of the developments in human civilizations?

Answers

1. It is common because rebelliousness is a natural aspect of adolescence. It assists them in giving them the comforts and security of home, achieving real independence, working for progress. 2. The first formative influences are parents. In early childhood, a boy strives to be like his father and the little girl to be like her mother. 3. He must break off his dependence on his parents. But to become an effective adult an individual must develop ideas and aims of his own.

DDE, GJUS&T, Hisar 73 | English (Compulsory) BA-101

4. The second element in determining his identity depends on how strong his rebellion is and what form it takes. To be effective, it must be directed into the right channels. 5. Many of the developments in human civilization were brought about by young people. They were able to have a vision of the future because they were impatient with the achievements of the past.

Answer the questions in 75-100 words

1. What does the word ‘identity’ mean? 2. Trace the stages through which a young person passes as he/ she matures into an individual. 3. Write about the positive side of rebelliousness.

Answers

1. Identity is a person’s conception and expression of his individuality in family or society. Three elements shape a person’s identity. The first is the influence of parents in childhood. A boy strives to be like his father and the little girl to be like her mother. The second is the rebelliousness against the old set up and concepts. The third element is the role of society in which he lives. The identity of a young person is formed by the fact of how he faces the challenges of life and society. 2. A young person goes through various stages of identity. In the beginning, children strive to be like their parents. A boy strives to be like his father and the little girl to be like her mother. In the second stage, they have to shake off the dependence of their dependence on their parents. They have to look out for new ideas to solve the problems of the day to day life. In the third stage, they are fully independent. They can take decisions for themselves. But at this stage, they develop according to nature and the needs of the times.

DDE, GJUS&T, Hisar 74 | English (Compulsory) BA-101

3. In this essay, the writer says that rebelliousness against the parents is not always negative. It is a natural aspect of adolescence. It assists them in achieving real independence for progress. His feeling of criticism and impatience towards his parents is not always bad. It puts him on the lookout for new ideas to solve the problems of life. It also helps him to show how the old are not progressing with time. It means that the kind of personality that he will finally achieve. It also means what final impression he will form upon his friends, family, and himself.

Comprehension Passage

Passage 1

Rebelliousness against parents is a natural, inevitable aspect of adolescence. It assists them in giving up the comforts and security of home, achieving real independence, working for progress.

The most basic problem for a young person, though he doesn’t usually think of it in these terms, is to find his own identity, to find out what sort of person he desires to be, and to get started being a person. This just doesn’t mean just the specific job he will take or the hobbies he will enjoy. It means the kind of personality he’ll end up with, how he’ll be thought of by his friends, family, and himself. It is not so much a matter of coming to conscious decisions- it’s gaining a sense of being an independent person, with a job to do.

Questions-

1. What is an inevitable aspect of adolescence? 2. What is the most basic problem for the young person? 3. What does the search for identity for the young man mean? 4. What does the gaining of identity mean?

Answers-

1. Rebelliousness against parents is a natural, inevitable aspect of adolescence.

DDE, GJUS&T, Hisar 75 | English (Compulsory) BA-101

2. The most basic problem for a young person is to find his own identity. 3. It means the kind of personality that he will acquire finally. 4. It means gaining a sense of being an independent person.

Passage 2

Three principal elements go to make up the identity an individual finally achieves. First and foremost is the character he was developing all through childhood. A boy, beginning at about the age of three, has been striving to be a man just like his father and the little girl to be like her mother. So they have had to cast themselves in the molds of their parents- they are made of their parents.

Questions-

1. Name the lesson from which this passage has been taken. 2. What is the first and foremost element? 3. How many elements go to make up the identity of an individual? 4. How do the very young children cast themselves?

Answers-

1. The Generation Gap. 2. The first and foremost element is the character, he was developing all through childhood. 3. Three principal elements go to make up the identity of an individual. 4. It means gaining a sense of being an independent person.

Passage 3

The third element is the nature of the times and the need of the times. These call on the youth for different qualities in different historical periods. As Erik Erikson explains in Young Man Luther, the corruption of the church in Martin Luther’s time, the real need for major reform, inspired in Luther fearlessness, clarity, eloquence, and perseverance of heroic proportions. In a less critical age, Luther might remain an undistinguished cleric.

DDE, GJUS&T, Hisar 76 | English (Compulsory) BA-101

Many of the advances of civilization- technical inventions, scientific discoveries- have been conceived by young people just on the threshold of adulthood. Because they we re impatient with the achievements of the past and because they did not need to defend the present, they were able to envision and bring on the future.

Questions-

1. What is the third element in determining the personality of a young man? 2. What inspired fearlessness, clarity, eloquence, and perseverance in Luther? 3. What has been conceived by young people? 4. Why were the young men able to envision and bring on the future?

Answers-

1. The third element is the nature of the times and the needs of the times. 2. The corruption of the church inspired fearlessness, clarity, eloquence, and perseverance in Luther. 3. Many of the advances in civilization have been conceived by young people. 4. They were able to envision and bring on the future because they were with the achievements of the past.

3.6 SUMMARY

In this essay, the author says that rebelliousness against parents in adolescence is a natural and unavoidable characteristic in children. It happens due to their desire in giving up the comforts and security of home, to get real inde pendence and working for progress. The most basic desire of the young person in the age of adolescence is to find his own identity to know what type of person he wishes to be in life.

 And his desire to be considered as a young person.  But these desires are concerns not only just specific jobs or hobbies. It means the kind of personality he will end up with.  How he will be thought of by his friends, family, and himself.  Gaining a sense of being an independent person

DDE, GJUS&T, Hisar 77 | English (Compulsory) BA-101

Three principal elements affect the personality of a child.

 First and foremost is the character he was developing all through childhood up to the age of three, a boy strives to be a man like his father and the little girl to be like her mother. So they try to cast themselves in the mold of their parents. But to become an effective adult, an individual must break off most of his dependence on his parents not only to free enough to leave home but to develop ideas and aims of his own so that he can help solve the existing problems of the society in which he will live. This feeling of impatience and criticism towards his parents put him on the lookout for new ideas; new methods that may better solve the problems of the day.  The second element that affects the personality of an individual is the power of rebelliousness in each individual. This power determines his eventual identity.  The third element is the nature of the times and the needs of the times. These needs bring in youth different qualities in different qualities in different historical periods

Erik Erikson explains in Young Man Luther, the corruption of the church in Martin Luther’s time the real need for major reforms, inspired by Luther's fearlessness, Darity, eloquence, and perseverance of heroic proportions.

Many of the advances of civilizations, technical inventions, scientific discoveries- all are the results of the inventive minds of young people just on the threshold of adulthood. They could do so because they were impatient with the achievements of the past and because they had no need to defend the present, they were able to envision and bring on the future.

3.7 KEYWORDS

Transcribe the following words-

1. Person

DDE, GJUS&T, Hisar 78 | English (Compulsory) BA-101

2. Home 3. Sense 4. Age 5. Need 6. Defend 7. People 8. Able 9. Luther 10. Future 11. Enough 12. Ghost

3.8 SELF ASSESSMENT QUESTIONS (SAQ’S)

Find the words in the chapter that are synonymous with the italicized words in the following sentences.

 Revolt against elders is common amongst young people: Rebelliousness……….  She has oratorical skills: Eloquence  Patience is a great quality: Perseverance

Find the words in the chapter that are antonyms with the italicized words in the following sentences.

 The incident was avoidable: Inevitable  Everyone likes to be praised: Censured  He is such a coward: Fearless

Find one-word substitutes for the following

 Teenage- Adolescent  Activities to nurture extracurricular skills-Hobbies  One who opts for a job in the church-Cleric  A starting point-Threshold

DDE, GJUS&T, Hisar 79 | English (Compulsory) BA-101

Find antonyms of the following words-

Word Antonyms

Assist Obstruct

Comfort Inconvenience

Gain Loss

Strive Slacken

Eloquent Tongue-tied

Give the noun forms of the following words-

Words Noun

Inevitable Inevitability

Progress Progression

Desirable Desirability

Decide Decision

Develop Development

3.9 ANSWERS TO YOUR PROGRESS

Adjective

An adjective is a word used to qualify a noun or a pronoun.

For example-

 Anil is an intelligent boy. (quality)  Arun ate two apples. (numeral)  I want some milk. (quantity)

3.9.1 Language Activity

DDE, GJUS&T, Hisar 80 | English (Compulsory) BA-101

Kind of Adjectives

Proper Distributive Interrogative Adjective Adjective Adjective

Descriptive Demonstrative Exclamatory Adjective Adjective Adjective

Quantitative Numeral Possessive Adjective Adjective Adjective

Proper Adjective- I love Shakespearean plays. (proper noun)

I love Italian food. (proper noun)

Descriptive Adjective- She is wearing a pink dress. (description)

He is a brave soldier. (description)

Quantitative Adjective- I have no money. (quantity)

She has enough time. (quantity)

Numeral adjective- I have ten rupees. (number)

Two boys were laughing at me. (number)

Demonstrative Adjective- This is my bag. (demonstrate)

Those flowers are beautiful. (demonstrate)

Distributive Adjective- Each boy has to present. (distribution)

Either she or her mother will come. (distribution)

DDE, GJUS&T, Hisar 81 | English (Compulsory) BA-101

Interrogative Adjective- Whose books are these? (question)

Which road should be taken? (question)

Exclamatory Adjective- What a beautiful dress! (exclamation)

What a piece of art! (exclamation)

Possessive Adjective- The dog is waging its tail. (possessive)

This is my bag. (possessive)

Comparison of adjective

Degree

Comparitive Degree

Superlative Degree

Madhu is a clever girl. (adjective)

Madhu is cleverer than her friend. (comparison between two people)

Madhu is the cleverest girl in our class. (comparison between more than two people)

Fill the blanks with a proper degree of adjectives given in the bracket

 Suresh is my old friend. (Old)

DDE, GJUS&T, Hisar 82 | English (Compulsory) BA-101

 Of these two books, this is better. (good) (Comparison between the two)  Preeti is the brightest of all the students. (bright) (Comparison between more than two)  Iron is more useful than any other metal. (useful) (Comparison)  Geeta is the best friend I have. (good) (Comparison between more than two)

Tenses

Present Perfect Tense

It shows an action done in the past and has its effect at present.

For example-

 I have done my work.  They have eaten their food.

Present Perfect Progressive Tense

It shows an action that began at some point in the past and is continuing at present

For example-

 Ritu has been working here since 2012.  Mukesh has been playing cricket for the last two hours.  I have been living in Hisar since I started schooling.

Fill the blanks with the present perfect continuous tense of the verbs-

 My brother is a writer. He writes a novel. He has been writing since 1920. (write)  I love learning languages. I have learned different languages. I have been learning languages since childhood. (learn) 3.9.2 Extended Composition

DDE, GJUS&T, Hisar 83 | English (Compulsory) BA-101

Prepare a report on the adventure trip you went on during the summer vacation.

I was very eager to go on a picnic. Some of my friends consented and we made a program to go near a lake. My brother also accompanied us. There we saw a very beautiful palace surrounded by water. Two strangers were talking there. The two men were talking about a hidden treasure. When they went for lunch, unfortunately, the map for the passage where the treasure was hidde n dropped from their belongings. To reach the palace we had to pass through much narrow passage and ultimately we succeeded in reaching the spot. Thus we found the treasure. Then we informed our parents and the police. That summer vacations are memorable for me.

3.10 REFERENCES/ SUGGESTED READINGS

 Hidalgo, Louise (August 23, 2011). "Dr. Spock's Baby and Child Care at 65". BBC News. Retrieved October 24, 2019.  Bart Barnes, Pediatrician Benjamin Spock Dies, The Washington Post, March 17, 1998; Page A01.  "Benjamin Spock -New Netherland Institute". New Netherland Institute. Retrieved October 24, 2019.  Biography of Spock at drspock.com  Kochakian, Mary Jo (June 14, 1998). "Public vs. Private: Dr. Spock, Mr. Hyde". The Washington Post.  Pace, Eric (March 17, 1998). "Benjamin Spock, World's Pediatrician, Dies at 94". The New York Times.

**************************

DDE, GJUS&T, Hisar 84 | English (Compulsory) BA-101

Course Code: BA101 Author: Dr.NutanYadav

Lesson No:04

Language and National Identity

Author Nirmal Verma

Lesson Structure

4.1 Learning Objectives

4.2 Introduction

4.3 Main Body of the Text

4.3.1 About the Author

4.3.2 About the Essay

4.4 Further Body of the Text

4.5 Check Your Progress

4.6 Summary

4.7 Keywords

4.8 Self -Assessment Questions (SAQs)

4.9 Answers to Your Progress

4.9.1 Language Activity

4.9.2 Extended Composition

4.10 References and Suggested Readings

DDE, GJUS&T, Hisar 85 | English (Compulsory) BA-101

4.1 LEARNING OBJECTIVES

After going through this lesson you will be able to-

 To develop reading, writing, speaking, and listening skills of the English language.  To enrich the taste of literature.  To develop a reading habit  Know the difference between poetry and prose.  To give grammar practice to students

4.2 INTRODUCTION

Language can be used in many ways; one of them is in specific purpose area such as social or cultural communication, government decisions, political debate, media which is linked to the ability to express the relevant content. Learne r or user of language is connected in time and space to cultural tradition which is considered as a significant process connected with the unique form of access to other traditions.

4.3 MAIN BODY OF THE TEXT

This text is an excerpt from his essay Language and National Identity, which reflects Verma’s concern with the language and the identity that is vested in it for the people of the nation. Discussing the necessity of language for constructing and asserting identity at all levels, the author, begins with the smallest unit of the individual language is the basis of self-discovery and analysis. In short, we need words to objectify ourselves and to share our feeling with others, as much as we need them to

DDE, GJUS&T, Hisar 86 | English (Compulsory) BA-101 think. We need language for our dreams and the shaping of our culture. As such it looms large over us, enveloping us in multiple ways.

4.3.1 About the author

Nirmal Verma (1929-2005)

He was a writer, novelist, activist, and translator. He is credited as being one of the pioneers of the Nai Kahani literary movement of Hindi literature. In his career spanning five decades and various forms of literature like a story, travelogue, and essays he penned fine novels, 8 short story collections, and nine books of nonfiction including essays and travelogues. He worked freely across genres and various ideological positions. He lived in Prague for ten years during which he translated nine world classics into Hindi. As a young man, he used to attend Gandhi’s prayer meetings in Berla House though he was also a card-holding member of the communist party. On his return to India, he shed his communist beliefs and his mind turned towards more philosophical and reflective concerns.

4.3.2 About the essay

This text is an excerpt from his essay Language and National Identity, which reflects Verma’s concern with the language and the identity that is vested in it for the people of the nation. Discussing the necessity of language for constructing and asserting identity at all levels, he begins with the smallest unit of the individual language is the basis of self-discovery and analysis. In short, we need words to objectify ourselves and to share our feeling with others, as much as we need them to think.

DDE, GJUS&T, Hisar 87 | English (Compulsory) BA-101

We need language for our dreams and the shaping of our culture. As such it looms large over us, enveloping us in multiple ways.

The important question is not how essential is language, but what colonialism has done to it diverting it from its natural path. Western thinkers and physiologists pushed the language of culture into the past but strangely enough also projected a picture of its continuity in other languages. The history/ histories of language are significant for understanding our own cultures.

4.4 FURTHER BODY OF THE TEXT

To say anything about language is like chasi ng your own shadow. It may be more appropriate to assert that we are its shadow; we create our reality with it and define ourselves through it, and with it. Man can never be defined fully, not only because there is something in him that is still unknown, but also because he has not yet completed the process of recreating and reinventing himself. We are unable to accept the man as a finished product. There has been much speculation about the manner, in which he may fulfill himself, the direction this fulfillment they may take, and the form it will acquire, and how he will project his innate potentialities in the future. Various scholars- from Nietzsche to Sri Aurobindo- have made predictions about his innate capacity. The question is not how to correct or relevant these prophecies are; the point to consider is that man being what he is, all these predictions are born of a painful disillusionment with what he is.

Man is the only creature- between a god and an animal- who finds himself in a strange situation of being incomplete; he lacks the self, the involvement, of the animal and does not possess the fullness of God. Even in the historical framework, the man appears as a figure half-drawn and incomplete, as if thwarted in the process of acquiring a proper shape, its colors lackluster and faded. The ‘full figure’ of man- if there is such a thing- is not without, but buried deep within him. To realize himself, a man digs ‘within’ and raises himself from his own grave.

DDE, GJUS&T, Hisar 88 | English (Compulsory) BA-101

Language is the most efficient weapon of self-discovery or self-analysis. It is that invisible part of the human body, which gives man an inner sight. The link between language and self-awareness puts man in a separate and unique category from all other living beings. This consciousness of his incompleteness makes him yearn for fulfillment…. Thus, the determination to attain it is also intrinsic to its realization. No culture is limited only to its reality, but also reveals itself through its dreams, and that is why language plays a very significant role in the shaping of culture.

The vision of a culture is largely formed by the memories that it carries. If the word is symbolic of a dream, it also has the hint of the memory. No language, as long as it is, can, therefore, be a dead language. If all else our past were to perish, even then the languages would remain through which the members of society communicate with each other, and though living in the present, are unconsciously linked with their past, and the past flows imperceptibly through the present. Language, in this manner, plays a double role; besides being the vehicle of communication, it is also the carrier of culture. Historical upheavals may impair the culture of a country, but its truth and cohesion persist in its language.

Each language, as it develops, establishes a special constitution and structure, which, once having come into existence, influences the inner responses of man. Both processes go on simultaneously; language determines the identity and the special characteristics of a race, and the collective experiences, memories, and traditions of a group mold the unique character of its language. This is, in a way, a circular process, in which the question of what comes first remains unanswered, like in the case of the chicken and egg. What is clear is that there is nothing like one language; there are numerous languages. And the various races, groups, and nationalities are nurtured by their languages; languages are their residing places. In this sense, each language in its ultimate form is a mother tongue.

Man is drawn to the world for the first time through language, opens up, and is linked to it. It is not only a part of the external day-to-day activity of a group, a part

DDE, GJUS&T, Hisar 89 | English (Compulsory) BA-101 of its supra structure (although it has a specific role in achieveme nts of its worldly goals), it also decides its inner nature, impulses, and sensibility, and demonstrates its unique spiritual character. One language differs from another, not only in its sounds, intonations, and the meaning its words convey, but also in its ‘world view’.

Each language looks at the universe in a particular manner, tests it, and translates it into its meanings in a special way. The connotations that the Rig Veda conveys through the symbols , , Dhenu, and Space to the Indian mind, and the images of certain divine forces that it conjures up, are distinct in their import from the Greek and Latin symbols, different because mythical sources and references are distinct and separate. Human languages are born in the same world, but each has its incomparable quality; manifest and well- defined. When we translate from one language to another, it is not just a translation of the words alone, but the transformation of a whole universe, with all its baggage, into another language.

Each language has its history, and also a geography of its own. We cannot separate its symbolic character from its natural environment. If symbols like Ganga, , Himalaya, ocean, and space are deeply entrenched in the Indian languages and profoundly influence our sensibilities, it is not because they are the physical attributes of our natural habit, but because they are the carriers of our legendary memories which at one time linked Indians to their deities and lent a divine grace to their material existence. The meaning of the word, ‘Bharat’ was its essence- a region surrounded by divinity. The reason why ‘Bharat’ was called ‘Dev ’- the land of gods- and was ‘Dev Bhasha’- the language of gods- was to underline the eternal and sacred relationship betwe en the land and language.

Language, with all its metaphors and imagery, played a significant role in kindling the national awakening in the 19th century in India. The novels of Bankim Babu, Sri Aurobindo’s philosophy, Gandhi’s Hind Swaraj, and Coomaraswamy's thoughts on art are ample evidence of this. When a handful of historians call the concept of Indian nationalism a mere ‘construct’, using the vocabulary of French philosophers,

DDE, GJUS&T, Hisar 90 | English (Compulsory) BA-101 where the very concept of ‘Bharat’ is labeled an imaginary product or ‘invented construct’ of some nationalist historians, then the linguistic and cultural foundations of Indian nationalism are overlooked, which is a living example of colonial forgetfulness. In India, the desire to seek freedom was inspired not only by political factors, but there was also the desire to retrieve our cultural identity from the darkness of oblivion.

From this point of view, it was different from the aggressive, egoistic sentiments that lay behind the modern nation-states, which came into existence only after the various races, nationalities, and folk languages were annihilated. That the Indian national awareness was free of self-involved rigidity right from its inception was because it had been forcibly imposed from above, but its roots lay deep in the Indian cultural traditions. The various dialects and languages of India, despite their distinctiveness, shared an inherent unity, and this could never obstruct the growth of national unification. Their diversity gave them openness- they lived in the same house; though the windows were different, their casements opened on a full panorama of the interrelationship of Indian culture. It is often said that we seek ourselves in our languages; perhaps, it happens in some cultures that they tell their stories in the language of dreams. The reality of the national identity of Bharat is perhaps reflected in such dream languages.

4.5 CHECK YOUR PROGRESS

Answer the following questions in 25-30 words

1. Why does the author say that it may be more appropriate to assert that we are the shadow of language? 2. Why can a man be not accepted as a finished product? 3. Why does the author say that a language cannot be a dead language while it exists? 4. How does one language differ from another? 5. How do symbols like ‘Ganga’ influence people ?

DDE, GJUS&T, Hisar 91 | English (Compulsory) BA-101

Answers

1. The writer says that man is the shadow of language. We define ourselves through it. Man creates his reality with it. So, man can never be defined fully because there is something in him that is still unknown. 2. Man can never be defined fully because there is something in him that is still unknown. Secondly, he has not yet completed the process of recreating and reinventing himself. So we are unable to accept the man as a finished product. 3. The vision of a culture is largely formed by the memories that it carries. If the world is symbolic of a dream, it also has the hint of the memory. No language, as long as it is, can, therefore, be a dead language. 4. One language differs from another, not only in its sounds, intonations, and the meanings its words convey, but also in its ‘world view’. Each language looks at the universe in a particular manner, tests it, and translates it into its meanings in a special way. Human languages are born in the same world, but each has its incomparable quality, manifest and well- defined. 5. Each language has its history, and also a geography of its own. Symbols like Ganga, Saraswati, Himalaya, ocean, and space are deeply entrenched in the Indian languages and profoundly influence our sensibilities.

Answer the following questions in 75-100 words

1. What role does language play in the national awakening of India? 2. What relation does the author establish between man and language? 3. Recount the author’s arguments about the diversity of cultures in India.

Answers

1. According to the author, language plays an important role in a nation’s major social and political events. Language played a significant role in the national awakening in the 19th century in India. The novels of Bankim Babu, Sri Aurobindo’s philosophy, Gandhi’s Hind Swaraj, and

DDE, GJUS&T, Hisar 92 | English (Compulsory) BA-101

Coomaraswamy's thoughts on art are evidence of this. Often the linguistic and cultural foundations of Indian nationalism are overlooked. In India, the desire to seek freedom was inspired not only by political factors. But there was also the desire to bring out our cultural identity from the darkness of ignorance. 2. The writer says that man is the shadow of language. We define ourselves through our languages. There is a deep relation between man and language. Each language influences the inner responses of man. Both processes go on simultaneously. Language determines the identity and the special characteristics of a race The collective experiences, memories, and traditions of a group mold the unique character of its language. What is clear is that there is nothing like one language; there are numerous languages. And the various races, groups, and nationalities are nurtured by their languages. 3. India has a diversity of cultures. This cultural diversity makes India a unique country. divers culture in India uses diverse languages. Different languages and dialects are spoken in the different parts of our country. But the various dialects and languages of India shared an inherent unity, and this could never obstruct the grown of national unification. Their diversity gave them openness. Our languages provide a full panorama of the inter- relationship of Indian cultures. We seek ourselves in our languages. Perhaps, it happens in some cultures that they tell their stories in the language of dreams.

Comprehensive Passage

Passage 1

To say anything about language is like chasing your own shadow. It may be more appropriate to assert that we are its shadow; we create our reality with it and define ourselves through it, and with it. Man can never be defined fully, not only because

DDE, GJUS&T, Hisar 93 | English (Compulsory) BA-101 there is something in him that is still unknown, but also because he has not yet completed the process of recreating and reinventing himself. We are unable to accept the man as a finished product. There has been much speculation about the manner, in which he may fulfill himself, the direction this fulfillment they may take, and the form it will acquire, and how he will project his innate potentialities in the future. Various scholars- from Nietzsche to Sri Aurobindo- have made predictions about his innate capacity. The question is not how to correct or relevant these prophecies are; the point to consider is that man being what he is, all these predictions are born of a painful disillusionment with what he is.

Questions-

1. What does the writer say about language? 2. Why can a man never be defined fully? 3. Who have made speculations about man’s innate capacity? 4. What are the predictions about man made of?

Answers-

1. The writer says that man is the shadow of language. 2. Man can never be defined fully because there is something in him that is still unknown. 3. Various scholars have made predictions about man’s innate capacity. 4. These predictions are born of a painful disillusionment with what man is.

Passage 2-

Man is the only creature- between a god and an animal- who finds himself in a strange situation of being incomplete; he lacks the self, the involvement, of the animal and does not possess the fullness of God. Even in the historical framework, the man appears as a figure half-drawn and incomplete, as if thwarted in the process of acquiring a proper shape, its colors lackluster and faded. The ‘full figure’ of man- if

DDE, GJUS&T, Hisar 94 | English (Compulsory) BA-101 there is such a thing- is not without, but buried deep within him. To realize himself, a man digs ‘within’ and raises himself from his own grave.

Questions

1. Why is the man in a strange position? 2. How is the man between an animal and a god? 3. How does man appear in the historical framework? 4. Where is the full figure of a man?

Answers-

1. Man is in a strange position of being incomplete. 2. He lacks the involvement of the animal and does not possess the fullness of God. 3. In the historical framework, a man appears as a figure half-drawn and incomplete. 4. The ‘full figure’ of man is not without but buried deep within him.

Passage 3-

Man is drawn to the world for the first time through language, opens up, and is linked to it. It is not only a part of the external day-to-day activity of a group, a part of its superstructure (although it has a specific role in achievements of its worldly goals), it also decides its inner nature, impulses, and sensibility, and demonstrates its unique spiritual character. One language differs from another, not only in its sounds, intonations, and the meaning its words convey, but also in its ‘world view’.

Questions-

1. How is a man drawn to the world for the first time? 2. What decides man’s inner nature, impulses, and sensibility? 3. What has a specific role in the achievements of man’s worldly goals? 4. How does one language differ from another?

Answers-

DDE, GJUS&T, Hisar 95 | English (Compulsory) BA-101

1. Man is drawn to the world for the first time through language. 2. Language decides man’s inner nature, impulses, and sensibility. 3. Language has a specific role in the achievements of man’s worldly goals. 4. One language differs from another, not only in its sounds, intonations but also in its world view.

Passage 4

The vision of a culture is largely formed by the memories that it carries. If the word is symbolic of a dream, it also has the hint of the memory. No language, as long as it is, can, therefore, be a dead language. If all else our past were to perish, even then the languages would remain through which the members of society communicate with each other, and though living in the present, are unconsciously linked with their past, and the past flows imperceptibly through the present. Language, in this manner, plays a double role; besides being the vehicle of communication, it is also the carrier of culture. Historical upheavals may impair the culture of a country, but its truth and cohesion persist in its language.

Questions-

1. How is the vision of a culture formed? 2. What would happen if everything else perished? 3. How does language play a double role? 4. What persists in the language of a country?

Answers-

1. The vision of a culture is largely formed by the memories that it carries. 2. If everything else perished, even then language would survive. 3. Besides being the vehicle of communication, it is also the carrier of culture. 4. A country’s truth and cohesion persist in its language.

4.6 SUMMARY

DDE, GJUS&T, Hisar 96 | English (Compulsory) BA-101

The author says that self-identity and language are like the shadow of each other. We defined our self through it. It creates a reality with it. Man can never be defined fully because there is something in him that is still unknown. He has not yet completed the process of recreating and reinventing himself. Man is unaccepted as a finished product. Various scholars have made predictions about his innate capacity. Man is the only creature who finds himself in a strange situation of being incomplete. Even in the historical framework, the man appears as a figure half drawn and incomplete.

Language is the most efficient weapon of self-discovery or self-analysis. It gives man an inner sight. The link between language and self-awareness puts man in a separate and unique category from all other living beings. No culture is limited only to its reality but also reveals itself through its dreams. That is why language plays a very significant role in the shaping of culture.

The vision of culture is largely formed by the memories that it carries. No language as long as it is, can therefore be a dead language. Through language, we are unconsciously linked with the past. Thus language plays a double role. It is the vehicle of communication as well as the carrier of culture.

There is a deep relationship between man and language. Each language influences the inner response of man. Both processes go on simultaneously. Language determines the identity and the special characteristics of a race.

The collective experiences, memories, and traditions of a group mold the unique character of its language. What is clear is that there is nothing like one language, there are numerous languages. And the various races, groups, and nationalities are nurtured by their language.

One language differs from others not only in its sounds, intonations, and meaning but also in its ‘world view’. Each language looks at the universe in a particular manner, tests it, and translates it into its meaning especially. Human languages are

DDE, GJUS&T, Hisar 97 | English (Compulsory) BA-101 born in the same world but each has its incomparable quality, manifest, and well- defined.

Each language has its history and geography. Symbols like Ganga, Saraswati, Himalaya, ocean, and space are deeply entrenched in the Indian languages and profoundly influence our sensibility. Indian languages are the carriers of our legendary memories which at one time Indian to their duties and lent a divine grace to their material existence. The meaning of the word ‘Bharat’ was its essence - a region surrounded by divinity.

The reason Bharat was called ‘Dev Bhumi’- the land of gods- and Sanskrit was ‘Dev Bhasha’- the language of gods- was the underline the eternal and sacred relationship between the land and the language.

Language played an important role in the national awakening in the nineteenth century in India. The novels of Bankim Babu, Sri Aurobindo’s philosophy, Gandhi’s Hind Swaras, and Coomaraswamy's thoughts on art are amply evidence of this. Often the linguistic and cultural foundations of Indian nationalism are overlooked. In India, the desire to seek freedom was inspired not only by political factors, but there was also the desire to bring out our cultural identity from the darkness of ignorance.

India has a diversity of cultures. But the various dialects and languages of India shared an inherent unity and this could never obstruct the growth of national unification.

India has a diversity of cultures. But the various dialects and languages of India shared an inherent unity. This diversity gave them openness. Our language provides a full panorama of the interrelationship of Indian culture. We seek ourselves in our languages perhaps it happens in some cultures that they tell their stories in the language of dreams.

4.7 KEYWORDS

Phonetic Transcription of following words-

DDE, GJUS&T, Hisar 98 | English (Compulsory) BA-101

 Shadow  Create  Manner  Define  Fulfill  Point  Capacity  Form  Project  Grave  Creature  What  God  Reality  Historical  Drawn  Figure  These  Recreating  Direction  Disillusion  consider

4.8 SELF ASSESSMENT QUESTIONS-(SAQ’S)

Find words from the essay that is synonymous with the italicized words in the following sentences.

Words Synonymous

Run after Chase

DDE, GJUS&T, Hisar 99 | English (Compulsory) BA-101

Obtain Acquire

Inborn Intrinsic

Well and successfully Perfectly

Disclose Reveal

Damaged Impaired

Find words from the essay that are antonyms with the italicized words in the following sentences.

Words Antonyms

Inappropriate Appropriate

Destroy Created

Incorrect Correct

Perceptible Imperceptibly

Mix Separate

Dropped Risen

Find words from the essay that are antonyms with the italicized words in the following sentences

Words Antonyms

More Less

Accept Reject

Strange Common

Unique Ordinary

Flow Stagnant

DDE, GJUS&T, Hisar 100 | English (Compulsory) BA-101

Expand Contract

4.9 ANSWERS TO YOUR PROGRESS

Explain the following words/terms-

 Metaphor- a figure of speech in which an expression is used to refer to something that it doesn’t denote to suggest a similarity  Imagery-the formation of mental images, figures, or likenesses of things, or of such images collectively.  Eternal-without beginning or end; lasting forever  Divinity-the quality of being divine  Culture –the behaviors and belief characteristic of a particular social, ethnic, or age group  Egoistic-being centered in or preoccupied with oneself  Intrinsic-belonging to or part of the real nature of something or somebody  Innate –inborn  Symbol- a thing which stands for something else  Habitat-the place where a particular type of animal and plant is normally found 4.9.1 Language activity

Verb-

DDE, GJUS&T, Hisar 101 | English (Compulsory) BA-101

sentence (Clause)

Verb Group NounGroup (It tells what sort of (It refers to the person or action,process or state we thing we are talkingabout) are talking about)

For Example-

 I play.  She plays.  They are playing.  She has played.  They have played.  They have been playing.  She was playing.  She had played.  She had been playing.  She will play.  She will be coming.  She will have come.  She will have been playing.  She may play.

DDE, GJUS&T, Hisar 102 | English (Compulsory) BA-101

 She would play.

Verb

Main Verb Auxillary Verb For example For Example see, sees, seeing, saw, Do, Be, Have seen

Forms of Verb

Transitive/ Intransitive Verb

Phrasal Verb Reflexive Verb

Finite/ Non Compound finite Verb Verb

DDE, GJUS&T, Hisar 103 | English (Compulsory) BA-101

Transitive Verb- The election raised several issues.

- I put my hand on the door.

Intransitive Verbs- My whole body ached.

- Everything you see here belongs to me.

Reflexive Verb- He prided himself on his tidiness.

- After the meeting, he introduced himself to the members.

Finite Verb- She is a good painter.

- She sells her painting in Europe.

Non-Finite Verb- They want to hire a trainer.

- The invitations were sent printed in gold.

Phrasal Verb- They agreed to let him into their secret.

- Come on. I’ll show you around.

Compound Verbs-I babysits when my wife goes out.

- It is not advisable to hitchhike on your own.

Main and Auxiliary Verb- Do you read a newspaper?

- Does he drive carefully? - Did she come yesterday?

4.9.2 Extended Composition

Write a paragraph on the importance of one’s mother tongue

“Mother Tongue” is the language that a person learns from his infancy. It is the first language that a person learns so that he could socially engage with his parents and other relatives. Language is not only used to communicate but also to a huge extent; it is used to preserve the culture and refinement of their race.

DDE, GJUS&T, Hisar 104 | English (Compulsory) BA-101

Nowadays, the mother tongue is losing its importance against other languages. Although English is incomparable with any other languages, it is causing the death of some of the languages. The inseparable part of one’s personality that should never be lost is one mother language. To preserve the rich cultural aspect of a person’s existence the mother tongue should be preserved at any cost. Mother's tongue is very crucial for framing one’s thinking and emotion. Learning one’s mother tongue gives them a sense of self-worth and cultivates confidence in them. For a child to be able to participate in their classes and grow communication skills, the child must learn his mother tongue from his parents.

4.10 REFERENCES/ SUGGESTED READINGS  Nirmal Verma, 1929–2005 Archived 24 June 2008 at the Wayback MachineFrontline, Volume 22 – Issue 24, Nov. 19 – Dec. 02, 2005.  Dwivedi, Ram Prakash (16 October 2020). "CCGS International Journal". Journal.globalculturz.org. Retrieved 17 October 2020.  Ode to Nirmal Verma The Hindu, 6 November 2005.  Nirmal Verma, India. Lettre-ulysses-award.org. Retrieved on 22 May 2016.  AUTHOR SPEAKS: "I cater to several layers of sensibilities" The Tribune, 10 March 2002.

*********************

DDE, GJUS&T, Hisar 105 | English (Compulsory) BA-101

Course Code: BA101 Author: Dr.NutanYadav

Lesson No:05 Wounded Plants

Wounded Plants

Author Jagadish Chandra Bose

(B) Playing the English Gentleman

Lesson Structure

5.1 Learning Objectives

5.2 Introduction

5.3 Main Body of the Text

5.3.1 About the Author

5.3.2 About the Essay

5.4 Further Body of the text

5.5 Check your Progress

5.6 Summary

5.7 Keywords

5.8 Self -Assessment Questions (SAQs)

5.9 Answers to Your Progress

5.10 References and Suggested Readings

5.1 LEARNING OBJECTIVES

After going through this lesson you will be able to-

DDE, GJUS&T, Hisar 106 | English (Compulsory) BA-101

 To develop reading, writing, speaking, and listening skills of the English language.  To enrich the taste of literature.  To develop a reading habit  Know the difference between poetry and prose.  Trained in critically analyzing phonetic transcription of a sound system.

5.2 INTRODUCTION

It’s a hard life being a plant! From pathogens and herbivores to unfavorable weather, they are constantly injured or wounded and their sessile lifestyle only adds to the trouble. Despite all this, they heal and survive. How did they evolve such a great repair mechanism? What are the molecules and proteins aiding it? These were some of the questions an international team of researchers set out to answer and have now found some interesting mechanisms. Though it was known that plants communicate within their different parts and with other plants during injury, not much was known about how they handle the injury.

Wounded Plants

5.3 MAIN BODY OF THE TEXT

DDE, GJUS&T, Hisar 107 | English (Compulsory) BA-101

Introduction

Sir Jagadish Chandra Bose was a pioneer of modern science and scientific experiments in India. Dr. Bose realized the pain of wounded plants, the effect of one part of the wound on the other part of wounds, and how they heal themselves. He studied throughout his life and documented it in his various re search papers and books. This chapter gives a brief description of his findings in this field.

5.3.1 About the Author

Sir Jagadish Chandra Bose was a pioneer of modern science and scientific experiments in India. He was a biologist, physicist, botanist, and an early writer of science fiction. He pioneered the investigation of radio and microwave optics made significant contributions to plant science and laid the foundations of experimental science in the Indian subcontinent. IEEE named him one of the fathe rs of radio science. He is also considered the father of Bengali science fiction and invented the crescograph, a device used to measure the growth of plants. He founded Bose Institute, a premier research institute of India and also one of its oldest. Established in 1917, the institute was the first interdisciplinary research center in Asia. He served as the Director of Bose institute from its inception until his death. His contributions to plant sciences remain significant today. His books include ‘Response in the Living and Non Living’(1902), ‘The Nervous Mechanism of Plants’(1926). In a 2004 BBC Poll, Bose was voted seventh Greatest Bengali of all time.

He compared the response of metals, plants, and animals to electrical, chemical, and mechanical stimulations and documented them in his book ‘Response in the Living and Non Living’ (1902). He proved that the plants feel pain and understand affection and other feelings.

5.3.2 About the Essay

DDE, GJUS&T, Hisar 108 | English (Compulsory) BA-101

The author starts his essay with a description of the tragedy during the First World War. This tragedy stalked over Western Europe. In this tragedy, a great number of human lives were lost. The fair fields of France and the bright sky was under a pall of battle smoke.

After giving the example of this tragedy, the writer says that we feel sympathy for those who died. Sympathy is that bond by which we realize the unity of all life. Multitudinous plants are surrounding us. Like the loss of human lives, there is the loss of many trees and plants also. Plants also suffer many shocks in life. But plants don’t make an outcry because they are mute.

When a man receives a shock or a shock of any kind, his answering cry makes us realize that he is hurt, but a mute make no outcry. His pain can be realized by the convulsive movement of his limbs or his agonized looks. We can also feel this pain through fellow feeling (sympathy). The author gives the example of a frog. But it is a pity that we don’t realize the pain of some living being because we have no sympathy for those creatures that are lower than us. Human beings doubt whether the lowly and the depressed also possess the fine sense of the exalted to feel the same joy and sorrow. Another feature that differentiates between them is the level of sympathy. Our sympathy is extended to those who are higher or equal to us. But it is hardly ever extended to the inferior. In this way, human attitude is discriminative to different grades of his species.

Test of livingness

The author gives a test to measure the livingness of creatures. He says that it depends on the test of Responsive movements. This responsive movement shows the difference between the living and the dead. The living answers to the shock from within, the liveliest gives the most energy. The answer of the strong is violent but the weakling will barely protest. But we are subject to change. Someday we are in a state of high exuberance and other times in a state of lowest depression. The total of these entire characteristic differentiates one individual from another.

DDE, GJUS&T, Hisar 109 | English (Compulsory) BA-101

Three separate investigations were carried out on the effect of a wound on plants-

1. First is the shock effect of the wound on growth. It retards or stops growth. 2. The second series of investigations show that death begins to spread from the cut end to the leaflet, and reaches the throbbing tissue which becomes permanently stilled on death. 3. The third investigation shows the experiments made to show the march of death. The cut leaflet which died in 24 hrs has now been kept alive for more than one week.

Paralysis of sensibility

Another series of investigations were carried out on the paralyzing effect of a severe wound. The paralysis of sensibility means that a plant remains depressed and irresponsive for several hours. The author describes an experiment on the Mimosa plant. A leaf of Mimosa was cut off from the plant remains depressed and irresponsive for several hours. The cutting of one leaf has caused a great shock to the parent plant, and an intense excitation spreads over to the distant organs. All the leaves remained depressed and irresponsive for several hours. But the detached leaf when placed in a nourishing solution soon recovers and held up his head with an attitude indicative of defiance, and the responses it gave were energetic. This lasted for 24 hours and after which a curious change crept in and the vigor of its responses began rapidly to wane. The leaf hither to erect fell over and at last, it died.

5.4 FURTHER BODY OF THE TEXT

It is a little over four years now that the Embodiment of World Tragedy stalked over Western Europe. The fair field of France and the bright sky was under a pall of battle- smoke. Our sight could not penetrate through the dense gloom, and the mortal cry of the wounded and dying, drowned by the hoarse roar of a thousand did not reach our ear. But from the time the Sikh and the Pathan, the Gurkha and the

DDE, GJUS&T, Hisar 110 | English (Compulsory) BA-101

Bengali, the Mahratta and the Rajput flung themselves in front of battle from that day our perception has become intensified. The distant cry of those whose lifeblood has crimsoned the white field of snow has found a reverberating echo in our hearts.

What is that subtle bond by which all distances are bridged over, and by which an individual life becomes merged in larger life? Sympathy is that bond by which we come to realize the unity of all life. We see the spread of multitudinous plants, silent and seemingly impassive. They too like us are actors in the Cosmic drama of life, the plaything of destiny. In their checkered life, light, and darkness, the warmth of summer and frost of winter, drought and rain, the gentle breeze and whirling tornadoes, life and death alternate. Various shocks impinge in them, but no cry is raised in the answer. I shall nevertheless try to decipher some chapters of their life history.

When a man receives a blow or shock of any kind, his answering cry makes us realize that he is hurt, but a mute make no outcry. How do we realize his sufferings? We know it by agonized look by the convulsive movement of his limbs, and through fellow- feeling realize his pain. When a frog is struck it does not cry, but its limbs show convulsive movement. But from this, it does not follow that the frog is not hurt, for some would urge that there is a great gap between us and the lower animals. One who feels for the humblest of His creatures alone knows whether the frog is hurt or not. Human sympathy always aspires: it is sometimes extended to equals, hardly ever to inferiors. And so it happens that many would doubt, whether the lowly and the depressed possess the fine sense of the exalted to feel the same joy and sorrow and to resent social tyranny.

When the human attitude is so finely discriminative as regards different grades of his species, it might be extravagant to believe that the frog could have any conscious of pain. A concession might however be made that the frog perceives a shock to which it responds by convulsive movements. It is as well that we should be careful about the use of terms for an eminent biologist insisted that animals never felt pain: when

DDE, GJUS&T, Hisar 111 | English (Compulsory) BA-101 an oyster is swallowed alive, it did not, according to him, feel any pain but rather a sensation of grateful warmth at contact with the alimentary tract. The question will remain undecided for no one has as yet returned from the gastric cavity of the tiger to expatiate on the exquisite sensation.

Responsive movements being a test of life, we shall try to construct a scale with which the height of livingness may be measured. What is the difference between the living and the dead? The living answers to the shock from within; the livelie st gives the most energetic, the torpid or dying the feeblest, and the dead no answer at all. Thus life may be tested by shocks from within, the size of the answer is the gauge of vitality. The answer of the strongest will be violent and almost explosive i n intensity, while the weakling will barely protest. The responsive movements may be recorded by a suitable apparatus. The successive responses to similar shocks will remain uniform if the living tissue always remained the same.

But the living organism is always in a state of change for the environment is always building us anew, and we are changing every day of our life. We are thus subject to change, someday we are in a state of high exuberance, and ‘other time in a state of lowest depression: we pass through numerous phases between two extremes. Not merely does the present modify, but there is also the subtle impression of memory of the past. The total of all these characterizes one individual from another. How is the hidden to be made manifest? To test the genuineness of a coin, we strike it and the sound response betrays the true from the false. The genuine rings true and the other gives a false note. In this way perhaps the inner history of different lives may be revealed by shocks and the resulting response.

Three separate investigations have been carried out on the effect of the wound on plants: The first is the shock effect of the wound on growth: this generally speaking retards or arrests growth. In the second series of investigations, the change of spontaneous pulsation of the leaflet of the Telegraph plant was recorded. Death begins to spread from the cut end of the leaflet, reaches the throbbing tissue which

DDE, GJUS&T, Hisar 112 | English (Compulsory) BA-101 becomes permanently stilled on cessation of life. Experiments are in progress of arrest their march of death, and the cut, leaflet which died in 24 hours has now been kept alive for more than a week.

Another series of investigations were carried out on the paralyzing effect of a severe wound. A leaf of Mimosa was cut off from the plant, and the subsequent histories of the wounded plant and the detached leaf are curiously different. The cutting of one of its leaves had caused a great shock to the parent, and an intense excitation spreads over to the distant organs. All the leaves remained depresse d and irresponsive for several hours. From this state of the paralyzed sensibility, the plant gradually recovers and the leaves begin to show returning sensitiveness. The detached leaf, when placed in a nourishing solution soon recovers, and holds up its head with an attitude indicative of defiance, and the responses it gives are energetic. This lasts for twenty-four hours, after which a curious change creeps in the vigor of its responses begins rapidly to wane. The leaf hitherto erect falls over; death had, at last, asserted its mastery.

5.5 CHECK YOUR PROGRESS

Answer the following questions in 25-30 words

1. How can we realize the pain of the sufferings of the mute? 2. Why do we not realize the pain of some living beings? 3. What characterizes individuals from one another?

Answers-

1. When a man receives a shock or blow of any kind, his answering cry makes us realize that he is hurt. But a mute make no outcry. We can know by his agonized look or by the convulsive movement of his limbs. We can also realize this pain by fellow-feeling. 2. We do not realize the pain of some living beings because we have no sympathy for those creatures that are lower than us. So the human beings doubt

DDE, GJUS&T, Hisar 113 | English (Compulsory) BA-101

whether the lowly or the depressed possess the fine sense of the exalted to feel the same joy and sorrow, and to resent social tyranny. 3. The degree of shock and reaction to pain characterizes individuals from one another. Another feature that differentiates between them is the level of sympathy. Our sympathy is extended to those who are higher or equal to us. But it is hardly extended to the inferiors.

Answer the questions in 75-100 words

1. What is the ‘second series of investigation’ referred to in the essay? 2. What is ‘paralysis of sensibility’? How is it measured? 3. What is the significance of these three series of investigations?

Answers-

1. The scientists carried out three separate series of investigations to study the effect of the wound on plants. In the second series of investigations, the change of spontaneous pulsation of the leaflet of the Tele graph plant was recorded. Death begins to spread from the cut end of the leaflet and reaches the throbbing tissue which becomes permanently stilled on death. Experiments are being made to slow the march of death. The cut leaflet which died in 24 hours had been kept alive for more than one week. 2. The paralysis of sensibility means that a plant remains depressed and irresponsive for several hours. Several investigations were carried out to see the paralyzing effect of a severe wound on a plant. A leaf of Mimosa was cut off from a plant, and the subsequent histories of the wounded plant and the detached leaf curiously. The cutting of one of its leaves had caused a great shock to the parent plant, and an intense excitation spreads over to the distant organs. All the leaves remained depressed and irresponsive for several hours. 3. Three separate investigations were carried out on the effect of wounds on plants. These experiments were conducted to see the effect of the wound on

DDE, GJUS&T, Hisar 114 | English (Compulsory) BA-101

plants. The first investigation was the shock effect of the wound on growth. In the second investigation, a change of spontaneous pulsation on a leaf was recorded. The third investigation was carried out to see the effect of the paralyzing effect of the wound on plants. This series of investigati ons showed that plants also have life and the response to wounds or shocks. The experiments showed that the various parts of a plant experience death just as living beings experience.

Comprehension passage

Passage 1

Three separate investigations have been carried out on the effect of the wound on plants: The first is the shock effect of the wound on growth: this generally speaking retards or arrests growth. In the second series of investigations, the change of spontaneous pulsation of the leaflet of the Telegraph plant was recorded. Death begins to spread from the cut end of the leaflet, reaches the throbbing tissue which becomes permanently stilled on cessation of life. Experiments are in progress of arrest their march of death, and the cut, leaflet which died in 24 hours has now been kept alive for more than a week.

Questions-

1. How many separate investigations have been carried out on the effect of the wound on plants? 2. What was the first investigation about? 3. On which plant was the investigation carried on? 4. When did the death become permanently stilled?

Answers-

1. Three separate investigations have been carried out on the effect of wound plants 2. The first investigation was about the shock effect of the wound on growth.

DDE, GJUS&T, Hisar 115 | English (Compulsory) BA-101

3. It was carried out on the Telegraph plant. 4. The death becomes stilled permanently on cessation of life.

Passage 2

What is that subtle bond by which all distances are bridged over, and by which an individual life becomes merged in larger life? Sympathy is that bond by which we come to realize the unity of all life. Before we are spread multitudinous plants, silent and seemingly impassive. They too like us are actors in the Cosmic drama of life, the plaything of destiny. In their checkered life, light, and darkness, the warmth of summer and frost of winter, drought and rain, the gentle breeze and whirling tornadoes, life and death alternate. Various shocks impinge in them, but no cry is raised in the answer. I shall nevertheless try to decipher some chapters of their life history.

Questions-

1. What is the bond with which all distances are bridged over? 2. How do we realize the unity of life? 3. How did the plants look like? 4. Who does not raise a cry in answer to shocks?

Answers-

1. Sympathy is the bond with which all distances are bridged over. 2. We realize the unity of life through sympathy. 3. They looked like the actors in the Cosmic drama of life. 4. Plants do not raise a cry in answer to shocks.

Passage 3

When the human attitude is so finely discriminative as regards different grades of his species, it might be extravagant to believe that the frog could have any conscious of pain. A concession might however be made that the frog perceives a shock to which it responds by convulsive movements. It is as well that we should be careful about

DDE, GJUS&T, Hisar 116 | English (Compulsory) BA-101 the use of terms for an eminent biologist insisted that animals never felt pain: when an oyster is swallowed alive, it did not, according to him, feel any pain but rather a sensation of grateful warmth at contact with the alimentary tract. The question will remain undecided for no one has as yet returned from the gastric cavity of the tiger to expatiate on the exquisite sensation.

Questions-

1. What makes us realize that a man is hurt? 2. How do we know the suffering of a mute? 3. Does a frog cry when it is struck? 4. What does the author say of human sympathy?

Answers-

1. His outcry makes us realize that a man is hurt. 2. We know the suffering of a mute by his convulsive movements. 3. No, it does not cry. 4 He says that human sympathy always aspires

5.6 SUMMARY

The writer says that we feel sympathy for those who died. Sympathy is that bond by which we realize the unity of all life. Multitudinous plants are surrounding us. Like the loss of human lives, there is the loss of many trees and plants also. Plants also suffer many shocks in life. But plants don’t make an outcry because they are mute.

When a man receives a shock or a shock of any kind, his answering cry makes us realize that he is hurt, but a mute make no outcry. His pain can be realized by the convulsive movement of his limbs or his agonized looks. We can also feel this pain through fellow feeling (sympathy). The author gives the example of a frog. But it is a pity that we don’t realize the pain of some living being because we have no sympathy for those creatures that are lower than us. Human beings doubt whether the lowly and the depressed also possess the fine sense of the exalted to

DDE, GJUS&T, Hisar 117 | English (Compulsory) BA-101

feel the same joy and sorrow. Another feature that differentiates between them is the level of sympathy. Our sympathy is extended to those who are higher or equal to us. But it is hardly ever extended to the inferior. In this way, human attitude is discriminative to different grades of his species. The author gives a test to measure the livingness of creatures. He says that it depends on the test of Responsive movements. This responsive movement shows the difference between the living and the dead. The living answers to the shock from within, the liveliest gives the most energy. The answer of the strong is violent but the weakling will barely protest. But we are subject to change. Someday we are in a state of high exuberance and other times in a state of lowest depression. The total of these entire characteristic differentiates one individual from another. The paralysis of sensibility means that a plant remains depressed and irresponsive for several hours. The author describes an experiment on the Mimosa plant. A leaf of Mimosa was cut off from the plant remains depressed and irresponsive for several hours. The cutting of one leaf has caused a great shock to the parent plant, and an intense excitation spreads over to the distant organs. All the leaves remained depressed and irresponsive for several hours. But the detached leaf when placed in a nourishing solution soon recovers and held up his head with an attitude indicative of defiance, and the responses it gave were energetic. This lasted for 24 hours and after which a curious change crept in and the vigor of its responses began rapidly to wane. The leaf hither to erect fell over and at last, it died.

5.7 KEYWORDS

Transcribe the following words-

 Alternate  Distance  Battle  Drought  Dense

DDE, GJUS&T, Hisar 118 | English (Compulsory) BA-101

 Fair  Hoarse  Perception  Sky  Sympathy  Warmth  Field  Intensity  Realize  Smoke  Thousand  White  Bright  Eco  Gentle  Echo  Merge  Silent  Subtle  Too

5.8 SELF ASSESSMENT QUESTIONS(SAQ’S)

Find out the synonymous of the following words-

 Deadly-fatal  Rough –hoarse  Cried- Roared  Go through-penetrate  Far off-distance

DDE, GJUS&T, Hisar 119 | English (Compulsory) BA-101

Find out the antonyms of the following words-

 Dead –living  Unconscious-conscious  Superior-inferior  Inhuman-human

Find out the antonyms of the following words-

 Highest quality-supreme  Providing nourishment-nourishing  Do not comply with-follow  Made evident-manifest  Shocking and serious-agonizing  Natural and instinctive -spontaneous

Elaborate on the following terms

 Cosmic-related to the whole universe  Tornados-violently destructive wind stormed occurring over land  Impinge-to make an impression  Decipher –to read with difficulty  Convulsive-affective by involuntary jerky muscular contraction

5.9 ANSWERS TO YOUR PROGRESS

5.9.1 Language activity

What is an adverb?

The adverb is a word that modifies the meaning of a verb, an adjective, or another adverb. An adverb can also modify a sentence.

DDE, GJUS&T, Hisar 120 | English (Compulsory) BA-101

Types of Adverbs

Adverb of time, Sentence Broad negative frequency and adverbs adverbs duration

Focusing Adverb of place Linking adverbs adverbs

Adverb of Adverb of manner degree

Formation of adverbs-

 The final y changes to i: happy, happily  The final ‘e’ is retained: extreme, extremely  Adjectives ending in able/ ‘ible’ drop the final e and add y: capable, capably; sensible, sensibly.  Adjectives ending in a vowel +l follow the usual rule: beautiful, beautifully; final, finally.

Position of Adverbs

 Mid position (before the verb, or part of the verb) 1. Mahesh suddenly drove off. 2. We have never been stopped by the police.  End position (at the end of a clause) 1. Mahesh drove off suddenly. 2. Mahesh reached late.

DDE, GJUS&T, Hisar 121 | English (Compulsory) BA-101

 Initial position (at the beginning of a clause) 1. Suddenly Mahesh drove off. 2. Tomorrow we shall have a party.

Exercise

Pick out the adverbs in the following sentences

 This is a very sweet mango.  The came again.  The puppy followed Mahesh everywhere.  You may be partly right but does it solve our problem.  Where is Raju?

5.9.2 Extended Composition

Write a short story of about 150 words narrating the life of a plant, from its birth to its death, incorporating its joyful and painful experiences.

Plants are the natural resource of our life. Without plants, we cannot survive. Apart from giving food, plants are important in many other ways. If there are no plants, we will face a shortage of oxygen in the future. Lack of oxygen in the atmosphere may lead to global warming. In addition to this, plants add beauty to the landscape. The life of a plant passes through many stages. The first is the seed growth. For a seed to grow into a healthy plant it needs plenty of water and light. The plant life cycle begins with a seed. The seed will sprout and produce a tiny, immature plant called a seedling. The seedling will grow to adulthood and form a mature plant. The mature plant will reproduce by forming new seeds which will begin the next cycle. This life cycle is generally used by flowering plants. The flowering plant include grasses, trees, shrubs, and, of course, flowers. There are also lower types of plants including ferns and mosses. These also have a life cycle but they do not produce seeds. These produce other reproductive cells called spores.

DDE, GJUS&T, Hisar 122 | English (Compulsory) BA-101

Compare J.C. Bose’s views with those of Vandana ’s and write a paragraph of about 150 words on the need to have a greater sensitivity towards plants and the environment.

All living things that live on this earth come under the environment. Whether they live on land or water they are part of the environment. The environment also includes air, water, sunlight, plants, animals, etc. Moreover, the earth is considered the only planet in the universe that supports life. The environment can be understood as a blanket that keeps life on the planet sage and sound. We truly cannot understand the real worth of the environment. But we can estimate some of its importance that can help us understand its importance. It plays a vital role in keeping living things healthy in the environment. Likewise, it maintains the ecological balance that will keep check of life on earth. It provides food, shelter, air, and fulfills all the human needs whether big or small. Moreover, the entire life support of humans depends wholly on the environmental factors. Also, it helps in maintaining various life cycles on earth. Most importantly, our environment is the source of natural beauty and is necessary for maintaining physical and mental health.

5.10 REFERENCES/SUGGESTED READINGS

o "Bose". Random House Webster's Unabridged Dictionary. o Editorial Board (2013). Sir Jagdish Chandra Bose. Edinburgh, Scotland: Encyclopædia Britannica, Inc. ISBN 9781593392925. o "A versatile genius". Frontline. Vol. 21 no. 24. The Hindu. 20 November 2004. o Chatterjee, Santimay and Chatterjee, Enakshi, Satyendranath Bose, 2002 reprint, p. 5, National Book Trust, ISBN 81-237-0492-5 o Sen, A. K. (1997). "Sir J.C. Bose and radio science". Microwave Symposium Digest. IEEE MTT-S International Microwave Symposium.

DDE, GJUS&T, Hisar 123 | English (Compulsory) BA-101

Denver, CO: IEEE. pp. 557– 560. doi:10.1109/MWSYM.1997.602854. ISBN 0-7803-3814-6.

******************************

DDE, GJUS&T, Hisar 124 | English (Compulsory) BA-101

Course Code: BA101 Author: Dr.NutanYadav

Lesson No:05 Playing about the English gentleman

Playing about the English Gentleman

Author –M. K. Gandhi

Lesson Structure

5.1 Learning Objectives

5.2 Introduction

5.3 Main Body of the Text

5.3.1 About the Author

5.3.2 About the Essay

5.4 Further Body of the text

5.5 Check your Progress

5.6 Summary

5.7 Keywords 5.8 Self -Assessment Questions (SAQs) 5.9 Answers to Your Progress

5.9.1Language Activity

5.9.2Extended Composition

5.10 References/ Suggested Readings

5.1 LEARNING OBJECTIVES

DDE, GJUS&T, Hisar 125 | English (Compulsory) BA-101

After going through this lesson you will be able to-

 To develop reading, writing, speaking, and listening skills of the English language.  To enrich the taste of literature.  To develop a reading habit  Know the difference between poetry and prose.  Trained in critically analyzing phonetic transcription of sound system

5.2 INTRODUCTION

In Playing the English Gentleman by we have the theme of identity, acceptance, change, perception, and tradition. Taken from his autobiography ‘The Story of My Experiments with Truth’ the reader realizes after reading the essay that Gandhi may be exploring the theme of identity. There is a sense that Gandhi while a student in England wishes to fit in with his peers even though he may live a lifestyle that would be deemed unfamiliar to an English gentleman (being a vegetarian for example). Regardless of this Gandhi does go some way to give off the appearance of an English gentleman. Going as far as getting dancing lessons, changing his attire, learning the violin, and getting elocution lessons. All of which Gandhi hopes will help him to fit in with his surroundings and be accepted by others. However, there does come a point when Gandhi realizes that he is living a lie and that he is not only fooling himself but also denying his tradition or the fact that he is an Indian and not an Englishman.

5.3 MAIN BODY OF THE TEXT

At the beginning of the essay, Gandhi says that he was vegetarian from his birth. His faith in vegetarian food becomes deeper after reading many books on it. After reading books like Howard William’s The Ethics of Diet. Dr. Anna Kingsford’s, The Perfect way in Diet’, Dr. Allison’s Writing On Health and Hygiene, he found that all philosophers and prophets from the past to the present age were vegetarians. As a result of reading all this literature, dietetic experiments came to take an important

DDE, GJUS&T, Hisar 126 | English (Compulsory) BA-101 place in his life. But in his later life, religion became the supreme motive. Gandhi’s friend in England had a great love for him. He was worried for him for various reasons.

He thought-

1. Gandhi’s health would remain weak if he did not eat meat in England. 2. He might be considered a duffer in English society. 3. He was afraid lest Gandhi should remain busy in experiments and forget his works a he was busy reading books on vegetarians.

So he started his efforts to reform him. He tried to make Gandhi Ji give up vegetarian food. But he did not succeed. However, this did not affect their relationship and they continued to be good friends.

Gandhi narrates some incidents in his autobiography ‘The Story of My Experiments with Truth’ chapter 16. He says that one day; his friend invited him to a big restaurant. The friend had planned to take him to this restaurant. The friend had planned to take him to this restaurant so that he would not ask questions because of his modesty. The first course was soup. He dared not ask his friend so he called the waiter to inquire if the soup was a vegetable soup. His friend became angry and told him to go to some other restaurant if he wanted vegetable soup. Gandhi Ji came out and went to some vegetarian restaurant close by but it was closed. So he had to remain hungry that night.

That was the last tussle he had with his friend. But it did not affect their friendship. Gandhi Ji could see and appreciated the love by which all his friend’s efforts were actuated and Gandhi’s respect for him was all the greater on account of their differences in thoughts and action.

Gandhi did not want to give up vegetarianism. But he decided to make up for his vegetarianism by cultivating other accomplishments that fitted the English society. He tried to become an English gentleman. He got English clothes. He

DDE, GJUS&T, Hisar 127 | English (Compulsory) BA-101

purchased a hat. He wasted 10 pounds on a fashionable suit. He asked his brother to send him a double watch chain of gold. He learned the art of using a tie and correctly parting his hair. He started taking lessons on dancing, learning French, playing the violin, and learning elocution. But his efforts did not prove successful and he gave up his attempts.

Gandhi Ji remembered the fable of the recluse. In the fable, the recluse kept a cat to keep off the rats and then a cow to feed the cat with milk and a man to keep the cow and so on. He felt that his ambitions also grew like that recluse. He thought to learn violin and for this, he spent money on violin. He engaged another teacher to teach him e locution. He made him buy a book on elocution. And at last, he realized the futility of these things and gave up his efforts to become an English gentleman.

Gandhi Ji thought that he had not to spend a lifetime in England. So there was no use of learning elocution. And dancing could not make a gentleman of him. He was a student and ought to go on with his studies. He should qualify himself to join the Inns of Court. He thought that if his character made a gentleman of him, so much the better.

After that, he gave up these useless attempts and concentrated on his studies and thus became a student. He expressed his thoughts in a letter to his elocution teacher. He requested his teacher to excuse him from further lessons. He wrote a similar letter to the dancing teacher. He made the same request to his violin teacher. In this way, he freed himself from the lessons.

5.3.1 About the Author

DDE, GJUS&T, Hisar 128 | English (Compulsory) BA-101

Mohandas Karam Chand Gandhi, also known as Mahatma Gandhi, was an Indian lawyer, anti-colonial nationalist, and political ethicist, who employed non-violent resistance to lead the successful campaign for India’s independence from British rule and also inspired movements for civil rights and freedom across the world.

The honorific Mahatma (great soul) first applied to him in 1914 in South Africa is now used throughout the world.

Gandhi Ji led nationwide campaigns for easing poverty, expanding women’s rights, building religious and ethnic amity, ending untouchability, and above all, for achieving ‘Swaras’ or self-rule. He was a good writer also. His autobiography is entitled ‘The Story of my Experiments with Truth’.

Some Indians thought Gandhi was too accommodating. Among them was Nathuram Godse a Hindu nationalist, who assassinated Gandhi on 30 Jan 1948 by firing three bullets in his chest.

Gandhi’s birthday on 2nd October is commemorated in India as Gandhi Jayanti a national holiday and worldwide as the international day of Non-Violence. Gandhi is commonly though not formally, considered the father of the Nation in India and was commonly called Bapu. Gandhi did not receive the Nobel Peace prize although he was nominated five times between 1937 and 1948.

5.3.2 About the essay

DDE, GJUS&T, Hisar 129 | English (Compulsory) BA-101

This essay has been taken from Mahatma Gandhi’s autobiography ‘The Story of My Experiments with Truth’. It was published in 1927. This excerpt appears in the fifteenth chapter of the first part of his autobiography. Here Gandhi narrates his experiences during the time he was in England.

5.4 FURTHER BODY OF THE TEXT

My faith in vegetarianism grew on me from day today. Salt’s book whetted my appetite for dietetic studies. I went for all the books available on vegetarianism and read them. One of these, Howard Williams’ The Ethics of Diet, was a ‘biographical history of literature and humane dietetics from the earliest period to the present day’. It tried to make out, that all philosophers and prophets from Pythagoras and Jesus down to those of the present age were vegetarians.

Dr. Anna Kingsford’s The Perfect Way in Diet was also an attractive book. Dr. Allison’s writings on health and hygiene were likewise helpful. He advocated a curative system based on the regulation of the dietary of patients. Himself a vegetarian, he prescribed for his patient a strictly vegetarian diet. The result of reading books on vegetarianism was that experiments with my diet came to take an important place in my life. Health was the principal consideration of those experiments, to begin with. But later on, religion became the supreme motive.

Meanwhile, my friend had not ceased to worry about me. His love for me led him to think that, if I persisted on my objections to meat-eating, I should not only develop a weak constitution but should remain a duffer, because I should never feel at home in English society. When he came to know that I had begun to interest myself in books of vegetarianism, he was afraid lest these studies should muddle my head; that I should fritter my life away in experiments, forgetting my work, and become a crank. He, therefore, made one last effort to reform me.

He one day invited me to go to the theater. Before the play, we were to dine together at the Holborn Restaurant, to be a palatial place and the first big restaurant I had been to since the Victoria Hotel. The stay at that hotel has scarcely been a helpful

DDE, GJUS&T, Hisar 130 | English (Compulsory) BA-101 experience, for I had not lived there with my wits about me. The friend had planned to take me to this restaurant imagining that modesty would forbid any questions. And it was a very big company of diners in the midst of which my friend and I sat sharing a table between us.

The first course was soup. I wondered it may be made up of, but dared not ask the friend about it., therefore summoned the waiter. My friend saw the movement and sternly asked across the table what the matter was. With considerable hesitation, I told him that I wanted to inquire if the soup was vegetable soup. ‘You are too clumsy for a decent society,’ he passionately exclaimed. ‘If you cannot behave yourself, you had better go. Feed-in some other restaurant and await me outside.’ This delighte d me. Out I went. There was a vegetarian restaurant close by, but it was closed. So I went without food that night. I accompanied my friend to the theatre, but he never said a word about the scene I had created. On my part of course there was nothing to say.

That was the last friendly tussle we had. It did not affect our relations in the least. I could see and appreciate the love by which all my friend’s efforts were actuated, and my respect for him was all the greater on account of our differences in thought and action.

But I decided that I should put him to ease, that I should assure him that I would be clumsy no more, but try to become polished and make up for my vegetarianism by cultivating other accomplishments which fitted one for polite society. And for this purpose, I undertook the all too impossible task of becoming an English gentleman.

The clothes after the Bombay cut that I was wearing were, I thought, unsuitable for English society, and got new ones at the Army and Navy Stores. I also went for a chimney-pot hat costing nineteen shillings- an excessive price in those days. Not content with this, I wasted ten pounds on an evening suit made in Bond Street, the center of fashionable life in London; and got my noble -hearted brother to send me a double watch- chain of gold. It was not correct to wear a ready-made tie and I

DDE, GJUS&T, Hisar 131 | English (Compulsory) BA-101 learned the art of tying one for myself. While in India, the mirror had been a luxury permitted on the days when the family barber shaved me. Here I wasted ten minutes every day before a huge mirror, watching myself arranging my tie and correctly parting my hair. My hair was by no means soft, and every day it meant a regular struggle with the brush to keep it in position. Each time the hat was put on and off, the hand would automatically move towards the head to adjust the hair, not to mention the other civilized habits of the hand now and then operating for the same purpose when sitting in polished society.

As if all this were not enough to make me look the thing, I directed my attention to other details that were supposed to go towards the making of an English gentleman. I was told I needed to take lessons in dancing, French and elocution. French was not only the language of neighboring France, but it was the lingua franca of the Continent over which I had desired to travel. I decided to take dancing lessons at a class and paid down 3 Euros as fees for the term. I must have taken about 6 lessons in three weeks. But it was beyond me to achieve anything like rhythmic motion. I could not follow the piano and hence found it impossible to keep time. What then was I to do?

The recluse in the fable kept a cat to keep off rats, and then a cow to feed the cat with milk, and a man to keep the cow, and so on. My ambition also grew like the family of a recluse. I thought I should learn to play the violin to cultivate an ear for western music. So I invested 3 Euros in a violin and something more in fees. I sought a third teacher to give me elocution lessons and paid him a preliminary fee of a guinea. He recommended Bell’s Standard Elocutionist as the textbook, which I purchased. And I began with a speech of Pitt’s.

But Mr. Bell rang the bell of alarm in my ear and I awoke.

I had not to spend a lifetime in England, I said to myself. When then was the use of learning elocution? And how could dancing make a gentleman out of me? The violin I could learn in India. I was a student and ought to go on with my studies. I should

DDE, GJUS&T, Hisar 132 | English (Compulsory) BA-101 qualify myself to join the Inns of Court. If my character made a gentleman of me, so much the better. Otherwise, I should forego the ambition.

These and similar thoughts possessed me, and I expressed them in a letter which I addressed to the elocution teacher, requesting him to excuse me from further lessons. I had taken only two or three. I wrote a similar letter to the dancing teacher and went personally to the violin teacher with a request to dispose of the violin for any price it might fetch. She was rather friendly to me, so I told her how I had discovered that I was pursuing a false idea. She encouraged me in the determination to make a complete change.

This infatuation must have lasted about three months. The punctiliousness in dress persisted for years. But henceforward I became a student.

5.5 CHECK YOUR PROGRESS

Answer the questions in 25-30 words

1. How did Gandhi Ji's faith in vegetarianism grow? 2. How did Gandhiji extricate himself from all the lessons?

Answers-

1. At the beginning of the essay, Gandhi says that he was a vegetarian. But his faith in vegetarianism grew because of some books. After reading Salt’s book he acquired more books on vegetarianism and read them. He found that all philosophers and prophets from the past to the present age were vegetarians. 2. Gandhiji expressed his thoughts in a letter that he wrote to his elocution teacher. He requested his teacher to excuse him from further lessons. He wrote a similar letter to his dance teacher. He made the same request to the violin teacher. In this way, he extricated himself from the lessons.

Answer the questions in 75-100 words

1. Describe the relationship that Gandhiji shared with his friend.

DDE, GJUS&T, Hisar 133 | English (Compulsory) BA-101

2. Describe some of the habits that Gandhiji had cultivated which were at variance with what he encountered in Britain. 3. Describe Gandhiji’s attempts to inculcate the qualities of an English gentleman. How did these attempts turn out? 4. ‘But henceforth I became a student’. Trace the course of events and thoughts that led Gandhiji to arrive at this.

Answers-

1. Gandhiji’s friend loved him deeply. He was worried that if Gandhiji did not eat meat, his he alth would remain weak. Moreover, he would be considered a duffer in English society. When he knew Gandhiji was reading books on vegetarianism, he was afraid lest Gandhiji should remain busy in these experiments and forget his work. So he tried to one last effort to reform him. He had a great love for Gandhiji. He tried to make Gandhiji give up vegetarianism. But did not succeed. However, this did not affect their friendship and they continued to be good friends. 2. Gandhiji did not like eating meat. He was a strict vegetarian. He would remain hungry but would not eat a non-vegetarian diet. He followed his habit even when he was in England. Gandhiji was particularly influenced by Dr. Anna Kingsford’s The Perfect Way in Diet and Dr. Allison’s writings on health and hygiene. As a result of reading all this literature, dietetic experiments came to take an important place in his life. But later on, religion became the supreme motive. Gandhiji acquired these habits and these were at variance with what he encountered in England. 3. He tried to become an English gentleman. His clothes were unsuitable for English society. He got English clothes. He also purchased a hat. He wasted 10 pounds on a fashionable suit. He asked his brother to send him a double watch chain of gold. He learned the art of using a tie. He wasted ten minutes every day in various fields. These fields were

DDE, GJUS&T, Hisar 134 | English (Compulsory) BA-101

dancing, learning French, learning to play the violin, and learning elocution. But his efforts did not prove successful and he gave up his attempts.

Comprehensive passages

Passage-1

Gandhiji realized that he had not to spend a lifetime in England. So, there was no use of learning elocution. And dancing could not make a gentleman of him. He could learn the violin even in India. He thought that he was a student and ought to go on with his studies he should qualify himself to join the Inns of Court. If his character made a gentleman of him, so much the better. Otherwise, he should forego the ambition. He gave up these attempts and concentrated on his studi es. Thus he became a student. He one day invited me to go to the theater. Before the play, we were to dine together at the Holborn Restaurant, to be a palatial place and the first big restaurant I had been to since the Victoria Hotel. The stay at that hote l has scarcely been a helpful experience, for I had not lived there with my wits about me. The friend had planned to take me to this restaurant imagining that modesty would forbid any questions. And it was a very big company of diners in the midst of which my friend and I sat sharing a table between us.

Questions-

1. To which restaurant did Gandhiji’s friend take him? 2. How was the restaurant? 3. What had his friend thought? 4. Where did Gandhiji and his friend sit?

Answers-

1. He took Gandhiji to the Holborn Restaurant. 2. It was a palatial restaurant. 3. He had thought that modesty would forbid any questions.

DDE, GJUS&T, Hisar 135 | English (Compulsory) BA-101

4. His friend and Gandhiji sat sharing a table between them.

Passage 2-

The clothes after the Bombay cut that I was wearing were, I thought, unsuitable for English society, and got new ones at the Army and Navy Stores. I also went for a chimney-pot hat costing nineteen shillings- an excessive price in those days. Not content with this, I wasted ten pounds on an evening suit made in Bond Street, the center of fashionable life in London; and got my noble-hearted brother to send me a double watch- chain of gold. It was not correct to wear a ready-made tie and I learned the art of tying one for myself. While in India, the mirror had been a luxury permitted on the days when the family barber shaved me. Here I wasted ten minutes every day before a huge mirror, watching myself arranging my tie and correctly parting my hair. My hair was by no means soft, and every day it meant a regular struggle with the brush to keep it in position. Each time the hat was put on and off, the hand would automatically move towards the head to adjust the hair, not to mention the other civilized habits of the hand now and then operating for the same purpose when sitting in polished society.

Questions-

1. From where did Gandhiji buy new clothes? 2. What kind of hat did he purchase? 3. Who sent him a double watch- chain of gold? 4. Where did he waste ten minutes daily?

Answers-

1. He bought new clothes at the Army and Navy Stores. 2. He purchased a chimney-pot hat. 3. His brother sent him a double watch- chain of gold. 4. He wasted ten minutes daily before the mirror.

Passage 3-

DDE, GJUS&T, Hisar 136 | English (Compulsory) BA-101

The recluse in the fable kept a cat to keep off rats, and then a cow to feed the cat with milk, and a man to keep the cow, and so on. My ambition also grew like the family of a recluse. I thought I should learn to play the violin to cultivate an ear for western music. So I invested 3 Euros in a violin and something more in fees. I sought a third teacher to give me elocution lessons and paid him a preliminary fee of a guinea. He recommended Bell’s Standard Elocutionist as the textbook, which I purchased. And I began with a speech of Pitt’s.

Questions-

1. Why did the recluse keep a cat? 2. How did Gandhiji’s ambitions grow? 3. How much did he invest in the violin? 4. Why did he seek a third teacher?

Answers-

1. The recluse in the fable kept a cat to keep off the rats. 2. His ambitions grew to like the family of the recluse. 3. He invested 3 Euros in violin. 4. He sought a third teacher to give him elocution lessons.

Passage 4-

These and similar thoughts possessed me, and I expressed them in a letter which I addressed to the elocution teacher, requesting him to excuse me from further lessons. I had taken only two or three. I wrote a similar letter to the dancing teacher and went personally to the violin teacher with a request to dispose of the violin for any price it might fetch. She was rather friendly to me, so I told her how I had discovered that I was pursuing a false idea. She encouraged me in the determination to make a complete change.

This infatuation must have lasted about three months. The punctiliousness in dress persisted for years. But henceforward I became a student.

DDE, GJUS&T, Hisar 137 | English (Compulsory) BA-101

Questions-

1. What did he write to his elocution teacher? 2. What request did he make to his violin teacher? 3. What was the response of the violin teacher? 4. How long did the infatuation last?

Answers-

1. He requested him to excuse him for further lessons. 2. He requested her to dispose of the violin for any price it might fetch. 3. She encouraged him in the determination to make a complete change. 4. This infatuation must have lasted about three months.

5.6 SUMMARY

This essay has been taken from Mahatma Gandhi’s autobiography ‘The Story of My Experiments with Truth’. It was published in 1927. This excerpt appears in the fifteenth chapter of the first part of his autobiography. Here Gandhi narrates his experiences during the time he was in England.

5.7 KEYWORDS

Give the phonetic transcription of the following words-

 Faith  Vegetarianism  Ethics  Biographical  Literature  Human  Period  Philosopher  Prophet

DDE, GJUS&T, Hisar 138 | English (Compulsory) BA-101

 Attractive  Hygiene  Advocate  System  regulation

5.8 SELF ASSESSMENT QUESTION(SAQ,S)

 What is a Noun?  What is a Verb?  How a noun can be converted into a verb or a verb can be converted into a noun.

5.9 ANSWERS TO YOUR PROGRESS

Some words can be used both as a noun and verbs. Here are some such words are taken from an essay. Use them in sentences of your own, in both their noun and verb forms

Stay

Stay (Noun) I met Ravi during my stay in Delhi.

Stay (Verb) He stayed in my house when he came here.

Advocate

Advocate (Noun) He is an advocate.

Advocate (verb) He advocated his case.

Result

Result (Noun) My result is late.

Result (Verb) His nonsense resulted in a hot dispute.

Interest

Interest (Noun) He is showing interest in studies.

DDE, GJUS&T, Hisar 139 | English (Compulsory) BA-101

Interest (Verb) I am not interested in gossips.

Muddle

Muddle (Noun) His theory has created a muddle in my mind.

Muddle (Verb) His nonsense has muddled my mind.

Change the following words into adverbs.

 Persistent------persistently  Deciding------Decidedly  Evident------Evidently  Modest------Modestly  Hesitant-----Hesitantly  Passionate-----Passionately  Easy ------Easily  Excessive------Excessively  Soft ------Softly  Direct------Directly  Supposed-----Supposedly  Personal-----Personally

Give synonyms and antonyms of the following words-

Words Synonyms Antonyms

Duffer Fool Wise

Interest Concern Indifferent

Appreciate Praise Criticize

Polite Courteous Rude

Tie Join Untie

DDE, GJUS&T, Hisar 140 | English (Compulsory) BA-101

Explain the following terms/words/phrases.

1. Vegetarianism- A person who does not eat meat. 2. All-too-impossible-Which is completely impossible. 3. Creating a scene- Creating a confusing situation. 4. Gentleman- well-cultured person 5. Now and then- frequently

Make nouns from the following verbs-

Verb Noun

Grow Growth

Persist persistence

Sit Seat

Behave Behavior

Appreciate appreciation

Decide Decision

Learn Learning

Teach Teacher

Write Writer

Direct Direction

Consider Consideration

Dine Dinner

Discover Discovery

Direct Directly

DDE, GJUS&T, Hisar 141 | English (Compulsory) BA-101

5.9.1 Language Activity

Tense –when we make a statement, we usually need to indicate whether we are referring to a situation that exists now, existed in the past, or is likely to exist in the future. This point of time is usually indicated by the verb group in the sentence and it is usually called Tense.

Past tense-

Past simple Past Continuous Past Perfect tense Past Perfect

Tense Continuous Tense

The simple past The most common We use the past The past perfect tense is used to perfect tense to go continuous tense Use of the past say that event continuous is to back to an earlier is not used with occurred at a past time, to talk verbs that are not talk about what particular time in about things that in continuous was already the past had happened at forms, except with happening at a the time we are want and particular past moment talking about. sometimes.

For example- For example- 1. I explained For example- that I had 1. My friend 1. Ram was The boy was forgotten visited our house dancing, but delighted with his my keys. last year. when she new air gun. He saw me he had wanted one

stopped. for a long time.

2. The prime 2. The phone rang 2. When he had 2. He had tried minister of while I was having shut the windows, five times to get Bhutan flew into my bath, as usual. we opened the her on the phone. Delhi yesterday to door of the cage.

DDE, GJUS&T, Hisar 142 | English (Compulsory) BA-101

start his visit to India.

3. Throughout 3.when Ramnish 3. When I had He had been 1999, he studied arrived I written all my trying to get her with us in a telephoned letters, I did some on the phone. central library. Harish(comparison) gardening.

About fifty miles 4. It happened 4. When the By 5 pm he had from the while I was living officer had written the story.

university there in the capital last learned the work was one of India’s year. of the department, most famous and he was moved. ancient Hindu temples.

5. We walked a 5. It was getting When Shyama He had been great deal in my dark. The wind was had known Gita writing the story. boyhood. rising for a year, she invited her to tea.

6. Actors used to 6. I was talking to be considered Shiv the other day. Gods.

Complete the sentences, using the simple present or simple past forms of the verbs given in blanks

1. I understood exactly what he meant (mean). 2. I don’t believe everything he tells me. (not believe) 3. Listen I think there’s someone at the door. (think) 4. Four plus four make eight. (make) 5. This sweeter does not match her suit(match)

DDE, GJUS&T, Hisar 143 | English (Compulsory) BA-101

6. Ramesh resembled his father when he was young. (resembles) 7. What exactly consists of. 8. This tea tastes nice. (taste)

Answer the following questions, using the simple past tense forms of the verbs in italics, together with adverbs wherever they are suggested in the bracket.

 Where did you spend your holiday last year?

I spent my holiday with my mother.

 Why did you choose this place for a holiday?

I love this place since my childhood.

 How did you travel?

I traveled by train.

 How did you spend your time?

I spent my time with my friend.

 What sort of luggage did you carry?

I carried woolen clothes.

 At what time did you get up?

I got up at 5 am.

 What sort of food did you eat?

I had rice and dal.

 How did you feel when you got back home?

I felt refreshed when I came back home.

5.9.2 Extended Composition

Write a paragraph on Self Discipline in about 150-200 words

DDE, GJUS&T, Hisar 144 | English (Compulsory) BA-101

Self-discipline is one of the most important and useful skills everyone should possess. This skill is essential in every area of life, and though most people acknowledge its importance, very few do something to strengthen it. Contrary to common belief, self-discipline does not mean being harsh toward you or living a limited, restrictive lifestyle. Self-discipline means self-control, which is a sign of inner strength and control of yourself, your actions, and your reactions. Self- discipline gives you the power to stick to your decisions and follow them through, without changing your mind, and is, therefore, one of the important requirements for achieving goals. The of this skill enables you to persevere with your decisions and plans until you accomplish them. It also manifests as inner strength, helping you to overcome addictions, procrastination, and laziness, and to follow through with whatever you do.

5.10 REFERENCES/SUGGESTED READINGS

 Gandhi, Mohandas K. (2009). An Autobiography: The Story of My Experiments With Truth. p. 21. ISBN 978-1775414056.  Malhotra, S.L (2001). Lawyer to Mahatma: Life, Work, and Transformation of M. K. Gandhi. p. 5. ISBN 978-81-7629-293-1.

*********************

DDE, GJUS&T, Hisar 145 | English (Compulsory) BA-101

Course Code: BA101 Author: Dr.NutanYadav

Lesson No:06

Great Books Born out of Great Minds

Author-A. P. J. Abdul Kalan

Lesson Structure

6.1 Learning Objectives

6.2 Introduction

6.3 Main Body of the Text

6.3.1 About the Author

6.3.2 About the Essay

6.4 Further Body of the text

6.5 Check your Progress

6.6 Summary

6.7 Keywords

6.8 Self -Assessment Questions (SAQs)

6.8.1 Language Activity

6.8 Answers to Your Progress 6.9.1 Extended Composition

6.10 References/ Suggested Readings

6.1 LEARNING OBJECTIVES

After going through this lesson you will be able to-

DDE, GJUS&T, Hisar 146 | English (Compulsory) BA-101

 To develop reading, writing, speaking, and listening skills of the English language.  To enrich the taste of literature.  To develop a reading habit  Know the difference between poetry and prose.  Trained in critically analyzing phonetic transcription of sound system

6.2 INTRODUCTION

This essay is a part of a speech delivered by Dr. Kalam. He delivered this speech on 7th December 2013 at the Lucknow Carnival. In this speech, he gives useful advice to young persons on how to forge ahead in life. This address is highly motivating and inspirational for young authors and building writers. Former Indian President Dr. A. P. J. Abdul Kalam spoke at three sessions at the 32nd Sharjah International Book Fair (SIBF) on Thursday, inspiring thousands of UAE schoolchildren, parents, educators, and readers.

6.3 MAIN BODY OF THE TEXT

6.3.1 About the Author

DDE, GJUS&T, Hisar 147 | English (Compulsory) BA-101

Dr. APJ Abdul Kalam is a famous name in the whole world. He is counted among the greatest scientists of the 21st century. Even more, he becomes the 11th president of India and served his country. He was the most valued person in the country as his contribution as a scientist and as a president is beyond compare. Apart from that, his contribution to the ISRO (Indian Space Research Organization) is remarkable. He headed many projects that contributed to society also he was the one who helped in the development of Agni and Prithvi missiles. For his involvement in the Nuclear power in India, he was known as the “Missile Man of India”. And due to his contribution to the country, the government awarded him with the highest civilian award. The great missile man becomes the President of India in 2002. During his presidency period, the army and country achieved many milestones that contributed a lot to the nation. He served the nation with an open heart that’s why he was called ‘people’s president’. But at the end of his term period, he was not satisfied with his work that’s why he wanted to be the President a second time but later on forfeited his name. After leaving the presidential office at the end of his term Dr. APJ Abdul Kalam again turn to his old passion which is teaching students. He worked for many renowned and prestigious institutes of India located across the country. Above all, according to his the youth of the country is very talented but need the opportunity to prove their worth that’s why he supported them in their every good deed.

6.3.2 About the Essay

Dr. Kalam began his talk titled “Great Books Born out of Great Minds” by applauding the audience for being self-proclaimed book lovers. The main thread of his speech was that human beings are wired for stories and the brain uses literature to make sense of life’s experiences. He firmly believes that storytelling is a central component of leadership and cited India’s own Mahatma Gandhi and South Africa’s Nelson Mandela for using the power of the written word to reach out to the people with their respective autobiographies.

DDE, GJUS&T, Hisar 148 | English (Compulsory) BA-101

Dr kalam begins his essay by talking about the culture of excellence. He believes that excellence is not by accident. It is a continuous struggle for people, organizations, and nations to make themselves better. They should set the performance goals themselves. Then they should work on these goals. If they meet with failures, they should not lose heart. Achieving high standards should be the aim of life. This is the culture of excellence.

Dr. Kalam talks about his interaction with young people. He says that he has met 16 million young people. They have aspirations in their lives. They want to become soldiers, doctors, and so on. They live in a world of imagination. When they grow up they take interest in novels, films, etc. The author feels that there is an author in the mind of every person.

According to Dr. Kalam, stories have a great influence on the mind of people. Stories help us grow through life’s complex social problems. Imagination allows people to transport mentally into ideal worlds. Imaginations give us the magical ability to experience what we do not have but wish to possess. In this matter, a story has a great utility.

According to the author, storytelling has a darker side too. It makes us a victim of conspiracy theories, advertisements, and narratives about ourselves that are not wholly true. Ideas like right, wrong, sin, good, duty, responsibility, love, hate, all depend on who is reading the story and how he is interpreting it. We must consider the impact of storytelling in a serious manner. We should ensure that stories remain authentic human experiences.

Dr. Kalam praises Goethe, Shakespeare, and Valmiki. Goethe wrote the German epic ‘Faust’. Shakespeare wrote great plays. He could see the past, the present, and the future through his magical plays. Valmiki evolved the story of , which stands for the past, the present, and the future. The biographies of Goethe and Shakespeare claimed that their minds were inspired by the Life Force. Valmiki says that a divine force helped him evolve Ramayana.

DDE, GJUS&T, Hisar 149 | English (Compulsory) BA-101

Dr. Kalam gives a message to the young authors. He tells them that they also have a mighty force of inspiration in their hearts. They should find out this force. They should concentrate on it and then they should use it for their literary imagination. This will help them in creating great works.

6.4 FURTHER BODY OF THE TEXT

Original Text

Friends, reading habits from a young age and continuous acquisition of knowledge through books indeed make human beings appreciate multiple aspects of social life and become autonomous learners. First, let me talk about the culture of excellence.

Friends, excellence is not by accident. It is a process, where individuals (or an organization or a nation) continuously strive to better themselves. They set the performance standard themselves, they work on their dreams with focus and are prepared to take calculated risks and do not get detected by failures as they move towards their dreams. Then they step up their dreams as they tend to reach the original targets. They strive to work to their potential, and in the process, they increase their performance, thereby multiplying further their potential. This is an unending life cycle phenomenon. They are not in competition with anyone else, but themselves. That is the culture of excellence.

Friends, I have so far met 16 million young men. Interacting with the young is like living in a world of stories. For children, everything is a story. They want to become astronauts, soldiers, doctors, sailors, and so on. They live in landscapes of make- believe. They spin fantasies. After growing up they absorb novels, films, and plays. Even sporting events and criminal trials unfold as narratives. It suddenly strikes to me now, after living for 82 years, that I am also living a story. Am I not, every day, conveying events and thoughts in words? Are we not ‘making’ and ‘beautifying’ the Reality around us? Can we say that humans are ‘wired’ for stories? Is there an author alive in our minds creating narratives about the past and the future?

DDE, GJUS&T, Hisar 150 | English (Compulsory) BA-101

Perhaps, stories help us navigate life’s complex social problems- just as flight simulators prepare pilots for difficult situations. Storytelling has evolved, like other behaviours, to ensure our survival. Imagination is an awesome evolutionary adaption that allows people to teleport mentally into alternative worlds. Imagination gives us, in other words, the near-magical ability to experience what we did not experience. A story is a thing that does’ rather than a thing that is’. It is a tool with measura ble utility rather than an object for aesthetic admiration. Attention is the reward that listeners bestow on the storyteller. This makes Storytelling a central component of leadership. When we wrote India 2020 in 1998, it was a story that stirred the Indian economy.

Of course, storytelling has a darker side too. It makes us a victim of conspiracy theories, advertisements, and narratives about ourselves that are more ‘truthy’ than true. Everything is in the eye of the beholder. Right, wrong, sin, good, duty, responsibility, love, hate, all of these depend on who is looking. New forms of stories generate their own new set of problems, which require yet more solutions. Alternate Reality Games (ARG) are interactive networked narratives that use the real world as a platform and uses trans-media Storytelling to deliver a story that may be altered by participants’ ideas or actions.

The human brain has been on a slower evolutionary trajectory than technology and our brains still respond to content by looking for the story to make sense out of the experience. It is very important that was incredibly visit storytelling and ensure that stories remain authentic human experiences. Let the story leapfrog the technology and bring us to the core of human experience about collaboration and connection.

In this connection, let me share an observation. I was reading a book on the great author Johann Wolfgang Goethe, whose mind evolved Faust, the great German epic. Shakespeare could see the past, the present, and the future through his great musical plays. Whereas, the great Indian epic master Valmiki evolved the story of Ramayana, which stands for the past, the present, and the future. I was asking

DDE, GJUS&T, Hisar 151 | English (Compulsory) BA-101 myself what could have actuated their minds to evolve such great ideas. In the biographies of Goethe and Shakespeare, they claimed that their minds were actuated by the Life Force. They claim that every human being has intellectual energy, this Life Force. Valmiki says that a divine force helped him evolve Ramayana. The message friends, particularly the young authors, is this: You have a mighty force within you- assemble it, concentrate, and use it for imagination and create great works.

6.5 CHECK YOUR PROGRESS

Answer the questions in 25-30 words

Questions

1. What makes human beings autonomous learners? 2. How can one achieve excellence in life? 3. What are the career prospects for children according to Dr. Kalam?

Answers-

1. Acquisition of knowledge makes human beings autonomous learners. If we develop reading habits from a young age and acquire knowledge from books, we become learners. We should be able to appreciate the multiple aspects of social life. 2. Excellence is not an accident. It is a process. One can achieve excellence in life by striving to better oneself. A man should set the performance standard himself. Then he should strive to reach that standard. 3. According to the author, there are many career prospects for children. They can choose any field in which they are interested. They can become astronauts, soldiers, doctors, sailors, and so on.

Answer the questions in 75-100 words

1. What is the culture of excellence, according to Dr. Kalam? 2. What message does Dr. Kalam give at the end of his speech?

DDE, GJUS&T, Hisar 152 | English (Compulsory) BA-101

Answers-

1. Dr. Kalam begins his essay by talking about the culture of excellence. He believes that excellence is not an accident. It is a process. This is an unending life-cycle phenomenon. It is a continuous struggle for individuals, organizations, and nations to make themselves better. They should set performance goals for themselves. Then they should work hard to achieve these goals. If they meet with failures, they should not lose heart. Achieving higher standards should be the aim of life. This is the culture of excellence. 2. Dr kalam gave the example of three great writers: Goethe, Shakespeare, and Valmiki. Their minds were inspired to write great epics. Goethe wrote the German epic ‘Faust’. Shakespeare wrote great plays. Valmiki evolved the story of ‘The Ramayana’. By giving their examples, Dr. Kalam gives a message to young authors. He tells them they have a mighty force of inspiration in their hearts. They should find out this force. They should concentrate on it and then they should use it for their literary imagination. This will help them in creating great works.

Comprehensive passages

Passage 1

Friends, reading habits from a young age and continuous acquisition of knowledge through books indeed make human beings appreciate multiple aspects of social life and become autonomous learners. First, let me talk about the culture of excellence.

Friends, excellence is not by accident. It is a process, where individuals (or an organization or a nation) continuously strive to better themselves. They set the performance standard themselves, they work on their dreams with focus and are prepared to take calculated risks, and do not get deterred by failures as they move towards their dreams. Then they step up their dreams as they tend to reach the original targets. They strive to work to their potential, and in the process, they increase their performance, thereby multiplying further their potential. This is an

DDE, GJUS&T, Hisar 153 | English (Compulsory) BA-101 unending life cycle phenomenon. They are not in competition with anyone else, but themselves. That is the culture of excellence.

Questions-

1. What makes human beings appreciate multiple aspects of societal life? 2. What is excellence, according to the author? 3. Who is not deterred by failures? 4. What is an unending life-cycle phenomenon?

Answers-

1. Reading habits make human beings appreciate multiple aspects of societal life. 2. Excellence is a process, where individuals continuously strive to better themselves. 3. Those who strive for excellence are not deterred by failures. 4. The struggle for achieving excellence is an unending life-cycle phenomenon.

Passage 2

Friends, I have so far met 16 million young men. Interacting with the young is like living in a world of stories. For children, everything is a story. They want to become astronauts, soldiers, doctors, sailors, and so on. They live in landscapes of make- believe. They spin fantasies. After growing up they absorb novels, films, and plays. Even sporting events and criminal trials unfold as narratives. It suddenly strikes to me now, after living for 82 years, that I am also living a story. Am I not, every day, conveying events and thoughts in words? Are we not ‘making’ and ‘beautifying’ the Reality around us? Can we say that humans are ‘wired’ for stories? Is there an author alive in our minds creating narratives about the past and the future?

Questions-

1. How many young men have the author met? 2. What does the author say about interacting with young men? 3. What are the career prospects for children according to the author?

DDE, GJUS&T, Hisar 154 | English (Compulsory) BA-101

4. What suddenly strikes the author, after living 82 years?

Answers-

1. The author has met 16 million young men. 2. He says that interacting with young men is like living in a world of stories. 3. They can become astronauts, soldiers, doctors, sailors, and so on. 4. It suddenly strikes the author that he is also living a story.

Passage 3

Of course, storytelling has a darker side too. It makes us a victim of conspiracy theories, advertisements, and narratives about ourselves that are more ‘truthy’ than true. Everything is in the eye of the beholder. Right, wrong, sin, good, duty, responsibility, love, hate, all of these depend on who is looking. New forms of stories generate their own new set of problems, which require yet more solutions. Alternate Reality Games (ARG) are interactive networked narratives that use the real world as a platform and uses trans-media Storytelling to deliver a story that may be altered by participants’ ideas or actions.

Questions-

1. What is the dark side of storytelling? 2. What does the author say about the new form of stories? 3. On who do right, wrong, sin, good, etc. depend? 4. What does ARG mean?

Answers-

1. It makes us a victim of conspiracy theories, advertisements, and narratives about ourselves that are less true than they seem. 2. New forms of stories generate their own new sets of problems. 3. These things depend on who is looking. 4. It means Alternate Reality Games. 6.6 SUMMARY

DDE, GJUS&T, Hisar 155 | English (Compulsory) BA-101

Dr. kalam begins his essay by talking about the culture of excellence. He believes that excellence is not by accident. It is a continuous struggle for people, organizations, and nations to make themselves better. They should set the performance goals themselves. Then they should work on these goals. If they meet with failures, they should not lose heart. Achieving high standards should be the aim of life. This is the culture of excellence.

Dr. Kalam talks about his interaction with young people. He says that he has met 16 million young people. They have aspirations in their lives. They want to become soldiers, doctors, and so on. They live in a world of imagination. When they grow up they take interest in novels, films, etc. The author feels that there is an author i n the mind of every person.

According to Dr. Kalam, stories have a great influence on the mind of people. Stories help us grow through life’s complex social problems. Imagination allows people to transport mentally into ideal worlds. Imaginations give us the magical ability to experience what we do not have but wish to possess. In this matter, a story has a great utility.

According to the author, storytelling has a darker side too. It makes us a victim of conspiracy theories, advertisements, and narratives about ourselves that are not wholly true. Ideas like right, wrong, sin, good, duty, responsibility, love, hate, all depend on who is reading the story and how he is interpreting it. We must consider the impact of storytelling in a serious manner. We should ensure that stories remain authentic human experiences.

Dr. Kalam praises Goethe, Shakespeare, and Valmiki. Goethe wrote the German epic ‘Faust’. Shakespeare wrote great plays. He could see the past, the present, and the future through his magical plays. Valmiki evolved the story of Ramayana, which stands for the past, the present, and the future. The biographies of Goethe and Shakespeare claimed that their minds were inspired by the Life Force. Valmiki says that a divine force helped him evolve Ramayana.

DDE, GJUS&T, Hisar 156 | English (Compulsory) BA-101

Dr. Kalam gives a message to the young authors. He tells them that they also have a mighty force of inspiration in their hearts. They should find out this force. They should concentrate on it and then they should use it for their literary imagination. This will help them in creating great works.

6.7KEYWORDS

Word-meaning

Appreciate- value

Strive- to make every effort

Autonomous- independent

Ensure- to make sure

Aesthetic- artistic

Alter- change

6.8 SELF ASSESSMENT QUESTIONS (SAQ’S)

Find words from the essay that is synonymous with the italicized words in the following sentences.

Word Synonym

Continuous Regular

Right Proper

Awesome Fearsome

Admiration Praise

Suddenly At once

Find words from the essay that are antonyms with the italicized words in the following sentences.

DDE, GJUS&T, Hisar 157 | English (Compulsory) BA-101

Word Antonym

Difficult Easy

Love Hatred

Sin Piousness

Real Unreal

Work Laziness

6.9 ANSWERS TO YOUR PROGRESS

6.9.1 Language activity

A preposition is a word placed before a noun or a pronoun to demonstrate to show in what relation the person or thing denoted by its stands regarding something else. Example: at, in, of, on, etc.

Compound Simple Prepositions Prepositions

Prepositions

Group Prepositions Participal Prepositions

Exercise

Fill in the blank with the correct preposition

DDE, GJUS&T, Hisar 158 | English (Compulsory) BA-101

 When will you go to University?  The child threw the toy on the floor.  The boys live in the hostel.  Parul is jealous of me.  Drinking is injurious to health.

6.9.2 Extended Composition

You are to play an interschool cricket match. Describe how will you select the team and prepare for the match.

Cricket is celebrated and practiced as a sport in many countries. It is played within a 22-yard oval field with two teams competing against each other. Each team includes 11 players who can be batters, all-rounder as well as bowlers. The game starts with a toss and is played based on over where each over has 6 balls to be bowled. Runs can be scored by hitting the ball out of the boundary in the form of fours and sixes or by running between the wickets to take singles. The total number of over depends on the format of the game. India plays cricket both as a sport and a recreation. Youngsters can be seen with their bats and balls in every nook and corner of the country. The Indian Cricket teams of both men and women represent the nation in different tournaments. The dedication and popularity of this game make it larger than life everywhere!

6.10 REFERENCES/ SUGGESTED READINGS o Pruthi, R. K. (2005). "Ch. 4. Missile Man of India". President A.P.J. Abdul Kalam. Anmol Publications. pp. 61–76. ISBN 978-81-261-1344-6. o "India's 'Mr. Missile': A man of the people". 30 July 2015. Archived from the original on 25 September 2015. Retrieved 30 July 2015. o "Kalam's unrealized 'Nag' missile dream to become reality next year". 30 July 2015. Archived from the original on 3 January 2017. Retrieved 30 July 2015.

DDE, GJUS&T, Hisar 159 | English (Compulsory) BA-101

o Sen, Amartya (2003). "India and the Bomb". In M. V. Ramana; C. Rammanohar Reddy (eds.). Prisoners of the Nuclear Dream. Sangam Books. pp. 167–188. ISBN 978-81-250-2477-4.

*************************

DDE, GJUS&T, Hisar 160 | English (Compulsory) BA-101

Course Code: BA101 Author: Dr.NutanYadav

Lesson No:07

The Responsibility of Young Men

Author-Lal Bahadur Shastri

Bharat Mata

Author Jawaharlal Nehru

Lesson Structure

7.1 Learning Objectives

7.2 Introduction

7.3 Main Body of the Text

7.3.1 About the Author

7.3.2 About the Essay

7.4 Further Body of the text

7.5 Check your Progress

7.6 Summary

7.7 Keywords

7.8 Self -Assessment Questions (SAQs)

7.8.1Language Activity

7.9 Answers to Your Progress 7.9.1 Extended Composition

7.10 References/ Suggested Readings

DDE, GJUS&T, Hisar 161 | English (Compulsory) BA-101

7.1 LEARNING OBJECTIVES

After going through this lesson you will be able to-

 To develop reading, writing, speaking, and listening skills of the English language.  To enrich the taste of literature.  To develop a reading habit  Know the difference between poetry and prose.  Trained in critically analyzing phonetic transcription of sound system

7.2 INTRODUCTION

The essay ‘Responsibility of Young Men ’is an abridged version of the speech that Shastriji delivered at the convocation of the Aligarh Muslim University. In this speech, he emphasizes the role that the Indian youth has to play in the development and progress of the nation. Shastriji says that our citizenship gives certain rights. They are guaranteed by the constitution. But this gives us certain responsibilities, which also have to be clearly understood. Our democracy gives freedom to us. But this freedom has to be subjected to several voluntary restraints in the interests of organized society. A good citizen is one who obeys the law. He must take delight in performing his civic duties. He must have a sense of self-restraint and discipline.

7.3 MAIN BODY OF THE TEXT

This essay is the abridged version of the speech that Shastriji delivered at the convocation of the Aligarh Muslim University. In this speech, he emphasizes the role that the Indian youth has to play in the development and progress of the nation.

7.3.1 About the Author

DDE, GJUS&T, Hisar 162 | English (Compulsory) BA-101

Lal Bahadur Shastri

Lal Bahadur Shastri (2 October 1904 – 11 January 1966) was an Indian politician who served as the second Prime Minister of India. He promoted the White Revolution – a national campaign to increase the production and supply of milk – by supporting the Amul milk co-operative of Anand, Gujarat, and creating the National Dairy Development Board. Underlining the need to boost India's food production, Shastri also promoted the Green Revolution in India in 1965. This led to an increase in food grain production, especially in Punjab, Haryana, and Uttar Pradesh.

7.3.2 About the Essay

Shastriji says that our citizenship gives certain rights. They are guaranteed by the constitution. But this gives us certain responsibilities, which also have to be clearly understood. Our democracy gives freedom to us. But this freedom has to be subjected to several voluntary restraints in the interests of organized society. A good citizen is one who obeys the law. He must take delight in performing his civic duties. He must have a sense of self-restraint and discipline.

In the past, there used to be a personal touch between the teacher and the student. But at present, in our country, the number of students and the educational institutions have grown very large. So there cannot be any personal touch contact

DDE, GJUS&T, Hisar 163 | English (Compulsory) BA-101 between the teacher and the taught. The economic stresses of present-day life, unfortunately, do not leave enough time for the parents to look after the children. Often this creates problems for young persons. They are often left much to their resources.

According to the writer, work culture is very important. In his view, every station in life is important in itself. Work has its dignity. There is great satisfaction in doing one’s job to the best of one’s ability. Whatever the duties, we should apply ourselves with sincerity and devotion. This approach is good in itself. But it has been added advantage of opening avenues for further advancement. We have to see whether we have done our job well before thinking of criticizing anybody else.

Shastriji says that we should never forget that loyalty to the country is more important than all other loyalties. And this is absolute loyalty. We should never think about what we get in return for this loyalty. It is essential to remember that the entire country is one and that anyone who fosters or promotes separatism is not our true friend. Separatism is very dangerous for the country. The country cannot progress if there are separatist tendencies among the citizens.

According to the author, loyalty to the country comes ahead of all other loyalties. A democratic country is sustained not only by the greatness of a few. It is sustained by the co-operative effort of the many. The future of the country is in the hands of young people. When the older generations complete their task the new generations come to take their place. If they are well loyal to the country as individuals and as citizens, the country’s future will be bright indeed.

Secularism means that there is equal respect for all religions. There are equal opportunities for all citizens whether they follow any caste, creed, or faith. Our position about secularism is known very well. It is embodied in our Constitution. According to the writer, the seed of secularism has to be sown at the earliest stages. It should be sown in the field of education. Only then the plant of secularism can be carefully nurtured as it grows.

DDE, GJUS&T, Hisar 164 | English (Compulsory) BA-101

7.4 Further Body of the Text

Original Text

Whatever your station in the future life, each one of you should first of all think of yourselves as citizens of the country. This confers on you certain rights, which are guaranteed by the Constitution, but it also subjects you to certain responsibilities, which also have to be clearly understood. Ours is a democracy, which enjoins freedom to the individual, but this freedom has to be subjected to several voluntary restraints in the interests of organized society. And these voluntary restraints have to be exercised and demonstrated in every-day life.

A good citizen is one who obeys the law, whether there is a policeman around or not, and who takes delight in performing his civic duties. In the olden days' se nse of self- restraint and discipline was inculcated by the combined effort of the family and the teacher. The economic stresses of present-day life, unfortunately, do not leave enough time for the parents to look after the children.

In educational institutions, the numbers have grown so large as to take away the benefits, which used to accrue formerly by personal contact between the teacher and the taught. Inevitably, our young students are often left much to their resources. Often this creates problems, which we all know about. This is an important aspect, which needs thorough examination. The responsibility of our young citizens is great. In my view, every station in life is important in itself. Work has its dignity and there is great satisfaction in doing one's job to the best of one's ability.

Whatever the duties, we should apply ourselves with sincerity and devotion. Such an approach, apart from being good in itself, also has the added advantage of opening avenues for further advancement. We have to see whether we have done our job well before thinking of criticizing anybody else. All too often, we succumb to the temptation of decrying others without bothering to look to ourselves.

DDE, GJUS&T, Hisar 165 | English (Compulsory) BA-101

Never forget that loyalty to the country comes ahead of all other loyalties. And this is an absolute loyalty since one cannot weigh it in terms of what one receives. It is essential to remember that the entire country is one and that anyone who fosters or promotes separatism or fissiparous tendencies are not our true friends. What I have said stems from a desire to see that the youth of our country prepares itself in a disciplined and determined manner for the responsibilities of tomorrow. A democratic country is sustained not by the greatness of a few but by the co-operative effort of the many. The future of the country is in your hands and as the older generations complete their task the new ones come along to take their place. If they are well equipped as individuals and as citizens, the country's future will be bright indeed. At a time when you are at the threshold of a new period in your life, I would urge you to play your role with confidence and dignity.

Our position about secularism is known so well that it hardly needs any reiteration. It is embodied in our Constitution, which ensures equal respect for all religions and equal opportunities for all citizens, irrespective of their caste and creed and the faith which they profess. Despite a seeming diversity, there is a fundamental unity in India, which we all cherish and it has to be our constant endeavor to maintain and strengthen this unity.

The country can progress only if it does away completely with fissiparous tendencies and emerges as an integrated whole. And it is in the field of education that the seed of secularism has to be sown at the earliest stages so that the plant can be carefully nurtured as it grows.

7.5 CHECK YOUR PROGRESS

Answer the questions in 25-30 words

1. What does the Constitution of India guarantee every citizen? 2. According to the writer, who is a good citizen?

Answers-

DDE, GJUS&T, Hisar 166 | English (Compulsory) BA-101

1. The Constitution of India guarantees certain rights to every citizen. But this gives us certain responsibilities also, which also have to be clearly understood. 2. According to the writer, a good citizen is one who obeys the law. He must take delight in performing his civic duties. He must have a sense of self-restraint and discipline.

Answer the questions in 75-100 words

Questions-

1. Why are voluntary restraints important for an organized society? 2. Explain the term ‘secularism’. Why is it necessary?

Answers-

1. Shastriji says that our citizenship gives certain rights. They are guaranteed by the constitution. But this gives us certain responsibilities, which also have to be clearly understood. Our democracy gives freedom to us. But this freedom has to be subjected to several voluntary restraints in the interests of organized society. A good citizen is one who obeys the law. He must take delight in performing his civic duties. He must have a sense of self-restraint and discipline. Thus the voluntary restraints are better than those restraints which are put on us by the society or government. 2. In this essay, Shastriji tells us about the necessity of secularism in our life. Secularism is the need of time. Secularism means that there is equal respect for all religions. There are equal opportunities for all citizens whether they follow any caste, creed, or faith. Our position about secularism is known very well. It is embodied in our Constitution. According to the writer, the seed of secularism has to be sown at the earliest stages. It should be sown in the field of education. Only then the plant of secularism can be carefully nurtured as it grows.

DDE, GJUS&T, Hisar 167 | English (Compulsory) BA-101

Comprehensive passages

Passage 1

Whatever your station in the future life, each one of you should first of all think of yourselves as citizens of the country. This confers on you certain rights, which are guaranteed by the Constitution, but it also subjects you to certain responsibilities, which also have to be clearly understood.

Ours is a democracy, which enjoins freedom to the individual, but this freedom has to be subjected to several voluntary restraints in the interests of organized society. And these voluntary restraints have to be exercised and demonstrated in every-day life.

Questions-

1. What should we first think of? 2. What is guaranteed by our Constitution? 3. What does our democracy give us? 4. What is this freedom subjected to?

Answers-

1. We should first all think of ourselves as citizens of this country. 2. Our rights are guaranteed by our Constitution. 3. Our democracy gives us freedom. 4. This freedom was subjected to several voluntary restraints.

Passage 2

Never forget that loyalty to the country comes ahead of all other loyalties. And this is an absolute loyalty since one cannot weigh it in terms of what one receives. It is essential to remember that the entire country is one and that anyone who fosters or promotes separatism or fissiparous tendencies are not our true friends. What I have said stems from a desire to see that the youth of our country prepares itself in a disciplined and determined manner for the responsibilities of tomorrow. A

DDE, GJUS&T, Hisar 168 | English (Compulsory) BA-101 democratic country is sustained not by the greatness of a few but by the co-operative effort of the many. The future of the country is in your hands and as the older generations complete their task the new ones come along to take their place. If they are well equipped as individuals and as citizens, the country's future will be bright indeed. At a time when you are at the threshold of a new period in your life, I would urge you to play your role with confidence and dignity.

Questions-

1. What does the writer say about loyalty? 2. What is absolute loyalty? 3. Who is not our true friend? 4. Where is the future of the country?

Answers-

1. He says that loyalty to the country comes ahead of all other loyalties. 2. Loyalty to the country is absolute loyalty. 3. Anyone who promotes separatism is not our true friend. 4. The future of the country is in the hands of young people. 7.6 SUMMARY Shastriji says that our citizenship gives certain rights. They are guaranteed by the constitution. But this gives us certain responsibilities, which also have to be clearly understood. Our democracy gives freedom to us. But this freedom has to be subjected to several voluntary restraints in the interests of organized society. A good citizen is one who obeys the law. He must take delight in performing his civic duties. He must have a sense of self-restraint and discipline. 7.7 KEYWORDS  Enjoin- to direct  Restraint- control  Accrue- to increase over some time

DDE, GJUS&T, Hisar 169 | English (Compulsory) BA-101

 Dignity- pride  Advancement- development  Criticize- condemn 7.8 SELF ASSESSMENT QUESTIONS (SAQ’S)

Find words from the essay that is synonymous with the italicized words in the following sentences. Synonym Word Just claims Rights Belief Faith Capability Ability Joy Delight

Satisfaction Contentment

Find words from the essay that are antonyms with the italicized words in the following sentences.

Word Antonym

Problem Solution

Obey Disobey

Combined Separated

Best Worst

Former Latter

7.9 ANSWERS TO YOUR PROGRESS

7.9.1 Language Activity

Conjunction-

DDE, GJUS&T, Hisar 170 | English (Compulsory) BA-101

Conjunctions are words that join together words or sentences. Example: and, but, etc.

Exercise

Fill in the correct conjunctions in the following sentences

 Gravit and Torres play football.  Sachit is rich but honest.  Jayant plays cricket well, yet his favorite sport is football.  Snakes crawl and elephants walk.  I came back after he had gone home.  Akshit will play if Amit plays.

Tense

Future Continuous Tense

This tense is used to express an action that is going on at a given point in time in the future.

Affirmative Negative Interrogative

I shall be singing. I will not be singing Will I be singing?

We shall be singing. We shall not be singing. Shall we be singing?

They shall be singing. They will not be singing Shall they be singing?

7.9.2 Extended Composition

Write a paragraph on how you will spend your summer vacations

Summer Vacation is quite a refreshing time for me. This summer get-away will be unique for me, as my parents have arranged an outing to Manali. It will be seven days trip. I am truly eager to watch the scenic beauty of the mountains. We have booked our tickets to Manali. The arrangement is separated into three little journeys of Chandigarh and Delhi. We will go through one day in Chandigarh and Delhi, there

DDE, GJUS&T, Hisar 171 | English (Compulsory) BA-101 we will go for touring, and after the side tour of Chandigarh, we will take a vehicle for Manali. For the following six days, we will be in Manali, and we have wanted to visit Rohtang Pass, Manikaran, Hidimba Devi Temple, and Manali markets. A year ago, we went to visit Mumbai and Lonavala. So for a change, my dad arranged an outing to the mountains. I have planned to capture lots of pictures. I love photography; catching the snowcapped mountains is my ultimate wish. For the remaining summer vacation, I will join the skateboarding classes. My mom has recruited a Maths teacher for me, as I have a little fear of arithmetic. I will spend a lazy afternoon in the summer with my comics. Before my next academic session, I need to gather all the great memories with my loved ones.

7.10 REFERENCES/ SUGGESTED READINGS

Bakshi, Shiri Ram (1991), Struggle for Independence: Lal Bahadur Shastri, Anmol Publications, ISBN 9788170411420

 Dhawan, S. K. (1991), Bharat Ratnas, 1954-1991, Wave Publications

 Grover, Verinder (1993), Political Thinkers of Modern India: Lala Lajpat Rai, Deep & Deep Publications, ISBN 978-81-7100-426-3

 Guha, Ramachandra (2008), India After Gandhi: The History of the World's Largest Democracy, Pan Macmillan, ISBN 978-0-330-39611-0

 Gupta, U.N. (2003), Indian Parliamentary Democracy, Atlantic Publishers & Distributors, ISBN 978-8126901937

 Prasad, Rajeshwar (1991), Days with Lal Bahadur Shastri: Glimpses from The Last Seven Years, Allied Publishers, ISBN 978-81-7023-331-2

 Srivastava, C.P. (1995), Lal Bahadur Shastri, Prime Minister of India; a life of truth in politics (1st ed.), Delhi: Oxford University Press, ISBN 978-0-19-563499-0

 Verma, Krant M. L. (1978), Lalita Ke Ansoo on WorldCat, OCLC 60419441

*************************

DDE, GJUS&T, Hisar 172 | English (Compulsory) BA-101

Course Code: BA101 Author: Dr.NutanYadav

Lesson No:07

Bharat Mata

Author-Jawaharlal Nehru

Lesson Structure

7.1 Learning Objectives

7.2 Introduction

7.3 Main Body of the Text

7.3.1 About the Author

7.3.2 About the Essay

7.4 Further Body of the text

7.5 Check your Progress

7.6 Summary

7.7 Keywords

7.8 Self -Assessment Questions (SAQs)

7.8.1 Language Activity

7.9 Answers to Your Progress

7.9.1 Extended Composition

7.10 References/ Suggested Readings

DDE, GJUS&T, Hisar 173 | English (Compulsory) BA-101

7.1 LEARNING OBJECTIVES

After going through this lesson you will be able to-

 To develop reading, writing, speaking, and listening skills of the English language.  To enrich the taste of literature.  To develop a reading habit  Know the difference between poetry and prose.  Trained in critically analyzing phonetic transcription of sound system

7.2 INTRODUCTION

Nehru wrote his book ‘The Discovery of India’ in 1944, while he was in Ahmadnagar Fort Prison. This essay is taken from Chapter 3 of this book. In the book, the chapter is titled ‘The Quest’. It was written simply. Here Nehru presents a vivid portrait of the innocence of Indian peasants. He enriches the reader’s general awareness by introducing them to landmarks events from around the world

7.3 MAIN BODY OF THE TEXT

In this essay, Nehru describes his meeting with the peasants and other people in his public meetings during the freedom struggle of India. He says that the name ‘Bharat’ is derived from the mythical founder of India. Nehru preferred to speak to the villagers about ‘Bharat Mata’. In the cities, people do not want to hear about such simple things. But he spoke to the peasants with their limited outlook. He spoke of his great country for whose freedom they were struggling.

7.3.1 About the Author

DDE, GJUS&T, Hisar 174 | English (Compulsory) BA-101

Jawaharlal Nehru

Jawaharlal Nehru, also known as Pandit Nehru, was a political leader of India and a freedom fighter. His contribution to the Indian History of independence is immense. He was also very fond of children and was commonly referred to as Chacha Nehru by them. Nehru was also a close associate of Mahatma Gandhi and thrived with Gandhi as his mentor. Nehru had also planned many movements demanding dominion status and total independence for India from the British. Jawaharlal Nehru organized the first Kisan March (the march organized for the Farmers) in Pratapgarh, then in Uttar Pradesh, then known as the United Provinces.

He was imprisoned twice from 1920 to 1922 for his active role in the Non- cooperation Movement. He was the baton-charged at Lucknow and also played an essential role in opposing the Simon commission. In the year, 1928, Jawaharlal Nehru formed the “Independence of India League” and was appointed as its General Secretary.

Nehru became the President of the Lahore session of the Indian National Congress in 1929 and passed the resolution of Purna Swaraj. Again from the period of 1930 to 1935, Jawaharlal was imprisoned numerous times for his role in Salt Satyagraha, or the Dandi March, and other similar movements. With the untiring efforts into the

DDE, GJUS&T, Hisar 175 | English (Compulsory) BA-101

Indian freedom struggle, India finally attained independence on 15th August 1947, and he became the first Prime Minister of independent and free India.

2.3.2 About the Essay.

Nehru wrote his book ‘The Discovery of India’ in 1944, while he was in Ahmadnagar Fort Prison. This essay is taken from Chapter 3 of this book. In the book, the chapter is titled ‘The Quest’. It was written simply. Here Nehru presents a vivid portrait of the innocence of Indian peasants. He enriches the reader’s general awareness by introducing them to landmarks events from around the world.

In this essay, Nehru describes his meeting with the peasants and other people in his public meetings during the freedom struggle of India. He says that the name ‘Bharat’ is derived from the mythical founder of India. Nehru preferred to speak to the villagers about ‘Bharat Mata’. In the cities, people do not want to hear about such simple things. But he spoke to the peasants with their limited outlook. He spoke of his great country for whose freedom they were struggling.

Nehru spoke to the peasants about many things. He told them how each part of India differed from the other. Yet there is unity in India. There are common problems of India from north to south and from e ast to west. He told them about the ‘Swaraj’ which could not be for one person only but for all the Indians. He told them about journeying from one corner of India to the other. He told them wonderful things in other countries.

Nehru said that the peasants all over India were facing similar problems. Every peasant asked him identical questions. It was because their were the same. They were suffering from poverty, debt, vested interests, landlords, and moneylenders. They were also suffering from heavy rents and taxes and police harassment. All these sufferings of the farmers were because of the structure that the foreign government had imposed upon the people.

DDE, GJUS&T, Hisar 176 | English (Compulsory) BA-101

Sometimes, when Nehru reached a gathering, a great roar of welcome would greet him, people would shout, ”Bharat Mata ki Jai” (Victory to ). Nehru asked them what the meaning of ‘Bharat Mata’ was. But they could not answer that question adequately. Then he explained to them the meaning of ‘Bharat Mata’. India indeed meant great fields, mountains, rivers, forests, etc. but ultimately Bharat meant the dharti or land as well as the people of India.

According to the writer, the people of India are the ‘Bharat Mata’. These are the people who lived in the villages and cities and the people like Nehru and the peasants. They were spread over this vast land which is called ‘Bharat Mata’ or ‘Mother India’. There are millions of people. India is essentially these people. And when they say ‘Victory to Mother India’, it means victory to them, the people of India.

Nehru told people at his gatherings that only lands, forests, mountains, and rivers do not make a country. a country is made by its people. When people asked him the meaning of ‘Bharat Mata’, he told them that they were the Bharat Mata. He told them that “You are parts of this Bharat Mata.” At first, the people could not understand him, but then slowly they grasped the meaning of Nehru’s words. Then their eyes would light up as if they had made a great discovery.

7.4 FURTHER BODY OF THE TEXT

Original Text

Often, as I wandered from meeting to meeting, I spoke to my audience of this India of ours, of Hindustan, and of Bharata, the old Sanskrit name derived from the mythical founder of the race. I seldom did so in the cities, for there the audiences were more sophisticated and wanted stronger fare. But to the peasant, with his limited outlook, I spoke of this great country for whose freedom we were struggling, of how each part differed from the other and yet was India, of common problems of the peasants from north to south and east to west, of the Swaraj that could only be for all and every part and not for some.

DDE, GJUS&T, Hisar 177 | English (Compulsory) BA-101

I told them of my journeying from the Khyber Pass in the far north-west to or Cape Comorin in the distant south, and how everywhere the pe asants put me identical questions, for their troubles were the same—poverty, debt, vested interests, landlords, moneylenders, heavy rents and taxes, police harassment, and all these wrapped up in the structure that the foreign government had imposed upon us—and relief must also come for all. I tried to make them think of India as a whole, and even to some little extent of this wide world of which we were a part. I brought in the struggle in China, in Spain, in Abyssinia, in Central Europe, in Egypt and the countries of Western Asia. I told them of the wonderful changes in the Soviet Union and of the great progress made in America.

The task was not easy; yet it was not so difficult as I had imagined, for our ancient epics and myths and legends, which they knew so well, had made them familiar with the conception of their country, and some there were always who had traveled far and wide to the great places of pilgrimage situated at the four corners of India. Or some old soldiers had served in foreign parts in World War I or other expeditions. Even my references to foreign countries were brought home to them by the consequences of the great depression of the ’30s.

Sometimes as I reached a gathering, a great roar of welcome would greet me: Bharat Mata ki Jai—Victory to Mother India. I would ask them unexpectedly what they meant by that cry, who was this Bharat Mata, Mother India, whose victory they wanted? My question would amuse them and surprise them, and then, not knowing exactly what to answer, they would look at each other and me. I persisted in my questioning. At last, a vigorous Jat wedded to the soil from immemorial generations, would say that it was the dharti, the good earth of India, that they meant. What earth? Their particular village patch, or all the patches in the district or province, or the whole of India?

And so question and answer went on, till they would ask me impatiently to tell them all about it. I would endeavor to do so and explain that India was all this that they

DDE, GJUS&T, Hisar 178 | English (Compulsory) BA-101 had thought, but it was much more. The mountains and the rivers of India, and the forests and the broad fields, which gave us food, were all dear to us, but what counted ultimately were the people of India, people like them and me, who were spread out all over this vast land. Bharat Mata, Mother India, was essentially these millions of people, and victory to her meant victory to these people. You are parts of this Bharat Mata, I told them, you are in a manner yourselves Bharat Mata, and as this idea slowly soaked into their brains, their eyes would light up as if they had made a great discovery.

7.5 CHECK YOUR PROGRESS

Answer the questions in 25-30 words

1. According to Nehru, from where did India get its name, Bharat? 2. Where is Khyber Pass located? 3. Where is Cape Comorin located?

Answers-

1. In this essay, Nehru talks of India and its culture. He tells us that India got its name ‘Bharat’ from the Sanskrit name derived from the mythical founder of our country. His name was Bharat. 2. Before the partition, Khyber Pass was a part of India. After the partition, it went to Pakistan. Now it is on the border of Pakistan and Afghanistan. 3. Cape Comorin is known as ‘Kanyakumari’ these days. It is the southern end of our country. it is situated on the southern extremity of India. It is in the state of Tamil Nadu.

Answer the questions in 75-100 words

1. Enumerate the various things that Nehru spoke to peasants about. 2. What were the problems faced by the peasants all over India?

DDE, GJUS&T, Hisar 179 | English (Compulsory) BA-101

Answers-

1. Nehru spoke to the peasants about many things. . He told them how each part of India differed from the other. Yet there is unity in India. There are common problems of India from north to south and from east to west. He told them about the ‘Swaraj’ which could not be for one person only but all the Indians. He told them about journeying from one corner of India to the other. He told them wonderful things about other countries. He asked the people of India to serve their motherland and to free it from the foreign rule. 2. Nehru said that the peasants all over India were facing similar problems. Every peasant asked him identical questions. It was because their troubles were the same. They were suffering from poverty, debt, vested interests, landlords, and moneylenders. They were also suffering from heavy rents and taxes and police harassment. All these sufferings of the farmers were because of the structure that the foreign government had imposed upon the people. He told them that they must struggle to get rid of these problems. Nehru tried to make them think of India as a whole.

Comprehensive passages

Passage 1

Often, as I wandered from meeting to meeting, I spoke to my audience of this India of ours, of Hindustan, and of Bharata, the old Sanskrit name derived from the mythical founder of the race. I seldom did so in the cities, for there the audiences were more sophisticated and wanted stronger fare. But to the peasant, with his limited outlook, I spoke of this great country for whose freedom we were struggling, of how each part differed from the other and yet was India, of common problems of the peasants from north to south and east to west, of the Swaraj that could only be for all and every part and not for some.

Questions-

DDE, GJUS&T, Hisar 180 | English (Compulsory) BA-101

1. About what thing did the author speak to the audience? 2. From where has the word ‘Bharat’ been derived? 3. What does the author say about the city audience? 4. What does the author say about the peasants?

Answers-

1. He spoke to them about ‘Bharat Mata’ 2. It has been derived from the mythical founder of his country. 3. He says that the city audience was more sophisticated. 4. He says that the peasants have a limited outlook.

Passage 2

Sometimes as I reached a gathering, a great roar of welcome would greet me: Bharat Mata ki Jai—Victory to Mother India. I would ask them unexpectedly what they meant by that cry, who was this Bharat Mata, Mother India, whose victory they wanted? My question would amuse them and surprise them, and then, not knowing exactly what to answer, they would look at each other and me. I persisted in my questioning. At last, a vigorous Jat wedded to the soil from immemorial generations, would say that it was the dharti, the good earth of India, that they meant. What earth? Their particular village patch, or all the patches in the district or province, or the whole of India?

Questions-

1. How did the audience greet the author? 2. What did the author ask them? 3. What did a Jat say? 4. Who is the writer of this passage?

Answers-

1. The audience greeted the author with the shout ‘Bharat Mata ki jai’ 2. He asked them who Bharat Mata was.

DDE, GJUS&T, Hisar 181 | English (Compulsory) BA-101

3. He said that this dharti (land) was Bharat Mata. 4. The writer of the passage is Jawaharlal Nehru.

Passage 3

And so question and answer went on, till they would ask me impatiently to tell them all about it. I would endeavor to do so and explain that India was all this that they had thought, but it was much more. The mountains and the rivers of India, and the forests and the broad fields, which gave us food, were all dear to us, but what counted ultimately were the people of India, people like them and me, who were spread out all over this vast land. Bharat Mata, Mother India, was essentially these millions of people, and victory to her meant victory to these people. You are parts of this Bharat Mata, I told them, you are in a manner yourselves Bharat Mata, and as this idea slowly soaked into their brains, their eyes would light up as if they had made a great discovery.

Questions-

1. What did the author endeavor to do? 2. What essentially was Bharat Mata? 3. Who are the parts of Bharat Mata? 4. What would happen when the villagers understood the idea?

Answers-

1. He endeavored to explain what India was. 2. Bharat Mata was essentially the millions of people. 3. The people of India are parts of Bharat Mata. 4. Their eyes would light up as if they had made a great discovery.

7.6 SUMMARY

Nehru describes his meeting with the peasants and other people in his public meetings during the freedom struggle of India. He says that the name ‘Bharat’ is derived from the mythical founder of India. Nehru preferred to speak to the villagers

DDE, GJUS&T, Hisar 182 | English (Compulsory) BA-101 about ‘Bharat Mata’. In the cities, people do not want to hear about such simple things. But he spoke to the peasants with their limited outlook. He spoke of his great country for whose freedom they were struggling.

Nehru spoke to the peasants about many things. He told them how each part of India differed from the other. Yet there is unity in India. There are common problems of India from north to south and from east to west. He told them about the ‘Swaraj’ which could not be for one person only but all the Indians. He told them about journeying from one corner of India to the other. He told them wonderful things about other countries.

Nehru said that the peasants all over India were facing similar problems. Every peasant asked him identical questions. It was because their troubles were the same. They were suffering from poverty, debt, vested interests, landlords, and moneylenders. They were also suffering from heavy rents and taxes and police harassment. All these sufferings of the farmers were because of the structure that the foreign government had imposed upon the people.

Sometimes, when Nehru reached a gathering, a great roar of welcome would greet him, people would shout, ”Bharat Mata ki Jai” (Victory to Mother India). Nehru asked them what the meaning of ‘Bharat Mata’ was. But they could not answer that question adequately. Then he explained to them the meaning of ‘Bharat Mata’. India indeed meant great fields, mountains, rivers, forests, etc. but ultimately Bharat meant the dharti or land as well as the people of India.

According to the writer, the people of India are the ‘Bharat Mata’. These are the people who lived in the villages and cities and the people like Nehru and the peasants. They were spread over this vast land which is called ‘Bharat Mata’ or ‘Mother India’. There are millions of people. India is essentially these people. And when they say ‘Victory to Mother India’, it means victory to them, the people of India.

Nehru told people at his gatherings that only lands, forests, mountains, and rivers do not make a country. a country is made by its people. When people asked him the

DDE, GJUS&T, Hisar 183 | English (Compulsory) BA-101 meaning of ‘Bharat Mata’, he told them that they were the Bharat Mata. He told them that “You are parts of this Bharat Mata.” At first, the people could not understand him, but then slowly they grasped the meaning of Nehru’s words. Then their eyes would light up as if they had made a great discovery-

7.7 KEYWORDS

 Seldom- rarely  Imposed- forced  Amuse- interest  Conception- idea  Myth- a traditional story accepted as history.  Sophisticated- refined

7.8 SELF ASSESSMENT QUESTIONS (SAQ’S)

Convert the following nouns into adjectives

Noun Adjective

Abdomen Abdominal

Intellect Intellectual

Nonsense Nonsensical

Shade Shady

Additional Additional

Make words using the following prefixes

Prefix Word

Eco- Ecology

Ant- Antique

Styl- Stylistics

DDE, GJUS&T, Hisar 184 | English (Compulsory) BA-101

De - Deflate

Chrono- Chronology

7.9 ANSWERS TO YOUR PROGRESS

7.9.1 Language activity

Future Perfect Tense

This tense form is used to talk about a future action that can be completed at a particular time. This tense also consolidates the determination of the speaker.

Affirmative Interrogative Negative

I shall have played. Will I have played? I shall not have played.

We shall have played. Shall we have played? We shall not have played.

They will have played Shall they have played? They will not have played.

Future Perfect Continuous Tense

This tense form is used to express actions that began in the past and will be in progress over some time that will end in the future.

Affirmative Negative Interrogative

I shall have been playing I shall not have been Will I have been playing?

playing

We shall have been We shall not have been Shall we have been playing playing playing?

They will have been They will not have been Shall they have been playing playing playing?

DDE, GJUS&T, Hisar 185 | English (Compulsory) BA-101

7.9.2 Extended Composition

Write a paragraph on ‘The Need to Ban Plastic.’

Five hundred billion used globally and one hundred billion of them end up in U.S. landfills, taking about one thousand years to decompose, but only 5.2 percent were recycled (Borrud, 2007, p.75).-These are the figures plastic bags have produced every year. Human beings invented plastic bags for the convenience of carriers and packers. However, just as other great inventions, say, nuclear energy and biotechnology, plastic bags are causing serious issues like global warming, environmental pollution, and energy consumption. They are gradually becoming a sword towards us. In responding to this problem, the city of San Francisco has become the trailblazer to prohibit non-biodegradable plastic bags in its supermarkets and pharmacies.

In other words, in this era of high oil prices and energy scarcity, petroleum-based bags are costing considerable natural resources and ultimately turn them into a huge pile of useless garbage. Significant energies like fuel, from which ethylene gas is obtained and plastic bags are made, are nonrenewable, while human beings are still consuming these precious treasures recklessly. What's more, the plastic bags easily catch the wind and blow, causing the difficulty of confining them to were disposed of (Borrud, 2007, p.75). This leads to difficulty in recycling, which hinders the circulation and reuse of the material to a large extent. Moreover, Ross Mirkarimi, the author of the ban, also notes that only 1% of the plastic bags in San Francisco are recycled, despite the city's perfect local recycling system ("100 Billion Reasons," 2007, p.77). People's poor awareness of energy-saving has brought the oil-shortage crisis one step further. A prohibitory edict on petroleum-based bags might be the right way out before we ultimately realize that the last calorie on the earth comes from our body temperature.

DDE, GJUS&T, Hisar 186 | English (Compulsory) BA-101

Another reason for supporting the prohibition on petroleum-based bags is that they are economically inefficient. Angela Spivey reports the city of New York once suspended their services in plastic recycling.

7.10 REFERENCES/ SUGGESTED READINGS

 Nehru: The Invention of India by Shashi Tharoor (November 2003) Arcade Books ISBN 1-55970-697-X

 Jawaharlal Nehru (Edited by S. Gopal and Uma Iyengar) (July 2003) The Essential Writings of Jawaharlal Nehru Oxford University Press ISBN 0-19-565324-6

 Autobiography: Toward freedom, Oxford University Press

 Jawaharlal Nehru: Life and work by M. Chalapathi Rau, National Book Club (1 January 1966)

 Jawaharlal Nehru by M. Chalapathi Rau. [New Delhi] Publications Division, Ministry of Information and Broadcasting, Govt. of India [197

******************************

DDE, GJUS&T, Hisar 187 | English (Compulsory) BA-101

LITERATURE AND

LANGUAGE-II

DDE, GJUS&T, Hisar 188 | English (Compulsory) BA-101

Vetter: Subject : English

Lesson No.-01 Author: Dr. Pallavi

Pigeons at Daybreak By Anita Desai

1.0 Learning Objectives

1.1 Introduction

1.2 Main Body of the Text

1.2.1 Very Short Question Answers

1.2.2 Short Question Answers

1.3 Further Main Body of the Text

1.3.1 Long Question Answers

1.3.2 Punctuation Marks

1.4 Check your Progress

1.5 Answers to check your Progress

1.6 Keywords

1.7 Summary

1.8 Self Assessment Test

1.9 References/ Suggested Readings

1.0 LEARNING OBJECTIVES

1. The students will understand the theme that love doesn't wither away at illness.

DDE, GJUS&T, Hisar 189 | English (Compulsory) BA-101

2. It emphasizes the reading of literature as an active enterprises involving thought and involving thought and invoking feeling. 3. The student will learn how well-told stories involve us emotionally in the lives of the character. 4. The student will learn the new style of writing of Anita Desai writing skills.

1.2 MAIN BODY OF THE TEXT

This is a touching story of an old couple, Mr. Basu and his wife Otima. The husband is a patient of Asthma and often has attacks of this disease. When the story starts, Mr. Basu is sitting in his chair in the balcony. He cannot read the newspaper himself. His wife reads the paper to him. He calls his wife to come and read out the news to him. She is in the kitchen and is busy with some work. She takes her time. In the meantime, Mr. Basu gets impatient and irritable.

When Otima is free from her work, she is ready to read out the newspaper. She comes to him and reads out the news items one by one. There is nothing special in the news. Mr. Basu gets irritated. Finally, Otima reads out the news that there would be electricity cut in their area during the night. This news alarms Basu. He complains that he would not be able to sleep properly.

The thought of having to sleep in the heat without the electricity fan makes Basu so worried that he has an attack of asthma. He calls out to his wife that he is short of breath. She comes running. She gives him the inhaler. He inhales from it and feels some relief. The postman brings a letter from their son who is at Bhilai. Otima reads out the letter that he is fine and doing work well. This also gives him some relief.

At night Basu's wife suggest that they would move their beds on the roof top and sleep in the open air. As they are old, they cannot move up the beds, themselves, She calls their neighbour, Bulu who helps in taking the beds upstairs.

Otima makes beds, brings pillows, adjusts them on Basu's bed and tries to make him comfortable. But Basu is too distracted by the sound of traffic to sleep. All

DDE, GJUS&T, Hisar 190 | English (Compulsory) BA-101 through the night he moaned and gasped for air. Towards dawn it is so bad that his wife has to get up and massage his chest and it seems to relieve him. She goes down to bring water for her husband. To her surprise, there is light on in their flat. She goes up the stairs and asks Basu to come down.

But he tells her to leave him alone and says that it is cooler now. The morning has brought cool breeze from Jamuna river. Basu is lying flat and still, gazing up. It's mouth is open as if to let it pour into him, as cool and fresh as water. Then a flock of pigeons suddenly flies in the sky and soon disappeared. The writer hints that Basu had died.

1.2.1 Very Short Answer Question

Q.1What is Otima's attitude towards Basu?

Ans. Otima's attitude towards Basu is tolerant and caring.

Q.2 Why was Basu impatient with the news?

Ans. He did not find the news interesting.

Q.3What brings on Amul Basu's asthma attack?

Ans. The imagined fear of spending a night in scorching heat without electricity brings on Amul Basu's asthma attack.

Q.4 How old was Otima?

Ans.Otima was fifty six years old.

Q.5Who is Bulu?

Ans:- Bulu is one of Basu's and Otima's neighbours.

Q.6Who helped them take their beds to the terrace?

Ans:-The friendly neighbours helped them.

Q.7 Are all of Basu's fears real? Are some of them concocted?

Ans:- The fears were not real. Some of them are imaginary.

DDE, GJUS&T, Hisar 191 | English (Compulsory) BA-101

Q.8What is 'one of Basu's worst afflictions'? Why?

Ans:- Mr Basu's inability to read the newspaper at his own is one of his worst pains. It is because of his illness.

Q.9What was the subject of consuming interest to Otima?

Ans:- Films, was the subject.

Q.10Why does the old man insist on staying on the terrace?

Ans:- He feels that terrace is cooler.

1.2.2 Short Answer Questions

Q.1 Comment on Anita Desai's prose style, with reference to the story, 'Pigeons at Daybreak'.

Ans:- Anita Desai uses the method of third person narrative. She makes use of descriptions of the characters internal thoughts. She uses symbolism to express her themes. She also makes use of brief but clear descriptions to imply the condition of Mr. Basu. For example, when the electricity stopped, ' Basu saw the sky shrouded with summer dust, and it seemed as airless as the room below.' The imagery of the flying pigeons conveys the release of soul from the body.

Q.2 Describe the bond between the old couple.

Ans:- There is strong bond between Basu and his wife. Otima is fifty six years old and her husband is sixty one. Otima loves & cares her husband. Basu is a patient of asthma and cannot survive without his wife's constant attention.

Q.3 What happens to Basu when he hears the news of the proposed electricity cut?

Ans:- Basu's wife reads to her the news that there would be a power cut that night. Basu is a patient of asthma. Suddenly he has an attack of asthma. He feels choked. His wife brings his inhaler. He inhales from it. But he feels that he is not feeling well.

DDE, GJUS&T, Hisar 192 | English (Compulsory) BA-101

Q.4 Discuss the relevance of the title.

Ans:- The title of the story is very relevant. Both the pigeons and day break occur at the end of the story. The flight of pigeons stands for the flying away of man's soul after death. The daybreak refers to the comfort or freedom which man's soul find after death. A feeling of comfort and relief are conveyed though.

Q.5 Write a note on the ending of the story.

Ans:- The end of the story is not very clear. The writer does not make a clear ending. Basu has an attack of asthma and does not feel well. In the morning, Otima asks him to come down. But Basu does not want to come. His gaze is fixed. The writer hints that Basu has died.

Q.6 How did Mrs. Basu arrange for her husband Mr. Basu to spend the night when there was no light?

Ans. Otima called for help a boy named Bulu who lived next door. They both took Mr. Basu up the stairs. They hauled him up as though he were a bag containing something fragile and valuable. Upon the terrace, he was lowered onto the bed that had been carried up earlier and spread with many pillows.

1.3 FURTHER MAIN BODY OF THE TEXT

1.3.1 Long Answer Type Question

Q.1 What is the theme of the story ' Pigeons at Daybreak'?

Ans. The story underlines the deep devotion of an Indian wife towards her husband. She gives herself heart and soul in the service of her husband. She never cares for her own comfort or joy. All her thoughts remain focused in her husband. And if the husband becomes afflicted with some pain or disease, she doesn't want to leave him for a moment.

This is what we find in the story. Mr. Basu is a sixty one year old but hasn't a wrinkle on her face. She is quite devoted to her husband and takes all care of him.

DDE, GJUS&T, Hisar 193 | English (Compulsory) BA-101

However, Mr. Basu remains grumbling all the time about his illness and doesn't let his wife even a moment's rest. One night, there is no electricity, and it is too hot to sleep in the room. With the help of a boy, named Bulu in the neighbourhood, she arranges to take Mr Basu and his bed up to the terrace. Even there in the open, Mr. Basu remains restless. Poor Otima has to keep adjusting the pillows for the old man. She has to keep fanning him with a palm leaf. She has to get downstairs again and again to bring him his medicine. Finally, dead tired, she collapses onto her own string bed. In the morning, when she gets up, she finds that the old man's soul has already taken wings. Now the only thing the old woman's soul can long for is to join her husband in the heavens.

Grammar for Knowledge

1.3.2 Punctuation Marks & Capital Letters

Punctuation is the art of dividing literary composition by means of the points or marks into clauses, members of sentences, and sentences so as to readily convey the sense. The punctuation represent to some extent the pauses the emphasis which assist in making clear the meaning and relative dependence of the sentences in the spoken language.

1. Full stop or Period (.)

2. Colon (:)

3. Semi- Colon (;)

4. Comma(,)

5. Note of Exclamation (!)

6. Note of Interrogation / Question Mark (?)

7. Dash(-)

8. Inverted Commas (" ")

9. Parenthesis ( )

DDE, GJUS&T, Hisar 194 | English (Compulsory) BA-101

10. Hypen (-)

11. Apostrophe (')

12. Captial letters.

Detailed study

 The Full Stop:- The Full Stop is used at the end of a sentence that is not exclamatory or interrogative, to separate it completely from the sentence that follow it ex:- It is better to have loved and lost than never to have loved at all .

2. After Abbreviations:- M.A., L.L.B., P.W.D

 The Colon:- The colon is used to introduce a direct statement or a direct question:-

Bacon says:" Reading maketh a full man; writing an exact man; and conference a ready man."

2. To introduce a list, or an enumeration of detail:- ex- Shakespeare's four great tragedies are: Hamlet, Macbeth, Othello, and King Lear.

3. Between two complete sentences not joined by conjunctions, yet so closely connected in thought that the second is felt to be an illustration. ex- This man is an habitual criminal: he has been four times convicted of felony, and is still leading a dishonest life.

 The Semi-colon (;) 1. To distinguish co-ordinate clauses joined by such coordinating conjunctions as express position, or introduce an inference; as , therefore, otherwise, so , then, for , still, yet ex- Eat more fruit; otherwise you will regret it.

2. To separate parallel clauses which the writer desires to emphasize by providing a longer pause between them.

DDE, GJUS&T, Hisar 195 | English (Compulsory) BA-101 ex- The coconut tree gives us food, it gives us drink; it gives us rafters; it gives us material for roofing; it gives us oil; it gives us ropes; it supplies us, with nearly all the necessaries of life.

3. Before such words and phrases as namely, that is, as, viz., introducing an example, explanation etc, as- ex:- There are three cardinal virtues; namely, faith, hope, and charity.

 The Comma (,) The Comma is used to mark off the Vocative ( Nominative of Address): Milton, thou shouldst be living at this hour- Wordsworth

When I am dead, my dear

Sing so sad songs for me - Rossetti

Fraility, they name is woman! - Shakespeare.

2. To separate a series of words in the same construction:- ex- We live by admiration, faith and hope.

Friends, Romans, Countrymen, lend me your ears.

It was a long, dull and wearisome journey

Earth, air, water, teem with life.

3. To separate such successive pair of words connected by and or or ex - Live or die, sink or swim, survive or perish,

I shall support him.

4. To mark off a Noun or a Phrase ex- Napolean, the man of Destiny, as some have called him, was a most wonderful general.

5. After Nominative Absolute ex- I am not, to tell the truth, very much impressed by your proposal.

DDE, GJUS&T, Hisar 196 | English (Compulsory) BA-101

6. To mark off two or more Adverbs or Adverbial phrase coming together:- ex - Then, at length, tardy justice was done to the memory of Oliver.

7. To mark off a series of Noun clause, or Adjective clause. ex- I did not know when he did it, or why he did it.

8. Before and after words, phrases or clauses ex- Her behaviour, to say the least, was very rude.

9. To indicate the omission of a word, especially a verb: ex- To have knowledge is good, but to possess wisdom, better.

10. After a Verb of saying, which introduces something in Direct Speech. ex- He said to his disciples, "Watch and pray".

11. After each preposition, when there are two prepositions to one object. ex- Your car is superior to , and cheaper than, the one I say yesterday.

12. Words and phrases indicating comparison. ex- The water is deep, yet it is clear.

13. When the natural order of a sentence is inverted ex- By attention to details, good printing is produced.

14. Independent elements in a sentence ex- I am, Dear Sir, Your sincerely, I refer, Mr. President, to yesterday's meeting.

15.When a word is understood, but not expressed, a comma is inserted in its place . ex- The characteristic of Chaucer is intensity; of spencer; remoteness; of Milton, of Shakespear, everything.

 The Question Mark:- The question mark is used at the end of a direct question:-

ex- Have you written your essay?

DDE, GJUS&T, Hisar 197 | English (Compulsory) BA-101

"If you prick us, do we not bleed? - Shakespear

 The Exclamation Mark:- The Exclamation mark is used after a vocative case a usage now almost exclusively confined to verse.

ex- Oh world! Oh life! Oh Time!

After interjections and exclamatory sentences:-

ex- Alas! they had been friends in youth Coleridge.

 The Dash:- The Dash is used to indicate an unexpected turn or sudden break in a sentence.

ex- He counted them at daybreak of day:-

To summarize a multiple subject:-

ex- And ships by thousands , lay below

And men in thousands ------all were his------

To denote agitated Speech:-

ex- I - er - think - I mean - er - I feel.

4. To show the intentional omission of a name.

DDE, GJUS&T, Hisar 198 | English (Compulsory) BA-101

 Inverted commas:- The inverted commas are used to indicate the exact words of a speaker or a quotation.

ex - He said to John, " May God Bless You."

Parentheses or Double Dashes- are used to separate from the main part of the sentence, a phrase or clause which does not grammatically belong to it.

ex- She gained from heaven ------a son

 The Hyphen:- The Hyphen - a shorter line than the Dash - is used to form compound words-

ex- Brother - in - law, booking - clerk;

DDE, GJUS&T, Hisar 199 | English (Compulsory) BA-101

 The Apostrophe :- The Apostrophe is used to denote the omission of some letter or letters from a word.

ex - I've, don't.

To show the Genivine Case:-

ex- Lies show upon the desert's dusty face.

In the plurals

ex - Do's and Don't's

 Capital Letters-Capital letters are used to indicate the first word of a sentence.

ex - The door to success is hardwork.

DDE, GJUS&T, Hisar 200 | English (Compulsory) BA-101

The pronoun I, and the interjection O

Nouns and pronouns relating to the Deity;

ex - Let us thank God.

 The Sentence.

We express our thoughts and feelings through words. Single words do not make the meaning clear without their context. So, when we write or speak, we use words in groups to express our thoughts. The group of words is 'sentences'. Each of the group of words has a subject and a verb.

A sentence is a group of words which makes complete sense.

ex - I have two sisters.

Cats like mice.

Components of a sentence

DDE, GJUS&T, Hisar 201 | English (Compulsory) BA-101

Every sentence has two essential parts.

1. A Subject 2. A predicate.

The subject of a sentence is the part about which something is said.

Predicate:- A perdicate proclaims or says something about the subject.

Subject is a noun or pronoun with or without determiners and other words. The predicate has a finite verb with or without object, complement, adverb etc.

DDE, GJUS&T, Hisar 202 | English (Compulsory) BA-101

Kinds of sentences

The main Kinds are:-

1. Affirmative:- A statement;(Mohan has a big house)

2. Imperative- A command; request,(Close the door)

3. Exclamatory- Sudden response ( What a tall building!)

4. Interrogative - A question ( Is it raining?)

5. Negative - Negation ( Do not close the door)

DDE, GJUS&T, Hisar 203 | English (Compulsory) BA-101

DDE, GJUS&T, Hisar 204 | English (Compulsory) BA-101

Exercise for Practice

Separate the Subject and predicate

1. It is raining outside.

2. They are never late.

3. Mohan is flying a kite.

4. The clown made us laugh.

5. God helps good children.

6. The priest sang the prayer.

7. He cannot solve this problem.

8. The old man greeted me with a smile.

Simple Sentence:-

A simple sentence consists of a subject and a predicate. A simple sentence consists of a single clause. ex - lives in Panipat.

Compound Sentence :- A compound sentence has two or more main clauses, joined together by co-ordinating conjuctions such as and, but , or , yet, still, therefore, so , for, etc. ex- She must eat or she will become weak.

- She worked hard and got success in her work.

Complex Sentence:- A complex sentence has one main clause and two or more subordinate clause. ex- I met him before going to Delhi.

He is the girl who stood first.

DDE, GJUS&T, Hisar 205 | English (Compulsory) BA-101

Sample paragraph

Life in a small village

Life in a small village has a charm of its own. There is peace, calm and quiet all around. The air is fresh, pure and free from pollution. There is neither smoke nor noise. People live and work together and share their joys and sorrows. Village life is now changing fast. Even small village are, provided with electricity and drinking water. They have been connected to cities by roads. People use cooking gas in their kitchens. But village life has its own problems. Poverty and ignorance have made people superstitious, jealous and narrow minded. They try to harm others as they cannot bear the progress of their neighbours. There are constant quarrels and family feuds over trifles. Litigation, elections and party politics have spoiled the peace, unity and charm of village life.

Need for Education

Education is the primary right of every child in a democratic society. We have made a law to provide free, compulsory education up to the age of fourteen, up to middle

DDE, GJUS&T, Hisar 206 | English (Compulsory) BA-101 standard level. An uneducated person can neither be a good citizen nor a good parent. He is ignorant and superstitious. Deprived of knowledge, he falls in the darkness of ignorance and becomes a victim of evil social practices. The problem of drop outs at primary level is quite serious. Poor parents force their children to stay at home and look after the younger kids. Some are compelled to work as child labourers in homes, shops, restaurants and factories. More community centres and Adult education centres have to be opened to educate men and women who were deprieved of the fruits of education during their formative years. Our aim should be to provide education to all.

1.4 CHECK YOUR PROGRESS

a. Fill in the blanks with correct form of verb

1. My luggage...... (have) arrived.

2. My friends...... (arrive) here tomorrow.

3. Respect your teachers(Command/request/advice)

4. My sister writes poetry(Interrogative/Simple)

5. Hurrah! We have won the match. ( Negative/ Imperative/ Interrogative Exclamatory.)

1.5 ANSWERS TO CHECK YOUR PROGRESS

1. has, will arrive, advice, Simple sentence, Exclamatory sentence.

b. Arrange the following statement

1. Nikhil, grandson a had they named.

2. Reliance for reading, Mr Basu's irritated the newspaper him .

Answer to check you progress

1. They had a grandson named Nikhil.

2. Mr Basu's reliance on Otima for reading the newspaper irritated him.

DDE, GJUS&T, Hisar 207 | English (Compulsory) BA-101

1.6 KEYWORDS

1. Fear-Worried

2. Curiosity- to know about something

3. Anxiety- Worried about something

4. Harmony- a state of agreement

5. Arguing - to say things angrily

6. Considerate - always thinking of other people wish

7. Whims- a sudden idea

8. Reliance - being able to trust somebody

9. spirits - the part of person , not physical

10. Troubled- to get into a awkard situation

1.7 SUMMARY Anita Desai depicts the deep sincere efforts of an Indian wife towards her husband. She gives herself heart and soul in the service of her husband. She never cares for her own comfort or joy. All her thoughts remain set in her husband. And if the husband becomes impatient with some pain or disease, she doesn't want to leave him for a moment.. Anita Desai uses the method of third person narrative. She makes use of descriptions of the characters internal thoughts. She uses symbolism to express her themes. She also makes use of brief but clear descriptions to imply the condition of Mr. Basu. For example, when the electricity stopped, ' Basu saw the sky shrouded with summer dust, and it seemed as airless as the room below.' The imagery of the flying pigeons conveys the release of soul from the body.

1.8 SELF- ASSESSMENT TEST

Q.1 What is Otima's attitude towards Basu.

DDE, GJUS&T, Hisar 208 | English (Compulsory) BA-101

Q.2 On what note does the story 'Pigeons at Day break' end?

Q.3 Give phonetic transcription

1. damage

2. Waist

3. fashion

4.goat

5. master

6. paper

Q.4 Make the following sentences

1. Honour

2. Daybreak

3. feverish

4. Attitude

Q.5 Punctuate the following sentences

1. She is intelligent bold and beautiful

2. My brother in law is an advocate

1.9 REFERENCES & SUGGESTED READINGS

 Pigeon at daybreak. (n.d). 2012

 Siegel, K(n.d). Psychonanalytic Criticism.

DDE, GJUS&T, Hisar 209 | English (Compulsory) BA-101

Vetter: Subject : English

Course Code: BA 101 Author: Dr. Pallavi

Lesson 02

With the Photographer

By – Stephen Leacock

2.0 Learning Objectives

2.1 Introduction

2.2 Main Body of the Text

2.2.1 Very short Questions Answers

2.2.2 Short Questions Answers

2.3 Further Main Body of the Text

2.3.1 Long Questions Answers

2.3.2 Question Tags

2.4 Check your Progress

2.5 Answers to check your Progress

2.6 Keywords

2.7 Summary

2.8 Self Assessment Test

2.9 References/ Suggested Readings

2.0 LEARNING OBJECTIVES

DDE, GJUS&T, Hisar 210 | English (Compulsory) BA-101

1. The learners will read and analyze the literary text. 2. Identification of the theme. 3. The student will get sanitized to the issues posed. 4. Development of reference skills. 5. Development of creativity through writing conversations. 6. Will get acquainted with the new vocabulary given.

2.1 INTRODUCTION

With the photographer

Stephen Leacock describes the kind of occurence which could happen to a person in those days when people still went to studios to get themselves photographed. The humour and irony with which he describes this common place experience transform this ordinary event into a funny one. Stephen's style was simple and interesting. He is best known for his humorous book ' Laugh with Leacock' which provides delectable reading. Born in 1896, Stephen Leacock died in 1944.

2.2 MAIN BODY OF THE TEXT

The story records the amusing account of narrator's encounter with a photographer. The present story is in dialogic form as it is constituted by the conversation. The idea of going to studio in our contemporary times may appear be eccentric as our youth belongs to the age that is dominated by 'Selfie'. On being requested, the photographer looks at the narrator without any enthusiasm. The narrator starts to describe the physical appearance of the photographer. The photographer is described as man in grey suit, with dim eyes. He looks like a " natural scientist." Leacock introduces the photographer to his readers as a man who has no enthusiasm.

The narrator waits for an hour before the process of taking a photograph is started. While waiting for the photographer the narrator reads some magazines. The narrator considers his act of reading these magazines as an "impertinent thing". The

DDE, GJUS&T, Hisar 211 | English (Compulsory) BA-101 photographer reads, and his choice for woman centric magazines speaks volumes about the industry of photography. Here one may say that fashion among women is the axis on which the entire photography industry revolves. After an hour's waiting the photographer comes and opens the inner gate. In a tone of command he asks the narrator to come in. The narrator sits down in a beam of sunlight. Then the photographer rolls the machine in the middle of the room and goes behind it. He looks at the narrator and he removes the cotton sheet over the frosted window allowing the light and air to enter the studio. Again he goes back into the machine. Finally, the photographer comes in a grave look and declares, "The face is quite wrong." Therefore, he attempts to correct the face through two different methods. 1. By asking the narrator to change his facial expression and 2. by editing his photograph. He asked the photographer to stop that drama and let him go without having photographed. He started to rise from the seat. But the photographer took the photograph and asked him to come on Saturday to have a look at the proof of the photograph. On Saturday the narrator feels disheartened not to find his photograph bearing his real face. Therefore, he calls the profession of photography based on the editing as a " brutal work". it is pertinent to quote the following here:

Take your negative, or whatever you call it is you call it, dip it in anything you like; remove the eyes, correct the mouth, adjust the face, restore the lips, reanimate the necktie and reconstruct the waistcoat...... Then......

He did not like use of retouch or any other process on his face. He wanted to give gift to his friends. Thus the story ends at a satirical dimension. While leaving the studio, the narrator bursts into tears.

2.3 FURTHER MAIN BODY OF THE TEXT

Very short question- answers

Q.1What made the writer happy?

Ans: The writer was happy to know that the photographer had a human side.

DDE, GJUS&T, Hisar 212 | English (Compulsory) BA-101

Q.2What experience does the writer narrator in the story?

Ans: The writer narrates his experience with photographer.

Q.3What did the photographer look like?

Ans. He looked like a scientist.

Q.4What had the photographer done to the eyebrows?

Ans. The photographer had removed the eyebrows.

Q.5What did the narrator do while waiting for the photographer?

Ans. He read the ' Ladies Companion, the Girl's Magazine, and the Infants' Journal.

Q.6What did the photographer do with the writer's head?

Ans. He took the writer's head in his hands and twisted it sideways.

Q.7From where was the sunlight coming in the studio?

Ans. The sunlight was coming through the cotton sheet and the window panes.

Q.8 How long did it take the photographer to open the inner door?

Ans. It took him an hour to open the inner door.

Short Answer Type Question

Q.1Why did the photographer look 'very grave'?

Ans. He looked very grave because he found the narrator face as ' quite wrong' face to be photographed. Therefore, he said to the narrator that the face would be better if it were " three quarters full." The gravity of his being grave can at best be understood in the fact that he ceased in his hands and twisted it sideways. Even after twisting the face he remained frustrated with the narrator's face and declared:"I don't like the head".

Q.2 ' Is it me?' Was the narrator right in asking this question? Why?

DDE, GJUS&T, Hisar 213 | English (Compulsory) BA-101

Ans. Yes, the narrator was right in asking this question. He was right in asking this question because at the time of taking his photograph the narrator replied to the derogatory remarks of the photographer about his face with dignity. This implies that the narrator is not the victim of the fancy of having an artificial beautiful face in a photo. The photographer also needed the same to understand. Rather he edited his photo and changed it fundamentally.

Q.3 How much was the process lengthy for the writer?

Ans.The writer went to the photographer to have his photograph taken. The photographer made him wait for an hour. Then he took a lot of time adjusting his studio and camera. He went on giving instruction to the writer. He made a number of harsh comments about the writer's face. At last the photograph was taken. The writer went on Saturday to see the proof. He was shocked to find that his face had been totally changed. The photographer had touched it at many places. The writer could not recognize even his own face. Thus, the entire purpose of the photograph was waisted.

Q.4Had you been in the narrator's place, what would have been your reaction!

Ans. My reaction would not have been much different from that of the narrator. No person, however ugly likes to be told that he or she is ugly. And this is what the foolish photographer in the story does. If I were in the narrator's place, I would have asked the photographer to shut up and mind his own business. I would at once have seen that the man was a foolish person and could not be depended upon for a good photograph. So I would have gone to another photographer instead of waisting my time and my money.

Q.5Where does your sympathy lie - with the photographer or the narrator?

Ans. Our sympathy certainly lies with the narrator. The photograher has no sense. He has no quality. He doesn't know how to talk. he changes all the features of the narrator by his 'retouchng'. The poor fellow had intended to give his photograph to

DDE, GJUS&T, Hisar 214 | English (Compulsory) BA-101 his friends. He thought that thus they would remember him after his death. We feel true sympathy for him when he bursts into tears and leaves the hellish photographes.

2.3.1 Long Answer Type Question

Q.1 Discuss the elements of humour in the story.

Ans. The story, 'With the Photographer' is full of humour from the beginning to the end. In face, the very purpose of the writer in this story is to create humour. There is humour even in situations that are distressing and painful for the protagonist. The narrator's very first sentence - ' I want my photograph taken'- gives the reader a light smile. Then as the story progresses, the reader's smile continually remains there. Sometimes, it changes into giggles, chuckles and even peals of laughter.

First of all, the reader feels amused to see how the photographer prepares the narrator for the photograph. He makes him sit in a beam of light coming through the frosted window panes. Then he rolls his machine into the middle of the room and crawls into it from behind. He draws a little black cloth over himself and remains in there for quite sometime. And when at last he comes out, he shake s his head and says to the narrator, "The face is quite wrong". Some other observations that he makes are" " The ears are bad." " I don't like the head". While the narrator fumes at such humiliating comments, "The reader keeps chucking to himself.

Sample Paragraphs

The Pen is mightier than the sword

Here the pen is mightier than the sword is a famous saying. The pen stands for the authors, newspapers (print media). Thus they go a long way in forming the opinions and thoughts of the people at large, by conne cting their minds with others. Thus, we can say that the pen stands for peaceful means to advice and persuade the people to accept the writer’s idea or opinions. The power of pen has brought about big revolutions in the history of the world. But those who believed in the force of the

DDE, GJUS&T, Hisar 215 | English (Compulsory) BA-101 sword brought about nothing but violence and destruction. Thus these who used sword utterly failed to bring about desirable changes in the opinions and ideas of the people all around the world. The use of sword can be used only when it becomes inevitable. It can do work when the pen or persuation fails to bring about the desirable changes in the people's mind.

A friendly Neighbour

Blessesd are those who have good neighbours. Good neighbours sweeten everyday and render great help in emergency. They rush to our help even before any relative can. A friendly neighbour supports in every manner. They are the only one whom we can rely on. On every occasion they can be seen working and helping. Such are really gems. A friendly neighbour can come at one call when some one needs them in a worse situation. One can trust them blindly.

2.3.2 Grammar for Knowledge

Question Tags.

When in normal conversation a person makes a statement and then ask for its confirmation which is called a Question Tag. ex - You won't forget, will you?

You will be careful, won't you?

Rule:- When a positive statement is there, negative tag is used. when a negative statement is there, Positive Tag is used.

Statement

DDE, GJUS&T, Hisar 216 | English (Compulsory) BA-101

DDE, GJUS&T, Hisar 217 | English (Compulsory) BA-101

1. You won't forget 2. She will pass 3. They have won the match. 4. You live at Agra 5. I am a small girl. Answers  You won't forget, will you?  She will pass, won't she?  They have won the match, haven't they?  I am a small girl, am'nt I?

Exercise for Practice

 You are a fool.  He should work hard.  She will scold you.  They were making a noise.  I shall sing songs.  You are about to live.  The students are having fun.  She eats meat.  The earth is round.  She has no wealth.  Nobody shouted.  Are you not well.  We have no holiday today.

Change the statements into question

 He goes to school in time.  She had done her homework.

DDE, GJUS&T, Hisar 218 | English (Compulsory) BA-101

 Ram is knocking at the door.  He knows English.  The doctor has come.  You can solve this sum.  You will help me.  He is improving now.  I will go there this evening.  I read a book.

Auxiliary verb

Singular Plural

Be:- I/She/He We / They you

(am/are/is) (are)

In present

In past

I/she /He/You We/They

DDE, GJUS&T, Hisar 219 | English (Compulsory) BA-101 was/were (Were)

In present

Have:- I/She/He/YOu We /They

(Have) (Have)

In past

I / She/ He / You We/They

Had (had)

In present

Do:- I/She/He/You We/They

(do/does) (do)

In past

I/She/He/You We/They

Did (did)

DDE, GJUS&T, Hisar 220 | English (Compulsory) BA-101

2.4 CHECK YOUR PROGRESS.

1. I am ...... ( write) a letter.(Present)

2. He was ...... (teach) English in our school.(Past)

3. I ...... not watch her go to college.

4...... you enjoy parties?

5. I am ...... (water) the plants.(continous)

6. He...... (finish) the work.(Past Perfect)

7...... they play a football.

2.5 ANSWERS TO CHECK YOUR PROGRESS.

1. writing

2. taught

3. do

4. Do

5. watering

6. had finished

7. Do

2.6 CHECK YOUR PROGRESS

1. After ...... the photographer opened the inner door.

2. I have lived with it for forty years and I know ......

3. The photographer looked at me without......

Answers

1. an hour

2. its faults

DDE, GJUS&T, Hisar 221 | English (Compulsory) BA-101

3. enthusiasm

2.7 KEYWORDS

1. Conscious - Aware

2. Frosted - Ice - covered

3. Expand - Enlarge

4. Humble - Polite

5. Burst into - Eruption

6. Satire - Mockery

7. Encounter - Meeting

8. Ordinary - Usual

9. Recollect - Remember

10. Self Esteem - Self respect

2.8 SUMMARY

"With the photographer" is an amusing account of the narrator's encounter with a photographer. He describes the kind of occurency which could happen to a person in those days when people still went to studios to get themselves photographed. However, the harmony and irony with which he describes the experience transform this ordinary event into a funny one. This record of his reactions to having his face reshaped by the skilled photographer is humorous. But when the narrator gets his photo back, he is disappointed as the photo does not resemble him.

2.9 SELF-ASSESSMENT TEST

1. Use the following auxiliaries/helping verbs in sentences of your own.

a. do (negative)

b. Did (Question)

DDE, GJUS&T, Hisar 222 | English (Compulsory) BA-101

c. Am(Continuous)

d. Are ( Passive)

e. Were(Continuous)

f. Had (Past Perfect)

2. Choose the correct option.

1...... (Do/Does) you prefer tea to coffee?

2. They ...... ( Was/Were) defeated by the South Africans.

3. He...... (Does/Did) not drink milk.

4. They ...... ( Was/Were) writing a lesson.

5. She...... ( Was/ Were) elected chairman.

Q.1Why did the narrator 'feel bitter'?

Q.2What had the photographer done to the eyebrows?

Q.3Why did the narrator think that the eyes in the photograph were not his?

Q4.Why did the writer close his eyes?

2.10 REFERENCES /SUGGESTED READINGS

 “National Library of Canada: Stephen Leacock”

 Leacock, Stephen (1970) . Last leaves

DDE, GJUS&T, Hisar 223 | English (Compulsory) BA-101

Vetter: Subject : English

Course Code: BA 101 Author: Dr. Pallavi

Lesson no : 03

The Journey

By Temsula Ao

3.0 Learning Objectives 3.1 Introduction

3.2 Main Body of the Text

3.2.1 Very Short Question Answer

3.2.2 Short Question Answer

3.3 Further Main Body of the Text

3.3.1 Long Question Answer

3.3.2 Modal Auxiliaries

3.4 Check your Progress

3.5 Answers to check your Progress

3.6 Keywords

3.7 Summary

3.8 Self Assessment Test

3.9 References/ Suggested Readings

DDE, GJUS&T, Hisar 224 | English (Compulsory) BA-101

3.1 INTRODUCTION

Tensula is a poetess and a short story writer. She is the recipient of the Governor's Gold Medal in 2009 from the Government of Meghalaya. This story "The Journey" has been taken from the writer's collection of short stories entitled. These hills called home of an adolescent girl whose journey is both physical and psychological. She has poems, ethnographic pieces and short stories to her literary credit.

3.2 MAIN BODY OF THE TEXT

The story takes in a tribal village in the jungles of Nagaland. When the story starts it is very early morning. Tinula wakes up by the squealing of a piglet. The school is a few miles away and the most part of the journey has to be travelled on foot through a difficult jungle and mountainous area. After the journey to the town of Mariani, she spent the night in the loft of a shopkeeper. She saw the women in the morning of her group. They were cooking curry and rice. It was the custom to carry sufficient food for the journey. After a cold and chilly night, the girl and her brother was hungry. After the morning meal, the party set off briskly Women of the group drop some pieces of meat and rice on the leaf plate of the young girl. Therefore, she has abundant amount of rice and many pieces of meat. After eating the food, they enter the river, for the adults the water is knee deep but the water reaches up to the eyebrows of Tinula. She is taken to the other bank of the river by her brother and another man. Being totally tired she sits on a stone steps and starts crying. The brother becomes worried to think of the dangers of the jungles. In the morning she finds that one of her feet is swollen badly.

When Tinula and her brother reached the station, the winter sun was already setting. They boarded the train which was crowded. At one of the wayside stations, her brother brought tea and a local food called singaras. Finally, the train reached farkating. It was as nearly midnight. At the time the station was deserted. Luckily, they met who was going that way in his car. He offered to take them & drop to the school.

DDE, GJUS&T, Hisar 225 | English (Compulsory) BA-101

On reaching the school, Tinula is allowed to share the bed with her friend Winnie. TInula feels happy as she finds the warmth of the bed after her cold and long journey. The softness of the sheets reminds her of her last night where she has to fling for her share of the torn blanket. The transition thrills her and she begins to giggle. When Tinula was about to fall asleep, Winnie said to her that her boyfriend Hubert had a new friend now. Tinula was totally surprised by her statement. She knew that Hubert was a friend and had told her friends that Hubert was a good friend. However, these words hurt her. To conceal her real emotions, she started laughing softly.

As the time has passed Tinula could not say whether she laughed or cried on that night. But it was a night of transition in her life. But Winnie's remarks made her realize that the barrier's she faced are not merely physical. Tinula recalls Winnie's deliberate attempt to hurt her. Now she often wonders what happened to a boy named Hubert whom she had never met. Winnie's remarks had forced her, who was only thirteen year old, to embark on a different kind of journey. It was an adventurous journey with new emotions awakened in her heart and soul.

3.2.1 Very short Question Answers.

Q.1What does the story focus on?

Ans. The story focuses on the physical as well as journey of emotions.

Q.2What is the time when the story starts?

Ans. The story starts in the very early morning.

Q.3How is Tinula waken up early in the monring?

Ans. She is waken up by the sound of squealing of a piglet.

Q.4Why did Tinula's brother ask her to walk faster?

Ans. He said that the wild animals could attack them in the evening.

Q.5Who had given her the shoes?

DDE, GJUS&T, Hisar 226 | English (Compulsory) BA-101

Ans. A senior student had given her the shoes.

Q.6Where did the supritendent gave shelter to Tinula?

Ans. She gave her shelter in the infirmary.

Q.7 Why was the journey so difficult?

Ans. The journey was difficult because of the deep rivers and presence of wild animals.

Q.8 How do the travellers get provisions for their journey?

Ans. They get the provisions of their journey by means of selling oranges, ginger, jam and at times special sticky rice.

3.2.2 Short Answer Type Questions

Q.1 How does the story bring out the importance of the effort of the community in helping an individual?

Ans. The story brings out the importance of the efforts of the community in helping an individual. In this story Tinula and her brother have to undertake two difficult journeys. Each member has brought meals with them. The people of the group help Tinula one woman gives her a lot to eat. In the train man gives Tinula a seat. Another person gives Tinula and her brother lift in his car. Thus the members of the community help one other.

Q.2What is special about the landscape portrayed in the story?

Ans. The story is set in a village in the Naga Hills. The landscape is beautiful as well as difficult. This is a hilly area. There are dense forests also which abound in wild animals. In the rainy season it becomes difficult to cross. Life in this region is very tough & harsh. But there is a strong bond between the landscape and the people.

Q.3 How do the people of the hill community prepare for the journey?

DDE, GJUS&T, Hisar 227 | English (Compulsory) BA-101

Ans. In the morning Tinula saw the women of their group. They were cooking rice and curry enough for the whole day of the journey. It was the custom to carry sufficient provision for the journey. After an icy night, the girl along with her brother was very hungry.

Q.4 Describe the experience of Tinula and her brother from the station to the school.

Ans. Finally, the train reached farkating, the station which was nearest to her school. It was nearly midnight and the station was deserted. The school was three or four miles from there. Luckily they met a man who was going that way in his car. He offered them a lift. The brother and the sister squeezed themselves in the car as there were other people also in it.

3.3 FURTHER MAIN BODY OF THE TEXT

3.3.1 Long Answer Type Questions.

Q.1 With reference to the story, ' The Journey', writes about the natural beauty of the North East.

Ans. Assam is known as the Shangri-la of the North East India. That area is rich in natural beauty. The lakes and rivers of that area reflect the colours of the changing sky. The fields soothe and refresh the minds of the people. As the season changes, the beauty of the North East also undergoes certain changes. Drops of rain bathe the landscape and keep it fresh. Assam is famous for tea gardens and one horned rhinos.

There is a fine mixture of many cultures. Bihu is celebrated by all people. But the terrain is tough and difficult. Tinula and her brother undergo two difficult journeys. But after the night when the day breaks, the whole area is bathed as beauty.

Q.2 Describe Tinula's journey from her boarding school to her village.

Ans. Tinula's brother escorted her. They started their journey early, so they could reach home by night. Her brother kept telling her to walk faster. But the girl could

DDE, GJUS&T, Hisar 228 | English (Compulsory) BA-101 not keep pace with the other and remained behind them. They reached the banks of Disoi river. They opened their leaf packets and took their midday meal. When they entered the river, the water was knee deep for others but it reached upto the eyes of Tinula. So her brother and another man lifted her up. After crossing the river, the mountain area was difficult to head. Tinula could not walk and started crying. Tinula remembered how she struggled over every step and by the time she reached the village, the sun had set

Grammar for knowledge .

3.3.2 Modals

Modal Auxiliaries are:- shall/should/will/would/can,could,may, might, must, dare, need, ought to, used to.

These are used along with main verbs. The modal auxiliaries express the 'mode' or 'manner' of actions denoted by the probability, permission, obligation or duty,etc.

Main Points

1. The modal auxiliary does not change form according to the number or person of the subject.

ex - I can drive, You can drive, We can drive.

2. The modal auxiliary does not stand alone but always goes with a full verb. ex - You must work hard.

DDE, GJUS&T, Hisar 229 | English (Compulsory) BA-101

3. Only the first form of the verb can be used after a modal auxiliary. ex - He can write. Modals  Shall:- With the first person, is used to express simple future ex - we shall leave for Delhi tomorrow. Shall, with the second & third person is used, to express a command, a threat, a promise. ex - You shall do it. You shall die for it. You shall get leave today.  Will - the second and third persons, will be used to express Simple Future, as ex - He will leave for Bombay tomorrow. Will,with the first person is used, to express willingness, promise, threat, determination ex - I will help you as far as possible. I will come in time. I will beat you. I will do or die.  Should - should is the Past tense of shall. It is used to denote in indirect speech. ex - I said that I should go. to express duty,purpose, probability,inference ex - We should respect our elders. She works hard lest she should fail. Should they play well, they will win.

 Would - Would is the past tense of 'Will'. It is used to to denote the past tense of will or shall ex - She said that she would go.

DDE, GJUS&T, Hisar 230 | English (Compulsory) BA-101

to express a habitual activity in the past ex - He would go for a swim in the sea. to make a polite request a wish or a preference. ex - would you take a cup of tea. Would that I were a king. I would rather die than beg.  Can - is used for all persons in the present tense.

 Could is used for all persons in the past tense. Can is used

DDE, GJUS&T, Hisar 231 | English (Compulsory) BA-101

to express permission, ability, possibility ex - you can go now. I can swim Anyone can make mistakes. Could is used to express polite request. ex - Could you send me some money?  May, Might - May is used to express possibility,wish,permission. ex - It may rain. May he live long! You may go.  Might - must be used when the main verb of the sentence is in the past tense, To express permission, possibility, ex - He said that I might borrow his cycle. He thought that I might like it  Must - Must refers to the present or the future Tense. To denote complusion, certainity, determination, duty , possibility. ex - A servant must obey his master. All must die sooner or later. I must see you again before I leave for Delhi. We must practice Yoga. He must be at least thirty years old.  Ought to- Ought refers to Present, Past or Future. It is used to denote the sense of duty, strong probability. ex - We Ought to love our friends. She ought to pass this time.

DDE, GJUS&T, Hisar 232 | English (Compulsory) BA-101

 Dare , Need - Dare, Need are peculiar verbs and can be used as main verbs. They are followed by to + first form of the verb. ex - He dares to disobey his master. it is used to denote to venture, challenge ex - I dare not touch the wire. I dare you to face me.  Need- It is used to express necessity or obligation. ex - He need not buy a bike. Need I go there?

DDE, GJUS&T, Hisar 233 | English (Compulsory) BA-101

Exercise 1...... you teach me how to write? (Ans=will 2. They ...... be glad to hear of you win. (Ans=will 3. I ...... be glad if you suggest me in this matter.(Ans = shall 4. You...... go instantly. 5. he ...... be sorry to miss the train. 6...... you meet her, bring her here. 7...... you please write a letter for me? 8. I ...... teach you a lesson. 9...... I come in, Mam? 10. You...... now go. 11. You ...... to do this. 12. A teacher...... be up right.

DDE, GJUS&T, Hisar 234 | English (Compulsory) BA-101

13. She ...... work or starve. 14. He ...... not disobey. Answers :- 1. Will 2. Will 3. Shall 4. Should 5. Would 6. Should 7. Can 8. Could 9. May 10. may 11. Ought 12. must 13. Must 14. Dare

Modals at a glance

Expression Modal to be used

1. Present ability / capacity...... can 2. Permission...... can 3. Asking Permission...... may 4. Wish/Blessing...... may 5. Possibility...... May 6. Slight possibility...... Might 7. Purpose (after so that) ...... May 8. Purpose (of that(past)...... might 9. Past ability...... could

DDE, GJUS&T, Hisar 235 | English (Compulsory) BA-101

10. Polite request...... will/would/could 11. Duty/Compulsion...... must 12. Willingness/Determination...... will 13. Simple future(ii and 3rd person)...... will 14. Simple future (I/We) ...... shall 15. Habit...... will 16. Preference...... would 17. Promise/Threat...... shall 18. Absence lack of necessity...... need not 19. Absence of courage...... dare not. 20. Accustomed...... used to 21. Social/moral obligation...... ought to 22. Command...... Shall 23. Invitation/Offer...... would 24. Existence in the past...... used to 25. Certainity...... must

Exercise for Practice

Fill in the blanks with ought to, should , must , need used to(positive or negative) as the case may be.

1. We...... not be rude to our elders. 2. I...... surrender to his whims. 3. He ...... Keep his promise, otherwise people will not respect him. 4. You...... do as you are told. 5. We...... not spit on the floor. 6. Whom am I that I ...... give advice? 7. Little children...... be careful while crossing the road. 8. It is five minutes to seven. The brain...... have arrived at the station. 9. In the month of september, it ...... be raining in delhi.

DDE, GJUS&T, Hisar 236 | English (Compulsory) BA-101

10. He ...... control temper, otherwise he will be sick.

3.4 CHECK YOUR PROGRESS.

Give Synonyms

1. Holiday 2. Flow 3. Hospital 4. Firm 5. Brushed

Answer the questions

1. Who is Winnie? 2. Which place or region has been described here? 3. Why was the journey so difficult?

3.5 ANSWERS TO CHECK YOUR PROGRESS

1. Vacation 2. Current 3. Infirmary 4. determined 5. Jostled. Answers 1. Winnie is a girl whom Tinula considers as her good friend but her hurts Tinula very pathetically. 2. The region of Naga Hills has been described in the story. 3. The journey was difficult because of the deep rivers and presence of wild animals in the jungles.

3.6 KEYWORDS

DDE, GJUS&T, Hisar 237 | English (Compulsory) BA-101

 Piglet - Offspring of a pig.  Prospect - Possibility  Swift - fast  Garland - Wreath  Dangling - Hanging  Plight - Misery  Hurricane - storm  Boon - Blessing  Giggle - laugh lightly  Pretend - to show  Drastic - Severe  Sticky - Gummy  Swift - Rapid  Ensure - To Safeguard  Urge - longing  Hoist - Raise  Enormously - Hugely  Huddle - Cluster  Determined - Committed  Foliage - Greenery  Jargon - words that are used by particular profession  Annoy - Bother  Tatter - Piece  Muffle - Silence  Antipathy - Hatred  Piglet - Offspring of a pig.  Prospect - Possibility  Swift - fast

DDE, GJUS&T, Hisar 238 | English (Compulsory) BA-101

 Garland - Wreath  Dangling - Hanging  Plight - Misery  Hurricane - storm  Boon - Blessing  Giggle - laugh lightly  Pretend - to show

3.7 SUMMARY

The title of the story is loaded with deep semantic connotations. The narrator gives detailed description of two journeys. She presents a comparative view point of the plains of Assam and the natural scenes of Naga hills. Here people have to manage their meals in collaboration and have to face a number of problems. In the beginning, a vivid description of journey from school to the foothill town of Mariani is given. In the middle, the narrator memorizes the problems faced by the brother sister. In the end, the traumatic comment of Winnie makes Tinula realizes, " the barriers of life are not only the physical ones." But the value of life is only estimated when we jump into it.

3.8 SELF ASSESSMENT TEST

1. What is the time when the story starts? 2. What does the story focus on? 3. Where did Tinula spend her night in the town of Mariani? 4. Who had given her the shoes? 5. What kind of journey is referred to at the end. 6. How did Tinula and her brother reach school from the station. Self Assessment test A. India ...... make progress only by following the principles of peace and co-operation. Every citizen ...... realize that he...... co-operate with his fellow citizen if he ...... achieve success. Co-operation

DDE, GJUS&T, Hisar 239 | English (Compulsory) BA-101

...... to be the basis of our life. No individual...... benefit anything. If he ...... worry about his safety. On the other hand, we ...... fear if we are united. B. The old woman...... you kindly do me a favour? The Young woman - what...... I do for you mam? The old woman - I ...... not go across the road...... You give me a helping hand? The young woman - You are like my granny. Such a situation...... happen with anyone in old age. I ...... most gladly help you.

3.9 REFERENCES/SUGGESTED READINGS

 Temsula Ao, These Hills Called Home: Stories from a War Zone, Zubaan (2013 edition)

DDE, GJUS&T, Hisar 240 | English (Compulsory) BA-101

Vetter: Subject : English

Course Code:BA 101 Author: Dr. Pallavi

Lesson no 4

The Refugee

By Khwaja Ahmed Abbas

4.0 Learning Objectives

4.1 Introduction

4.2 Main Body of the Text

4.2.1 Very Short Question Answers

4.2.2 Short Question Answers

4.3 Further Main Body of the Text

4.3.1 Long Question Answers

4.3.2 Paragraph Writing

4.4 Check your Progress

4.5 Answers to check your Progress

4.6 Keywords

4.7 Summary

4.8 Self Assessment Test

4.9 References/ Suggested Readings

DDE, GJUS&T, Hisar 241 | English (Compulsory) BA-101

4.0 LEARNING OBJECTIVES

1. The student will learn the theme of conflict, independence, loyality and acceptance. 2. Students should be able to interpret and explain the content in English. 3. Understanding the reference to context and explanation of passages. 4. Students will be able to get the knowledge of paragraph writing.

4.1 INTRODUCTION

Born in 1914 in Panipat, Haryana,"Khwaja Ahmad Abbas" was a famous Indian novelist, journalist, film director and script writer. He began his career as a journalist first by joining the National call, and then later the Bombay Chronicle. "The Refugee' tells the story of an old woman from Rawalpindi. The old woman, also called Maanji, is evacuated from Rawalpindi. Finally she comes to Mumbai with the writer's Mother. She learns a lesson full of hatred and bitterness. She has seen how the neighbours, both Hindu and Muslim, turn against each other after the partition. Her story is tragic. As a film maker, he founded his production company, Naya Sansar, in 1951. The Government of India honoured him with the Padma Shri in 1969. Khwaja Ahmad Abbas represented a generation of the playwrights associated with IPTA( Theatre Association). The Associates of IPTA would use folk art forms and idioms to promote their radical ideas for social change. The progressive thinking of IPTA goes deep in making of Abbas as a literary figure.

4.2 MAIN BODY OF THE TEXT

In the year 1947, India was partitioned. There were wide spread riots between the Hindu and Muslims. The partition has been called a " tragic storm" by the narrator as it uprooted people from their ancestral home. In october of that year, two old women had to travel hundreds of miles away from their homes. One of them was the writer's mother. She came to Mumbai from Panipat. The other was an old Sikh woman. But after the partition first she was sent to Delhi and from there to Mumbai Maanji has lived in Rawalpindi before . The Muslim women of the

DDE, GJUS&T, Hisar 242 | English (Compulsory) BA-101 neighbourhood call her behanji. It has been the pattern of living not only in their neighbourhood, but all over the Punjab. In Rawalpindi, Maanji lived in her own house. The old couple had an only son. He first worked in Lahore, then Calcutta and finally in Mumbai. At home, the couple had a buffalo. She daily churned the milk and made curd and butter. The she distributed butter milk to the whole neighbourhood. They had income from the rent of shops also. They were a contented couple.

In June, there was the news of the impending partition of India. But it did not alarm the old woman. The old couple has concern with their neighbours with them their relations has always been cordial. But now there was violence and hatred everywhere. Then some Muslim came from East Punjab and spread false rumours. There were fierce communal riots. Some Muslim ne ighbours came to her. But even then ' Maanji' believed that everything will be alright. Rather she sends them donations of foodstuffs, clothes, blankets etc. Then one day, Maanji's faith in fellow beings was shattered. She witnessed the murder of a Tonga-Wallah in front of her house. She saw that was a Hindu. But the brutal killing of his horse pains her too deeply as a horse has no religion. Then she knew that the communal madness had gone too far. She felt that they were no longer safe in Rawalpindi.

Then she and her husband locked up the house and left everything behind. She still hoped to come back one day. By the time, she reached Delhi. She witnessed horrible scenes of communal madness. In Rawalpindi she has two male servants and a maid servant whereas in Mumbai she had to live in a single room house. In Rawalpindi, her house was spacious. But in Mumbai, there was a small kitchen, served as a bathroom and store room. After partition, her health can also be not called good. She cooked everything with her own hands. However she never sits idle and sleeps only for six hours at night. She never speaks about her tragedy. Maanji had a fair complexion. She had short stature and frail body. She got attacks of asthma. She got up first in the morning and was the last to go to bed. She still remembered her

DDE, GJUS&T, Hisar 243 | English (Compulsory) BA-101

Muslim neighbour of Rawalpindi. Her face gets brightened whenever her husband reads out a letter received from Rawalpindi.

At the end of the story she is depicted as a silent woman with " tears bubbling up in her tired old eyes. Her heart has “neither anger nor self- pity but memories." Thus the story depicts the traumatic psyche of a refugee that is representative of all the sufferers of partition.

4.2.1 Very Short Question Answer Type

Q.1What does the ' storm stand fore here?

Ans :- The word storm' stands for the partition of India in 1947.

Q.2Where were the ' two weak old women' blown to?

Ans :- The two weak old women were blown to Bombay.

Q.3Who reached Bombay the same day?

Ans. The narrator's mother and the mother of his sikh friend reached Bombay the same day.

Q.4 Before Partition, which place was the "whole world" for Maanji?

Ans. Before Partition, Rawalpindi was the 'whole world' for Maanji.

Q.5 Who is the writer of the story, ' The Refugee'?

Ans. K.A.Abbas is the writer of the story.

Q.6What was the steady source of Income for her?

Ans. Rent from the shops was the steady source of income for Maanji.

Q.7What frightful scene did 'Maanji' witness in front of her house?

Ans. Some Muslims killed a Hindu tonga-wallah. This scene made Maanji frightened.

Q.8 Whom did " Maanji" help with food, bedding and blankets?

Ans. She helped the Muslims with the facilities of food, bedding and blankets.

DDE, GJUS&T, Hisar 244 | English (Compulsory) BA-101

4.2.2 Short Answer Type Questions

Q.1. How did the 'two women' arrive in Bombay?

Ans. Writer's mother and Sikh friend's mother arrived in Bombay from Panipat and Rawalpindi respectively. By a strange chance they reached Bombay on the same day. Writer's mother, along with other women and children of her family, was evacuated from Panipat in a military truck and brought to Delhi. She stayed there for three weeks crowded in a small room with two other families, and then came to Bombay by place because it was still unsafe to travel by train. Writer's Sikh friend's mother, along with her old husband, came in a refugee caravan from' Pindi' to Amritsar, from there to a refugee camp in Delhi, and finally from there to Bombay.

Q.2 Prior to partition what did Maanji think of Muslims?

Ans. Before Partition, she thought that Muslim and Hindus were like brothers. She thought that a brother cannot kill his brother. She even helped the Muslim refugees from East Punjab. She distributed free food, beddings and blankets to them. When there was the news of the impending partition of India. She did not bother. But one incident changed her outlook. A Hindu Tonga wallah was killed by some Muslims. Even his horse was not spared.

Q.3 Describe the daily life of the old Sikh couple in Rawalpindi?

Ans. At home the couple had a buffalo. She daily churned the milk and made curt and butter. Then she distributed butter milk to the whole neighbourhood. They had their own land. It was leased out to some farmers. So they had wheat, maize, bajra, milk, butter and ghee in abundance. They had income from the rent of shops also. Thus they were a contented couple.

Q.4 How does the old ' Sardar' couple react to the news of the impending partition?

Ans. The first response of the old couple to the news of impending partition does not worry them. The old couple has concerns with their neighbours with them their

DDE, GJUS&T, Hisar 245 | English (Compulsory) BA-101 relations has always been cordial. They don't mind whether the country is called Hindustan or Pakistan. Maanji's son who works in Bombay asks his parents to come Bombay. But she refuses to leave her beloved Rawalpindi. Whenever anyone shows one's concerns for the safety of the old couple she says" who will harass us here? After all the Muslims who live around us are all like my own children - aren't they?

Q.5What was it that worried Maanji About her son?

Ans. Maanji had an only son. He didn't live with her in Rawalpindi. He worked first in Lahore, then in Calcutta and finally in Bombay. The mother remained worried to think about the food her son had to eat. While there was plenty of pure ghee, butter, curt and lassi at home, her son had to eat hotel food. And that would make her sad. She would often say to him: 'What is the use of earning money, my son, when in those cities you get neither pure milk nor ghee, neither apricots nor peaches, neither graphe nor apples. And baggogoshas? Why, in the city, they don't even know what this is!

Q.6 How did the old ' Sardar ' Couple react to the news of the impending Partition?

Ans. The old couple didn't make much of this news. They thought that politics was no concern of common peaceful people like them. It did not matter to them whether the country was called Hindusan or Pakistan. They thought their concern was with their neighbours only. And with their neighbours, their relations had always been friendly and cordial. As for communal riots, they had been there earlier also. The old couple called such riots a fever of the mind that always cooled off.

4.3 FURTHER MAIN BODY OF THE TEXT

4.3.1 Long Answer Type Questions

Q.1 "The Refugee" proves that men and women are capable of behaving both as uncouth animals and noble human beings. Discuss.

DDE, GJUS&T, Hisar 246 | English (Compulsory) BA-101

Ans. Simply having two hands and two legs does not make one a human being. It is only by having a kind and humane heart that one becomes a true human being. Bereft of this basic trait, one is no better than an uncouth animal. The old woman, called Maanji in the story, is a perfect specimen of a noble human being. She was a sikh and lived in Rawalpindi where most of her neighbours were Muslims. She never had any problem with them. The older Muslim women called her Behanji while the younger ones respectfully addressed her as Maanji or Chachi. The old woman reciprocated their love and regard in equal measure. She had a buffalo that gave no less than her seers of milk everyday. She churned the curd to take out butter and distributed the buttermilk to the whole neighbourhood. When some Muslim refugees from East punjab settled in her neighbourhood. She felt moved by their light. She sent for them donations of foodstuffs, clothes, blankets and bedding. She never thought of them as Muslims, supposed to be the enemies of her people.

Among Muslims also, there was no dearth of such noble souls. When the communal madness was at its worst, her Muslim neighbours came to her and pleaded with her to go away to place of safety. But still there were some who reassured her and promised that they would protect her life, honour and property with their own lives. A Muslim tailor, who was a tenant of hers, kept watch on her house day and night. He saw to it that no harm came to his landlady. There could be no greater example of a noble human being.

But there were some two legged uncouth animals also. They stabbed a tongs-wallah to death because he was a Hindu. And they did not stop at that. They did not spare even the horse who had no caste or religion. They went on stabbing the poor, dumb animal with their daggers tall it blem to death.

Q.2 Describe the life of 'Maanji' in Bombay.

Ans. In Rawalpindi, the house of Maanji had six spacious rooms. But in Mumbai she had to live in a single room house. There was a small kitchen which also served as a bathroom and store room. She kept everything clean. She cooked everything. But she

DDE, GJUS&T, Hisar 247 | English (Compulsory) BA-101 never forgot her North Indian hospitality. Maanji had a fair complexion. She had short stature and frail body. After partition, her hair almost turned white. She never sat idle, except for sleeping for six hours. She never felt sad that she had lost so much. Even after partition she still recalls her Muslim neighbours with affection. She remembered her friendly relationships with her neighbours in Rawalpindi. Sometimes there were eyes of maanji full of tears. But she never had any anger or self pity in her heart.

Q.3 How would you consider, ' The Refugee' as a story about displacements, geographical, social, political, moral and spiritual?

Ans:- The writer describes firstly the geographical displacement. He explains that the partition blows away people from one end of the country to the other from Delhi to Karachi , from Karachi to Bombay, from Lahore to Delhi, from Lyallpur to Panipat, from Rawalpindi to Agara and so on. Ten million people have been suffers of this " tragic storm" who have been compared with " autunm Leaves" very appropriately.

The writer shows the deep concern of old lady with her social miliers of Rawalpindi. It has been the pattern of living not only in the neighbourhood, not only in Rawalpindi, but all over the punjab. The old couple has small but steady income from the rent of the shops. The town of Rawalpindi has been the whole world for Maanji as she has never been elsewhere before partition.

In the end, the writer depicts the displacement. The old woman had a no. of servants in Rawalpindi. But in Bombay she does all her work by herself. Her hair had turned white. Her health was not also ve ry good. She gets attack of asthma and new religion.

The tragic event of the killing of a Hindu Tonga wallah and his horse in front of her house shatters the last thread of faith. She says that they killed him simply because he was a hindu. But the brutal killing of his horse pains her too deeply as a horse has no religion. Now she realizes the madness.

DDE, GJUS&T, Hisar 248 | English (Compulsory) BA-101

Grammar for Knowledge

4.3.2 Paragraph Writing

A paragraph is a unit of composition in which one idea or thought is developed by means of examples or supporting details. Every good paragraph has a controlling idea, to which its sentences are related. As a paragraph is build around a single idea, it is possible to make a brief statement of this idea in one sentence. This is the key sentence, topical sentence or the pilot sentence. The most common pattern of paragraph writing is to state the main idea by the very first sentence which begins the paragraph.

Main features of a Paragraph

1. Unity:- The most important thing of a paragraph is unity. It means that all the sentences in a paragraph should bring out only one thought, idea or subject. The unity of a paragraph is thus uphold by the facts logically arranged, the

DDE, GJUS&T, Hisar 249 | English (Compulsory) BA-101

sentences following in a natural sequence and all the points converging on the main idea. 2. Order:- All the sentences in a paragraph should be arranged in a logical and natural order. All the sentences are inter related keeping in view the order of time, place, cause and effect or the relative importance of the ideas.

How to write a Paragraph

1. Clear thinking is necessary to understand the main idea or title about which you are expected to write. 2. Think out the points. Jolt them down. In order to build your points, raise questions about the given topic.

DDE, GJUS&T, Hisar 250 | English (Compulsory) BA-101

3. Arrange the points in a logical and proper order. 4. Start with the most striking sentence which may express the main idea and awaken the reader's interest. 5. Vary the length & structure of your sentence. 6. Avoid repetition & monotony. 7. Use linking devices. 8. Every sentence should state a clear fact or idea. 9. End up with a sentence that summarizes what you have said. It should be interesting, attractive and impressive. 10. Do not fail to revise carefully what you have written. Correct all errors, spelling & unctuation.

Different ways

DDE, GJUS&T, Hisar 251 | English (Compulsory) BA-101

1. The Chronological order:- The facts or statements may be arranged in order of time or the chronological order. 2. The spatial Arrangement:- This arrangement can be used in describing objects, scenes, buildings etc. 3. The logical order:- Here, a statement is made at the outset. Reasons are given leading to a conclusion or a cause and effect relation is presented. This arrangement can be used in explaining a proverb, a scientific fact or observation. 4. The use of Connectives:- Here it is used to show relation between sentences by using connecting words. They are known linking devices. 5. Variety:- Variety is the third important quality of a paragraph. In a Paragraph, the sentences should be of different lengths. They should not always be of the same kind.

For example:- Diwali : The Festival of Lamps

Diwali is an important festival for Hindus. Preparations for celebrating Diwali are made quite in advance. Shops and houses are white washed. The walls are decorated with pictures. New clothes and furnishings are purchased. Children get new clothes and gifts. On the Diwali day, there is a festive mood. We go to market and buy sweet, toys, candles, crackers and calendars. In the evening, we light lamps and candles. Some people illuminate their homes with rows of electric lamps. After worshipping the goddess of wealth, . We fire crackers and then present gifts and sweets to our friends, relatives and neighbours. Diwali fills use with joy and light.

4.4 CHECK YOUR PROGRESS

Fill in The Blanks

1. The players of the team...... ( be) having their lunch. 2. A pair of socks ...... (be) lying on the bed. 3. Two notebooks or a textbook ...... (be) necessary. 4. Bread and Butter ...... (be) his usual breakfast.

DDE, GJUS&T, Hisar 252 | English (Compulsory) BA-101

5. Eighty kilograms...... (be) quite a load. 6. The material in the school ...... (be) outdated. 7. The owners of the factory ...... be(rich) 8. All the luggage...... (have) now been spent.

Complete the sentences.

1. You know a horse has neither...... 2. The town of Rawalpindi was the ...... 3. and suddenly she is silent, tears bubbling...... 4. I called my mother ' Amman' and my friend called......

4.5 ANSWERS TO CHECK YOUR PROGRESS.

1. are 2. is 3. is 4. is 5. is 6. is 7. are 8. have

Complete the sentences

1. religion nor caste 2. whole world 3. up in her eyes 4. his mother Maanji.

4.6 KEYWORDS

1. Ruthlessly - Cruelly 2. Ancestral - belongings of ancestor 3. Caravan - a convoy of travellers

DDE, GJUS&T, Hisar 253 | English (Compulsory) BA-101

4. Tenants - those who live on rent. 5. Steady - Continuous 6. Communal - based on caste and religion 7. Harass - to trouble 8. Frightful-fearful 9. Prevalent - Spread 10. Flavour - form 11. Dorning - Mending 12. Intensely - Deeply 4.7 SUMMARY The Refugee" tells the story of an old woman from Rawalpindi. She becomes a refugee as a consequence of the partition of India in 1947. The old woman known as " Maanji" is evacuated from Rawalpindi. Then she is sent to a refugee camp in Delhi. Finally she comes to Mumbai with the writer's mother. She learns a bitter lesson about her fellow Indians in Rawalpindi. She has seen that neighbours and friends, both Hindu and Muslim, living in perfect harmony, turn against each other after the partition. Her story is tragic. Like many others she was also compelled to leave her home and travel into alien lands.

4.8 SELF-ASSESSMENT TEST

Q.1What news was published in Newspapers in June 1947?

Q.2How was her Complexion and stature?

Q.3 Point out the simile used in the opening?

Q.4 In what kind of house did ' Maanji' live in Bombay?

Q.5What was the steady source of income for her?

B.) Answer the questions of the passage

In the October of the year of sorrowful memories, this very storm blew two weak old women into Bombay. One of them was my own mother and the other was as the

DDE, GJUS&T, Hisar 254 | English (Compulsory) BA-101 mother of a sikh friend and neighbour of mine. One had come from the East Punjab and the other from the west Punjab, one from Panipat and the other from Rawalpindi. My mother, along with other women and children of my family was evacuated from Panipat. My friend's mother along with her old husband, came in a refugee caravan from, Pindi to Amritsar

a.) Who were these two women? b.) How did the narrator's mother travel to Bombay? c.) How the old women came to Amritsar? d.) From where did the two women came to Bombay?

4.9 REFERENCES / SUGGESSTED READINGS

 https://doi.org/10.1017/CBO9780511760990.004

DDE, GJUS&T, Hisar 255 | English (Compulsory) BA-101

Subject: English

Course code: BA-101 Author: Dr. Narsingh Jangra

Lesson: 5 Vetter:

PANCHLIGHT

Structure

5.1 Learning Objectives

5.2 Introduction

5.3 Main Body I

1.2.1 Text of the story

1.2.2 Vocabulary of the text

5.4 Main Body II

1.3.1 Introduction to the author

1.3.2 Summary

5.5 Check your progress

5.6 Transcription

5.7 Comprehension

5.8 Examination style questions

1.8.1 Short questions

1.8.2 Long questions

5.9 Answers to check your progress

5.10 References/Suggested Reading

DDE, GJUS&T, Hisar 256 | English (Compulsory) BA-101

5.1 LEARNING OBJECTIVES

After going through this lesson you will be able to-

 Read, understand and enjoy the story.  Enrich your taste for literature.  Learn and understand the fundamentals of prose.  Learn about the biography of the writer.  Develop reading, writing, and speaking skills of the English language.  Enrich your vocabulary.  Learn the phonetic transcription of some English words.  Learn to answer the examination style questions.

5.2 INTRODUCTION

The story, Panchlight, is set in an unnamed village of Bihar. It revolves around the arrival of a petromax in the village. The petromax, called 'Panchlight' in the story, is a type of kerosene lamp which has paraffin in it and has a hand pump to pressurise the oil into a mantle and to light it up. There are eight Panchayats in the village, each based on a different caste, called a Toli. All other Panchayats have their own panchlight (Petromax), except the Mahto Toli. Mahto Toli buys its own Panchlight. The writer makes this Panchlight the central character and weaves the whole story around it. They plan to offer pooja to their Panchlight before they light it. But, in this Toli no one knows how to light it. In the end, to save the honour of their Toli, they had to request Godhan whom the panchayat had banished, to light the lamp.

5.3 MAIN BODY OF THE TEXT

5.2.1 Text of the story

The elders of Mehto Toli bought a petromax at this year’s Ramnavami fair from the fines and penalties collected over the past fifteen months. There are eight Panchayats in the village, one for each community. All the panchayats have

DDE, GJUS&T, Hisar 257 | English (Compulsory) BA-101

their own durries, jajim, mats and a petromax lamp, which the villagers call the panchlight.

After buying the Panchlight, the panchayat members decided to buy offerings for puja with the ten rupees that were left over, for it would be most inappropriate a technical object with nuts and bolts without first having it suitably blessed. After all, even in the time of the British Bahadur, a sacrifice would be offered to appease the gods before work could begin on building a bridge.

All the panchayat members returned home from the fair well before sunset— the orderly in front carrying the box with the Panchlight on his head, followed by the chief, the secretary and the other members. Phutangi Jha of the Brahmin Toli Stopped them at the outskirts of the village and asked, ‘How much did you buy this lantern for?’

‘Can’t you see? This is Panchlight! You people from the Brahmin Toli think no end of yourselves. You will call the flickering oil lamp in your home an electric bulb and when someone else buys a Panchlight, it looks like a lantern to you!’

By the time they reached their own toil, word had spread. Everyone dropped whatever they were doing and rushed to see the panchlight. ‘Come on, come on. Hurry! Our Panchlight has come!’

The orderly, Agnu Mahto, kept issuing stern warnings; ‘Watch out! Don’t come too close! Don’t touch it! See you don’t damage it!’

The chief of the Panchyat told his wife: ‘There will be a puja in the evening. Hurry up and cook my dinner, and you too take a bath and get ready.’

The head of the neighbourhood troupe of singers told his musicians: ‘Look here, tonight we shall be performing in the light of Panchlight. So all you out- of-tune types better watch out! One false note and you are out!’

DDE, GJUS&T, Hisar 258 | English (Compulsory) BA-101

Gulri Kaki, one of the leading lights of women’s choir, began to hum. Little girls and boys started a ruckus, shouting and crying for no reason save uncontrollable excitement.

There was still an hour to go before sunset, but people had already gathered at the chief’s door. The chat went up: ‘Panchlight! Panchlight!’

There was no other topic for either conversation or gossip, except the Panchlight. The chief gurgled his hukkah amd said, “ The shopkeeper asked for five cowries and five rupees. I said, “ Don’t take me for an ignorant rustic; I have seen lots of Panchlights.” At first, the shopkeeper could only gape at me. Then he said, “Looks like you are a chief by caste! All right, since the chief himself has come to my shop to buy a Panchlight, I will charge only five cowries only for it,”

The secretary of the Panchyat added: That shopkeeper could read faces. The shop’s servant didn’t want to give us a box for the Panchlight, but I said, “ Mr. Shopkeeper, how can we carry the Panchlight without its box?” the shopkeeper scolded his servant and said, “ What is this? Are you trying to fool the secretary? Give him the box.”

The people from the toil looked at the chief and the secretary with new respect. The orderly was telling the crowd of women, ‘It was making funny noises all the way- sannn- sannn...’

But ...at the precise moment a ‘but’ reared its ugly head. Three bottles of kerosene had been fetched from Rudal Shah’s shop, but now the question was: ‘Who would light the Panchlight?’

No one had thought of it so far. No one had thought of it before buying the Panchlight, and no one did after it was bought. The offerings for the puja were ready, the singers were waiting with their drums, cymbals and bells, while the Panchlight stood in the middle of it all. The villagers had never bought

DDE, GJUS&T, Hisar 259 | English (Compulsory) BA-101

anything like this before. There is a saying in the village: ‘Shall we buy a cow?’ Yes, but who will milk it?’...And so it was with this contraption of nuts and bolts... ‘Who will light it?’

It was not that no one in the entire village knew how to light a Panchlight. The issue here was: ‘After all the preparations, the ritual offerings and puja, will someone from another toil have to be brought in to help light our panchlight? Wouldn’t it be better to let it lie around? Who can bear the insults and mockery for the rest of one’s life? People from the other tolis will taunt us at the slightest pretext, “So, you got someone from another toil to light your...” No, no, it is a matter of our panchyat’s pride. Don’t even think of asking for help from other panchyats.’

Sadness descended on the crowd. The evening darkened. No one had lit even an oil lamp in his home...Who would want a flickering oil lamp now that the Panchlight had been bought?

But it had all come to nought! And, not a word from the chief, the secretary or the orderly1 the panchyat was in serious danger of losing face. Someone said in a feeble voice: ‘These technical things throw their own tantrum.’

A young man arrived with the news: ‘People from the Rajput Toli are going crazy laughing at us. They are saying, “Hold your years and do five sit-ups in front of the Panchlight and it will light up on its own,”

The panchyat members heared this report and said themselves: ‘God has given them the opportunity to laugh at us, so why wouldn’t they laugh? An old man came with the grim news that Rudal Shah, the grocer, was saying that pumping the Panchlight can be a tricky business and needs the utmost care.

Gulri Kaki’s daughter, Munri, wanted to say something. The words were itching to off her tongue. But how could she? She knew that Godhan knew how to light a Panchlight, but she also knew that he had been ostracised by

DDE, GJUS&T, Hisar 260 | English (Compulsory) BA-101

the panchayat. Munri’s mother had complained to the panchyat that Godhan would look at her daughter and sing romantic songs from the movies. The panchayat members had been itching to teach Godhan a lesson: he had come from another village and started living here without ever having given so much as a paisa to the panchayat. He couldn’t care less about the panchayat members. So now when the opportunity presented itself, they fined him ten rupees, and as he did not pay up, forced the villagers to ostracise him. The ban was still in place—Godhan was not allowed to talk to anyone from the toil or mingle socially with them. He could not even smoke from their hukkahs, so how could he be called now? How could Munri take his name? On the other hand, there was the question of the entire community losing face.

Munri dropped the suggestion cleverly in her friend Kaneli’s ear. Kaneli smiled, ‘but Godhan is debarred!’ Munri said, ‘Why don’t you tell the chief to send for him.’

‘Godhan knows how to light a Panchlight!’ Kaneli announced.

‘Who? Godhan? Does he? But...’

The chief looked at the secretary and the secretary looked at the other panchayat members. They had all taken a anonymous decision to ostracise Godhan. The entire village was fed up with Godhan singing lewd songs and making sheep’s eyes at the girls. But the chief said, ‘Why quibble over banishment from the community when the community’s honour is at stake?’

The secretary said, ‘True, True.’

The other members of the panchayat spoke in one voice, ‘All right. Let the ban be lifted from Godhan.’

The chief dispatched the orderly. The orderly returned and said, ‘Godhan refuses to come. He says there is no telling with the panchayat members;

DDE, GJUS&T, Hisar 261 | English (Compulsory) BA-101

tomorrow if something goes wrong with the nuts and bolts of the contraption, they will make me pay a fine.’

The orderly pleaded. ‘Please get him to agree somehow, or else it would be impossible show our face in the village.’

Gulari kaki spoke up, ‘Shall I go and try?’

She went towards Godhan’s hut and managed to mollify him. A new li ght of hope lit up everyone’s faces. Godhan began to quietly fill up the Panchlight with oil. The chief’s wife shooed off the cat circling around the offerings for the puja. The lead singer of the troupe began to smoothen the hairs of his flick in anticipation of whisking it vigorously. Godhan asked, ‘Where is the spirit? How can I light it without the spirit?’

Another spanner in the works! Everyone began to harbour serious doubts about the intelligence of the chief, the secretary and the orderly—they had set out to do things without any idea of how they were to done! Despair gripped the crowd once again. But Godhan was a clever young man. He would light the Panchlight without spirit... ‘Will someone please get a little coconut oil?’ Munri ran to do his bidding. Godhan began to pump the Panchlight.

The silken mesh of the Panchlight’s torch slowly began to glow with light. Godhan began to alternately blow at the torch and turn its key. A sibilant hiss emerged from the Panchlight and its light began to brighten and flare. All traces of ill will seeped out from everyone’s hearts. Godhan was a very bright young man.

Soon enough, entire toil was lit up in the Panchlight’s luminous glow. The singers sang in high, clear voice. The bright flare of the Panchlight shone on the smiling faces all around. Godhan had won everyone’s hearts. Munri looked longingly at him. Their eyes met and a message flashed between them: ‘Let bygone be bygones. Forgive me!’

DDE, GJUS&T, Hisar 262 | English (Compulsory) BA-101

The chief fondly called out to Godhan. Making him sit closely by his side, he said, ‘You have saved our community’s honour. You will be forgiven all your trespasses. You may sing as many songs from the movies as you wish!’

Gulri Kaki said, ‘You must eat dinner at my house tonight.’

Godhan looked at Munri once again. Munri coyly lowered her eyelashes.

The troupe of singers finished one song and launched into loud cries of jubilation. ‘Victory to... Long Live...’ Every leaf and twig on every plant and bush was trembling with happiness.

(translated from Hindi by Rakhshanda Jalil)

5.2.2 Vocabulary of the text

 Toli: An Indian version of a group of people  Petromax: Brand name for a type of pressurised paraffin/kerosene lamp that uses a mantle  Penalty: A punishment for violating rule s.  Sacrifice: The act of killing an animal in order to make an offering to a deity  Appease: To make somebody calmer or less angry by giving them what they want  Orderly: A person who works in an office, usually doing jobs that do not need any special training  Outskirts: The parts of a town or city that are furthest from the centre  Flicker: To burn or shine unsteadily, or with a wavering light.  Stern: Strict  Troupe: A group of actors, singers, etc., who work together  Note: A musical sound  Choir: A group of people who sing together

DDE, GJUS&T, Hisar 263 | English (Compulsory) BA-101

 Ruckus: A situation in which there is a lot of noisy activity, confusion or argument  Chant: Words or phrases that a group of people shout or sing again and again  Gossip: An idle talk in general  Gurgle: To make a sound like water flowing quickly through a narrow space  Ignorant: Not knowing, unaware  Rustic: A Person from a rural area.  Cowrie: An old Indian currency made of shell  Gape: To stare at somebody or something with your mouth open because you are shocked or surprised  Scold: To speak angrily to somebody because he/she has done something wrong; rebuke  Reared its ugly head: If something unpleasant rears its head or rears its ugly head, it appears or happens  Contraption: A machine or piece of equipment that looks strange  Mockery: Comments or actions that are intended to make somebody/something seem ridiculous  Taunt: To try to make somebody angry or upset by saying unkind things about them, laughing at their failures, etc.  Pretext: A false reason that you give for doing something, usually something bad, in order to hide the real reason; an excuse which has not been successful  Come to nought: Come to nothing, not bringing any fruit  lose face: Look stupid because of what you have done; be humiliated  Feeble: Very weak

DDE, GJUS&T, Hisar 264 | English (Compulsory) BA-101

 Throw tantrum: To display a sudden short period of angry, unreasonable behaviour  Grim: looking or sounding very serious  Utmost: Greatest; most extreme  Itching: to want to do something badly  To roll off tongue: Wanting to speak very eagerly  Fed up with: Bored or unhappy especially with the situation that has continued for too long  Lewd: Suggesting moral looseness; obscene  To make sheep’s eyes: To look at someone in a way that shows that you love him/her or are attracted to him/her  Banishment: The punishment of being sent away from a place, especially from a country  No telling: This expression is used to say that it is impossible to know what has happened or what will happen  Mollify: To make somebody feel less angry or upset; placate  Shoo off: To make somebody/something go away, especially by saying ‘shoo’ and waving your arms and hands  Flick: Hit something with a sudden quick movement, especially using your finger and thumb together, or your hand  Anticipation: A feeling about something (usually something good) that is going to happen  Whisk: Brush or wipe off lightly  Vigorously: Very actively; with determination; energetically  Spanner in the works: To cause a delay or problem with something that somebody is planning or doing  Set out: To begin a job  Sibilant : A long hissing sound such as ‘s’ or ‘sh’

DDE, GJUS&T, Hisar 265 | English (Compulsory) BA-101

 Emerge: To come out  Flare: A bright but unsteady light or flame that does not last long  Seep out: To flow slowly and in small quantities; trickle  Luminous: Very bright  Glow: A dull steady light  Longingly: A feeling or showing that you want something badly  Trespass: (here) Wrongdoings  Coyly: In a manner that is shy or pretending to be shy and innocent (especially in a playful or provocative way)  Twig: A small thin branch that grows out of a larger branch on a bush or tree

5.4 MAIN BODY OF THE TEXT II

5.3.1 Introduction to the author

Phanishwar Nath ‘Renu’ (1921—77) was one of the most significant writers of Hindi literature in its post-Premchand era. His novel Maila Anchal is considered to be one of the most significant Hindi novels of all times. Born in Araria district (then Purnea district), Bihar, in 1921, Renu spent a part of his growing up years in Nepal. He graduated from Kashi Hindu Vishvavidyalay (University) in 1942 and took part in the Indian freedom struggle. Renu introduced the ‘Anchalik. Katha’ (regional story) into Hindi writing, and brought local flavours into Hindi literature. He died in 1977.

5.3.2 SUMMARY

The story revolves around the arrival of Panchlight (Petromax) in Mahto Toli which is a caste-based group in a village in Bihar. The Panchayat of Mahto Toli decides to purchase a Panchlight from the fines collected over a period of seven months on the occasion of Ramnavami. Panchlight has

DDE, GJUS&T, Hisar 266 | English (Compulsory) BA-101

become some sort of prestige issue for Mahto Toli since it has already been purchased by the rival Tolis. So, there is a mood of celebration in Mahto Toli. They wanted to light the panchlight only after performing Puja. Since no one in the Mahto Toli knows how to light it, a very embarrassing situation arises. It would be humiliating if someone from a rival Toli is called to light the lamp. The news has spread to other Tolis about it. The Rajput Toli is already mocking at Mahto Toli. The Toli would then lose all honour and become a butt of ridicule among the other Tolis of the village. A young man named Godhan is the only one in the Mahto Toli who knows how to light a panchlight, but he has been expelled from the Toli for singing lewd film songs and teasing the girls of the village. Gulri Kaki, the mother of a girl named Munari, had complained to the Panchayat that Godhan sang lewd songs to her daughter. Munri drops the information in her friend Kaneli's ear that Godhan knows how to light a Panchlight. Kaneli announces it in the Panchayat. The chief sends his orderly to Godhan with the request to come and light the panchlight. But Godhan refuses to come. The chief does not know what to do. Seeing the honour of the Toli at stake, Gulri Kaki herself comes forward and suggests that Godhan should be called to light the Panchlight. At once, Godhan is invited to light the lamp and in a few minutes the Panchlight starts glowing. The entire Toli is lit up in the panchlight's luminous glow. The singers start singing. Godhan had won everyone's heart. Munri looks at him with loving eyes. The chief calls Godhan and asks him to sit by his side. He thanks him for saving the honour of the community. He declares that all his faults has been forgiven. Gulri Kaki invites him to take dinner at her house that night.

5.5 CHECK YOUR PROGRESS

1). Answer the following questions using a word, a phrase or one or two sentences each.

DDE, GJUS&T, Hisar 267 | English (Compulsory) BA-101

a)What is a Panchlight?

b)How much did the Panchlight cost?

c)Who bought the Panchlight?

d)What is the technical name of Panchlight?

e)How was the money arranged to buy a Panchlight?

f)On which occasion was the Panchlight bought?

g)Who gave the stern warnings not to touch the Panchlight?

h)How many bottles of kerosene had been fetched from Rudal Shah’s shop to light the Panchlight?

i)Which toil said, “Hold your ears and do five sit-ups in front of the Panchlight and it will light up on its own?”

j)Who had been ostracised by the panchayat?

k)How much fine was imposed upon Godhan?

l)Who announced that Godhan could light a Panchlight?

m)Who is Munri?

n)Who was Gulri Kaki?

o)Who was Kaneli?

p)Who succeded in mollifying Godhan?

q)What was used in place of spirit to light the Panchlight?

r)Who lighted the Panchlight?

s)Who invited Godhan for dinner in the end?

2). Give words that are similar in meaning to the following words:

a) Appease

DDE, GJUS&T, Hisar 268 | English (Compulsory) BA-101

b) Incorrect

c) Hurry

d) Chorus

e) Strict

3) Give words that are opposite in meaning to the following words:

a) Beautiful

b) Dishonour

c) Allow

d) Praise

e) Weak

5.6 Transcription

Transcription of important words taken from the story.

a. Penalty ˈpɛnlti

kəˈlɛkt b. Collect ɔːl əʊ c. All n ˈəʊ ə d. Own v məʊst e. Over juːz f. Most ˈɒbʤɪkt g. Use ˈsjuːtəbl

h. Object bɪˈfɔː ˈbɪldɪŋ i. Suitable

DDE, GJUS&T, Hisar 269 | English (Compulsory) BA-101

ɪʤ j. Before br feə k. Building wɛl l. Bridge ɪˈlɛktrɪk

m. Fair ˈwɔːnɪŋ wɒʧ n. Well kləʊs

o. Electric tʌʧ p. Warning ˈdæmɪʤ

q. Watch

r. Close

s. Touch

t. Damage

5.7 Comprehension

Read the following passage and answer the questions that follow.

a) The elders of Mehto Toli bought a petromax at this year’s Ramnavami fair from the fines and penalties collected over the past fifteen months. There are eight Panchayats in the village, one for each community. All the panchayats have their own durries, jajim, mats and a petromax lamp, which the villagers call the panchlight.

After buying the Panchlight, the panchayat members decided to buy offerings for puja with the ten rupees that were left over, for it would be most inappropriate a technical object with nuts and bolts without first having it suitably blessed. After all, even in the time of the British Bahadur, a sacrifice would be offered to appease the gods before work could begin on building a bridge.

DDE, GJUS&T, Hisar 270 | English (Compulsory) BA-101

All the panchayat members returned home from the fair well before sunset—the orderly in front carrying the box with the Panchlight on his head, followed by the chief, the secretary and the other members. Phutangi Jha of the Brahmin Toli Stopped them at the outskirts of the village and asked, ‘How much did you buy this lantern for?’

1. Who bought a Petromax?

Ans. The elders of Mahto Toli bought a petromax.

2. At which fair the Petromax was bought?

Ans. The Petromax was bought at Ramnavami fair.

3. How many Panchayats were there in the village?

Ans. There were eight Panchayats in the village.

4. What did the members of the Panchayat decide?

Ans. The members of the Panchayat decided to buy offerings for puja with the Ten rupees that were left over.

5. Who was carrying the Panchlight.

Ans. The orderly in the front was carrying the box with the panchlight on his head.

b) Gulri Kaki's daughter, Munri, wanted to say something. The words were itching to roll off her tongue. But, how could she? She knew that Godhan knew how to light a Panchlight, but she also knew that he had been ostracised by the panchayat. Munri's mother had complained to the panchayat that Godhan would look at her daughter and sing romantic songs from the movies. The panchayat members had been itching to teach Godhan a lesson: he had come from another village and started living here without ever having given so much as a paisa to the panchayat. He couldn't care less about the panchayat members. So now when the opportunity presented itself, they fined him ten

DDE, GJUS&T, Hisar 271 | English (Compulsory) BA-101

rupees, and as he did not pay up, forced the villagers to ostracise him. The ban was still in place—Godhan was not allowed to talk to anyone from the toli or mingle socially with them.

He could not even smoke from their hukkahs, so how could he be called now? How could Munri take his name? On the other hand, there was the question of the entire community losing face.

Munri dropped the suggestion cleverly in her friend Kaneli's ear. Kaneli smiled, 'But Godhan is debarred!'

Munri said, 'Why don't you tell the chief to send for him.'

'Godhan knows how to light a Panchlight!' Kaneli announced.

'Who? Godhan? Does he? But. . . '

The chief looked at the secretary and the secretary looked at the other panchayat members. They had all taken a unanimous decision to ostracise Godhan. The entire village was fed up with Godhan singing lewd songs and making sheep's eyes at the girls. But the chief said, 'Why quibble over banishment from the community when the community's honour is at stake?'

1. Who is Munri?

Ans. Munri is the daughter of Gulri Kaki.

2. Why was Godhan ostracised?

Ans. He was ostracised for singing lewd songs.

3. How did Kaneli ‘help Munri?

Ans. Kaneli announced that Godhan knew how to light a panchlight.

4. What was the Panchayat chief's reaction to Kaneli's announcement?

Ans. He was relieved to hear this news.

DDE, GJUS&T, Hisar 272 | English (Compulsory) BA-101

5. From the passage, what do you gather about Munri's attitude towards Godhan?

Ans. It appears that Munri loves Godhan.

5.8 Examination style questions

5.8.1 Short questions

Q1.What was the mood in the Mahto Toli?

Ans. Elders of the Mahto Toli had bought a Panchlight at that year's Ramnavmi fair. When the members of the Toli came to know of it, they came running to see it and it was decided to have a puja in the evening. Everyone was full of excitement and there was a mood of celebration in the toli.

Q2. How did the people react when the Panchlight could not be lighted?

Ans. Every memeber of the Mahto Toli was sad when the Panchlight could not be ilighted. Some commented that in the anticipati on of having the Panchlight's luminous glow, they had not lit even an oil lamp in their home. It was evening and there was darkness everywhere in the toli. The Panchayat was in the serious danger of losing face. People from the other tolis were full of taunts. Those from the Rajput Toli were saying: "Hold your ears and do five sit- ups in front of the Panchlight and it will light up from its own.”

Q3. Why was Godhan ostracised?

Ans. Godhan had come from another village and started living in the village where Munri lived with her mother Gulri Kaki. Godhan never paid even a paisa to the panchayat. He had no respect for the elders of the panchayat. Gulri Kaki complained to the Panchayat that Godhan often looked at her daughter and sang romantic songs from the movies. Godhan was fined ten rupees for this. He did not pay up the fine, so he was ostracised from the community.

DDE, GJUS&T, Hisar 273 | English (Compulsory) BA-101

Q4. How did the panchayat solve the issue of Godhan, who had been ostracised from the community, lighting the panchlight?

Ans. The panchayat was in a dilemma. None in the Mahto Toli knew how to light the Panchlight. Only Godhan knew it, and he had been ostracised. Calling someone from a rival toli would be humiliating. After much talks, the chief decided to call Godhan to save the honour of the community. Gulri Kaki, at whose complaint Godhan had been ostracised, herself offered to go and get Godhan to light the Panchlight. Thus, the issue was solved.

Q5. How did the musicians behave that evening when the Panchlight was to be lighted?

Ans. That evening, the musicians were very excited. The head of the troupe of singers was Very happy. He said that they would perform in the light of the panchlight. He asked his members to give their best. Gulri Kaki was one of the lead female singers. She started humming with joy. Then all the musicians sat before Panchlight with their musical instruments.

Q6. How did Godhan light the Panchlight?

Ans. Godhan first filled up the Panchlight with oil. He then asked for the spirit, but none had thought of it. However, Godhan was a clever young man. He asked for a little coconut oil. Munri at once ran to get it. Godhan began to pump the Panchlight. The silken mesh slowly began to glow with light. Soon the entire toli was lit up in the Panchlight's luminous glow.

Q7. How did Godhan win back his place in the community?

Ans. By lighting the Panchlight, Godhan had saved the community from humiliation. He was now the favourite of the community. He had won everyone's heart. The Chief forgave all his wrongdoings and made him sit closely by his side. He was also allowed to sing as many songs from the movies as he likes.

DDE, GJUS&T, Hisar 274 | English (Compulsory) BA-101

Q8. Comment on the ending of the story, 'Panchlight'.

Ans. The story ends on a happy note. Godhan, who was ostracised for 'teasing' Munri was forgiven. The Chief and other members of the Panchayat are happy with him. Munri looks lovingly at him. Gulri Kaki too realizes that there was nothing wrong in adolescents of opposite sexes feeling attracted towards each other. She invites Godhan to eat dinner at her house at ni ght.

Q9. Who was Kaneli? How did she help Munri?

Ans. Kaneli was Munri's friend. An embarrassing situation had arisen for the toli when there was none to light the Panchlight. Munri knew that Godhan could do it but she couldn't take his name because it was at the complaint of her mother that Godhan had been ostracised. Munri cleverly dropped the suggestion in Kaneli's ear. And Kaneli announced at once that Godhan knew how to light a Panchlight. It was oly then that the Panchlight could be lighted.

5.8.2 Long questions

Q. 1 What image of the rural society do we get from the story, 'Panchlight’?

Ans. In this story, the writer presents a realistic picture of a village of Bihar. We see here a society that is simple -hearted, poverty-stricken, superstitious, illiterate, narrow-minded but forgiving and God-fearing. The society in the village is sharply divided on the basis of caste. There are eight Panchayats, one for each of the eight castes, called the Toli. Every caste has its own congregation, sheets, carpets, rugs and a Petromax, which is called Panchlight by the villagers. With the caste is associated the question of honour. No one in the Mahto Toli knows how to light the panchlight. But they did not want to invite a member of some other Toli to light it. It was a matter of pride and honour. The members of the Rajput Toli were already taunting them for not being able to light the Panchlight. For the sake the bigger honour, the

DDE, GJUS&T, Hisar 275 | English (Compulsory) BA-101

Panchayat members were ready to forget the smaller honour. They had banished Godhan from their Toli for singing lewd songs. But when they came to knew that he knows how to light the Panchlight, the immediately forgave him and took him back into their Toli. Thus, the writer has presented a true picture of a rural society in India.

Q. 2 How did the Panchlight prove a boon for Godhan?

Ans. The Panchlight plays the central role in the story. It is the Panchlight that unites the two loving hearts in the story. Godhan had been ostracised from the community. It had been alleged by Munri's mother that he would look at her daughter and sing lewd songs from the movies. The panchayat fined him ten rupees for this, and he was ostracised from the community. When he did not pay up the fine, he was not allowed to talk to anyone in the Toli or mingle socially with them.

But then the fate took a turn and the Panchlight entered the scene. There was no one in the community who knew how to light it. Munri knew that Godhan could do it, but he had been ostracised. She couldn't dare to take his name since it was at her own mother's complaint that Godhan had been ostracised. Very cleverly, she whispered the suggestion in the ear of her friend Kaneli. When Kaneli announced this to the elders, Godhan was at once sent for. At first, he refused, but then agreed when Munri's mother Gulri Kaki herself went to him.

In no time the Panchlight was lit up. The bright flare of the Panchlight shone on the smiling faces all around. The chief forgave him and made him sit closely by his side for saving the honour of the community. He was allowed to sing as many songs from the movies as he wishes. Munri looked lovingly at Godhan and their eyes met. Even Gulri Kaki also invited him to eat dinner at her house at night. Thus, the Panchlight has proved a heavenly boon for Godhan.

DDE, GJUS&T, Hisar 276 | English (Compulsory) BA-101

Q. 3 In 'Panchlight’, Renu creates a narrative of rural society that remains valid even today, though the symbols have changed. Discuss.

Ans. 'Panchlight' is a realistic and comprehensive narrative of the Indian rural society. We see here a rural society with all its typical qualities, good as well as bad. We see here a society that is simple-hearted, innocent, illiterate, poverty- stricken, superstitious, caste -ridden, narrow minded, but forgiving and God- fearing. This portrayal of the rural society is as valid today as it was when the story was written about six decades ago.

Caste has always been the basic fibre of the Indian rural society. We are told in the story that there are eight different panchayats in the village. Each community has its own panchayat, and the communities entertain feelings of animosity towards each other. This divide and animosity continue even today, but the symbols of the divide have changed. Now the divide is more economic, social and political than caste -based. But it is to be noted that the communities remain as divided as ever they were.

The rural community is still simple -hearted, innocent, uneducated, economically backward, superstitious and narrow-minded, but not to the same extent as it was sixty years ago. The Panchlight which was then a symbol of change and development, has been replaced by mobiles and LCD's today.

To sum up, we may say that Renu's narrative of rural community as depicted in ‘Panchlight' is valid even today, though the symbols have changed.

Q. 4 Justify the title of the story, 'Panchlight'.

Ans. Panchlight is not just a petromax lamp, it is the central character in the story. The whole story revolves around the Panchlight. It is through the Panchlight that the theme of the story is developed. It is through the Panchlight that the various shades and traits of rural society are highlighted and the story reaches its climax and then finds the resolution also.

DDE, GJUS&T, Hisar 277 | English (Compulsory) BA-101

The story begins with the Mahto Toli of the village buying a petromax while all other Tolis have already their own petromaxes. It is an occasion of celebration for the Mahto Toli, as other Tolis are feeling jealous. They try to make a mockery of them. The Mahto Toli makes all arrangements are to light the Panchlight in the evening and offer puja before the lamp is lit. Three bottles of kerosene are fetched from the village shop, but no one from the Toli knows how to light the Panchlight.

The only one from the toli who knows how to light the Panchlight is Godhan but he is ostracised from the society. He often looked at a girl named Munri and sang songs from the movies. The girl's mother complained to the panchayat and Godhan was ostracised.

Thus, a very embarrassing situation arises for the Toli. Calling someone from a rival toli to light the Panchlight will be humiliating.

After much deliberation, it is decided that Godhan should be called to light the Panchlight. At first, Godhan refuses but agrees when Munri's mother herself goes to call him. The Panchlight is lighted and its bright flare shines on the smiling faces gathered all around. Godhan becomes the star of every eye and Munri’s mother Gulri kaki also invited him to eat dinner at her house at night.

Thus, all problems are solved with the lighting of the Panchlight, and we can conclude that no other title could be more suitable.

Q. 5 Why was Godhan ostracised and how was he taken back into the community?

Ans. Godhan had come from another village and had started living in the village where Munri lived with her mother, Gulri Kaki. He never paid a paisa to the panchayat and showed no regard for the panchayat members. The members were waiting for some opportunity to punish him and this opportunity came when Gulri Kaki complained to the Panchayat that Godhan

DDE, GJUS&T, Hisar 278 | English (Compulsory) BA-101

would look at her daughter and sing romantic songs from the movies. To teach Godhan a lesson, the panchayat fined him ten rupees. But Godhan did not pay up the fine and was thus ostracised from the community.

Godhan was not allowed to talk to anyone from the Toli or mingle socially with them. But then the things took a turn when the Mahto Toli bought a Panchlight for the community. All arrangements for the puja and the inaugural lighting of the lamp had been made but then it was found that there was no one who knew how to light the Panchlight. Munri knew Godhan could do it, but she couldn't dare to take his name. Very cleverly, she dropped the suggestion in the ear of her friend Kaneli. Kaneli announced this to the elders and Godhan's banishment was at once cancelled. He was called and the Panchlight was lighted.

Godhan at once became everyone’s favourite. Munri looked lovingly at him. The chief pardoned all his mistakes and made Godhan sit closely by his side for saving their community's honour. He was allowed to sing as many songs from the movies as he wishes."

Q. 6 Reproduce briefly and in your own words the story of 'Panchlight'.

Ans. The elders of the Mahto Toli in the village buy a Panchlight for their community at Ramnavami fair. Other Tolis of the village already have their own separate Panchlights. While the re is a mood of celebration in Mahto Toli, those from other tolis feel jealous and try to make fun of them. Everyone in the Mahto Toli prepares for the puja and the inaugural lighting of the lamp. Musicians are also ready with their drums, cymbals and bells for celebrations. But then suddenly it is discovered that there is no one in the Toli who knows how to light it. The only one who knows this is Godhan but he has been ostracised from the community on a complaint from Gulri Kaki. She had complained that Godhan would look at his daughter, Munri and sing songs

DDE, GJUS&T, Hisar 279 | English (Compulsory) BA-101

from the movies. To teach Godhan a lesson, he was ostracised from the community.

But now when the community's honour is at stake, Godhan's offence seems to be of no great seriousness. It will be a matter of great humiliation if someone from a rival toli is called to light the Panchlight. Therefore, all offences of Godhan are forgotten. Gulri Kaki herself goes and fetches Godhan to light the Panchlight. The lamp is lighted and there is joy on everyone's face. The singers sing in high and clear voices. Godhan becomes the apple of everyone’s eyes. Munri looks lovingly at him. Their eyes meet in a silent message and they forgive each other. The chief forgives all his wrongdoings and makes Godhan sit closely by his side and says for saving their community's honour. Even Gulri Kaki invites him to eat dinner at her house at night. Godhan looks at Munri once again and she coyly lowers her eyes.

5.9 ANSWERS TO CHECK YOUR PROGRESS

a) A Petromax lamp is called the Panchlight.

b) The Panchlight cost five cowries.

c) The elders of Mahto Toli bought the Panchlight.

d) Petromax lamp

e) The money was arranged from the fines and penalties by the Panchyat of Mehto Toli.

f) The Panchlight was bought on the occasion of Ramnavmi fair.

g) The orderly Angu Mahto.

h) Three bottles.

i) Rajput Toli.

j) Godhan.

DDE, GJUS&T, Hisar 280 | English (Compulsory) BA-101

k) Ten rupees.

l) Kaneli.

m) Daughter of Gulri Kaki.

n) An elderly woman and mother of Munri.

o) Kaneli was a friend of Munri.

p) Gulri Kaki.

q) Coconut oil.

r) Godhan.

s) Gulri Kaki

2). a) Calm

b) Inappropriate

c) Rush

d) Choir

e) Strict

3) a) Ugly

b) Honour

c) Ban

d) Insult

e) Feeble

5.10 REFERENCES/SUGGESTED READINGS

Hooda, Rana and Mohan (Eds). Literature and Language II. Hyderabad: Orient Blackswan, 2018.

DDE, GJUS&T, Hisar 281 | English (Compulsory) BA-101

Subject: English

Course code: BA-101 Author: Dr. Narsingh Jangra

Lesson: 06 Vetter:

THE CHILD

Structure

6.1 Learning Objectives

6.2 Introduction

6.3 Main Body I

1.2.1 Text of the story

1.2.2 Vocabulary of the text

6.4 Main Body II

1.3.1 Introduction to the author

1.3.2 Summary

6.5 Check your progress

6.6 Transcription

6.7 Comprehension

6.8 Examination style questions

1.8.1 Short questions

1.8.2 Long questions

6.9 Answers to check your progress

6.10 References/Suggested Reading

DDE, GJUS&T, Hisar 282 | English (Compulsory) BA-101

6.1 LEARNING OBJECTIVES

After going through this lesson you will be able to-

 Read, understand and enjoy the story.  Enrich your taste for literature.  Learn and understand the fundamentals of prose.  Learn about the biography of the writer.  Develop reading, writing, and speaking skills of the English language.  Enrich your vocabulary.  Learn the phonetic transcription of some English words.  Learn to answer the examination style questions.

6.2 INTRODUCTION

Premchand's story 'The Child' shows his unorthodox and progressive state of mind. It reveals how Premchand's thought was much ahead of his time. ‘The Child’ is a story narrated by the employer of a Brahmin servant named Gangu, who wants to marry Gomti. The author does not approve of this match because Gomti is a woman of loose morals. She has already run away from three of her husbands. Despite being warned by his employer and others, he marries a widow, Gomti and even adopts her child who is born of her previous husband. The narrator has a great inte rest in Gangu’s life and the story is told from his point of view. The story places Gangu’s compassion, simple heartedness and acceptance of human nature as the embodiment of goodness and shows the reader a way of putting aside all kinds of pressures that society exerts on us.

6.3 MAIN BODY THE TEXT I

6.2.1 Text of the story

People call Gangu a Brahman and he considers himself a Brahman. My groom and other servants salute me, even from a distance, Gangu never does.

DDE, GJUS&T, Hisar 283 | English (Compulsory) BA-101

Perhaps he expects me to greet him by saying, ‘I touch your feet.’ He never touches a glass front which I have drunk, I have never dared ask him to fan me. When I am drenched in sweat and none of the other servants is around. Gangu does pick up the fan, but the look on his face indicates that he i s doing me a favour. For some reason, I always take the fan from his hand immediately. He’s a man of extremes. He can’t tolerate people’s chatter. He must have very few friends. Perhaps he thinks it beneath his dignity to sit with the groom and the servants. I’ve never seen him socialize with anyone. Surprisingly he is not given to opium or hemp; this is an extraordinary virtue in men of his class. I have never seen him perform religious rituals or go for a sacred bath in the river. Despite being absolutely illiterate he is still a Brahman, and wants the world to respect and serve him in recognition of this. And why shouldn’t he? If people can not only retain control of property accumulated by their forefathers but also derive status from it as if they had generated it themselves, why should he give up the respect and honour gathered by his forefathers? After all, this is his only inheritance.

I am not one to talk too much with servants. I don’t want any of them to come and talk to me unless I call them. I don’t approve of having my servants at my beck and call to perform small chores. I find it much easier to pour my own drinking water, light my own lamp, wear my own shoes and take out books from the cupboard myself than to call for Ilingan and Maiku. Thus helps me feel autonomous and self- reliant. The servants are familiar with my temperament, rarely approach me unnecessarily. So one day, when Gangu appeared before me early in the morning, I was not at all pleased. Whenever these people come to me, they either want an advance on their wages or they want to complain about another servant. I dislike both these things. I pay everyone on the first of the month, and i get angry when anyone asks for an advance. Who has the time to keep detailed accounts of the advances given? When someone is paid for the whole month, what right does he or she have to

DDE, GJUS&T, Hisar 284 | English (Compulsory) BA-101

spend all the money in fifteen days, and then seek an advance or a loan? And I hate complaints. I consider complaints a sign of weakness or abase attempt at flattering the employer.

I said with a frown, ‘What is it? I didn’t call you.’

I was very surprised to see Gangu’s sharp, proud face looking mild, pleading and bashful. I reckoned he wanted to say something, but was unable to find the right words.

Somewhat more mildly, I asked, ‘What’s the matter? Why don’t you speak? You know that this is my time to go for a walk. I’m getting late.’

Gangu said, in a dejected tone, ‘Well, then you go...I’ll come later.’

This was more worrisome. Right now, since I was in hurry, he would have to rush through his story because he knew I didn’t have much time. If he came later, the wretch would spend hours complaining. Perhaps he realizes that I’m working when he sees me reading or writing; but he thinks I’m resting when I’m thinking, which is actually the hardest of all my tasks. That’s he’ll come and bother me.

I said, unkindly, ‘Have you come to ask for an advance? I don’t give advances.’

‘No indeed, sir, i have never asked for an advanced.’

‘Then do you want to complain about anyone? I hate complains.’

‘No, sir, I’ve never complained about anyone!’

Gangu pulled himself together. It was from his expression that he was summoning up his courage, when he said hesitantly, ‘Please give me permission to leave. I won’t be able to work for you anymore.’

This was the first proposal I had ever received. My- self respect was wounded. I consider myself an embodiment of humaneness; I never speak sharply to the

DDE, GJUS&T, Hisar 285 | English (Compulsory) BA-101

servants; I try my best to keep my dominance sheathed. Why wouldn’t I be surprised at this announcement!

I said silently, ‘Why, what do you have to complain of?

‘Sir, no one could be as good natured as you, but things have so developed that I cannot remain here any longer. I don’t want anything to happen to give you a bad name. I don’t want your reputation to be sullied because of me.’

This created a dilemma for me. The fire of curiosity grew fierce. Sitting down on a chair in the veranda, as if making me concession, I said, ‘You are talking in riddles. Why don’t you tell me clearly what the matter is.’

Gangu said very humbly, ‘That woman Gomti Devi, who has just been thrown out of the Widows home...’

He fell silent. I said impatiently, ‘yes, she’s been thrown out; so what? What has she to do with your job?’

Gangu seemed to throw a heavy burden off his head onto the ground: ‘I want to marry her, sir.’

I gaped at him in amazement. This stupid Brahman with his old-fashioned ideas, who has remained entirely untouched by the breezes of modernity, wants to marry that loose woman, whom no decent man would allow inside his house. Gomti had created much turbulence in our peaceful neighbourhood. She had come to the Widows’ Home many years ago. The Home’s administrators had gotten her married three times, but each time she ran away in a fortnight or a month’s time, and returned to the home. This time the head of the Home had thrown her out. She had rented a room in the neighbourhood, and had become a source of entertainment for the dissolute men of the locality.

I was agreed by Gangu’s simplicity but also pitied him. Is this the only woman in the world this donkey can find to marry? If he were a very rich man it might

DDE, GJUS&T, Hisar 286 | English (Compulsory) BA-101

be different. Perhaps she would have stayed for six months or a year. This fellow is as good as a blind man. They won’t get along for even a week.

I said in a warning tone, ‘Do you know this woman’s life history?’

Gangu said, as if speaking of events he had witnessed with his own eyes, ‘It’s all lies, sir; people have defamed her for nothing.’

‘What do you mean? Didn’t she run away from three husbands?’

‘They threw her out so what could she do?’

‘What a fool you are! Would anyone travel such long distances and spend thousands of rupees to marry a woman, just in order to throw her out?’

Gangu said with deep emotion, sir, a woman cannot live whe re there is no love. A woman wants more than food and clothes, she wants some love too. Those people think they have done a widow a great favour by marrying her. They want her to become theirs, body and soul; but to make another person one’s own, one has to first become the other person’s, sir. That’s the thing .And then she also suffers from an ailment. She is possessed by a spirit. Sometimes, she gets hysterical and falls down in a faint.’

‘And you will marry such a woman?’ I said, shaking my head in dismay. ‘Your life will be ruined.’

Gangu said, sounding like a martyr, ‘I think my life will be fulfilled, sir. The rest is God’s will!’

I said firmly, ‘Have you made up your mind?’

‘Yes sir.’

‘Then I accept your resignation.’

I am not a slave to meaningless conventions and useless principles, but it would certainly be problematic to employ a man who marries a wicked woman. Fresh complications arise every day, new dilemmas will crop up, the police

DDE, GJUS&T, Hisar 287 | English (Compulsory) BA-101

might come calling, lawsuits might ensue. Quite possibly, thefts might occur. It’s best to avoid this swamp. Liking a starving creature, Gangu had seen a piece of bread and was rushing towards it. He didn’t care that the bread was half- eaten, dry and unfit to eat. He was unable to use his mental faculties. I thought it in my best interest to serve connections with him.

2

Five months passed. Gangu had married Gomti and was living in a thatched house in the same neighbourhood. He eked out a living by selling snacks from a cart. Whenever I met him in the market, I would stop to enquire how he was doing. I had become very interested in his life. It was like an experiment on not just a social but also a psychological question. I wanted to see how things would turn out. I always saw Gangu looking cheerful. I could clearly see in him the energy and self-respect born of prosperity and freedom from worry. He sold goods worth a rupee or twenty annas. After deducting the investment, he got a profit of eight or ten annas. This was his income, but he was blessed by some God, because he showed no sign of the shamelessness and deprivation found in others of his class. The dignity and joy on his face could arise only from inner peace.

One day heard that Gomti had run away from Gangu’s house. I don’t know why I experienced a strange pleasure at the news. I had felt a certain envy of Gangu’s contented and happy life. I was waiting for something unexpected, some disaster, some shameful incident to occur. This news doused my envy. Finally, what I had believed would happen had happened. Finally, the idiot had suffered the punishment for his short- sightedness. Let’s see what face he puts on it! Now his eyes will be opened and he’ll realize that those who advised him against this marriage were his well-wishers. At that time, he behaved as if he had come across some rare treasure or as if the gates of liberation had opened to him. So many people told him that this woman was not worthy of trust, had

DDE, GJUS&T, Hisar 288 | English (Compulsory) BA-101

betrayed several others and would betray him too, but that advice was like water off a duck’s back. Now I’ll see how he’s doing. I’ll say, ‘well sir, are you happy with this boon from your goddess or not? You never tired of praising her, and said that people spoke against her merely from ill will. Now tell me, who ws mistaken?’

The same day, I ran into Gangu in the market. He was upset, agitated, completely lost. As soon as he saw me, his eyes were filled with tears. He come to me and said, not with embarrassment but with pain, ‘Sir, Gomti has betrayed me too.’

I said with cruel pleasure but pretended sympathy, ‘I told you so but you wouldn’t listen. Now you must endure the consequences. What else can you do? Did she take all your money or did she leave any for you?’

Gangu put his hand on his breast, as if his heart was wounded by my words, and said, ‘Oh no, sir, don’t say so. She hasn’t touch a single thing. She’s even left behind her own things. I don’t know what defect she saw in me. I was not worthy for her, what else can I say? She was educated, and I am completely illiterate. It was surprising that she stayed with me so long. If I had stayed with her a while longer, I would have become a man. What can I tell you about her, sir? Whatever she may have been to others, to me she was a gift sent by some God. I don’t know what mistake I made. But I swear she never so much as looked angry. What am I, sir? I am a labourer, worth ten or twelve annas; but she was so blessed that she made this little go a long way and we never felt that we lacked anything.’

I was deeply disappointed. I had expected him to recount a tale of her infidelity, and I would have sympathized with his blind devotion. But this fool’s eyes were not yet opened. He was still singing her praises. He definitely seemed mentally disturbed.

I started a cruel joke, ‘So she didn’t take anything from your house?’

DDE, GJUS&T, Hisar 289 | English (Compulsory) BA-101

‘No sir, not a single thing.’

‘And she loved you a lot too?’

‘What can I say, sir? I will remember that love till my dying day.’

‘And yet she left you and went away?’

‘That is what’s surprising, sir!’

‘Have you ever heard of a woman’s frailty?’

Oh, sir, don’t say so. Even if someone puts a knife to my throat, I won’t stop singing her praises.’

‘Then go find her.’

‘Yes, sir: I won’t rest until I find her. If only I can find out where she is, I’ll go immediately and fetch her. And, sir, my heart says that she’ll definitely return with me. You’ll see. She didn’t leave on account of being angry with me. My heart cannot accept this. I’ll go and roam around jungles and mountains for a couple of months. If I survive, I’ll see you again.’

So saying, he went off, looking like a madman.

3

After this, I had to go to Nainital for work, not pleasure. I returned after a month and had not even changed my clothes when I saw Gangu standing before me, carrying a newborn baby. Perhaps not even Nand was so enraptured when he adopted the baby . Gangu appeared to me bursting at the seams with joy. A hymn of gratitude and devotion seemed to rise from his face and eyes. His expression was like that of a starving beggar who has just had a hearty meal.

I asked, ‘Well, sir, have you found out anything about Madam Gomti? You had gone in search of her?’

DDE, GJUS&T, Hisar 290 | English (Compulsory) BA-101

Gangu, barely able to contain his joy, ‘Yes, sir, thanks to your blessings, I did manage to find her. She was in the women’s hospital in Lucknow, She had told a friend here to inform me if I became too upset. I immediately rushed to Lucknow and brought her back. I got this child too in the bargain.’

He held the child out towards me, like a sportsman showing off the trophy he has won.

I asked, jokingly, “Oh, so you got this now too. Perhaps that is why she ran away. You’re sure he’s your son?’

‘Why my son, sir—he’s yours, he’s God’s.’

‘So he was born in Lucknow?’

‘Yes, sir, he’s just one month old.’

‘How long ago did you get married?’

‘This is the seventh month.’

‘So he was born in the sixth month of your marriage?’

‘Yes sir.’

‘Yet he’s your son.’

‘Yes sir.’

‘What nonsense you are talking!’

I wasn’t sure if he understood me, or was pretending not to. He said, as guilelessly as before, ‘She narrowly escaped death, sir. It’s as if she was reborn. She suffered for three days and three nights.’

I said, somewhat sarcastically, ‘This is the first time I’ve heard of a child being born in six months.’

This barb struck home.

DDE, GJUS&T, Hisar 291 | English (Compulsory) BA-101

He smiled and said, ‘Oh, that’s what you mean! I didn’t realize you were talking of that. Gomti ran away because of this fear. I said, “Gomti, if you are not happy with me, leave me. I’ll go away and never come to you again. When you need me to do anything for you, write, and I’ll help as much as I can. I have no complaint against you. In my eyes, you are just as good as before. I still love you as much. No, I actually love you more. But if your heart has not turned away from me, then come with me. Gangu will not be unfaithful to you as long as he lives. I didn’t marry you because you are a goddess but because i loved you and thought you loved me too. This child is my child, my own child. I bought a field and will i refuse the crop because someone else sowed it?”

So saying, he burst out laughing.

I forgot that I had to go and change my clothes. I don’t know why my eyes filled with tears. Some unknown power subdued my mental disgust and made me extend my hands. I took that unblemished child in my lap and kissed it, perhaps more lovingly than I have ever kissed my own children.

Gangu said, ‘Sir, you are a very good person. I always praise you to Gomati, and tell her to come and meet you, but she is too shy.’

I, a good person! The veil of my goodness had been drawn away from my eyes. I said, in a voice steeped in devotion, ‘No, why should she come to see an impure man like me? You think I’m a good person? I appear to be good, but my heart is mean. True goodness is in you, and this child is a flower perfumed with your goddness.’

Holding the child to my breast, I went with Gangu.

(Translated by Ruth Vanita)

6.2.2 Vocabulary of the text

 Groom: Someone employed in a stable to take care of the horses  Greet: To welcome in a friendly manner

DDE, GJUS&T, Hisar 292 | English (Compulsory) BA-101

 Drenched: Completely wet  Indicate: To show  Favour: A kind or helpful deed  Extremes: In the greatest or highest degree, excessive  Dignity: The state of being worthy of praise  Socialise: to take part in social activities; interact with others  Opium: A yellow brown addictive drug  Hemp: a plant of the cannabis family which yields a narcotic drug  Virtue: An admirable quality  Ritual: A repeated set of actions  Sacred: Religious, spiritual, divine  Accumulate: Gather, collect  Inheritance: a title, property or estate that passes by law to the heir on the death of the owner  At someone’s back and call: Always ready to obey someone’s orders immediately  Chore: a specific piece of work required to be done as a duty or for a specific fee free  Autonomous: Free from external control and constraint  Temperament: A person’s usual manner of thinking, behaving, or reacting  Frown: a facial expression of dislike or displeasure  Bashful: shy  Reckon: Calculate, conclude, judge  Mildly: Slightly  Dejected: Sad, disheartened  Wretch: Someone you feel sorry for  Summon: (here) To rouse oneself to exert a skill  Sheathed: Hidden

DDE, GJUS&T, Hisar 293 | English (Compulsory) BA-101

 Sully: Attack the good name and reputation of someone  Gape: To look with amazement  Dissolute: Immoral  Swamp: A situation full of difficulties  eke out: Live from day to day, as with some hardship here, moral weakness  Frailty: (here) moral weakness  Dilemma: A confusing situation  Fierce: Violent  Concession: A privilege, grant  Turbulance: Disturbance  Defamed: Disgraced, accused  Ailment: Disease  Hysterical: (here) Uncontrollable  Faint: Weak  Martyr: One who sacrifices his or her life for the sake of a principle or cause  Thatched: Covered with straw or leaves  Deprivation: Dispossession  Envy: Jealousy  Contented: Satisfied  Douse: To put out  Betray: Deceive  Boon: Blessing  Embarrassment: Humiliation, discomfort  Consequences: Results  Infidelity: Unfaithfulness in a relationship  Enraptured: Filled with delight  Seams: Joints  Hymn: A song of worship

DDE, GJUS&T, Hisar 294 | English (Compulsory) BA-101

 Beggar: Pauper, someone who begs  Bargain: A gainful purchase  Barb: The point of an arrow  Subdued: Quieter  Disgust: A strong feeling of dislike  Unblemished: Not spoiled or damaged

6.4 MAIN BODY OF THE TEXT II

6.3.1 Introduction to the author

Premchand was one of the most distinguished writers of the Indian subcontinent and is considered to be one of the prominent Hindi-Urdu writers of the early twentieth century.

Born Dhanpat Rai Srivastav in 1880, Premchand began writing under the pen name ‘Nawab Rai’, but subsequently switched to ‘Premchand’ after his short story collection Soz-e-Watan was banned by the British administration. He was popularly known as ‘Munshi Premchand’. He wrote over a dozen novels, around 250 short stories and numerous essays, and translated a number of literary works into Hindi. Due to his versatility, Premchand has been referred to as the ‘Upanyas Samrat’ by Hindi writers.

Premchand's most important contribution to Hindi literature is the introduction of realism in it. He freed it from romanticism. Premchand had a progressive outlook and he brought a new sense of social awareness to fiction. The novels Prema, Vardaan, Nirmala, Gaban, Rangabhumi and Godaan are some of his well-known works.

6.3.2 Summary

Gangu is a Brahman servant of the writer. Although he is a servant, he does not show the same reverence to his master as is shown by other servants. He never mixes with the other servants of the household. Unlike other servants,

DDE, GJUS&T, Hisar 295 | English (Compulsory) BA-101

he is not addicted to opium. He is completely illiterate but being a Brahman, thinks highly of him due to the respect and honour of his forefathers.

One day, Gangu approaches the writer and tells him that he wants to resign from his job. The writer is mildly shocked because he has always treated his servants well. But Gangu tells him that he has no grudge against him and he is leaving him because he wants to marry Gomti. Now this comes as a real shock to the writer. Gomti is a woman of ill repute. The whole village knows that she is a woman of loose morals. She came to the Widow's house many years ago. They got her married three times. But each time she ran away from her husband. Because of her ill reputation, she was thrown out of the Widow's House. Now she has rented a room in the neighbourhood. She is visited by lecherous young men of the locality. The writer exclaims how Gangu can even think of marrying such a woman.

Gangu tells the writer that he knows everything about Gomti. Still he wants to marry her. He says that she left her previous husbands because they did not really love her. They loved only her body not the heart. He says that he will love Gomti from the core of his heart. When Gangu insists on marrying Gomti, the writer accepts his resignation. He does not want to invite people's criticism on having a servant who marries a wicked woman like Gomti. But he warns Gangu that Gomti will probably run away from him also in a few months.

Gangu goes and marries Gomti. He is living in a thatched house in the neighbourhood. He earns his living by selling snacks from a cart. Whenever the writer comes across him, he expects to see disappointment on Gangu's face. But he is always cheerful. He earns very little, yet he is not worried. His face does not show any feeling of guilt on having married Gomti. The writer thinks that the dignity and joy on his face can come only from inner peace.

Five months have passed. One day the writer learns that Gomti has run away from Gangu's house. He thinks that Gangu has suffered the punishment for

DDE, GJUS&T, Hisar 296 | English (Compulsory) BA-101

his foolish deed. Now he will realize the worth of his master's advice. The same day, the writer comes across Gangu in the market. He is upset and on seeing the writer his eyes are filled with tears. He tells him that Gomti has betrayed him. The writer pretends to be sympathetic to him. He asks him whether she taken away all his money also. Gangu says that she did not touch even a single thing. She has left behind even her own things. Gangu says that she looked perfectly happy and satisfied with her. The writer is disappointed. He has expected Gangu to tell a tale of Gomti's infidelity. But he was still singing her praises. He thinks that Gangu has become mentally disturbed. Gangu says that he will try to find Gomti and wherever she is, he will bring her again to his home.

For one month, the writer is away to Nainital. After his return, one day, Gangu comes to him. He is very happy. He is carrying a child in his arms. The writer asks him whether he has found Gomti and whose child he was carrying. Gangu replies that she was in a woman's hospital in Lucknow. She told a friend here to inform him about her. He immediately went to Lucknow and brought her. Gangu tells the writer that in the hospital Gomti gave birth to this child. It is only one month old. The writer asks him how a child can born in the sixth month of his marriage. Gangu catches the hint in the writer's statement. He agrees that the child is born of the union of Gomti with one of her previous husbands. But he says that he considers the child his own. He says that he will always remain faithful to Gomti.

The writer's eyes are filled with tears. Some unknown power subdues his orthodox thinking and moral disgust. He takes the child in his lap and kisses it. Now the writer thinks that the veil of goodness has been drawn from his eyes. He says that he will himself go and meet Gomti. Saying this, he goes with Gangu, holding the child to his breast.

6.5 CHECK YOUR PROGRESS

DDE, GJUS&T, Hisar 297 | English (Compulsory) BA-101

1) Answer the following questions:

a) Who was Gangu?

b) Does Gangu ever salute the narrator?

c) What type of man is Gangu?

d) Was Gangu illiterate?

e) What does gangu inherit from his forefathers?

f) What does the narrator think about the servants when they come early in the morning to him?

g) What does the narrator consider complaints?

h)What is the hardest of tasks for the narrator?

i) Who said to the narrator’ “Please give me permission to leave.”?

j) Who has been thrown out of the Widow’s home?

k)How many times had the Home’s administrators gotten Gomti married?

l) Who had become a source of entertainment for the dissolute men of the locality?

m) Whom does Gangu marry?

n) How did Gangu earn his living after marriage?

o) How much profit did Gangu make everyday?

p) Was Gomti educated?

q)Where did the narrator go for work?

r) Where did Gangu find Gomti?

s) Was Gangu the real father of the child?

2) Give the words which are similar in meaning to the following words:

DDE, GJUS&T, Hisar 298 | English (Compulsory) BA-101

a)Seen

b)Suffer

c)Taken aback

d)

e)Fallibility

3) Give the words which are opposite in meaning to the following words:

a)Evil

b)Faithful

c)Kindness

d)Fidelity

e)New

6.6 Transcription

Phonetic transcriptions of some important words taken from the story.

Drenched drɛnʧt ˈ ɜː ə servants s v nts səˈpraɪzɪŋli surprisingly rɪˈlɪʤəs religious ˈæbsəluːtli

absolutely əˈkjuːmjʊleɪtɪd ˈrɪʧʊəl accumulated sɛlf-rɪˈlaɪənt ritual ˈwiːknɪs self-reliant ˈmaɪldli ˈdɒmɪnəns Weakness ˈsʌmən

DDE, GJUS&T, Hisar 299 | English (Compulsory) BA-101

ˈ ɜː ə mildly s v nts dɪˈvɛləp dominance kənˈsɛʃən summon ˈfæʃən

servants ˈfɛləʊ ˈwɔːnɪŋ develop mɒˈdɜːnɪti concession rʊɪnd fashion ˈlɔːsjuːts

fellow ˌsaɪkəˈlɒʤɪkəl ˈsʌfəd warning ˈwɪdəʊ modernity bɪˈtre ruined

lawsuits

psychological

suffered

widow

betray

6.7 Comprehension

Read the passages carefully and answer the questions that follow.

1)People call Gangu a Brahman and he considers himself a Brahman. My groom and other servants salute me, even from a distance, Gangu never does. Perhaps he expects me to greet him by saying, ‘I touch your feet.’ He never touches a glass front which I have drunk, I have never dared ask him to

DDE, GJUS&T, Hisar 300 | English (Compulsory) BA-101

fan me. When I am drenched in sweat and none of the other servants is around. Gangu does pick up the fan, but the look on his face indicates that he is doing me a favour. For some reason, I always take the fan from his hand immediately. He’s a man of extremes. He can’t tolerate people’s chatter. He must have very few friends. Perhaps he thinks it beneath his dignity to sit with the groom and the servants. I’ve never seen him socialize with anyone. Surprisingly he is not given to opium or hemp; this is an extraordinary virtue in men of his class. I have never seen him perform religious rituals or go for a sacred bath in the river. Despite being absolutely illiterate he is still a Brahman, and wants the world to respect and serve him in recognition of this. And why shouldn’t he? If people can not only retain control of property accumulated by their forefathers but also derive status from it as if they had generated it themselves, why should he give up the respect and honour gathered by his forefathers? After all, this is his only inheritance.

a)What do the people call Gangu?

Ans. People call Gangu a Brahman.

b) What does Gangu expects from the narrator?

Ans. Gangu expects the narrator to greet him.

c) What kind of man is Gangu?

Ans. He’s a man of extremes. He can’t tolerate people’s chatte r.

d)What is virtuous about Gangu?

Ans. He is not given to opium or hemp.

e)Is Gangu illiterate?

Ans. Yes, Gangu is illiterate.

f)What is the only inheritance of Gangu?

DDE, GJUS&T, Hisar 301 | English (Compulsory) BA-101

Ans. Respect and honour gathered by his forefathers is the only inheritance of Gangu.

2)I gaped at him in amazement. This stupid Brahman with his old- fashioned ideas, who has remained entirely untouched by the breezes of modernity, wants to marry that loose woman, whom no decent man would allow inside his house. Gomti had created much turbulence in our peaceful neighbourhood. She had come to the Widows' Home many years ago. The Home's administrators had gotten her married three times, but each time she ran away in a fortnight or a month's time, and returned to the Home. This time, the head of the Home had thrown her out. She had rented a room in the neighbourhood, and had become a source of entertainment for the dissolute men of the locality.

I was angered by Gangu's simplicity but also pitied him. Is this the only woman in the world this donkey can find to marry? She has run away from three husbands; how long will she stay with him? If he were a very rich man it might be different. Perhaps she would have stayed for six months or a year. This fellow is as good as a blind man. They won't get along for even a week.

I said in a warning tone, 'Do you know this woman's life story?'

Gangu said, as if speaking of events, he had witnessed with his own eyes, 'It's all lies, sir; people have defamed her for nothing. '

'What do you mean? Didn't she run away from three husbands?'

'They threw her out so what could she do?'

'What a fool you are! Would anyone travel such long distances and spend thousands of rupees to marry a woman, just in order to throw her out?'

Gangu said with deep emotion, 'Sir, a woman cannot live where there is no love. A woman wants more than food and clothes, she wants some love too. Those people think they have done a widow a great favour by marrying her.

DDE, GJUS&T, Hisar 302 | English (Compulsory) BA-101

They want her to become theirs, body and soul; but to make another person one's own, one has to first become the other person's, sir. That's the thing. And then she also suffers from an ailment. She is possessed by a spirit. Sometimes, she gets hysterical and falls clown in a faint.' (a) Who did the narrator gape at in amazement?Why?

a)What is the narrator's opinion about Gomti?

Ans. The narrator thinks that Gomti is a woman of loose morals.

b)Why did the head of the Widow's Home throw Gomti out?

Ans. He threw her out because he thought that she was a woman of bad character.

c)What angered the narrator about Gangu?

Ans. The fact that Gangu knew everything and yet wanted to marry Gomti angered the narrator.

d)According to Gangu, who had thrown Gomti out?

Ans. According to Gangu, Gomti's previous husbands threw her out.

e)According to Gangu, what does a woman want?

Ans. A woman wants love.

f)What does one need to do to make another person one's own?

Ans. To make another person one's own, one has to become the other person.

g)Who is referred to in the passage as possessed by a spirit?

Ans. Gomti is referred to in the passage as possessed by a spirit.

6.8 Examination style questions

1.8.1 Short questions

DDE, GJUS&T, Hisar 303 | English (Compulsory) BA-101

1)How does Gangu convince the narrator that it is not wrong to marry Gomti? Is the narrator convinced?

Ans. Gangu tells the writer that he knows everything about Gomti and wants to marry her. He says that she left her previous husbands because they did not really love her. They loved only her body and not the heart. According to him, a woman wants only love. He says that he will love Gomti from the core of his heart. But the narrator is not convinced.

2)What is the impression the narrator has of Gangu's married life with Gomti?

Ans. The narrator thinks that Gangu's married life with Gomti will not be a happy one. He thinks that Gomti will run away from Gangu's house also. After his marriage with Gomti, Gangu starts living in a thatched house. To earn his living, he started to sell snacks from a cart. He earns enough to meet all his needs. Now he always looked cheerful. He seemed to have no worry at all. He looks full of energy. The dignity and joy on his face is a proof of his inner peace. One day he comes across Gangu in the market. He is surprised to see that Gangu is cheerful.

3)Is Gangu the hero of the story? If so, how does the story make him one?

Ans. Yes, Gangu is the hero of the story. In this story he acts like a reformer. He knows that Gomti has run away from the houses of her three previous husbands. Even now she is leading a life of ill-reputation. But Gangu believes that one should love one's heart, not body. So, despite the protest of his master, he marries Gomti. He even adopts Gomti's child which is born to her by her previous husband. Gangu does not believe in outdated conventions.

4)What is the role of the child in the story?

Ans. The story is appropriately named 'The Child.' The child in the title appears at the end of the story. But it plays an important role. This is the child of Gangu's wife, Gomti. But it is Gomti's child from a previous husband.

DDE, GJUS&T, Hisar 304 | English (Compulsory) BA-101

Despite knowing this fact, Gangu adopts the child. This incident moves the narrator to tears. He realizes that he has been following old and useless conventions. He takes the child from Gangu. He holds the child to his breast.

5)What is the 'middle class' morality that the narrator seems to represent here? How does this contrast with what Gangu believes?

Ans. The narrator of the story certainly represents middle -class morality which believes in false concepts of goodness, purity and morality. He points an accusing finger at a woman who runs away from her husband. He refuses to see that it is never the child but the parent that is illegitimate. On the other hand, Gangu finds nothing wrong in a woman who runs away if her husband does not love her. To him, every child, born in or out of wedlock, is God's child.

Q6. What did the narrator think about Gangu as a servant?

Ans. The narrator says that Gangu was different from other servants. The other servants bowed to him, but Gangu never greeted him like that. Being a Brahmin, he probably expected the narrator to bow to him. The narrator had not the courage to ask him to fan him in the hot weather. If ever Gangu did it, he behaved as if he were doing the narrator a great favour. Gangu never touched any of the used utensils.

Q7. What was the narrator's attitude towards his servants?

Ans. The narrator talked to his servants only when it was necessary. He had instructed them to come to him only when they were sent for. He never called them for such small matters as getting a glass of water or putting on the shoes or lighting the lamp. He preferred to do such things himself. They gave him a feeling of independence and self-reliance.

DDE, GJUS&T, Hisar 305 | English (Compulsory) BA-101

Q8. How did the narrator feel when Gangu said that he wanted to leave his job? Ans. The narrator was shocked when Gangu said that he wanted to leave his job. He was Considered an ideal employer. Servants considered it their good fortune to stay on with him. No servant had ever come to him with a request to leave. Naturally, the narrator felt hurt when Gangu came with such a request.

Q9. Why did Gangu want to leave his job?

Ans. Gangu had decided to marry a widow named Gomti Devi. She was a woman of bad reputation. If Gangu married her, people could raise their fingers at his master also. Gangu didn't want his master to get a bad name on his account. So, he decided to leave his job.

Q10.How had Gomti created a stir in the mohalla?

Ans. Gomti was a widow. She lived in the Widows' Home. The Home authorities got her married thrice. But each time she came back after a week or so. She was expelled from the Home. Now Gomti took a room in the mohalla on rent and started living there. She became an object of interest to all young men. It created quite a stir in the mohalla.

Q11. How did Gangu defend Gomti's leaving her previous husbands?

Ans. Gangu found fault with the people who had married Gomti. He said that those people had no love for Gomti. They thought they had done her a great favour by marrying a widow. But Gangu said, "Where there is no love, you cannot expect a woman to stay on. You cannot win a woman with mere boarding and lodging.

Q12. Describe Gangu's life after he had married Gomti.

Ans. Gangu had left his job with the narrator. After marrying Gomti, he started living in a thatched hut. He was now always happy. His face had a glow which showed that he had no worry at all. He earned a living as a hawker. He

DDE, GJUS&T, Hisar 306 | English (Compulsory) BA-101

earned about a rupee daily. After buying the stock, he was left with about ten annas. Thus, he lived a life of complete contentment.

Q13.Why did Gomti run away barely six months after her marriage?

Ans. Gomti was expecting a child. This child was not fathered by Gangu. Gomti feared that when the child was born, Gangu would not love it. The child could become a cause of quarrels between them. That was why Gomti ran away even before the child was born.

Q14.How did Gangu feel after Gomti had left him?

Ans. Gangu was very sad after Gomti had left him. He looked completely shattered. He felt sure that he was not good enough for her. While she was educated, he was an absolute illiterate. "I must have been at fault somewhere that she decided to leave." said Gangu very sadly.

Q15. What did the narrator do when he realized Gangu's true nobility?

Ans. The narrator was deeply touched by Gangu's nobility. He took the child from him and kissed it. He called Gangu the embodiment of goodness. He said that the child was adding charm to his goodness. Then he went with Gangu to meet Gomti at their house,

6.8.2 Long questions

Q1. Describe Gangu's married life and his reaction on his wife's running away.

Ans. Gangu marries Gomti. He lives in a thatched house in the neighbourhood. He earns his living by selling snacks from a cart. Whenever the writer comes across, he expects to see disappointment on Gangu's face. But he is always cheerful. He earns very little, yet he is not worried. His face does not show any feelingof guilt on having married Gomti. The writer thinks that the dignity and joy on her face can come only from inner peace. Five months have passed. One day the writer learns that Gomti has run away from Gangu's house. He thinks that Gangu has suffered the punishment for his foolish deed. Now he will

DDE, GJUS&T, Hisar 307 | English (Compulsory) BA-101

realize that his master's advice was worth accepting. The same day, the writer comes across Gangu in the market. He is upset and on seeing the writer his eyes are filled with tears. He tells him that Gomti has betrayed him. The writer pretends to be sympathetic to him. He asks him whether she taken away all his money also. Gangu says that she did not touch even a single thing. She has left behind even her own things. Gangu says that she looked perfectly happy and satisfied with her. The writer is disappointed. He has expected Gangu to tell a tale of Gomti's infidelity. But he was still singing her praises, He thinks that Gangu has become mentally disturbed. Gangu says that he will try to find her and wherever she is, he will bring her again to his home.

Q2. Who is Gangu? How does he try to justify his desire to marry Gomti?

Ans. Gangu is a Brahman servant of the writer. One day, Gangu approaches the writer and tells him he wants to resign from his job. The writer is slightly shocked because he has always treated his servants well. But Gangu tells him that he has no grudge against him. He is leaving him because he wants to marry Gomti. Now this comes as a real shock to the writer. Gomti is a woman of ill repute. The whole village knows that she is a woman of loose morals. She came to the Widow's House many years ago. They got her married three times. But each time she ran away from her husband. Because of her ill reputation, she was thrown out of the Widow's House. Now she has rented a room in the neighbourhood. She is visited by lecherous young men of the locality. The writer exclaims how Gangu can even think of marrying such a woman. But Gangu tells the writer that he knows everything about Gomti, still he wants to marry her. He says that she left her previous husbands because they did not really love her. They loved only her body not the heart. He says that he will love Gomti from the core of his heart.

DDE, GJUS&T, Hisar 308 | English (Compulsory) BA-101

Q3. How did Gangu defend his decision to marry Gomti?

Ans. Gangu did not see any fault in Gomti. He said that people had given her a bad name for nothing. It was true that she had left three husbands. But here also, Gangu found no fault in her. He said that those people had no love for Gomti. They thought they were doing her a great favour in marrying a widow. So, they expected her to do everything for them. But Gangu said, "Where there is no love, you cannot expect a woman to stay on• You cannot win a woman with mere board and lodging." Gangu believed that to win someone over, one has first to forget about oneself. And he was prepared to forget himself in order to win Gomti's love. He loved her from the core of his heart and could make any

sacrifice for her. He says, "I shall make something of myself, if I yet her." It shows the depth of Gangu's love and regard for Gomti. No defense could be greater than that!

Q4. Draw a character sketch of Gangu.

Ans. Gangu is completely illiterate, but he towers far above the so-called literates. He may have been old-fashioned but in matters of humanism, no one could be more enlightened. He may not deserve respect as a Brahmin, but as a man he is the embodiment of goodness. Whatever shortcomings he might have as a servant; as a husband he would do proud to any wife.

Gangu is an old-fashioned, illiterate Brahmin. But he shows better understanding of human character than the narrator. The narrator calls him short-tempered. But nowhere in the story do we see him reacting in that manner. The narrator blames Gomti for leaving her husbands. He never tries to analyse the reasons behind it. But Gangu puts it so rightly when he says, "Where there is no love, we cannot expect a woman to stay on. We cannot win a woman with mere board and lodging." To conclude, we can say that Gangu is an angel of love and compassion.

DDE, GJUS&T, Hisar 309 | English (Compulsory) BA-101

Q5. Is Gangu the hero of the story, 'The Child’? If yes, justify your argument.

Ans. Gangu is definitely the hero of the story. He towers over eve ry other character in the story. He is an old-fashioned, illiterate Brahmin, but he shows a better understanding of the human character than the narrator who is quite literate and experienced. For example, the narrator blames Gomti for leaving her husbands one after the other. He never tries to know or analyse the reasons behind it. But Gangu, in spite of being illiterate and orthodox, puts it very rightly when he says, "Where there is no love, we cannot expect a woman to stay on. We cannot win a woman with mere board and lodging.”

When Gangu starts living with Gomti, he gives her all his love. He knows that the child born to Gomti is not his child. Yet he proudly calls the child his own child. He says, "I would love it as my own. After all, when one takes a harvested field, one does not refuse the crop merely because someone else has sown it." If the purity and goodness of heart combined with moral courage were an indication of heroic qualities, Gangu definitely deserves to be called the hero of the story. He is a hero who wins all the praise and respect of the reader.

Q6. Narrate briefly the story, 'The Child', in your own words.

Ans. Gangu was one of the narrator's servants. He was completely illiterate. One day, he told the narrator that he wanted to leave his job. He wanted to marry a woman named Gomti Devi. She was a widow and had already married three times. But Gangu found no fault in her. He said, "Where there is no love, you can't expect a woman to stay on." When the narrator saw that Gangu was determined to marry Gomti Devi, he let him go. But he was certain that the marriage won't last more than a few days.

And he was proved right. One day he heard that Gomti had run away. Gangu looked completely shattered. He said that he must have been at fault somewhere that Gomti had left him. He called her a goddess. The narrator

DDE, GJUS&T, Hisar 310 | English (Compulsory) BA-101

called Gangu a fool who still had his eyes closed. He asked him to go and find her out if he was so much attached to her.

Gangu found Gomti in the Women's Hospital in Lucknow. She had gi ven birth to a male child. That was why she had run away. Gangu brought her back. The narrator knew that Gangu had been married for only six months. He was surprised at Gangu's Shamelessness. But Gangu said that he would love the child as his own. The important thing for him was that Gomti should love him. The narrator was deeply impressed by Gangu's sentiments. He took the child from Gangu and kissed it. Then he went with Gangu to meet Gomti at their house.

6.9 ANSWERS TO CHECK YOUR PROGRESS

a) Gangu was a Brahmin and a servant to the narrator.

b) No, Gangu never salutes the narrator.

c) Gangu is a man of extremes.

d) Yes, Gangu was illiterate.

e) Respect and honour gathered by his forefathers.

f) The narrator thinks that they have come either to complain about other servants or to get an advance.

g) The narrator considers complaints a sign of weakness or a base attempt at flattering the employers.

h) Thinking is the hardest task for narrator.

i) Gangu said this.

j) Gomti has been thrown out of Widow’s home.

k) The Home’s administrators had gotten her married three times.

l) Gomti had become a source of entertainment for the dissolute men of society.

DDE, GJUS&T, Hisar 311 | English (Compulsory) BA-101

m)Gangu marries Gomti.

n) Gangu earned his living by selling snacks from a cart.

o) Gangu made a profit of eight or ten annas.

p) Yes, Gomti was educated.

q) The narrator went to Nainital for work.

r) Gangu found Gomti in the women’s hospital in Lucknow.

s) No, Gangu was not the real father of the child.

2) Give the words which are similar in meaning to the following words:

a)Witness

b)Bear

c)Shocked

d)Embodiment

e)Unreliability

3) Give the words which are opposite in meaning to the following words:

a)Good

b)Unfaithful

c)Cruelty

d)Infidelity

e)Old

6.10 REFERENCES/SUGGESTED READINGS

Hooda, Rana and Mohan (Eds). Literature and Language II. Hyderabad: Orient Blackswan, 2018.

DDE, GJUS&T, Hisar 312 | English (Compulsory) BA-101

Course Code: BA101 Author: Dr.NutanYadav

Lesson No:07

Chapter-7 The Blind Dog

R.K.Narayan

Lesson Structure

7.1 Learning Objectives

7.2 Introduction

7.3 Main Body

7.3.1 About the Author

7.3.2 About the Essay

7.4 Further Body of the text

7.5 Check your Progress

7.6 Summary

7.7 Keywords

7.11 Self -Assessment Questions (SAQs)

7.8.1Language Activity

7.12 Answers to Your Progress 7.9.1 Extended Composition

7.10 References/ Suggested Readings

7.1 LEARNING OBJECTIVES

After going through this lesson you will be able to-

DDE, GJUS&T, Hisar 313 | English (Compulsory) BA-101

 To develop reading, writing, speaking, and listening skills of the English language.  Read, understand and enjoy the story  To enrich the taste of literature.  To develop a reading habit  Know the difference between poetry and prose.

7.2 INTRODUCTION

‘The Blind dog’ is one of the most famous stories of R.K Narayan. This is the story of a dog and his loyalty to a blind beggar. This story tells us about human gree d and cruelty to animals. But the most important message in the story is the loyalty of a dog to his master. The blind beggar treats the dog cruelly. A shopkeeper cut the cord which holds the dog and it becomes free. But after a few days, it returns to the beggar on his own accord.

7.3 MAIN BODY OF THE TEXT

"Blind Dog" is one of the most popular stories of R.K. Narayan from his collection 'Malgudi Days' that discusses the story of a dog and his loyalty towards his blind master who is a beggar.

The story begins with the everyday struggles of the blind man and Tiger (the dog). Tiger helps him in saving his food from others which the blind man appreciates. The blind man begins to tyrannize the dog and treat him merely as a working dog that is there only to serve him. The man gradually becomes greedy and makes Tiger work more and more to earn extra money. He begins to lend money to people on interest. With increasing greed, the blind man becomes cruel and he not only fails to acknowledge the hard work and efforts of the poor animal but also beats him often. Then, the tiger finally is set free by the Perfumer as he felt what the blind man was doing to him was devilish. The blind man repents his actions for mistreating him. The readers are given a shock when the tiger returns at the end despite all this

DDE, GJUS&T, Hisar 314 | English (Compulsory) BA-101 mistreatment and humiliation which might be his affection towards him or the loyalty.

The theme of the story is 'human greed and ungratefulness' in contrast to the 'loyalty and free-will of animals.'

About the Author

R. K. Narayan (born 1906) is one of the best-known Indo-English writers. He created the imaginary town of Malgudi, where realistic characters in a typically Indian setting lived amid unpredictable events.

Rasipuram Krishnaswami Narayanswami, who preferred the shortened name R.K. Narayan, was born in Madras, India, on Oct. 10, 1906. His father, an educator, traveled frequently, and his mother was frail, so Narayan was raised in Madras by his grandmother and an uncle. His grandmother inspired in young Narayan a passion for language and people. He attended the Christian Mission School, where, he learned to love the Hindu gods simply because the Christian chaplain ridiculed them. Narayan graduated from Maharaja's College in Mysore in 1930. In 1934 he was married, but his wife, Rajam, died of typhoid in 1939. He had one daughter, Hema. He never remarried.

R. K. Narayan’s s short stories are artistical as eminent as his novels, and in any general estimate of his writings, they cannot be ignored. One might go so far as to say that Narayan is essentially a short storyteller and the one element that stands out even in his novels is the story element. These stories belong to the Indian soil and are evocative of its culture. In the main, they represent South Indian life and clearly expressing Narayan’s view of the world and those who live in it. The simple but captivating plot, sparkling characterization, strict economy of narration, and graceful simplicity of language are features of these short stories. They serve as a good foreword to the foreigner who wants to know the Indian way of life. Narayan perceives the balance of power in human relations in every aspect of man’s life – social, political, and moral and the perception leads to his detached observation of

DDE, GJUS&T, Hisar 315 | English (Compulsory) BA-101 the human scene. It is this quality more than any other that distinguishes Narayan from the other writers

About the story-

In The Blind Dog by R.K. Narayan, we have the theme of struggle, connection, greed, freedom, control, and loyalty. Taken from his Malgudi Days collection, the story is narrated in the third person by an unnamed narrator and after reading the story the reader realizes that Narayan may be exploring the theme of struggle.

Both the blind man and Tiger have difficulties in their everyday lives. Each day the blind man struggles to get enough money to live on while Tiger struggles with getting food and ends up getting into fights with other dogs just to survive. It is as though both the blind man and Tiger have something in common or something that connects both of them. It is also interesting that at first, the blind man appreciates Tiger’s company and his diligence when it comes to people trying to steal from him. However, as time passes the reader soon realizes that the blind man is a tyrant when it comes to how he treats Tiger. Tiger for the blind man is not a pet or friend or a helping hand rather he is simply a working dog that is there to serve the blind man. Any type of life that Tiger had previously known is soon forgotten due to the tyranny of the blind man. If anything Tiger is there to serve the blind man just as a servant would serve their master.

It is also noticeable that the blind man starts to become greedy. He wishes to increase his daily income so he works Tiger more and more. He also starts to lend other people money while at the same time charging them interest. While some may pity the blind man because of the very fact he is blind others might suggest that the blind man is becoming greedy and taking advantage of Tiger’s good-nature. The blind man knows he will make more money by walking along the streets with Tiger leading the way and there is a sense that the main priority for the blind man is no longer just survival but he is beginning to be driven by a desire for more money. Rather

DDE, GJUS&T, Hisar 316 | English (Compulsory) BA-101 than treating Tiger with kindness and being grateful that Tiger is helping him. The blind man often beats Tiger which may suggest that the blind man is being cruel.

It may also be important that others notice how the blind man is treating T iger by having others notice what is happening. Narayan may be using their voices as a consensus to stop what is happening to Tiger. The cutting of the ribbon by the ribbon vendor may also be important as symbolically this action acts as a path to freedom for Tiger. He can live his life as he had previously lived it. It is also noticeable after Tiger has been set free how reliant the blind man was on Tiger. He is no longer able to walk along the streets and his income drops severely. It causes great anguish to the blind man. At no stage in the story does the reader suspect that the blind man is repentant about his treatment of Tiger. If anything he wants to beat Tiger should he end up finding him? This again suggests a servant and the master relationship between Tiger and the blind man. The relationship between Tiger and the blind man is one-sided. The blind man never realizes that he needs Tiger more than Tiger needs him.

The end of the story is also interesting as Narayan appears to be exploring the theme of loyalty. By returning to the blind man Tiger is showing his loyalty. Even though it is clear to the reader that nothing will change between Tiger and the blind man. Something noticeable by the fact that the blind man now has bought a chain to ensure that Tiger does not run away again. There is a sense that the blind man is in complete control of Tiger again. As to why Tiger has returned is difficult to say when as readers we are aware of how badly he has been treated by the blind man. However, Narayan may be suggesting that just as the blind man is blind so too is Tiger’s loyalty. He is willing to forgive the blind man’s actions towards him even though he has been unfairly treated. It is also possible that Tiger has sympathy for the blind man.

7.4 FURTHER BODY OF THE TEXT

Original Text

DDE, GJUS&T, Hisar 317 | English (Compulsory) BA-101

IT was not a very impressive or high -class dog; it was one of those commonplace dogs one sees everywhere, the color of white and dust, tail mutilated at a young age by God knows whom, born in the street, and bred on the leavings and garbage of the market-place. He had spotty eyes and undistinguished carriage and needless pugnacity. Before he was two years old he had earned the scars of a hundred fights on his body. When he needed rest on hot afternoons he lay curled up unde r the culvert at the eastern gate of the market. In the evenings he set out on his daily rounds, loafed in the surrounding streets and lanes, engaged himself in skirmishes, picked up edibles on the roadside, and was back at the market gate by nightfall.

This life went on for three years. And then occurred a change in his life. A beggar, blind of both eyes, appeared at the market gate. An old woman led him up there early in the morning, seated him at the gate, and came up again at midday with some food, ga thered his coins, and took him home at night.

The dog was sleeping nearby. He was stirred by the smell of food. He got up, came out of his shelter, and stood before the blind man, wagging his tail and gazing expectantly at the bowl, as he was eating his sparse meal. The blind man swept his arms about and asked: “Who is there?” At which the dog went up and licked his hand. The blind man stroked its coat gently tail to ear and said: “What a beauty you are. Come with me “He threw a handful of food which the dog ate gratefully. It was perhaps an auspicious moment for starting a friendship. They met every day there, and the dog cut off much of its rambling to sit up beside the blind man and watch him receive alms morning to evening. In course of time observing him, the dog understood that the passers-by must give a coin, and whoever went away without dropping a coin was chased by the dog; he tugged the edge of their clothes by his teeth

DDE, GJUS&T, Hisar 318 | English (Compulsory) BA-101 and pulled them back to the old man at the gate and let go only after some thi ng was dropped in his bowl.

Among those who frequented this place was a village urchin, who had the mischief of a devil in him. He liked to tease the blind man by calling him names and by trying to pick up the coins in his bowl. The blind man helplessly shouted and cried and whirled his staff. On Thursdays this boy appeared at the gate, carrying on his head a basket loaded with cucumber or plantain. Every Thursday afternoon it was a crisis in the blind man's life. A seller of bright colored but doubtful perfumes with his wares mounted on a wheeled platform, a man who spread out cheap story -books on a gunny sack, another man who carried colored ribbons on an elaborate frame these were the people who usually gathered under the same arch, On a Thursday when the young man appeared at the Eastern gate one of them remarked, "Blind fellow! Here comes your scourge"

"Oh, God is this Thursday?" he wailed. He swept his arms about and called: "Dog, dog, come here, where are you?" He made the peculiar noise that brought the dog to his side. He stroked his head and muttered: "Don't let that little rascal"

At this very moment, the boy came up with a leer on his face.

"Blind man! Still pretending you have no eyes. If you are blind, you should not know this either "He stopped, his hand moving towards the bowl. The dog sprang on him and snapped his jaws on his wrist. The boy extricated his hand and ran for his life. The dog bounded up behind him and chased him out of the market.

“See the mongrel's affection for this old fellow!” marveled the perfume -ve ndor.

One evening at the usual time the old woman failed to turn up, and the blind man waited at the gate, worrying as the evening grew into the night. As he sat

DDE, GJUS&T, Hisar 319 | English (Compulsory) BA-101 fretting there, a neighbor came up and said: "Sami, don't wait for the old woman. She will not come again. She died this afternoon"

The blind man lost the only home he had, and the only person who cared for him in this world. The ribbon-vendor suggested: "Here, take this white tape "He held a length of the white cord which he had been selling" I will give this to you free of cost. Tie it to the dog and let him lead you about if he is so fond of you"

Life for the dog took a new turn now. He came to take the place of the old woman. He lost his freedom completely. His world came to be circumscribed by the limits of the white cord which the ribbon -vendor had spared. He had to forget wholesale all his old life all his old haunts. He simply had to stay on forever at the end of that string. When he saw other dogs, friends, or foes, instinctively he sprang up, tugging the string, and this invariably earned him a kick from his master. "Rascal, want to tumble me down, have sense" In a few days the dog learned to discipline his instinct and impulse. He ceased to take notice of other dogs, even if they came up and growled at his side. He lost his orbit of movement and contact with his fellow-cre ature s.

To the extent of this loss his master gained. He moved about as he had never moved in his life. All-day he was on his legs, led by the dog. With the staff in one hand and the dog-lead in the other, he moved out of his home a corner in a country veranda a few yards off the market: he had moved in there after the old woman's death. He started early in the day. He found that he could treble hi s i ncome by moving about instead of staying in one place. He moved down the country street, and wherever he heard people's voices he stopped and held out his hands for alms. Shops, schools, hospitals, hotels he left nothing out. He gave a tug when he wanted th e dog to stop, and shouted like a bullock - driver when he wanted him to move on. The dog protected his feet from going into pits, or stumping against steps or stones, and took him up inch by inch

DDE, GJUS&T, Hisar 320 | English (Compulsory) BA-101 on safe ground and steps. For this sight, people gave coins a nd helped him. Children gathered around him and gave him things to eat. A dog is essentially an active creature who punctuates his hectic rounds with well-defined periods of rest. But now this dog (henceforth to be known as Tiger) had lost all rest. He had rest only when the old man sat down somewhere. At night the old man slept with the cord turned around his finger.

"I can't take chances with you," he said. A great desire to earn more money than ever before seized his master, so that he felt any resting a waste of opportunity, and the dog had to be continuously on his feet. Sometimes his legs refused to move. But if he slowed down even slightly his master goaded him on fiercely with his staff. The dog whined and groaned under this thrust. "Don't whine, you rascal. Don't I give you your food? You want to loaf, do you?" swore the blind man. The dog lumbered up and down and round and round the market-place on slow steps, tied down to the blind tyrant. Long after the traffic at the market ceased, you could hear the night stabbed by the far-off wail of the tired dog. It lost its original appearance. As months rolled on, bones stuck up at his haunches, and ribs were relieved through his fading coat.

T he ri bbon-seller, the novel-vendor, and the perfumer observed it one evening, when business was slack, and held a conference among them: "It rends my heart to see that poor dog slaving. Can't we do something? "The ribbon -se lle r remarked: "That rascal has started lending money for interest I heard it from that frui t-seller He is earning more than he needs. He has become a very devil for mone y"

At this point the perfumer's eyes caught the scissors dangling from the ri bbon-rack. "Give it here," he said and moved on with the scissors in hand.

The blind man was passing in front of the Eastern gate. The dog was straining the lead. There was a piece of bone lying on the way and the dog was straining

DDE, GJUS&T, Hisar 321 | English (Compulsory) BA-101 to pick it up. The lead became taut and hurt the blind man's hand, and he tugged the string and kicked till the dog howled. It howled, but could not pass the bone lightly; it tried to make another dash for it. The blind man was heaping curses on it. The perfumer stepped up, applied the scissors, and snipped the cord. The dog bounced off and picked up the bone. The blind man stoppe d dead where he stood, with the other half of the string dangling in his hand. "Tiger! Tiger! Where are you?" he cried. The perfumer moved away quietly, muttering: "You heartless devil! You will never get at him again! He has his freedom!" The dog went off at top speed. He nosed about the ditches happily, hurled himself on other dogs, and ran round and round the fountain i n the market-square barking, his eyes sparkling with joy. He returned to his favorite haunts and hung about the butcher's shop, tea-stall, and the bakery.

T he ri bbon-vendor and his two friends stood at the market gate and enjoyed the sight immensely as the blind man struggled to find his way about. He stood rooted to the spot waving his stick; he felt as if he were hanging in mid- air. He was wailing. "Oh, where is my dog? Where is my dog? Won't someone give him back to me? I will murder it when I get at it again!" He groped about, tried to cross the road, came near being run over by a dozen vehicles at different points, tumbled and struggled and gasped." He'd deserve it if he was run over, this heartless blackguard" they said, observing him. However, the old man struggled through and with the help of someone found his way back to his corner in the country veranda and sank on his gunnysack bed, h alf faint with the strain of his journey.

He was not seen for ten days, fifteen days, and twenty days. Nor was the dog seen anywhere. They commented among themselves. "The dog must be loafing over the whole earth, free and happy. The beggar is perhaps gone fore ve r" Hardly was this sentence uttered when they heard the familiar tap-tap of the blind man's staff. They saw him again coming up the pavement led by the dog.

DDE, GJUS&T, Hisar 322 | English (Compulsory) BA-101

"Look! Look!" they cried. "He has again got at it and tied it up." The ribbon - seller could not contain him. He ran up and said: "Where have you been all these days?"

"Know what happened!" cried the blind man. "This dog ran away. I should have died in a day or two, confined to my corner, no food, not an anna to earn imprisoned in my corner. I should have perished if it continued for another day. But this thing returned."

"When? When?"

"Last night. At midnight as I slept in the bed, he came and licked my face. I felt like murdering him. I gave him a blow which he will never forget again," said the blind man." I forgave him, after all, a dog! He loafed as long as he could pick up some rubbish to eat on the road, but real hunger has driven him back to me, but he will not leave me again. See! I have got this" and he shook the lead: it was a steel chain thi s ti me .

Once again there was the dead, despairing look in the dog's eyes. "Go on, you fool," cried the blind man, shouting like an ox-driver. He tugged the chain, poked with the stick, and the dog moved away on slow steps. They stood listening to the ta p-tap going away.

"Death alone can help that dog," cried the ribbon -seller, looking after it with a sigh. "What can we do with a creature who returns to his doom with such a free heart?"

7.5 CHECK YOUR PROGRESS

Answer the following questions in a word, a phrase, and one or two sentences.

1. How did the dog go to the blind man?

2. What did the perfumer do with the scissors?

DDE, GJUS&T, Hisar 323 | English (Compulsory) BA-101

Answers-

1. The dog was attracted by the smell of food the blind man was eating. He went up to the blind man and stood there, wagging his tail.

2. He snipped the cord which the blind man had tied to the dog.

Answer the following questions in 75-100 words

1. What happened to the dog once it became the blind man’s companion?

2. How did the friendship between the dog and the blind man begin?

3. How did the dog guard the blind man from the village urchin?

4. Write a character-sketch of the blind man.

5. What is your opinion about the dog’s behavior?

Answers-

1. It was a street dog and would go about the streets and lanes around the market-place. He fed himself on the leavings and other edibles on the roadside. But when he became the blind man’s companion, he cut off much of his rambling. He would sit beside the blind man all day and watch him receive alms. An old woman would bring food for the blind man at midday. The bli nd man would throw a handful of food for the dog, and the dog would eat it gratefully.

2. The blind man would beg at the city gate all day. At midday, an old lady would bring food for the beggar. One day, the dog was resting close by. He was stirred by the smell of food. He went and stood by the blind man, wagging his tail. The blind man asked, “Who is there?” the dog started licking the blind man’s hand. The blind man stroked its coat gently tail to ear and threw a handful of food which the dog ate gracefully. Thus the friendship between the two began.

3. The village urchin came every Thursday and teased the blind man by abusing him. He would also try to pick up coins from the blind man’s bowl. On

DDE, GJUS&T, Hisar 324 | English (Compulsory) BA-101

Thursday, someone told the blind man that the boy was coming. The blind man called out to the dog. At once, the dog sprang on the boy and snapped his jaws on the boy’s wrist. The boy extricated his hand and ran for his life. The dog bounded up behind him and chased him out of the market.

4. We can say that the blind man is blind not only in his eyes but in his soul also. His greed for money makes him dead in his soul. He makes the dog his prisoner and uses him as a slave. The poor animal has to lead him about all day. Seized with the desire to earn more and more money, the blind man allows him no rest. And if the dog slows down, he beats him mercilessly. We can say that the blind man is a thoroughly detestable fellow.

5. In this story, it is the beggar, not the dog, who is blind; yet the writer has titled his story, ‘The Blind Dog’. And he has done this very rightly. We can’t blame the dog when he loses his freedom for the first time. But having experienced all the pain and suffering of confinement, he willingly walks into capacity at the end of the story. What can we do with such a creature? Death alone can help him.

Comprehensive passage

Read the passage carefully and answer the following questions

One evening at the usual time the old woman failed to turn up, and the blind man waited at the gate, worrying as the evening grew into the night. As he sat fretting there, a neighbor came up and said: "Sami, don't wait for the old woman. She will not come again. She died this afternoon"

The blind man lost the only home he had, and the only person who cared for him in this world. The ri bbon-vendor suggested: "Here, take this white tape "He held a length of the white cord which he had been selling" I will give this to you free of cost. Tie it to the dog and let him lead you about if he is so fond of you"

DDE, GJUS&T, Hisar 325 | English (Compulsory) BA-101

Life for the dog took a new turn now. He came to take the place of the old woman. He lost his freedom completely. His world came to be circumscribed by the limits of the white cord which the ribbon -vendor had spared. He had to forget wholesale all his old life all his old haunts. He simply had to stay on forever at the end of that string. When he saw other dogs, friends, or foes, instinctively he sprang up, tugging the string, and this invariably earned him a kick from his master. "Rascal, want to tumble me down, have sense" In a few days the dog learned to discipline his instinct and impulse. He ceased to take notice of other dogs, even if they came up and growled at his side. He lost his orbit of movement and contact with his fellow-cre ature s.

Questions-

1. Why did the old man fail to turn up that evening? 2. When and how did the blind man lose the only home he had? 3. How did the ribbon-vendor help the blind man? 4. Whose place did the dog take in the blind man’s life? How? 5. How did the dog’s life become limited and circumscribed? 6. What earned the dog the blind man’s kick and abuses? 7. How did the dog lose contact with his fellow-creatures? 8. From the passage, do you think the dog enjoyed his new role and life? Why?

Answers-

1. She had died in the afternoon. 2. When the old woman died, the blind man lost the only home he had. He had none else in the world to take care of him. 3. He gave the blind man a length of white tape to tie the dog with it. The dog could then lead the blind man about. 4. The dog took the place of the old woman who had been taking care of the blind man and leading him about. 5. He had to stay on forever at the end of the tape tied around his neck.

DDE, GJUS&T, Hisar 326 | English (Compulsory) BA-101

6. The dog would spring upon seeing the other dogs and then the blind man would shower kicks and abuses on him. 7. The dog learned to discipline his instinct and impulse. He ceased to take notice of the other dog. 8. No, the dog had lost his freedom and there was no question of his enjoying his new role and life.

7.6 KEYWORDS

Transcribe the following words-

 High  Born  Curl  Gate  Change  Smell  Arms  Beauty  Pick  Blind  Stop  Bowl  Sprang  Snap  Life  Chase  Old  Fellow  Income  Active

DDE, GJUS&T, Hisar 327 | English (Compulsory) BA-101

 Cord  Refuse  Poor  Hurt  Round  Perish

7.7 SUMMARY

In The Blind Dog by R.K. Narayan, we have the theme of struggle, connection, greed, freedom, control, and loyalty. Taken from his Malgudi Days collection, the story is narrated in the third person by an unnamed narrator and after reading the story the reader realizes that Narayan may be exploring the theme of struggle.

Both the blind man and Tiger have difficulties in their everyday lives. Each day the blind man struggles to get enough money to live on while Tiger struggles with getting food and ends up getting into fights with other dogs just to survive. It is as though both the blind man and Tiger have something in common or something that connects both of them. It is also interesting that at first, the blind man appreciates Tiger’s company and his diligence when it comes to people trying to steal from him. However, as time passes the reader soon realizes that the blind man is a tyrant when it comes to how he treats Tiger. Tiger for the blind man is not a pet or friend or a helping hand rather he is simply a working dog that is there to serve the blind man. Any type of life that Tiger had previously known is soon forgotten due to the tyranny of the blind man. If anything Tiger is there to serve the blind man just as a servant would serve their master.

It is also noticeable that the blind man starts to become greedy. He wishes to increase his daily income so he works Tiger more and more. He also starts to lend other people money while at the same time charging them interest. While some may pity the blind man because of the very fact he is blind others might suggest that the blind man is becoming greedy and taking advantage of Tiger’s good-nature. The blind

DDE, GJUS&T, Hisar 328 | English (Compulsory) BA-101 man knows he will make more money by walking along the streets with Tiger leading the way and there is a sense that the main priority for the blind man is no longer just survival but he is beginning to be driven by a desire for more money. Rather than treating Tiger with kindness and being grateful that Tiger is helping him. The blind man often beats Tiger which may suggest that the blind man is being cruel.

It may also be important that others notice how the blind man is treating Tiger by having others notice what is happening. Narayan may be using their voices as a consensus to stop what is happening to Tiger. The cutting of the ribbon by the ribbon vendor may also be important as symbolically this action acts as a path to freedom for Tiger. He can live his life as he had previously lived it. It is also noticeable after Tiger has been set free how reliant the blind man was on Tiger. He is no longer able to walk along the streets and his income drops severely. It causes great anguish to the blind man. At no stage in the story does the reader suspect that the blind man is repentant about his treatment of Tiger. If anything he wants to beat Tiger should he end up finding him? This again suggests a servant and the master relationship between Tiger and the blind man. The relationship between Tiger and the blind man is one-sided. The blind man never realizes that he needs Tiger more than Tiger needs him.

The end of the story is also interesting as Narayan appears to be exploring the theme of loyalty. By returning to the blind man Tiger is showing his loyalty. Even though it is clear to the reader that nothing will change between Tiger and the blind man. Something noticeable by the fact that the blind man now has bought a chain to ensure that Tiger does not run away again. There is a sense that the blind man is in complete control of Tiger again. As to why Tiger has returned is difficult to say when as readers we are aware of how badly he has been treated by the blind man. However, Narayan may be suggesting that just as the blind man is blind so too is Tiger’s loyalty. He is willing to forgive the blind man’s actions towards him even though he has been unfairly treated. It is also possible that Tiger has sympathy for the blind man.

DDE, GJUS&T, Hisar 329 | English (Compulsory) BA-101

7.8 SELF ASSESSMENT QUESTIONS (SAQ’S)

Provide the antonyms of the following words taken from the story

Word Antonym

High Low

Back Front

Appear Disappear

Free Captive

Sense Nonsense

Friendship Enmity

Cheap Dear/ Expensive

Income Expenditure

Whole Partial

Provide the synonyms of the following words taken from the story

Word Synonym

Earn Gain/ Acquire

Grateful Thankful

Urchin Brat

Tease Annoy

Leer Smile

Freedom Independence

Active Lively

Tyrant Dictator/ Oppressor

DDE, GJUS&T, Hisar 330 | English (Compulsory) BA-101

Observe Watch

Hurt Injure

Find one-word equivalents of the following

Flesh-eating animals Carnivores

Cud- chewing animals Ruminants

Reptiles living in the water Crocodiles

Original inhabitants of a country Natives

Things of the same nature Homogenous

Something that cannot be repaired Irreparable

A dumb show Mime

One who leads the others Leader

7.9 ANSWERS TO YOUR PROGRESS 7.9.1 Language Activity

Punctuation

During verbal communication, from time to time, we drop our tone, sometimes we raise or lower our tones, sometimes we whisper shout. In writing, this function is performed by punctuation. Correct punction gives logic and meaning to words and sentences.

The following are the punctuation marks-

Capital letters

The full stop(.)

The comma (,)

Single Inverted Commas (‘ ’)

DDE, GJUS&T, Hisar 331 | English (Compulsory) BA-101

Double Inverted Commas (“ ”)

Dashes (--)

The hyphen (-)

Semi colon(;)

Colon (:)

Question mark(?)

Parentheses()

The exclamation mark (!)

Ellipses (……..)

Capital Letters-

 Capital letters are generally used in the beginning of sentences.

Example- Urvi is the best player of the team.

 They are used to start a sentence within inverted commas.

Example- Shakespeare says,’To be or not to be…….’

 Capital letters are used in the names of people, month, weeks, days, festivals etc.

Example- Wednesday, Christmas, Karnataka, John, etc

Full Stop-

 Full stops are used at the end of complete sentences.

Example- He is a good boy.

 It is used in abbreviations

Example- Urvi is an I.A.S. officer.

Comma-

DDE, GJUS&T, Hisar 332 | English (Compulsory) BA-101

 It is used to separate aseries of words. It suggest a pause in the writing.

Example- Yuvraj can do that; Dhoni, never.

 They are used between the words that do not belong together.

Example- the shirts are blue, and brown. (It means blue, and brown, separately.)

 It is used to separate two or more nouns in apposition.

Example- Hari, my cousin, is going to New Zealand tomorrow.

 It is used to address people.

Example- Madam, kindly pass the salt, please.

 It separate the coordinate clause in a compound sentence.

Example- I came, I saw, I conquered.

Double Inverted Commas

 They are used to quote the exactwords of person or from a text.

Example- It is said, “To err is human”

Dash-

 It is used in place of a colon or parenthess in a sentence. It is used to emphasise the idea anticipated in the sentence.

Example- Finally, the students got what they long desired- a computer.

Single Inverted Commas-

 They are used to indicate quoted material within a quotation.

Example- The trainer warned, “Are you not aware? ‘Out of sight is out of mind.’”

Hyphen-

 They are used to join two or more words in a compound word.

Example- My mother-in-law is coming to visit us.

DDE, GJUS&T, Hisar 333 | English (Compulsory) BA-101

 It is used with adjectival compounds of nouns and past participles.

Example- It left him a heart-broken boy.

Semi Colon

 It is used to replace commas in a long sentence. It is also used to separate clauses.

Example- The manager admires his aptitude; but hates his behavior.

Colon-

 It is used to join examples and enumeration.

Example- the various parts of speech are: Noun, Pronoun, Adjective, Adverb, Preposition, Conjunction, etc.

Question Mark/ Sign of Interrogation

 It is used after a question. It should not be put in brackets after any word.

Example- Will you sit here?

Parentheses-

 It is used to set off additional information or ideas related to the idea expressed in a sentence.

Example- The corrupt officer (and this is how he is known throughout the locality) has finally been suspended.

Exclamation Mark

 It is used to express a sudden strong emotion or a desire.

Example- Hurrah! We have won the match.

Ellipsis-

 It is indicated by using three spaced dots. It is used to indicate the missing of a quotation.

DDE, GJUS&T, Hisar 334 | English (Compulsory) BA-101

Example- The General roared… We shall not give up.

Exercise

Punctuate the following sentences appropriately

1 Houses that are built on the top of the hills need no coolers compared to house which is built on plains 2 Sandeep said why do you call me people from the villages are sure to come 3 Anu angrily retorted do you think bringing up children is only my responsibility 4 Maggie is intelligent bold and beautiful 5 Omprakash is not only a great player of chess but is good at tennis as well 6 Come here little boy 7 Mr. Ahmed's efforts went in vain 8 My brother in law is an advocate 9 Sapna kept her in good humor Vikram had made her angry and upset 10 Any boy who carries out such a task will be large -hearted

Answers-

1 Houses that are built on the top of the hills need no coolers, compared to house which is built on plains. 2 Sandeep said, “Why do you call me? People from the villages are sure to come.” 3 Anu angrily retorted, “Do you think bringing up children is only my responsibility?” 4 Maggie is intelligent, bold, and beautiful. 5 Omprakash is not only a great player of chess but is good at tennis as well. 6 Come here, little boy. 7 Mr. Ahmed’s efforts went in vain. 8 My brother-in-law is an advocate. 9 Sapna kept her in good humor. Vikram had made her angry and upset. 10 Any boy who carries out such a task will be large -hearted.

DDE, GJUS&T, Hisar 335 | English (Compulsory) BA-101

Punctuate the following paragraph appropriately.

Then he told me that it is not a good habit to take gifts for they are always accompanied by some purpose and are dangerous that way it is like touching a snake and getting the poison, in turn, this one lesson always stands out in my mind even now when I am in my seventies that one good incident taught me a valuable lesson for my entire life

Answer-

Then he told me that it is not a good habit to take gifts, for they are always accompanied by some purpose and are dangerous that way. It is like touching a snake and getting the poison in turn. This one lesson always stands out in my mind even now when I am in my seventies. That one good incident taught me a valuable lesson for my entire life.

7.9.2 Extended Composition

Write an essay of around 200 words on a visit to a historical monument.

During the last spring holidays, I went to Agra. There I visited the Taj. It is built outside the city on the bank of the Yamuna. The Taj was built by Shah Jahan in the sweet memory of his beloved wife, Mumtaz Mahal. It was built about three hundred years ago. But so far, the time has not in any way dimmed its glory or beauty. It is made of pure white marble. It took twenty thousand workmen twenty years to build it. It cost about three crores of rupees. It is a fitting memorial to conjugal love.

The Taj is a large and beautiful building. It stands on a raised platform. In the middle of the platform, there is a splendid white dome. At its four corners, there are fur safety towers. Underneath the white dome is the marble tombs of Mumtaz Mahal and Shah Jahan. These tombs are inlaid with precious stones. The Taj looks like a fairy dressed in white. It looks beautiful against the blue sky. But in the moonlight, the Taj is like a dream in marble. No words can describe its beauty. It is one of the wonders of the world. Visitors from foreign countries who come to India make it a

DDE, GJUS&T, Hisar 336 | English (Compulsory) BA-101 point to see the Taj. It is a superb piece of architecture. I stayed there for about two hours. All this time, I was lost in admiration and wonder. Then I left the place most unwillingly. Its memory is still fresh in my mind. A thing of beauty is a joy forever.

7.10 REFERENCES/ SUGGESTED READINGS

 Ram, N.; Ram, Susan (1996). R. K. Narayan. Allen Lane. ISBN 978-0-670- 87525-2. OCLC 36283859.  Rao, Ranga (2005). R. K. Narayan. Makers of Indian Literature (2nd ed.). New Delhi: Sahitya Akademi. ISBN 81-260-1971-9. *****************

DDE, GJUS&T, Hisar 337 | English (Compulsory) BA-101

GRAMMAR Comment [g3]: done

SECTION

DDE, GJUS&T, Hisar 338 | English (Compulsory) BA-101

Subject: English

Course Code- B.A.-101 Author: Dr. Astha Gupta

Lesson- 1 Editor: Dr. Shakuntla Devi

Introduction to Noun and Pronoun (Part of Speech)

Structure of Lesson

1.1 Learning Objectives 1.2 Introduction 1.3 Main Body of the Text 1.3.1 Definition of Noun

1.3.2 Types of Noun

a. Proper noun b. Common Noun c. Abstract Noun d. Concrete noun e. Collective noun f. Countable and Uncountable noun 1.4 Noun Usage 1.4.1 Number: Singular and Plural Noun 1.4.2 Noun: Gender 1.4.3 General rules of changing gender 1.5 Further Main Body of the Text 1.5.1 Pronoun - Persons 1.5.2 Gender of the Pronoun 1.5.3 Types of Pronoun- a. Personal Pronoun

DDE, GJUS&T, Hisar 339 | English (Compulsory) BA-101

b. Reflexive Pronoun c. Emphatic Pronoun d. Demonstrative Pronoun e. Indefinite Pronoun f. Distributive Pronoun g. Relative Pronoun 1.6 Check Your Progress 1.7 Summary 1.8 Keywords 1.9 Self-Assessment Test

1.10 Answers to Check Your Progress

1.11 References /Suggested Readings

1.1 Learning Objectives

After studying this unit, you will be able to:

• To identify various parts of the speech; choose their correct form and use them effectively.

• Define noun and pronoun.

• Explain all types of noun.

• Use all types of pronoun.

1.2- Introduction

Learning about the parts of speech is the first step in grammar study just as learning the letters of the Alphabet is the first step to being able to read and write. From learning the Parts of Speech, we begin to understand the use or function of words and how words are joined together to make meaningful communication. To understand what a part of speech is, you must understand the idea of putting

DDE, GJUS&T, Hisar 340 | English (Compulsory) BA-101 similar things together into groups or categories. Let’s look at some examples of categories. From learning the parts of speech, we begin to understand the use or function of words and how words are joined together to make meaningful communication. To understand what a part of speech is, you must understand the idea of putting similar things together into groups or categories.

1.3 Main Body of the Text

1.3.1- Definition of Noun

A Noun is a word used as the name of a person, place, or a thing. It includes the name of an object that we can perceive through our senses (sight, smell, touch, hearing, taste) as well as an object about which we can think but cannot perceive through the senses.

1.3.2 Types of Noun

a. Proper noun: They are the names of particular persons, places, animal, objects, etc. A proper name is always written with a capital letter in the beginning.

For Example: Himalaya is the tallest mountain in India. Here Himalaya and India are the names of one specific mountain and one specific country respectively and cannot be applied to any another mountain and country; and hence they are proper names.

b. Common noun: Nouns other than proper nouns are called common nouns.

For example -In the above given sentence, mountain is a general category indicating the geographical structure and hence it is a common noun. Common nouns can be Abstract nouns or Concrete nouns.

c. Abstract noun: Abstract nouns denote the ideas, qualities, and emotions about which we can think but which cannot be perceived by using our senses.

For Example: courage, kindness, beauty, honesty etc.

DDE, GJUS&T, Hisar 341 | English (Compulsory) BA-101

d. Concrete noun: Concrete nouns denote the animate or inanimate objects that can be perceived through our senses. For Example- book, road, cup, telephone water, noise etc. e. Collective noun: They stand for a group of persons, animals, birds etc and various other objects which are taken together as a unit.

For Example: Army, fleet, police, herd, swarm, flock and so on.

f. Countable and Uncountable noun: Countable nouns are the names of the objects that can be counted. Countable nouns have both singular and plural form.

For Example- cat, person, note, table etc.

Uncountable nouns are the names of materials, substances, concepts, qualities and such things which are not individual objects and cannot be divided into separate elements. For example: milk, sugar, cake, kindness, love, water, smoke, electricity. Uncountable nouns are always treated as singular form and they use singular verb.

1.4 Noun Usage

DDE, GJUS&T, Hisar 342 | English (Compulsory) BA-101

1.4.1: Number: Singular and Plural Noun

A noun that denotes single number is called singular For Example: boy, crow, bag,

A noun that denotes many numbers is called plural. For Example: boys, crows, bags,

General Rules to form plural: The plural form of most of the nouns can be obtained by adding‘s’ to the singular form. For Example: Boy, Tree, River, Cap Boys, Trees, Rivers, Caps . In nouns ending in s, sh, Ch., x, and z, the plural form is obtained by adding ‘es’ to the singular form.

For Example: Class, Bush, Watch, Box, Quiz, Fuzz Classes, Bushes, Watches, Boxes, Quizzes, Fuzzes

Note : In case of nouns ending in ‘z’ , if the noun has double z at the end in the singular form then ‘es’ is directly added to the singular form to form the plural ; but if the singular form has single ‘z’ at the end then ’z’ is added to the singular form before adding ‘es’ to form the plural i) Nouns ending in ‘y’ and preceded by consonant form the plurals by changing ‘y’ to ‘I’ and adding ’es ’to the singular form.

For Example: Lady Duty Story Berries, Ladies Duties Stories Berries ii) In nouns ending in ‘o’, plurals are formed by adding ‘es’ or ‘s’ to the singular form.

For Example: Mango Buffalo Piano Ratio

Mangoes, Buffaloes, pianos, Ratios iii) Nouns ending in ‘f’ or ‘fe’ form their plural by changing ‘f or ‘fe’ to ‘v’ and adding ‘es’ to the singular form.

For Example: Mango, Buffalo, Piano, Ratio

For Example: Leaf Self Life, Leaves Selves lives

There are exceptions to this rule. Some instances are given below.

For Example: Roof Chief Dwarfs

DDE, GJUS&T, Hisar 343 | English (Compulsory) BA-101

Roofs Chiefs Dwarfs iv) In few noun plurals are formed by changing inside vowel of the singular form.

For Example: Man, Tooth Mouse Foot

Men Teeth Mice Feet

Note: The plural forms of compound nouns given below Mother-in-law Step-son Passer-by Commander-in-chief Mothers-in-law Step-sons Passers-by Commanders- in-chief.

1.4.2: Noun: Gender: Gender means sex distinction. In English There are two genders – Masculine and Feminine. The absence of gender implies Neuter gender.

A Noun that expresses maleness is called masculine gender. A Noun that expresses femaleness is called feminine gender. A Noun that can be used to express maleness as well as femaleness is called a common noun. A Noun that is used to denote a genderless thing is called a neuter noun.

1.4.3: General rules of changing gender

 Some nouns have specially assigned words for feminine gender. For Example: Boy Father Son Boar Cock Bull Girl Mother Daughter Sow Hen Cow  In some nouns the feminine form is obtained by adding ‘ess’ to the masculine form. For Example: Host Lion Priest Baron, Hostess Lioness Priestess Baroness  In some nouns the inner vowel is dropped and ’ess’ is added to the masculine form to get feminine form. For Example: Actor Master Waite Emperor, Actress Mistress Waitress Empress  Though the suffix ‘ess’ is commonly used to change the gender there are some irregular suffixes used to form the feminine form. For Example: Czar Hero Signor, Czarina Heroine Signor.

DDE, GJUS&T, Hisar 344 | English (Compulsory) BA-101

 Sometimes a word is placed before or after the masculine noun to form the feminine form. For Example: Bull- calf He-goat Grand-father Milk –man Cow-calf She-goat Grand-mother Milk-maid General Rules to form plural Mothers-in-law Step- sons Passers-by Commanders.

1.5 Further Main Body of the Text

1.5.1 Pronoun: Persons

Pronoun is a word used in the place of a noun.

For Example: Neeta woke up late. Neeta slept late at night.

Neeta woke up late, because she slept late at night. Here ‘She’ is used instead of Neeta, so it is a Pronoun. Since nouns follow number and gender, pronouns too follow them. ‘He’ is masculine pronoun, ‘she’ is feminine and ‘it’ is neuter one ‘.’ He, She, It, I, singular pronouns while ‘we’, ‘our’, ‘us’, ‘they’ are plural pronouns. When we talk to others, we refer to ourselves as ‘I’ or ‘we’; the person to whom we are talking as you and the person or thing about we are talking as ‘he’, ’she ’or ‘it’. This is called, persons. Thus, the relationship between the addresser, the addressee and the subject of the address is called persons. There are three persons in English language. See the table below.

Persons Singular Plural

1st I we

2nd you

3rd he they, She, They, It, they

1.5.2 - Gender of the pronoun

 First Person Pronouns: Masculine or Feminine  Second Person Pronouns: Masculine or Feminine  Third Person Pronouns: He –Masculine

DDE, GJUS&T, Hisar 345 | English (Compulsory) BA-101

 She-Feminine

It-Neuter

They- Any gender can be applied as per the specification.

1.5.3- Types of Pronoun

a. Personal Pronoun

The pronouns that denote various persons are called personal pronoun. Generally, ‘it’ substitutes a noun representing inanimate object, so it is called impersonal pronoun.

Read the following sentences carefully to understanding the use of impersonal pronouns

It was at school that the two friends met.

It was raining heavily.

Note that ‘it’ does not work as a substitute for any noun.

b. Reflexive Pronoun: When ‘self’ is added to personal pronouns, compound personal pronouns are formed.

For Example- Myself, himself depending upon the use of the compound personal pronouns, there are two types.

Read the following sentences carefully. i) ram killed himself in despair. ii) At last they found themselves sitting in the stadium.

Here you will notice that, the compound personal pronouns are the objects of the respective verbs and they refer to the respective subjects. Here, ‘himself’ refers to ‘he’ and ‘ourselves’ refer to ‘we’. Thus, they help to reflect the action back to the subject. Hence, they are called Reflexive Pronouns.

DDE, GJUS&T, Hisar 346 | English (Compulsory) BA-101

c. Emphatic Pronoun: Here you will notice that compound personal pronouns (words in bold) are used to emphasize the action; and hence they are called Emphatic pronouns.

I myself saw the girl yesterday.

You yourself have to do it.

d. Demonstrative Pronoun: The pronouns that are used to refer to and point out the objects are called Demonstrative Pronouns.

For Example:

i) That is his house.

ii) These are my options for the extracurricular activities.

iii) Both the dresses are of reasonable price but this one is cheaper than that.

Here ‘that’, ‘these’ and’ both’ point out the objects that are the subjects of the respective sentences. e. Indefinite Pronoun: In all the above given sentences,’ one’, ’none’, ’they’, ’someone’ refer to person or persons in general but they do not stand for a particular person or persons. Hence, they are called Indefinite pronouns. Thus, the pronouns that refer to a person, persons or things in general are called Indefinite pronouns

Read the following sentences carefully. a) One should exercise regularly to stay healthy. b) None of her arguments were true. c) They believed him to be a generous man. d) Someone should come forward.

DDE, GJUS&T, Hisar 347 | English (Compulsory) BA-101

Note: In the sentence number (d) above, you will notice that ‘someone’ is a compound word and it is called compound indefinite pronoun. Other such pronouns are ‘anyone, ’anybody’, ’anything’, ’everyone’, ‘everybody’, ’everything’ ’none’,’ ‘nobody’, ’nothing’, ’somebody’, ’something’ ’another’. f. Distributive Pronoun: Here you will notice that ‘each’, ’either’, ’neither’ refer to the persons or things, one at a time; and hence they are called Distributive Pronouns. Thus, the pronouns that are used to denote singular person or thing one at a time are called Distributive Pronouns. They always use singular verb.

 Each of the boys took his turn on the guard duty.  Either of us should go at the railway station.  Neither of the key was fitting the lock. g. Relative Pronoun: Thus, the pronoun that refers to or relates to the antecedent is called Relative Pronoun ‘who’, ’whom’, ’what’, ’which’, ’whose’ and ‘that’ are the Relative Pronouns. Adding ‘ever’ and ‘so ever’ to the Relative Pronouns, (except ‘that’) you get Compound Relative Pronouns, ’whosoever’, ’whoever’, ’whomsoever’, ’whichever,’, ‘whatsoever’, ’whatever’.

Seema is the girl. We met Seema at the mall yesterday.

Seema is the girl whom we met at the mall yesterday

. Here you will observe that ‘whom, the pronoun stands for ‘Seema’ the antecedent.

For Example: Whoever comes first will get the seat.

1.6 Check Your Progress

Pick out Nouns and Pronouns in the following sentences.

1. Recognize the kinds of Noun-

a) An apple was lying on the table. b) Love begets love. c) We cannot live without Air.

DDE, GJUS&T, Hisar 348 | English (Compulsory) BA-101

d) The jury has given its verdict. e) The Hindus regards Krishna as an incarnation of Lord .

2. Use appropriate Pronoun-

a) When I have guests, I myself cook dinner for …………………… b) When she realized her mistake, she laughed at …………………… c) Susie prevented me from entering ……………. room. d) She yelled at ………………………… without any provocation. e) That is …………………………

3. Recognize Noun in the sentences-

a) They arrived early but at the wrong station. b) We counted only six different colours in the rainbow. c) The man was trying to steal a horse with a cart full of apple.

1.7 Summary

• A noun is a word used to name a person, animal, place, thing, and abstract idea.

• A common noun is a noun referring to a person, place, or thing in a general sense - usually, you should write it with a capital letter only when it begins a sentence.

• A concrete noun is a noun which names anything (or anyone) that you can perceive through your physical senses: touch, sight, taste, hearing, or smell.

• A countable noun (or count noun) is a noun with both a singular and a plural form, and it names anything (or anyone) that you can count.

• A collective Noun is a noun naming a group of things, animals, or persons.

• A personal Pronoun refers to a specific person or thing and changes its form to indicate person, number, gender, and case.

• A demonstrative pronoun points to and identifies a noun or a pronoun.

DDE, GJUS&T, Hisar 349 | English (Compulsory) BA-101

• An indefinite pronoun is a pronoun referring to an identifiable but not specified person or thing. An indefinite pronoun conveys the idea of all, any, none, or some.

1.8 Keywords

 Noun: It is name of person, place or things.  Interrogative Pronoun: Used to ask questions.  Intensive Pronoun: Used to emphasise its antecedent.

1.9 Self- Assessment Questions (SAQ)

State whether the following statements are true or false:

a) ‘He’ is an objective pronoun. b) Interrogative pronoun used to ask questions. c) Whomever is an Interrogative pronoun. d) Myself is a Reflexive pronoun.

1.10 Answers to Check your progress:

Ans.1

a) (Pen – Common Noun; table – Common Noun) b) (Love – Abstract Noun) c) (Water – Material Noun) d) (Jury – collective noun; verdict – Abstract Noun) e) (Hindus – Proper Noun; Krishna – Proper Noun; Incarnation – Common Noun; Vishnu – Proper Noun)

Ans.2

a) When I have guests, I myself cook dinner for them. b) When she realized her mistake, she laughed at herself. c) Sam prevented me from entering his room. d) She yelled at me without any provocation. e) That is mine.

DDE, GJUS&T, Hisar 350 | English (Compulsory) BA-101

Ans.3

a) They arrived early but at the wrong station. b) We counted only six different colours in the rainbow c) The man was trying to steal a horse with a cart full of apple

1.11 References /Suggested Readings:

 Books English for Competitive Exams, By DR. R.P. BHATNAGAR  Unique Quintessence of General English, Edited by DR. S. Sen & Others and revised by DR.G.S. MANSUKHANI.  A background to the Study of English Literature, D.K. Patnaik, Swastik Publications Online links http://www.writingcentre.uottawa.ca/hypergrammar/nounchar.html  http://grammar.ccc.commnet.edu/grammar/plurals.htm

DDE, GJUS&T, Hisar 351 | English (Compulsory) BA-101

Subject: English

Course Code- B.A.-101 Author: Dr. Astha Gupta

Lesson- 2 Editor: Dr. Shakuntla Devi

Introduction to Verbs and Adverbs (Part of Speech)

Structure of Lesson

2.1 Learning Objectives

2.2 Introduction

2.3 Main Body of the Text

2.3.0 Verbs and Its Classification

2.3.1 Helping Verbs

2.3.2 Main verbs

2.3.3 Transitive and Intransitive verbs

a. Transitive Verbs b. Intransitive Verbs

2.3.4 Linking Verbs

2.3.5 Dynamic and Stative verbs

2.3.6 Regular and Irregular Verbs

2.4 Application-Verbs and Tenses

2.4.1 Verbs and Tenses Usage

a. Present Indefinite Tense/ Simple Present b. Present Continuous Tense c. Present Perfect Tense

DDE, GJUS&T, Hisar 352 | English (Compulsory) BA-101

d. Present Perfect Continuous Tense e. Past Indefinite Tense/ Simple Past f. Past Continuous Tense g. Past Perfect Tense h. Past Perfect Continuous Tense i. Future Indefinite Tense/ Simple Future j. Future Continuous Tense k. Future Perfect Tense l. Future Perfect Continuous Tense 2.5 Further Main Body of the Text

2.5.0 Adverbs and Its Classification

2.5.1 Adverbs 2.5.2 Adverbs of Manner 2.5.3 Adverbs of Place or Location 2.5.4 Adverbs of Time 2.5.5 Adverbs of Degree 2.6 Usage of Adverbs 2.6.1 Adverbs Modifying Adjectives 2.6.2 Adverbs Modifying Adverbs 2.6.3 Adverbs Modifying Nouns 2.6.4 Adverbs Modifying Noun Phrases 2.6.5 Adverbs Modifying Determiners, Pronouns

2.7 Check Your Progress

2.8 Summary

2.9 Keywords

2.10 Self-Assessment Questions (SAQs)

2.11 Answers to check your progress

DDE, GJUS&T, Hisar 353 | English (Compulsory) BA-101

2.12 References/ Suggested Readings

2.1 Learning Objectives

After studying this unit, you will be able to:

 Know about verbs and adverbs  Explain helping verbs and main verbs  Explain present, past and future tenses.

2.2 Introduction

The verb is king in English. The shortest sentence contains a verb. You can make a one-word sentence with a verb, for example: “Stop!” You cannot make a one -word sentence with any other type of word. Verbs are sometimes described as “action words”. This is partly true. Many verbs give the idea of action, of “doing” something. For example, words like run, fight, do and works all convey action. But some verbs do not give the idea of action; they give the idea of existence, of state, of being”. For example, verbs like be, exist, seem and belong all convey state.

A verb always has a subject. (In the sentence “John speaks English”, John is the subject and speaks is the verb.) In simple terms, therefore, we can say that verbs are words that tell us what a subject does or is; they describe:

• action (Ram plays football.)

• state (Anthony seems kind.)

There is something very special about verbs in English. Most other words (adjectives, adverbs, prepositions etc. do not change in form (although nouns can have singular and plural forms). But almost all verbs change in form. For example, the verb to work has five forms:

• to work, work, works, worked, working

DDE, GJUS&T, Hisar 354 | English (Compulsory) BA-101

Of course, this is still very few forms compared to some languages which may have thirty or more forms for a single verb.

2.3 Main Body of the Text

2.3.0 Verbs and Its Classification

A word or phrase that describes an action, condition, or experience:

The words "run", "keep", and "feel" are all verbs.

Verbs Classification. We divide verbs into two broad classifications:

2.3.1 Helping Verbs –

Helping verbs have no meaning on their own. They are necessary for the grammatical structure of a sentence, but they do not tell us very much alone. We usually use helping verbs with main verbs. They “help” the main verb (which has the real meaning). There are only about 15 helping verbs in English, and we divide them into two basic groups:

Notes - Helping verbs are also called “auxiliary verbs. Primary he lping verbs (3 verbs).These are the verbs be, do, and have.

Note that we can use these three verbs as helping verbs or as main verbs. On this page we talk about them as helping verbs. We use them in the following cases:

• be

to make continuous tenses (He is watching TV.)

to make the passive (Small fish are eaten by big fish.)

• have

to make perfect tenses (I have finished my homework.)

• do

to make negatives (I do not like you.)

DDE, GJUS&T, Hisar 355 | English (Compulsory) BA-101

to ask questions (Do you want some coffee?)

to show emphasis (I do want you to pass your exam.)

to stand for a main verb in some constructions (He speaks faster than she does.)

Modal helping verbs (10 verbs)

We use modal helping verbs to “modify” the meaning of the main verb in some way. A modal helping verb expresses necessity or possibility, and changes the main verb in that sense. These are the modal verbs:

• can, could

• may, might

• will, would,

• shall, should

• must

• ought to

Here are examples using modal verbs:

• I can’t speak Chinese.

• John may arrive late.

• Would you like a cup of coffee?

• You should see a doctor.

• I really must go now.

Semi-modal verbs (3 verbs)

• need

• dare

• used to

DDE, GJUS&T, Hisar 356 | English (Compulsory) BA-101

Task Write down 10 sentences containing helping verb and main verb and mention the types of verb.

Imagine that A stranger walks into your room and says:

• I can.

• People must.

• The Earth will.

Do you understand anything? Has this person communicated anything to you? Probably not! That’s because these verbs are helping verbs and have no meaning on their own. They are necessary for the grammatical structure of the sentence, but they do not tell us very much alone. We usually use helping verbs with main verbs. They “help” the main verb. (The sentences in the above examples are therefore incomplete. They need at least a main verb to complete the, there are only about 15 helping verbs.

2.3.2 Main Verbs

Main verbs have meaning on their own (unlike helping verbs). There are thousands of main verbs, and we can classify them in several ways:

Notes - Main verbs are also called “lexical verbs”.

Now imagine that the same stranger walks into your room and says:

• I teach.

• People eat.

• The Earth rotates.

Do you understand something? Has this person communicated something to you? Probably yes! Not a lot, but something. That’s because these verbs are main verbs and have meaning on their own. They tell us something. Of course, there are

DDE, GJUS&T, Hisar 357 | English (Compulsory) BA-101 thousands of main verbs. In the following table we see example sentences with helping verbs and main verbs.

Notice that all of these sentences have a main verb. Only some of them have a helping verb.

John likes coffee.

You lied to me.

They are happy.

The children are playing.

We must go now.

I do not want any.

2.3.3 Transitive and intransitive verbs

A transitive verb takes a direct object: Somebody killed the President. An intransitive verb does not have a direct object: He died. Many verbs, like speak, can be transitive or intransitive.

Look at these examples: a. Transitive:

• I saw an elephant.

• We are watching TV.

• He speaks English. b. Intransitive:

• He has arrived.

• John goes to school.

2.3.4 Linking verbs

DDE, GJUS&T, Hisar 358 | English (Compulsory) BA-101

A linking verb does not have much meaning in itself. It “links” the subject to what is said about the subject. Usually, a linking verb shows equality (=) or a change to a different state or place (>). Linking verbs are always intransitive (but not all intransitive verbs are linking verbs).

• Mary is a teacher. (Mary = teacher)

• Tara is beautiful. (Tara = beautiful)

• That sounds interesting. (that = interesting)

• The sky became dark. (the sky > dark)

• The bread has gone bad. (bread > bad)

2.3.5 Dynamic and stative verbs

Some verbs describe action. They are called “dynamic”, and can be used with continuous tenses. Other verbs describe state (non-action, a situation). They are called “stative”, and cannot normally be used with continuous tenses (though some of them can be used with continuous tenses with change in meaning).

Dynamic verbs (examples):

• hit, explode, fight, run, go

Stative verbs (examples):

• be

• like, love, prefer, wish

• impress, please, surprise

• hear, see, sound

• belong to, consist of, contain, include, need

• appear, resemble, seem

2.3.6 Regular and Irregular Verbs

DDE, GJUS&T, Hisar 359 | English (Compulsory) BA-101

This is more a question of vocabulary than of grammar. The only real difference between regular and irregular verbs is that they have different endings for their past tense and past participle forms. For regular verbs, the past tense ending and past participle ending is always the same: . For irregular verbs, the past tense ending and the past participle ending is variable, so it is necessary to learn them by heart. regular verbs: base, past tense, past participle

• look, looked, looked

• work, worked, worked irregular verbs: base, past tense, past participle

• buy, bought, bought

• cut, cut, cut

• do, did, done

 Regular verbs-

English regular verbs change their form very little (unlike irregular verbs). The past tense and past participle of regular verbs end in – ed, for example: work, worked, worked But you should note the following points:

1 Some verbs can be both regular and irregular, for example: learn, learned, learned learn, learnt, learnt.

2. Some verbs change their meaning depending on whether they are regular or irregular, for example “to hang”: regular hang, hanged, hanged to kill or die, by dropping with a rope around the neck irregular hang, hung, hung to fix something (for example, a picture) at the top so that the lower part is free

3. The present tense of some regular verbs is the same as the past tense of some irregular verbs: regular found, founded, founded

DDE, GJUS&T, Hisar 360 | English (Compulsory) BA-101 irregular find, found, found

 Irregular verbs –

Irregular verbs are an important feature of English. We use irregular verbs a lot when speaking, less when writing. Of course, the most famous English verb of all, the verb “to be”, is irregular. What is the difference between regular verbs and irregular verbs? Base Form Past Simple, Past Participle With regular verbs, the rule is simple...

The past simple and past participle finish finished always end in-ed: stop stopped, stopped work, worked, worked

But with irregular verbs, there is no rule...

Sometimes the verb changes completely: sing, sang, sung

Sometimes there is “half” a change: buy, bought, bought

Sometimes there is no change: cut, cut, cut.

DDE, GJUS&T, Hisar 361 | English (Compulsory) BA-101

2.4 Application- Verbs and Tenses

2.4.1 Verbs and Tenses Usage a. Present Indefinite Tense/ Simple Present -Subject + main verb (V1)

Ram plays cricket. Rita swims.

Negative - Subject + don’t/ doesn’t main verb (V1)

Ram does not play cricket.

They do not swim.

b. Present Continuous Tense - Subject + (Is/Are) + main verb + ing.

She is staying in London.

Rita is playing.

Negative - Subject + (Is/Are) + not + main verb + ing.

Sita is not staying in London.

They are not playing football. c. Present Perfect Tense -Subject + auxiliary verb (Has/Have) + past participle (V3)

I have done.

She has played tennis.

Negative - Subject + auxiliary verb (Has/Have) + not + past participle (V3)

I have not done.

She has not played tennis. d. Present Perfect Continuous Tense - subject + Has/have + auxiliary verb (been) + main verb + ing

I have been doing.

You have been talking too much.

DDE, GJUS&T, Hisar 362 | English (Compulsory) BA-101

Negative - Subject + Has/have + not + auxiliary verb (been) + main verb + ing

You had not been playing tennis.

It had not been working well. e. Past Indefinite Tense/ Simple Past - Subject + V2

I sang.

He went to school.

Negative - Subject + did not + V1

He did not come.

They did not speak English. f. Past Continuous Tense - Subject + auxiliary verb BE (Was/Were) + main verb + ing

I was doing. You were working

Subject + auxiliary verb BE (Was/Were) + not + main verb + ing

I was not doing.

They were not playing football. g. Past Perfect Tense - Subject + (Had) + past participle (V3)

I had sung.

Ram had finished his work.

Negative- Subject + (Had) + not + past participle (V3)

I had not sung.

Ram had not finished his work. h. Past Perfect Continuous Tense - Subject + Had + Been + Main verb + ing

I had been doing.

DDE, GJUS&T, Hisar 363 | English (Compulsory) BA-101

You had been playing tennis.

Negative - Subject +Had + not + Been + Main verb + ing

You had not been playing tennis.

Reema has not been running. i. Future IndefiniteTense/ Simple Future - subject + auxiliary verb (will) + main verb (V1)

I will do.

Mohan will play cricket.

Negative - Subject + auxiliary verb (will) not + main verb (V1)

She will not play. They will not play j. Future Continuous Tense - subject + auxiliary verb (will) + auxiliary verb BE + main verb +Ing

I will be doing.

You will be lying on a beach tomorrow.

Negative - subject + auxiliary verb (will) + not + auxiliary verb BE + main verb) + Ing

I will not be doing.

You will not be lying on a beach tomorrow. k. Future Perfect Tense - Subject +auxiliary verb (will) + auxiliary verb have/ + past participle (V3)

I will have done.

You will have forgotten me by then.

Negative - Subject + (will) + not have + past participle (V3)

I will not have done.

DDE, GJUS&T, Hisar 364 | English (Compulsory) BA-101

She will not have gone to school. l. Future Perfect Continuous Tense - Subject + will + have + auxiliary verb been + main verb + Ing

I will have been doing.

You will have been travelling for two days.

Negative - Subject + will + not + have + auxiliary verb been + main verb + Ing

I will not have been doing.

You will not have been travelling for two days.

2.5 Further Main Body of the Text

2.5.0 Adverbs and Its Classification

2.5.1 Adverbs:

An adverb is a word that modifies the meaning of a Verb; an Adjective; another adverb; a Noun or Noun Phrase; Determiner; a Numeral; a Pronoun; or a Prepositional Phrase and can sometimes be used as a Complement of a Preposition.

Adverbs Spelling Notes-

• Adjectives ending-l still take -ly; careful-carefully.

• Adjectives ending-y change to-ily; lucky-luckily.

• Adjectives ending “ble”, change to-bly responsible-responsibly.

2.5.2 Adverbs of Manner-

Adverbs of manner modify a verb to describe the way the action is done.

Example: She did the work carefully.

(‘Carefully’ modifies the verb to describe the way the work was done, as opposed to quickly, carelessly, etc.)

DDE, GJUS&T, Hisar 365 | English (Compulsory) BA-101

2.5.3 Adverbs of Place or Location-

Adverbs of place show where the action is done.

Example: They live locally.

2.5.4 Adverbs of Time-

Adverbs of time show when an action is done, or the duration or frequency. Example: He did it yesterday. (When)

They are permanently busy. (Duration)

She never does it. (Frequency)

2.5.5 Adverbs of Degree-

Adverbs of degree increase or decrease the effect of the verb.

Example: I completely agree with you. (This increases the effect of the verb, whereas ‘partially would decrease it.

2.6 Usage of Adverbs

2.6.1 Adverbs Modifying Adjectives-

An adjective can be modified by an adverb, which precedes the adjective, except ‘enough’ which comes after.

DDE, GJUS&T, Hisar 366 | English (Compulsory) BA-101

Example: That’s really good.

It was a terribly difficult time for all of us. It wasn’t good enough. (‘Enough’ comes after the adjective.)

2.6.2 Adverbs Modifying Adverbs- An adverb can modify another. As with adjectives, the adverb precedes the one it is modifying with ‘enough’ being the exception again.

Example: She did it really well.

He didn’t come last night, funnily enough.

Task How adverb is different with adjective, explain with example.

2.6.3 Adverbs Modifying Nouns-

Adverbs can modify nouns to indicate time or place. Example: The concert tomorrow

Example: The room upstairs

2.6.4 Adverbs Modifying Noun Phrases-

Some adverbs of degree can modify noun phrases.

Example: We had quite a good time.

They’re such good friends. Quite; rather; such; what (What a day!) can be used in this way.

2.6.5 Adverbs Modifying Determiners, Pronouns-

Adverbs such as almost; nearly; hardly; about, etc., can be used:

Example: Almost everybody came in the end.

2.7 Check your progress- a. Do exercises related to verb and adverb-

1. She saw a ghost. (Transitive/Intransitive)

2. My vase dropped and broke into pieces. (Transitive/Intransitive)

DDE, GJUS&T, Hisar 367 | English (Compulsory) BA-101

3. The deer _____ fighting.

4. The natives of this island _____ a friendly people.

5.A pair of gloves _____ what they have been looking for the whole hour.

6. He was taken _____ in the middle of the night. (bad, badly)

7. My mother works _____ to support the family. (hard, hardly)

8. I carried the statue _____ in both hands. (careful, carefully) b. Choose the most suitable adverb from the following to fill in each blank:

across, anywhere, around, backwards, over

1. In the park, you can go _____ and find a place to sit.

2. We thought it was fun to swim _____ the river.

3. They are there but we are not going _____ to talk to them.

4. There were many people walking _____ in the park. c. Choose the most suitable adverb from the following to fill in each blank:

always, ever, just, nearly, never, only, slightly, still, unusually, quietly.

1. We hardly_____ see him go to church.

2. My father is _____late for work. d. Place the adverbs in bracket in the correct position in the sentence.

1. We walked to get there on time. (quickly)

2. The rain had stopped when I arrived. (already)

3.I have finished my exercise book. (nearly).

2.8 Summary

• The verb is king in English. The shortest sentence contains a verb.

DDE, GJUS&T, Hisar 368 | English (Compulsory) BA-101

• Most adverbs in English are forme d by adding -ly to an Adjective. An adverb is a word that modifies the meaning of a Verb; an Adjective; another adverb; a Noun or Noun Phrase; Determiner; a Numeral; a Pronoun; or a Prepositional Phrase and can sometimes be used as a Complement of a Preposition.

2.9 Keywords

Auxiliary verbs: Helping verbs also called ‘Auxiliary verbs.

Lexical verbs: Main verbs also called ‘Lexical verbs.

Ought to: It is a modal helping verb.

Used to: It is semi-modal helping verb and partly like main verb.

Adverb: The word that modifies the meaning of a verb.

2.10 Self- Assessment Questions (SAQs)

– State whether the following statements are true or false:

1. Helping verb is also called ‘lexical verb, why?

2 Difference between Main Verb and Helping Verb?

3. What is adverb? Explain with examples.

4. What is others name of helping verb and main verb?

5. Explain different types of verbs.

2.11 Answers to Check Your Progress- a. 1. (Transitive verbs – object: ghost)

2. (Intransitive verbs – no object)

3. The deer are fighting.

4. The natives of this island are a friendly people.

5. A pair of gloves is what they have been looking for the whole hour.

DDE, GJUS&T, Hisar 369 | English (Compulsory) BA-101

6. He was taken bad in the middle of the night

7. My mother works hard to support the family.

8. I carried the statue carefully in both hands.5 b. - Ans. Choose the most suitable adverbs from the following to fill in each blank:

across, anywhere, around, backwards, over .

1. In the park, you can go anywhere and find a place to sit.

2. We thought it was fun to swim across the river.

3. They are there but we are not going over to talk to them.

4. There were many people walking around in the park c. Ans. Choose the most suitable adverbs from the following to fill in each blank:

always, ever, just, nearly, never, only, quiet, slightly, still, unusually

1. We hardly ever see him go to church.

2. My father is never late for work. d. Place the adverbs in bracket in the correct position in the sentence answers. 1.We walked quickly to get there on time.

2. The rain had already stopped when I arrived.

3. I have nearly finished my exercise book.

2.12 References / Suggested Readings-

 Books English for Competitive Exams, By DR. R.P. Bhatnagar  Unique Quintessence of General English, Edited by DR. S. Sen & Others and revised by DR. G.S. Man Sukhmani.

DDE, GJUS&T, Hisar 370 | English (Compulsory) BA-101

 A Background to the Study of English Literature, D.K. Patnaik, Swastik Publications .  Online links o http://grammar.ccc.commnet.edu/grammar/adverbs.htm o http://www.english-language-grammar-guide.com/types-of-verbs.html

DDE, GJUS&T, Hisar 371 | English (Compulsory) BA-101

Subject: English

Course Code- B.A.-101 Author: Dr. Astha Gupta

Lesson – 3 Editor: Dr. Shakuntla Devi

Introduction to Adjectives and Prepositions (Part of Speech)

Structure of Lesson

3.1 Learning Objectives 3.2 Introduction 3.3 Main Body of the Text

3.3.0 Adjectives and Its Classification

3.3.1 Adjectives

3.3.2 Kinds of Adjectives

a. Proper Adjective b. Descriptive Adjective c. Quantitative Adjective d. Numeral Adjective e. Demonstrative Adjective f. Distributive Adjectives g. Interrogative Adjectives h. Exclamatory Adjectives i. Possessive Adjective j. Emphasising Adjective 3.3.3 Comparison of Adjectives 3.3.4 Position of Adjectives 3.3.5 Degrees of Adjectives

3.4 Usage of Adjectives

DDE, GJUS&T, Hisar 372 | English (Compulsory) BA-101

3.4.1 Adjectives subject to subjective measure 3.4.2 Capitalizing Proper Adjectives 3.4.3 Collective Adjectives 3.4.4 Adjectival Opposites

3.5 Further Main Body of the Text

3.5.0 Prepositions and Its Types

3.5.1 Prepositions

3.5.2 Prepositions of Time: at, on, and in

3.5.3 Prepositions of Place: at, on, and in

3.5.4 Prepositions of Location: in, at, and on and No Preposition

3.5.5 Prepositions of Time: for and since

3.6 Preposition and Its Usage

3.6.1 Prepositions with Nouns, Adjectives, and Verbs

3.6.2 Unnecessary Prepositions-

3.6.3 Important Words with Preposition

3.7 Check Your Progress

3.8 Summary

3.9 Keywords

3.10 Self-Assessment Questions (SAQs)

3.11 Answers to check your progress

3.12 Answers to check your progress

3.13 References/ Suggested Readings

DDE, GJUS&T, Hisar 373 | English (Compulsory) BA-101

3.1 Learning Objectives

After studying this unit, you will be able to:

• Explain about parts of speech

• Understand the adjectives

• Use preposition in the sentences.

3.2 Introduction-

Based on their use and functions, words are categorized into several types or parts of speech. The parts of speech explain how a word is used in a sentence. There are eight parts of speech such as Nouns, pronouns, adjectives, verbs, adverbs, preposition, conjunction and interjection. Adjective and Preposition are the important parts of grammar to qualify noun and pronoun. Adjective is a word, when used with a noun or pronoun add something to the meaning of noun or pronoun.

3.3 Main Body of the Text

3.3.0 Adjectives and Its Classification

3.3.1 Adjectives-

Adjectives are words that describe or modify another person or thing in the sentence. The Articles—a, an, and the—are adjectives. It is a word, when used with a noun or pronoun add something to the meaning of noun or pronoun.

1. She is a good singer.

2. I want some milk.

3. Radha gave me five rupees.

4. This is my book.

3.3.2 Kinds of Adjectives

1. Proper Adjective

DDE, GJUS&T, Hisar 374 | English (Compulsory) BA-101

2. Descriptive Adjective/Adjective of Quality

3. Quantitative Adjective/Adjective of Quality

4. Numeral Adjective/ Adjective of Number

5. Demonstrative Adjective

6. Distributive Adjective

7. Interrogative Adjective

8. Exclamatory Adjective

9. Possessive Adjective

10. Emphasizing Adjective a. Proper Adjective:

These Adjectives are formed from proper Nouns. They always begin with a capital letter. For e.g. , I enjoy Italian food. b. Descriptive Adjective:

These adjectives show the quality or condition of a person or a thing.

Delhi is a big city.

I have a black dog.

She is a beautiful lady. c. Quantitative Adjective:

These adjectives tell us about the quantity or degree of a thing for eg. Any, all, whole, no, none, some, little, enough, sufficient, full, entire etc. d. Numeral Adjective:

Adjective of number show how many persons or things are meant, or in what order a person or thing stands. For e.g. one, two, first, second, all, none, some, any, several, sundry, certain, every, either, neither, each etc.

DDE, GJUS&T, Hisar 375 | English (Compulsory) BA-101 e. Demonstrative Adjective: These adjectives point out which person or thing is meant e.g. this, that, the se, those, such some, etc. f. Distributive Adjectives: These adjectives show that the persons or things are to be taken independently or in separate lots. E.g. each, every, either, neither etc. g. Interrogative Adjectives: What, which and whose, when, they are used with nouns to ask questions, are called Interrogative adjectives; as,

Whose pen is this?

Which road should we take? h. Exclamatory Adjectives: These adjectives are used in an exclamatory sense.

What a beautiful garden!

What a piece of art! i. Possessive Adjective: My, his, her, your, their, our, are possessive adjectives which indicate possession, when used before a noun to qualify it. j. Emphasising Adjectives: These adjectives are used before the noun to emphasize the noun. For e.g. He took his own car. Formation of Adjectives. Many Adjectives are formed from Nouns:

Noun Adjective Noun Adjective

Boy Boyish Dirt Dirty

Care Careful Courage Courageous

Sense Senseless Envy Envious

Silk Silken King Kingly

Gold Golden Gift Gifted

Play Playful Storm Stormy

Hope Hopeful Pardon Pardonable

DDE, GJUS&T, Hisar 376 | English (Compulsory) BA-101

Some Adjectives are formed from verbs:

Verb Adjective Verb Adjective

Tire Tireless Move Moveable

Talk Talkative Cease Ceaseless

Some Adjectives are formed from other Adjectives:

Adjective Adjective Adjective Adjective

Tragic Tragical White Whitish

Three Threefold Black Blackish

Sick Sick Brave Bravely

3.3.3 Comparison of Adjectives

Comparison of adjective can be shown by changing the forms of adjectives. Look at the following sentences:

1. Radha’s mango is sweet.

2. Ram’s mango is sweeter than Radha’s.

3. Gopal’s mango is the sweetest of all.

The Adjective sweet is said to be in positive degree.

The Adjective sweeter is said to be in comparative degree.

The Adjective sweetest is said to be in superlative degree.

Comparative degree is used when comparison is made between two things. Superlative degree is used when more than two things are compared.

Formation of Comparative and superlative:

Positive Comparative Superlative

Clever Cleverer Cleverest

DDE, GJUS&T, Hisar 377 | English (Compulsory) BA-101

Kind Kinder Kindest

Young Younger Youngest

Great Greater Greatest

Fast Faster Fastest

Large Larger Largest

Busy Busier Busiest

Easy Easier Easiest

Wealthy Wealthier Wealthiest

Beautiful More beautiful Most beautiful

Learned More learned Most learned

Difficult More difficult Most difficult

Good Better Best

Bad Worse Worst

Much (quantity) More Most

Many (Number) More Most

Little Less Least

Exercises:

Pick out the adjectives in the following sentences, also identify the type of adjective.

1. He is a man of few words.

2. Every student has a book.

3. Alwar is a small city.

4. He gave me ten books.

5. Raman is a good boy.

DDE, GJUS&T, Hisar 378 | English (Compulsory) BA-101

6. Ankita won second prize.

7. My brother lives in next house.

8. He was absent last week.

9. This boy is stronger than Ravi.

10. I have taught you many things

3.3.4 Position of Adjectives

Adverbs which often seem capable of popping up almost anywhere in a sentence, adjectives nearly always appear immediately before the noun or noun phrase that they modify. Sometimes they appear in a string of adjectives, and when they do, they appear in a set order according to category. When indefinite pronouns—such as something, someone, and anybody— are modified by an adjective, the adjective comes after the pronoun: Anyone capable of doing something horrible to someone nice should be punished.

3.3.5 Degrees of Adjectives:

Adjectives can express degrees of modification:

• Gladys is a rich woman, but Josie is richer than Gladys, and Sadie is the richest woman in town.

Notes- The degrees of comparison are known as the positive, the comparative, and the superlative.

(Actually, only the comparative and superlative show degrees.) We use the comparative for comparing two things and the superlative for comparing three or more things. Notice that the word than frequently accompanies the comparative and the word the precedes the superlative. The inflected suffixes-Er and- Est suffice to form most comparatives and superlatives, when a two-syllable adjective ends in

DDE, GJUS&T, Hisar 379 | English (Compulsory) BA-101

(happier and happiest); otherwise we use more and most when an adjective has more than one syllable.

Positive, Comparative, Superlative

Rich Richer Richest

Lovely lovelier loveliest

Beautiful, more beautiful, most beautiful

Certain adjectives have irregular forms in the comparative and superlative degrees:

Irregular Comparative and Superlative Forms good, better, best bad, worse, worst

Note- Premodifiers with Degrees of Adjectives Both adverbs and adjectives in their comparative and superlative forms can be accompanied by premodifiers, single words and phrases, that intensify the degree.

• We were a lot more careful this time.

• He works a lot less carefully than the other jeweller in town.

• We like his work so much better.

• You’ll get your watch back all the faster.

The categories in the following table can be described as follows:

(i) Determiners—articles and other limiters.

(ii) Observation—post determiners and limiter adjectives (e.g., a real hero, a perfect idiot)

3.4 Usage of Adjectives

3.4.1 Adjectives subject to subjective measure

(e.g., beautiful, interesting)

DDE, GJUS&T, Hisar 380 | English (Compulsory) BA-101

(iii) Size and Shape—adjectives subject to objective measure (e.g., wealthy, large, round)

(iv) Age—adjectives denoting age (e.g., young, old, new, ancient)

(v) Colour—adjectives denoting colour (e.g., red, black, pale)

(vi) Origin—denominal adjectives denoting source of noun (e.g., French, American, Canadian)

(vii) Material—denominal adjectives denoting what something is made of (e.g., woolen, metallic, wooden)

(viii) Qualifier—final limiter, often regarded as part of the noun (e.g., rocking chair, hunting cabin, passenger car, book cover) Determiner Observation Physical Description Origin Material Qualifier Noun .Size Shape Age Colour ,a beautiful old Italian touring car , an expensive antique silver mirror, four gorgeous long- red silk roses, stemmed, her short black hair , our big old English sheep, dog, those square wooden hat boxes that dilapidated little hunting cabin several enormous young American basketball players ,some delicious Thai food.

.3.4.2 Capitalizing Proper Adjectives:

When an adjective owes its origins to a proper noun, it should probably be capitalized. Thus we write about Christian music, French fries, the English Parliament, the Ming Dynasty, a Faulknerian style, democracy. Some periods of time have taken on the status of proper adjectives: the Nixon era, a Renaissance/Romantic/Victorian poet (but a contemporary novelist and medieval writer). Directional and seasonal adjectives are not capitalized unless they’re part of a title: We took the northwest route during the spring thaw. We stayed there until the town’s annual Fall Festival of Small Appliances.

3.4.3 Collective Adjectives

DDE, GJUS&T, Hisar 381 | English (Compulsory) BA-101

When the definite article is combined with an adjective describing a class or group of people, the resulting phrase can act as a noun: the poor, the rich, the oppressed, the homeless, the lonely, the unlettered, the unwashed, the gathered, the dear departed. The difference between a Collective Noun (which is usually regarded as singular but which can be plural in certain contexts) and a collective adjective is that the latter is always plural and requires a plural verb:

• The rural poor have been ignored by the media.

• The rich of Connecticut are responsible.

• The young at heart are always a joy to be around.

3.4.4 Adjectival Opposites –

The opposite or the negative aspect of an adjective can be formed in a number of ways. One way, of course, is to find an adjective to mean the opposite—an antonym. The opposite of beautiful is ugly, the opposite of tall is short. A thesaurus can help you find an appropriate opposite. Another way to form the opposite of an adjective is with a number of prefixes. The opposite of fortunate is unfortunate, the opposite of prudent is imprudent, the opposite of considerate is inconsiderate, the opposite of honorable is dishonorable, the opposite of alcoholic is non- alcoholic, the opposite of being properly filed is misfiled. If you are not sure of the spelling of adjectives modified in this way by prefixes (or which is the appropriate prefix), you will have to consult a dictionary, as the rules for the selection of a prefix are complex and too shifty to be trusted. The meaning itself can be tricky; for instance, flammable and inflammable mean the same thing.

A third means for creating the opposite of an adjective is to combine it with less or least to create a comparison which points in the opposite direction. He swims well. He knows only too well who the murderer is. However, when using a linking verb or a verb that has to do with the five human senses, you want to use the adjective instead.

DDE, GJUS&T, Hisar 382 | English (Compulsory) BA-101

• We were annoyed by the elephants.

• We were bored by the ringmaster.

 Adjectives: The most common of the so-called a- adjectives are ablaze, afloat, afraid, aghast, alert, alike, alive, alone, aloof, ashamed, asleep, averse, awake, aware. These adjectives will primarily show up as predicate adjectives (i.e., they come after a linking verb).

• The children were ashamed.

• The professor remained aloof.

• The trees were ablaze.

Occasionally, however, you will find a- adjectives before the word they modify: the alert patient, the aloof physician. Most of them, when found before the word they modify, are themselves modified: the nearly awake student, the terribly alone scholar. And a- adjectives are sometimes modified by “very much”: very much afraid, very much alone, very much ashamed, etc.

3.5 Further Main Body of the Text

3.5.0 Prepositions and Its Types

3.5.1 Prepositions-

a) The dog sat under the table. b) The book is on the table.

In sentence (a) ‘under’ join the words ‘sat’ and ‘the table’ and shows the relationship between them. Also ‘on’ join the words ‘is’ and’ the table. Thus, a Preposition is defined as a word that is written before a noun or pronoun, and which shows the relation of the person or thing or quality denoted by that noun or pronoun to something else.

DDE, GJUS&T, Hisar 383 | English (Compulsory) BA-101

Prepositions: In, out, off, of, up, down, for, at, to, from, by, till, with, within, without, through , above, below, beneath, beside, beyond, between, underneath, across, outside.

Read the following sentences carefully. a) Come in b) The biscuits are in the jar.

Here in the first sentence, ’in’ gives the information about the verb ‘come’, so it is an adverb. But in the second sentence ‘in’ join the words ‘are’ and ‘the jar’ and it shows the relationship between them and therefore is –

A preposition describes a relationship between other words in a sentence. In itself, a word like “in” or “after” is rather meaningless and hard to define in mere words.

Notes- When you do try to define a preposition like “in” or “between” or “on,” you invariably use your hands to show how something is situated in relationship to something else.

Prepositions are nearly always combined with other words in structures called preposition phrases. Prepositional phrases can be made up of a million different words, but they tend to be built the same: a preposition followed by a determiner and an adjective or two, followed by a pronoun or noun (called the object of the preposition). This whole phrase, in turn, takes on a modifying role, acting as an adjective or an adverb, locating something in time and space, modifying a noun, or telling when or where or under what conditions something happened. Is it any wonder that prepositions create such troubles for students for whom English is a second language? We say we are at the hospital to visit a friend who is in the hospital. We lie in bed but on the couch. We watch a film at the theatre but on television. For native speakers, these little words present little difficulty, but try to learn another language, any other language, and you will quickly discover that prepositions are troublesome wherever you live and learn.

DDE, GJUS&T, Hisar 384 | English (Compulsory) BA-101

3.5.2 Prepositions of Time: at, on, and in

We use at to designate specific times.

The train is due at 12:15 p.m.

We use on to designate days and dates.

My brother is coming on Monday.

We’re having a party on the Fourth of July.

We use in for nonspecific times during a day, a month, a season, or a year.

She likes to jog in the morning.

It’s too cold in winter to run outside.

He started the job in 1971.

He’s going to quit in August.

3.5.3 Prepositions of Place: at, on, and in

We use at for specific addresses.

We use on to designate names of streets, avenues, etc.

Her house is on Tughlaq Road.

And we use in for the names of land-areas (towns, counties, states, countries, and continents).

She lives in Durham.

Durham is in Windham County.

Windham County is in Connecticut.

3.5.4 Prepositions of Location: in, at, and on and No Preposition

The bedroom home, the ceiling downstairs, the car, the library, the floor downtown, the class, the office, the horse inside.

DDE, GJUS&T, Hisar 385 | English (Compulsory) BA-101

*The library*, school*, the plane, outside, school, work, the train upstairs, uptown

* You may sometimes use different prepositions for these locations. Prepositions of Movement: to and No Preposition like use to, in order to, express movement toward a place. They were driving to work together. She’s going to the dentist’s office this morning. toward and towards are also helpful prepositions to express movement. These are simply variant spellings of the same word; use whichever sounds better to you. We’re moving toward the light. This is a big step towards the project’s completion. With the words home, downtown, uptown, inside, outside, downstairs, upstairs, we use no preposition.

Grandma went upstairs

Grandpa went home.

They both went outside.

3.5.5 Prepositions of Time: for and since

We use for when we measure time (seconds, minutes, hours, days, months, years).

He held his breath for seven minutes.

She’s lived there for seven years.

The British and Irish have been quarrelling for seven centuries.

DDE, GJUS&T, Hisar 386 | English (Compulsory) BA-101

3.6 Prepositions and Its Usage

3.6.1 Prepositions with Nouns, Adjectives, and Verbs-

Prepositions are sometimes so firmly wedded to other words that they have practically become one word. This occurs in three categories: nouns, adjectives, and verbs.

Summary - Nouns and Prepositions, approval of fondness for need for ,awareness of grasp of participation in, belief in hatred of reason for, concern for hope for respect for confusion about interest in success, in desire for love of understanding of Adjectives and Prepositions, afraid of fond of proud of angry at happy about similar to aware of interested in sorry for capable of jealous of sure of careless about made of tired of familiar with married to worried about Verbs and Prepositions apologize for give up prepare for ask about grow up study for ask for look for talk about belong to look forward to think about bring up look up trust in care for makeup work for C GT.;/A combination of verb and preposition is called a phrasal verb. The word that is joined to the verb is then called a particle. Please refer to the brief section we have

DDE, GJUS&T, Hisar 387 | English (Compulsory) BA-101 prepared on phrasal verbs for an explanation. Idiomatic Expressions with Prepositions.

• agree to a proposal, with a person, on a price, in principle

• argue about a matter, with a person, for or against a proposition

• compare to to show likenesses, with to show differences (sometimes similarities)

• correspond to a thing, with a person

• differ from an unlike thing, with a person

• live at an address, in a house or city, on a street, with other people

3.6.2 Unnecessary Prepositions

In everyday speech, we fall into some bad habits, using prepositions where they are not necessary. It would be a good idea to eliminate these words altogether, but we must be especially careful not to use them in formal, academic prose.

• The book fell off of the desk.

• He threw the book out of the window.

• She wouldn’t let the cat inside of the house. [or use “in”]

• Where did they go to?

• Put the lamp in back of the couch. [use “behind” instead]

• Where is your college at?

Prepositions in Parallel Form When two words or phrases are used in parallel and require the same preposition to be idiomatically correct, the preposition does not have to be used twice. The female was both attracted by and distracted by the male’s dance. Did u know? However, when the idiomatic use of phrases calls for different prepositions, we must be careful not to omit one of them. The children were interested in and disgusted by the movie. It was clear that this player could both

DDE, GJUS&T, Hisar 388 | English (Compulsory) BA-101 contribute to and learn from every game he played. He was fascinated by and enamored of this beguiling woman.

3.6.3 Important Words with Preposition abhorrence of abound with abounding in absolve from abundant with /on accord with acquiesce in acquit of adapted for (by nature) adapted to (purpose) admit of affinity between afflict with agree on (a matter or point) agree to (a proposal) agree with (a person, statement, opinion) aggravate by alien to amenable to angry at / about a thing

DDE, GJUS&T, Hisar 389 | English (Compulsory) BA-101 angry with a person (or at to stress the anger itself) associated with averse to/ from begin by doing something begin from a point begin with an act benefits of the benefactor benefits to the beneficiary bestow upon bored with or by (not of) capable of capacity for

Centre on change for (a thing) change with (a person) chary of cognizant of compare to (to show similarities) compare with (to note points of difference) compatible with comply with under the conditions confer on (to bestow upon)

DDE, GJUS&T, Hisar 390 | English (Compulsory) BA-101 confer with confide in (trust in) confide to (entrust to) conform to in conformity with congratulate on connive at consequent upon content oneself with content others by contrast (v.) with contrast (n.) to or with convenient for (purpose) convenient to (person) conversant with correspond to (a thing; resemble) correspond with (a person) deficient in derogatory to die for (a cause) die of (an illness) differ from (be different) differ with (disagree)

DDE, GJUS&T, Hisar 391 | English (Compulsory) BA-101 different from disappointed of (what we cannot have) disappointed in (what we have) dissent from (not to) distaste for embark money in embark on a ship, career essential to exception to (a rule, statement) exempt from forbid to do free from or of glad at (a piece of news) glad of (a possession) guiltless of identical with impatient of (things) impatient with (persons) impervious to immune from an obligation immune to a disease incidental to inculcate upon someone (not with)

DDE, GJUS&T, Hisar 392 | English (Compulsory) BA-101 independent of indicative of ineligible for infected with infested with inflict upon influence over/ with (a person) influence on /upon/ in (persons or things) infuse into someone

(take the) initiative in

(on one’s own) initiative insensible to insight into inspired by instill into invest in a business invest with an office involve in irrespective of join in a game join with a person or thing labor at a task labour for a person, an end

DDE, GJUS&T, Hisar 393 | English (Compulsory) BA-101 labour in a cause labour under disadvantages live by labour live for riches martyr for (a cause) martyr to (a disease, etc.) at a moment’s notice on the spur of the moment negligent of (noun) oblivious of (not to) parallel with or to part from (persons) part with (things) perpendicular to point at a thing point to a fact possessed of wealth possessed with an idea prefer one to the other preferable to preference for prevail against (things) prevail upon (persons)

DDE, GJUS&T, Hisar 394 | English (Compulsory) BA-101 provide (someone) with (something) profuse in profusion of provide against ill luck provide for an emergency pursuant to in pursuance of recommend that she do (not “recommend her to do”) reconcile to (thing) reconcile with (person) regard as with reference to regard for a person with regard to a subject relevant to bring relief to suffering replace with (but “is replaced by “) report on or to (not into) resentful of responsible for (something) responsible to (someone) result from an event result in a failure

DDE, GJUS&T, Hisar 395 | English (Compulsory) BA-101 the result of an investigation find satisfaction in an improvement the satisfaction of knowing give satisfaction to a person satisfied of (fact) satisfied with (things) secure against attack secure from harm secure in a position sensitive to substitute for suggest that he do (not “suggest him to do”) susceptible to taste for (art, etc.) taste of (food) thirst for/after (knowledge) unconscious of (not to) at variance on topics at variance with a person

3.7 Check your progress

Fill in the blanks using correct Adjective:

1. Of the two sisters, Susie is the ……………………

DDE, GJUS&T, Hisar 396 | English (Compulsory) BA-101

Older oldest

2. Mumbai is …………………………. from Kochi than Bangalore.

Further farther

3. She could become an actor or a model – she chose the …………………….. option.

Later latter

4. She is still on good terms with her ……………………………. boyfriend.

Former then

5. I cannot walk any ………………………farther/further

2. Fill in the blanks with an appropriate preposition.

(1). The child ran ……………………….. the road.

Over across above

2. The news from …………………………. the borders is quite disquieting.

Along across over

3. I got up early …………………………the morning. on in at

4. He accused me ……………………. stealing his money.

About for of

5. I am afraid ………………………… spiders.

About of at

6. Do you agree …………………………… his decision to quit?

About with to

3.8 Summary

DDE, GJUS&T, Hisar 397 | English (Compulsory) BA-101

• Adjectives are words that describe or modify another person or thing in the sentence.

• If a group of words containing a subject and verb acts as an adjective, it is called an Adjective Clause.

• The most common of the so-called a- adjectives are ablaze, afloat, afraid, aghast, alert, alike, alive, alone, aloof, ashamed, asleep, averse, awake, aware.

• A combination of verb and preposition is called a phrasal verb.

3.9 Keywords

Articles: A, An, and The are the articles.

Degrees: Used to compare the adjectives.

Comparative: Used for comparing two things.

Superlative: Used for comparing three or more things.

Determiners: Articles and other limiters.

Qualifiers: Find limiter, often regarded as part of the noun.

A-Adjectives: The words ablaze, afloat, afraid, aghast, alert, alike, alive, alone, asleep etc.

3.10 Self- Assessment Test

Fill in the blanks:

1. Adjectives have ...... types of degrees.

2. Biggest is the ...... degree.

3. The word ...... comes after comparative degree.

4. Prepositions describes a ...... between other words in a sentence.

DDE, GJUS&T, Hisar 398 | English (Compulsory) BA-101

1. Write down all parts of speech and explain.

2. Define adjectives and explain with examples

3. What is preposition and how it is used?

3.11 Answers: Self -Assessment

1. three 2. superlative

3. than 4. relationship

3.12 Answers to check your progress

Adjectives-

1. Of the two sisters, Susie is the older.

2. Mumbai is farther from Kochi than Bangalore.

3. She was happy when she got the job but two weeks later she was fired.

4. She is still on good terms with her former boyfriend.

5. I cannot walk any farther / further.

Prepositions-

1. The child ran across the road.

2. The news from across the borders is quite disquieting.

3. I got up early in the morning.

4. He accused me of stealing his money.

5. I am afraid of spiders.

6. Do you agree with his decision to quit?

3.13 References/ Readings

 Books English for Competitive Exams, By DR. R.P. Bhatnagar

DDE, GJUS&T, Hisar 399 | English (Compulsory) BA-101

 Unique Quintessence of General English, Edited by DR. S. Sen & Others and revised by DR. G.S. Mansukh ani.  A Background to the Study of English Literature, D.K. Patnaik, Swastik Publications  Online links : o www.writingcentre.uottawa.ca/hypergrammar/preposit.html o http://www.dl.ket.org/latin2/grammar/ch34-degofadj.htm

DDE, GJUS&T, Hisar 400 | English (Compulsory) BA-101

Subject: English

Course Code- B.A.-101 Author: Dr. Astha Gupta

Lesson- 4 Editor: Dr. Shakuntla Devi

Introduction to Modals and Conjunctions

Structure of Lesson

4.1 Learning Objectives

4.2 Introduction

4.3 Main Body of the Text

4.3.0 Modals

4.3.1 Modals and Modal Phrases (Semi-Modals)

4.3.2 Ability/Lack of Ability

a. Present and Future: - can/can’t + base form of the verb

b. Past: could / couldn’t + base form of the verb

4.3.3 Possibility / Impossibility-can / q of the verb

4.3.4 Ask Permission / Give Permission

a. Can + Subject + base form of the verb (informal)

b. Could + subject + base form of the verb (polite)

4.3.5 Make a suggestion – To make a suggestion use:

Could + base form of the verb (informal)

4.3.6 May, Might

Formal Permission / Formal Prohibition

DDE, GJUS&T, Hisar 401 | English (Compulsory) BA-101

may / may not + base form of the verb

4.3.7 Polite Request

May + subject + base form of the verb

4.3.8 Polite Suggestion

might + base form of the verb

4.4 Modal Usage

4.4.1 Shall, Should, Ought to,

To Offer of Assistance or Polite Suggestion

a. (When you are quite sure of a positive answer)

Shall + subject + base form of the verb

b. (When you are not sure of a positive answer)

Should + subject + base form of the verbS

4.4.2 To Give Advice

should / ought to + base form of the verb

4.4.3 Necessity or Requirement

Present and Future: -must / have to / need to + base form of the verb

4.4.4 must + base form of the verb /To Persuade

4.4.5 Lack of Necessity

don’t /doesn’t /didn’t + have to + base form of the verb

4.4.6 Modals: Will / Would Polite Request or Statement

Will / Would + base form of the verb

4.4.7 List of modal verbs

DDE, GJUS&T, Hisar 402 | English (Compulsory) BA-101

4.4.8 Use of modal verbs

4.5 Further Main Body of the Text

4.5.0 Conjunctions and Its Types

4.5.1 Conjunctions

4.5.2. Conjunctions: Its Types

a) Coordinating Conjunctions

b) Subordinating Conjunctions

4.5.3 Coordinating Conjunctions

• and, but, or, yet, for, nor, so

4.5.4 Subordinating clause

after, before, since—

4.5.5 Common Subordinating Conjunctions

4.6 Usage of Conjunctions

4.6.1 Correlative Conjunctions

4.6.2 Conjunctive Adverbs

4.6.3 Other Conjunctions

4.6.4 Omitting That

4.7 Check Your Progress

4.8 Summary

4.9 Keywords

4.10 Self-Assessment Questions (SAQs)

4.11 Answers to Check your Progress

4.12 References/ Suggested Readings

DDE, GJUS&T, Hisar 403 | English (Compulsory) BA-101

4.1 Learning Objectives

After studying this unit, you will be able to:

• Use of Modals

• Use of conjunctions

• Write sentences using conjunctions.

4.2 Introduction. A modal is a type of auxiliary (helping) verb that is used to express: ability, possibility, permission or obligation. Modal phrases (or semi-modals) are used to express the same things as modals, but are a combination of auxiliary verbs and the preposition to. Coordinating

Conjunctions The simple, little conjunctions are called coordinating conjunctions (you can click on the words to see specific descriptions of each one Coordinating Conjunctions• and, but, or, yet, for, nor, so

(It may help you remember these conjunctions by recalling that they all have fewer than four letters. Also, remember the acronym FANBOYS: For-And-Nor-But-Or-Yet- So. Be careful of the words then and now; neither is a coordinating conjunction, so what we say about coordinating conjunctions’ roles in a sentence and punctuation does not apply to those two words). When a coordinating conjunction connects two independent clauses, it is often (but not always) accompanied by a comma:

4.3 Main Body of the Text

4.3.0 Modals

4.3.1 Modals and Modal Phrases (Semi-Modals)

A modal is a type of auxiliary (helping) verb that is used to express: ability, possibility, permission or obligation. Modal phrases (or semi-modals) are used to express the same things as modals, but are a combination of auxiliary verbs and the preposition to. The modals and semi-modals in English are:

Can/could/be able to

DDE, GJUS&T, Hisar 404 | English (Compulsory) BA-101

May/might

Shall/should

Must/have to

Will/would

Can, Could, Be Able To

Can, could and be able to are used to express a variety of ideas in English:

Modal verbs are used to express functions such as, Permission, Ability, Obligation, Prohibition, Lack of necessity, Advice, Possibility, Probability.

4.3.2 Ability/ Lack of Ability a. Present and Future: - can/can’t + base form of the verb

Tom can write poetry very well.

I can help you with that next week.

Lisa can’t speak French. am / is / are / will be + able to + base form of the verb am not/ isn’t / aren’t/ won’t be + able to + base form of the verb b. Past: could / couldn’t + base form of the verb

When I was a child, I could climb trees. was / were + able to + base form of the verb wasn’t / weren’t + able to + base form of the verb hasn’t / haven’t + been able to + base form of the verb

I wasn’t able to visit her in the hospital.

He hasn’t been able to get in touch with the client yet.

DDE, GJUS&T, Hisar 405 | English (Compulsory) BA-101

Note: Can and could do not take an infinitive (to verb) and do not take the future auxiliary will.

Correct: I can help you this afternoon.

Correct: I will (I’ll) be able to help you this afternoon.

4.3.3 Possibility / Impossibility-can / can’t + base form of the verb

You can catch that train at 10:43.

He can’t see you right now. He’s in surgery. could + base form of the verb

I could fly via if I leave the day before.

4.3.4 Ask Permission / Give Permission a. Can + Subject + base form of the verb (informal)

Can you lend me ten dollars?

Can + base form of the verb (informal)

You can borrow my car. b. Could + subject + base form of the verb (polite)

Could I have your number?

Could I talk to your supervisor please?

4.3.5 Make a suggestion – To make a suggestion use:

Could + base form of the verb (informal)

You could take the tour of the castle tomorrow.

Exercises: Can, Could, Be able to

Fill in the correct form of can, could or be able to as in the examples.

Berlin could not help his little brother with his homework yesterday.

DDE, GJUS&T, Hisar 406 | English (Compulsory) BA-101

Can I call you later tonight?

______Tony run long distances when he was a boy?

______you please call a tow truck for me? My car broke down. (polite)

The students ______to buy their textbooks today. The bookstore is all out of them.

Answers:

Could, aren’t able, Can,

4.3.6. May, Might

Formal Permission / Formal Prohibition may / may not + base form of the verb

You may start your exam now.

You may not wear sandals to work.

4.3.7 Polite Request

May I help you?

Possibility / Negative Possibility may/ might + base form of the verb

We may go out for dinner tonight. Do you want to join us?

Our company might get the order if the client agrees to the price. may not / might not + base form of the verb

Adam and Sue may not buy that house. It’s very expensive.

They might not buy a house at all.

To Make a Suggestion (when there is no better alternative) may as well / might as well + base form of the verb

You may as well come inside. John will be home soon.

DDE, GJUS&T, Hisar 407 | English (Compulsory) BA-101

We might as well take Friday off. There’s no work to be done anyway.

4.3.8 Polite Suggestion might + base form of the verb

Fill in the correct form of may or might as in the example.

May I sit here?

They ______finish the project on time. The main engineer is ill.

You _____ want to stop by the museum gift shop on your way out.

Answers: might not, might.

4.4 Modal Usage

4.4.1 Shall, Should, Ought to

To Offer of Assistance or Polite Suggestion a. (When you are quite sure of a positive answer)

Shall + subject + base form of the verb

Shall we go for a walk?

Note: Shall is only used with I or we. It is used instead of will only in formal English. b. (When you are not sure of a positive answer)

Should + subject + base form of the verb

Should I call a doctor?

A Prediction or Expectation that Something Will Happen should/shouldn’t + base form of the verb

The proposal should be finished on time. I shouldn’t be late. The train usually arrives on time.

4.4.2. To Give Advice

DDE, GJUS&T, Hisar 408 | English (Compulsory) BA-101 should / ought to + base form of the verb

You should check that document before you send it out.

You ought to have your car serviced before the winter.

To Give Advice (about something you think wrong or unacceptable) shouldn’t + base form of the verb

James shouldn’t teach him words like those.

Exercises: Should, Shouldn’t, Ought To

Fill in should, shouldn’t or ought in the following sentences as in the example.

He shouldn’t encourage such bad behaviour.

You _____ get your teeth cleaned at least once a year.

The house ______be ready to move into by next month. It’s almost finished.

Answers: Should, ought to

4.4.3 Necessity or Requirement

Present and Future: -must / have to / need to + base form of the verb

You must have a passport to cross the border.

Elisabeth has to apply for her visa by March 10th.

I need to drop by his room to pick up a book.

Past: -had to / needed to + base form of the verb

I had to work late last night.

I needed to drink a few cups of coffee in order to stay awake.

Note: have to and need to are often used in the same context, but many times, need to is used to express something that is less urgent, something in which you have a choice. Almost 100% Certain

DDE, GJUS&T, Hisar 409 | English (Compulsory) BA-101

4.4.4 must + base form of the verb -Thomas has lived in Paris for years. His French must be very good.

To Persuade must / have to + base form of the verb

You must try this wine. It’s excellent.

You have to visit us while you’re in town.

Prohibited or Forbidden must not / mustn’t + base form of the verb

You must not drive over the speed limit.

You mustn’t leave medicines where children can get to them.

4.4.5 Lack of Necessity don’t /doesn’t /didn’t + have to + base form of the verb

You don’t have to park the car. The hotel valet will do it for you.

Tim doesn’t have to go to school today. It’s a holiday.

You didn’t have to shout. Everyone could hear you. needn’t + base form of the verb

You needn’t worry about me. I’ll be fine.

Exercises: Must, have to, need to, Don’t Have to, Needn’t

Fill in the blanks with one of these Modals: must, must not, have to, has to, don’t have to, doesn’t have to, needn’t as in the examples.

There may be more than one correct answer.

Shira doesn’t have to drive to the airport. She’s going by taxi.

DDE, GJUS&T, Hisar 410 | English (Compulsory) BA-101

You must speak politely to the customers. will / won’t + base form of the verb

John will pick you up at 7:00am.

Beth won’t be happy with the results of the exam.

4.4.6 Modals: Will / Would Polite Request or Statement

Will / Would + base form of the verb

Will you please take the trash out?

Would you mind if I sat here?

I’d (I would) like to sign up for your workshop.

Habitual Past Action

Would/Wouldn’t + base form of the verb -When I was a child, I would spend hours playing with my train set. Peter wouldn’t eat broccoli when he was a kid. He loves it now.

Exercises: will, would

Fill in the blanks with one of the following words: will, won’t, would, wouldn’t.

Will you please help me lift this box?

I ______like to order the onion soup please.

The manager ______be pleased to hear that a customer slipped on the wet floor.

Answers: Would, won’t

Here are some characteristics of modal verbs: They never change their form. You can't add "s", "ed", "Ing"...They are always followed by an infinitive without "to" (. the bare infinitive.) They are used to indicate modality allow speakers to express certainty, possibility, willingness, obligation, necessity, ability

DDE, GJUS&T, Hisar 411 | English (Compulsory) BA-101

4.4.7 List of modal verbs - Here is a list of modal verbs: can, could, may, might, will, would, shall, should, must ,The verbs or expressions dare, ought to, had better, and need not, behave like modal auxiliaries to a large extent and may be added to the above list.

4.4.8 Use of modal verbs

Here is a list of modals with examples:

Modal Verb Expressing Example

Must Strong obligation You must stop when the traffic lights turn red.

Logical conclusion / He must be very tired. He's been working Certainty all day long.

Must not Prohibition You must not smoke in the hospital.

Can Ability I can swim.

Permission Can I use your phone please?

Possibility Smoking can cause cancer.

Could Ability in the past When I was younger, I could run fast.

Polite Permission Excuse me, could I just say something?

Possibility It could rain tomorrow!

May Permission May I use your phone please?

Possibility/ Probability It may rain tomorrow!

Might Polite Permission Might I suggest an idea?

Possibility/ Probability I might go on holiday to Australia next year.

DDE, GJUS&T, Hisar 412 | English (Compulsory) BA-101

Need Not Lack of I need not buy tomatoes. There are Necessity/Absence of plenty of tomatoes in the fridge. Obligation

Should/Ought 50 % Obligation I should / ought to see a doctor. I to have a terrible headache.

Advice You should / ought to revise your lessons

Logical Conclusion He should / ought to be very tired. He's been working all day long.

Had Better Advice You 'd better revise your lessons

Note- Modal verbs are followed by an infinitive without "to", also called the bare infinitive.

Examples:

You must stop when the traffic lights turn red.

You should see to the doctor.

DDE, GJUS&T, Hisar 413 | English (Compulsory) BA-101

4.5 Further Main Body of the Text

4.5.0 Conjunctions and Its Types

4.5.1 Conjunctions

Some words are satisfied spending an evening at home, alone, eating ice-cream right out of the box, watching Seinfeld re -runs on TV, or reading a good book. Others aren’t happy unless they’re out on the town, mixing it up with other words; they’re joiners and they just can’t help themselves. A conjunction is a joiner, a word that connects (conjoins) parts of a sentence. In sentence a), ‘and’ join the two words. In sentence b), ‘but’ join the two sentences. Thus, Conjunction is a word which merely joins together two sentences or two words. Conjunctions: and, or, if, that, but, where, or, also, still, since, only, after, before, because, as, neither, nor, either, or, not only, but also

DDE, GJUS&T, Hisar 414 | English (Compulsory) BA-101

4.5.2. Conjunction: Its Type a) Coordinating Conjunctions b) Subordinating Conjunctions

Read the following sentences carefully. i) I waited on the station but the train did not arrive. ii) Since you asked, I am going to tell you the story.

Here in the first sentence, ‘I waited on the station’ and ‘the train did not arrive’ are two independent clauses; joined by the conjunction ‘but’. They can stand as the sentences of complete meaning if separated, by removing the joining conjunction. They are called main/ principal clauses. Thus, conjunctions which join together the two or more independent clauses are called

4.5.3 Coordinating Conjunctions. The simple, little conjunctions are called coordinating conjunctions (you can click on the words to see specific descriptions of each one): In the second sentence, ‘since’ is conjunction and ‘since you asked ‘and ‘I am going to tell you the story.’ are the two clauses. Second clause can stand on its own as an independent sentence and it is the main clause. However, the first clause depends on the second clause for its complete meaning.

Coordinating Conjunctions

• and, but, or, yet, for, nor, so

(It may help you remember these conjunctions by recalling that they all have fewer than four letters. Also, remember the acronym FANBOYS: For-And-Nor-But-Or-Yet-So. Be careful of

DDE, GJUS&T, Hisar 415 | English (Compulsory) BA-101 the words then and now; neither is a coordinating conjunction, so what we say about coordinating conjunctions’ roles in a sentence and punctuation does not apply to those two words).

When a coordinating conjunction connects two independent clauses, it is often (but not always) accompanied by a comma:

• Ulysses wants to play for UConn, but he has had trouble meeting the academic requirements.

When the two independent clauses connected by a coordinating conjunction are nicely balanced or brief, many writers will omit the comma:

• Ulysses has a great jump shot but he isn’t quick on his feet.

The comma is always correct when used to separate two independent clauses connected by a coordinating conjunction.

A comma is also correct when and is used to attach the last item of a serial list, although many writers (especially in newspapers) will omit that final comma:

• Ulysses spent his summer studying basic math, writing, and reading comprehension.

When a coordinating conjunction is used to connect all the elements in a series, a comma is not used:

• Presbyterians and Methodists and Baptists are the prevalent Protestant congregations in

Oklahoma.

DDE, GJUS&T, Hisar 416 | English (Compulsory) BA-101

A comma is also used with but when expressing a contrast:

AND

(a) To suggest that one idea is chronologically sequential to another: Tashonda sent in her applications and waited by the phone for a response.”

(b) To suggest that one ide a is the result of another: “Willie heard the weather report and promptly boarded up his house.”

(c) To suggest that one idea is in contrast to another (frequently replaced by but in this usage): “Juanita is brilliant and Shalimar has a pleasant personality.

(d) To suggest an element of surprise (sometimes replaced by yet in this usage): “Hartford is a rich city and suffers from many symptoms of urban blight.”

(e) To suggest that one clause is dependent upon another, conditionally (usually the first clause is an imperative): “Use your credit cards frequently and you’ll soon find yourself deep in debt.” (f) To suggest a kind of “comment” on the first clause: “Charlie became addicted to gambling — and that surprised no one who knew him.”

BUT

(a) To suggest a contrast that is unexpected in light of the first clause: “Joey lost a fortune in the stock market, but he still seems able to live quite comfortably.”

(b) To suggest in an affirmative sense what the first part of the sentence implied in a negative way (sometimes replaced by on the contrary): “The club never invested foolishly, but used the services of a sage investment counsellor.”

(c) To connect two ideas with the meaning of “with the exception of” (and then the second word takes over as subject): “Everybody but Golden breath is trying out for the team.”

OR

(a) To suggest that only one possibility can be realized, excluding one or the other: “You can study hard for this exam or you can fail.”

DDE, GJUS&T, Hisar 417 | English (Compulsory) BA-101

(b) To suggest the inclusive combination of alternatives: “We can broil chicken on the grill tonight, or we can just eat leftovers.

(c) To suggest a refinement of the first clause: “Smith College is the premier all- women’s college in the country, or so it seems to most Smith College alumnae .”

(d) To suggest a restatement or “correction” of the first part of the sentence: “There are no rattlesnakes in this canyon, or so our guide tells us.”

(e) To suggest a negative condition: “The New Hampshire state motto is the rather grim “Live free or die.”

(f) To suggest a negative alternative without the use of an imperative

(g) “They must approve his political style or they wouldn’t keep electing him mayor.”

4.5.4 Subordinating clause. subordinating conjunction and the clause ‘since you asked’ is, Thus, conjunctions which join one or more dependent or subordinate clauses to the independent or main clause are called subordinating conjunctions.

1.The sentence in which two or more principal or main clauses are joined together by the coordinating conjunctions is called Compound sentence. ii) The sentence in which one or more subordinate clauses are joined to the principal or main clause, by the subordinating conjunctions, is called Complex sentence. iii) The sentence made of only one clause, one subject and one predicate with one verb,) is called simple sentence. Notice that some of the subordinating conjunctions in the table below—4 are also prepositions, but as subordinators they are being used to introduce a clause and to subordinate the following clause to the independent element in the sentence. as though rather than whenever because since where

DDE, GJUS&T, Hisar 418 | English (Compulsory) BA-101 before so that whereas even if than wherever even though that while

4.5.5 Common Subordinating Conjunctions

Strictly speaking, the word like is a preposition, not a conjunction. It can, there fore, be used to introduce a prepositional phrase (“My brother is tall like my father”), but it should not be used to introduce a clause (“My brother can’t play the piano as he did before the accident” or “It looks like as if basketball is quickly overtaking baseball as America’s national sport.”).

4.6 Usage of Conjunctions

4.6.1 Correlative Conjunctions:

Some conjunctions combine with other words to form what are called correlative conjunctions. They always travel in pairs, joining various sentence eleme nts that should be treated as grammatically equal.

DDE, GJUS&T, Hisar 419 | English (Compulsory) BA-101

• She led the team not only in statistics but also by virtue of her enthusiasm.

• Polonius said, “Neither a borrower nor a lender be.”

• Whether you win this race or lose it doesn’t matter as long as you do your best.

Correlative conjunctions sometimes create problems in parallel form. both . . . and neither . . . nor not only . . . but also whether . . . or not . . . but as . . . as either . . . or

4.6.2 Conjunctive Adverbs

The conjunctive adverbs such as however, moreover, nevertheless, consequently, as a result are used to create complex relationships between ideas.

4.6.3 Other Conjunctions

The conjunction NOR is not extinct, but it is not used nearly as often as the other conjunctions, so it might feel a bit odd when nor does come up in conversation or writing. Its most common use is as the little brother in the correlative pair, neither- nor.

• He is neither sane nor brilliant.

• That is neither what I said nor what I meant. It can be used with other negative expressions:

• That is not what I meant to say, nor should you interpret my statement as an admission of guilt. It is possible to use nor without a preceding negative element, but it is unusual and, to an extent, rather stuffy: Be careful of the conjunction SO. Sometimes it can connect two independent clauses along with a comma, but sometimes it can’t. For instance, in this sentence,

DDE, GJUS&T, Hisar 420 | English (Compulsory) BA-101

• Soto is not the only Olympic athlete in his family, so are his brother, sister, and his Uncle Chet. where the word so means “as well” or “in addition,” most careful writers would use a semicolon between the two independent clauses. In the following sentence, where so is acting like a minor-league “therefore,” the conjunction and the comma are adequate to the task:

• Soto has always been nervous in large gatherings, so it is no surprise that he avoids crowds of his adoring fans.

4.6.4 Omitting That

The word that is used as a conjunction to connect a subordinate clause to a preceding verb. In this construction that is sometimes called the “expletive that.” Indeed, the word is often omitted to good effect, but the very fact of easy omission causes some editors to take out the red pen and strike out the conjunction that wherever it appears. In the following sentences, we can happily omit the that (or keep it, depending on how the sentence sounds to us):

• Isabel knew [that] she was about to be fired.

• She definitely felt [that] her fellow employees hadn’t supported her.

• I hope [that] she doesn’t blame me.

4.7 Check your progress-

Fill in the blanks using appropriate modal and conjunction

3. I ______like to order the onion soup please.

4.The manager ______be pleased to hear that a customer slipped on the wet floor.

DDE, GJUS&T, Hisar 421 | English (Compulsory) BA-101

5.______it be okay if I slept here tonight?

6.If you are sick, you ______go to work. You’ll infect everyone there.

7.Drivers ______stop at red lights.

8.You ______finish the proposal today. You can finish it tomorrow.

9.She ______hear much better with her new hearing aids.

10.______I order us a bottle of wine?

11.This is the place ______we stayed last time we visited. (where, when, how)

12.______you win first place, you will receive a prize. (wherever, if, unless)

13.You won’t pass the test ______you study. (when, if, unless)

14.Calculus is ______easy ______difficult ______(not / but, both / and, either / or)

15.It’s ______going to rain ______snow tonight. (as / if, either / or, as / as)

4.8 Summary

• Some words are satisfied spending an evening at home, alone, eating ice -cream right out of the box, watching Seinfeld re -runs on TV, or reading a good book. Others aren’t happy unless they’re out on the town, mixing it up with other words; they’re joiners and they just can’t help themselves. A conjunction is a joiner, a word that connects (conjoins) parts of a sentence.

• Some conjunctions combine with other words to form what are called correlative conjunctions.

• The conjunctive adverbs such as however, moreover, nevertheless, consequently, as a result are used to create complex relationships between ideas.

Modal verbs are used to express functions such as: Permission, Ability, Obligation, Prohibition, Lack of necessity, Advice, possibility, probability.

4.9 Keywords

DDE, GJUS&T, Hisar 422 | English (Compulsory) BA-101

Conjunction: A word that connects parts of a sentence.

Yet: It is coordinating conjunction.

Modal verbs: are followed by an infinitive without "to", also called the bare infinitive

4.10 Self- Assessment Questions (SAQs)

State whether the following statements are true or false:

1.When we use modals in sentences.

2.. Write down 10 modals.

3. Write down 10 conjunctions and use it in sentence

4.11 Answers to Check your Progress-

3. Would 4. won’t 5. would 6. shouldn’t 7. must 8. don’t have to 9. can 10 – where, 11 – if, 12 unless 13.not / but, 14– either / or, 15 – neither / nor

4.12 References/ Suggested Readings

 Books English for Competitive Exams, By DR. R.P. Bhatnagar  Unique Quintessence of General English, Edited by DR. S. Sen & Others and revised by DR. G.S. Mansukh ani.  A Background to the Study of English Literature, D.K. Patnaik, Swastik Publications  Online links: o www.writingcentre.uottawa.ca/hypergrammar/preposit.html o http://www.dl.ket.org/latin2/grammar/ch34-degofadj.htm

DDE, GJUS&T, Hisar 423 | English (Compulsory) BA-101

Subject: English

Course Code- B.A.-101 Author: Dr. Astha Gupta

Lesson- 5 Editor: Dr. Shakuntla Devi

Introduction to Tenses and Active and Passive voice

Structure of Lesson

5.1 Learning Objectives

5.2 Introduction

5.3 Main Body of the Text

5.3.0Tense and Its Types

5.3.1 Tense

5.3.2 Types of Tenses

a. Present Tense b. Past Tense c. Future Tense

5.3.3 Present Tense

a. Simple present Tense.

b. Present Continuous Tense.

c. Present Perfect Tense.

d. Present Perfect Continuous Tense

5.3.4 Past Tense

a. Past Indefinite Tense/ Simple Past

b. Past Continuous Tense:

DDE, GJUS&T, Hisar 424 | English (Compulsory) BA-101

c. Past Perfect Tense

d. Past Perfect Continuous Tense:

5.3.5 Future Tense

a. Simple future / Future Indefinite Tense

b. Future Continuous Tense

c. Future Perfect Tense

d. Future Perfect Continuous Tense

5.4 Further Main Body of the Text

5.4.0 Voice- Active and Passive

5.4.1 Active Voice and Passive Voice

5.4.2 Difference between Active Voice and Passive Voice:

5.4.3 General Rules to change the Voice:

5.4.4 Transitive and Intransitive Verbs:

5.5 Check Your Progress

5.6 Summary

5.7 Keywords

5.8 Self-Assessment Questions (SAQs)

5.9 Answers to check your progress

5.10 References/ Suggested Readings

5.1 Learning Objectives:

The objectives of this chapter are: Students will be able to recognize the past, present and future tenses of regular and irregular verbs. In this lesson we shall study about

DDE, GJUS&T, Hisar 425 | English (Compulsory) BA-101

Active Voice and Passive Voice. We will also study general rules of changing the verbs from Active voice to Passive voice and vice versa. Students will understand that verbs change form depending on their relationship to time the objectives of this lesson are:

1. To give an introduction to Voice in Language.

2. To learn about Transitive Verbs and Intransitive Verbs.

3. To make students aware of general rules of changing the Verbs in Active and Passive Voice.

5.2 Introduction:

In English grammar, verbs are often used in a way that it indicates or denotes the time when an event occurred. These verbs that take up different forms to indicate the time of an action, event or condition by changing its form are called as Tenses. The word “tense” means “time”. It is used in relation to a verb. It tells us about an action which occurs or about the state of a thing which exists. “Tense is the form of a verb which shows the time of an action and its degree of completeness.” Along with the knowledge of correct grammar, it is important to know its correct practical use. In this part, you will study sentences in larger context for social as well as academic purposes. You will learn to construct and synthesize the sentences. You have learned earlier that the verb denotes action. The action may be happening in the present time or it may have happened in the past time or it may happen in the future. Thus, depending upon the time of the action or state there are three tenses of the verbs.

a) Present Tense: In this tense the verb shows that the action is in the present time b) Past tense: In this tense the verb indicates that the action has happened in the past.

DDE, GJUS&T, Hisar 426 | English (Compulsory) BA-101

c) Future tense: In this tense the verb in dictates that the action will happen in the future. Depending upon the completeness or incompleteness of the action each tense is further divided into four types. i) Simple: In this the action is simply mentioned. ii) Continuous: In this the action is incomplete and ongoing. iii) Perfect: In this the action completes. iv) Perfect Continuous: In this the action is ongoing at the time of reporting the speech. Voice is that property of the Verb which shows whether the subject is performing or experiencing the action. We have two types of Voice:

(a)Active Voice (b) Passive Voice.

Active Voice describes a sentence where the subject performs the action stated by the verb. Whereas in Passive Voice sentences, the subject is acted upon by the verb.

An action of a subject, in relation to an object, is expressed in two ways. These two ways expressing action of a subject are known as voice i.e. Active Voice and Passive Voice. Let us try to understand it with the help of some examples:

Ram kills a snake. (Active Voice)

A snake is killed by . (Passive Voice)

5.3 Main Body of the Text

5.3.0 Tense and Its Types

5.3.1 Tense:

Tense means time. The content of this lesson shall be presented as below

- Tenses

- Types of Tenses

- Present Tense

DDE, GJUS&T, Hisar 427 | English (Compulsory) BA-101

- Use of Present tenses

- Exercises for practice

5.3.2 Types of Tenses:

There are three types of Tenses i.e. a. The Present Tense, b. The Past Tense and c. The Future Tense

Depending upon the present, past or future event. With each of these tenses, there are four aspects associated with it. An aspect here refers to the nature of action performed by the verb. Each of these three tenses has further four forms:

1. Indefinite or Simple

2. Continuous or Progressive

3. Perfect

4. Perfect Continuous

The students must also know about the forms of the verbs before they are told about tenses in detail. There are three form of the verb.

1. Present or the first form

2. Past or the second form

3. Past Participle or the third form for e.g.

Go Went Gone

Called Called Called

There is another form also known as present participle form which is formed by adding “Ing” to the first form of the verb. e.g.:

Smile + Ing = Smiling

DDE, GJUS&T, Hisar 428 | English (Compulsory) BA-101

Go + Ing = Going

Play +Ing = Playing

Now, let us study the use of Tenses in detail.

5.3.3 Present Tense

This tense is used to refer or indicate to something that occurs in the present. It is of four types:

a. Simple present Tense. b. Present Continuous Tense. c. Present Perfect Tense. d. Present Perfect Continuous Tense. a. Simple Present Tense:

The simple present or indefinite present tense is used to describe an action, event or condition that is occurring in the present while being spoken about or written for e.g.

The boys play.

Cows eat grass.

The present indefinite tense is formed with the first form of the verb i.e.

Sub + First form of the verb

Sentence Subject: I, We, You, They He, She, It, Noun (Singular)

Noun (Plural)

Sentence Ist form of Verb Ist form + s/es

Negative do not + 1st form does not + 1st form

Interrogative do+Subject+1st form+.? Does + Subject + 1st form…?

DDE, GJUS&T, Hisar 429 | English (Compulsory) BA-101

Use of Simple Present Tense:

The simple present tense is used to express:

(a) Habitual action e.g. i) The School starts at 8 a.m. ii) Mohan goes to office every day at 9 a.m.

(b) The Simple present tense is used to describe universal truth, scientific facts and what is always stated as true e.g. i) The earth moves round the sun. ii) Water boils at 100degree C.

(c) Planned future action which must happen in the near future, e.g.

1) Our examination begins tomorrow.

2) We leave Delhi at 9 a.m. on Monday.

(d) It is also used in newspaper headlines. e.g.

1. PM congratulates victorious Indian Team.

(e) The simple present is also used to describe processes, to give instructions or to give running commentary. e.g.

1. Sachin hits the six.

(f) Conditional sentences when they refer to some future action, e.g.

1) If you work hard, you will pass.

2) Unless you run fast, you will miss the train.

(g) Adverbs and adverbial phrases like usually, frequently, occasionally, often, seldom, sometimes, generally, rarely, daily, everyday etc. e.g.

1) I go to my friend’s house every week.

DDE, GJUS&T, Hisar 430 | English (Compulsory) BA-101

2) Mohan seldom goes to play cricket.

EXERCISE – I

Rewrite the following sentences into their negative and interrogative forms: a) I come to school by bus. b) Mohit studies every day. c) My sister lives in Mumbai. d) Radha is a singer.

Use Simple Present tense in the following sentences:

1) My sister…………… (receive) the letter daily.

2) Mohan always………… (speak) the truth.

3) Most trains ……………. (not run) on time.

4) If it ……………… (rain), we will not play the match.

5) If you………… (go) to Delhi, tell him to ring me.

b. Present Continuous Tense

) Present continuous tense is used to denote an action which is going on at the time of speaking. Look at the following examples:

He is playing the piano.

She is dancing on the stage.

These sentences show that the actions are in progress at the present moment. In other words, we can say that Present Continuous tense is used for the action which is happening simultaneously at the time of action. This tense is also used with words like now, still, today, while, at the present moment, when they refer to the action in progress. Look at the following table:

Affirmative sentences is/am/are + 1st form + Ing

DDE, GJUS&T, Hisar 431 | English (Compulsory) BA-101

Negative sentences is not / am not / are not +1st form + Ing

Interrogative sentences is/am/are + subject… And question mark in the end.

For e.g. I am singing a song now.

I am not singing a song now.

Am I singing a song now?

Am I not singing a song now?

Use of Present Continuous tense:

It is used to describe definite and immediate future action. For example

- I am going to Delhi tonight.

- We are attending the party tonight.

It is used to say that we are in the middle of an action. For example

- I am waiting for the bus.

- The wind is blowing.

It is also used when we are in the middle of something, but not necessarily/actually at the moment of speaking. For example.

- He is reading a novel these days.

- We are working on a new project.

Note: There are number of verbs which are not normally used in the present continuous tense. These are: Verbs of perception: See, taste, smell, notice, etc. Verbs used to express feelings or states of mind: love, want, desire, wish, refuse, forgive, care, hate, adore, like, dislike admire etc.

Incorrect Correct

These grapes are tasting sour. These grapes taste sour.

I am thinking you are wrong. I think you are wrong.

DDE, GJUS&T, Hisar 432 | English (Compulsory) BA-101

EXERCISE 1

Change the following into the Present Continuous tense: - a) The airplanes fly in the sky. b) The doctor examines the patient. c) The gardener waters the plants. d) Mohan swims in the tank at this time.

EXERCISE 2

Fill in the blanks with Present Continuous tense: a) Rita …………. (comb) her hair. b) You can’t see him, he………… (sleep) at the moment. c) The train……………… the platform. (leave) d) The washing ……. on the line. (hang

C. Present Perfect Tense

Present Perfect Tense is used in the sentences which began its action in the past but it will be completed in the present. Present perfect Tense is used to express a recently completed action which can be denoted by words like first now, already, yet, recently, lately e.g.

Study the following table:

Affirmative sentences Has/Have + 3rd form

Negative sentences Has not + Have not + 3rd

Interrogative sentences Has/Have before subject and Question mark in the end.

For e.g. I have done my work.

I have not done my work.

DDE, GJUS&T, Hisar 433 | English (Compulsory) BA-101

Have I done my work?

Have I not done my work?

Note: Special care is to be taken in case of sentences where past time like yesterday, last week, last month, last year, ago, etc. are mentioned. You are required to use here Past Indefinite Tense and not Present Perfect Tense.

USES OF PRESENT PERFECT TENSE

The present perfect Tense is used to show that an action done in the past has an effect or relevance at the present time.

For example –

1) Seema, are you still reading the book?

2) He has read The Discovery of India. (So, he knows the story)

 To talk about activities completed in the immediate past:

1) I have just heard the news.

2) She has just gone out.

 To indicate actions where the time of the action is not given (but is not indefinite)

1) Suman has gone to Amritsar.

2) Have you seen him walking by?

Exercise – I

Use the Present Perfect tense: a) The cat ………… (steal) the fish. b) You……. (see) him lately. c) His flight ………. (take off) already. d) I …………… (know) him to be a serious man.

DDE, GJUS&T, Hisar 434 | English (Compulsory) BA-101

Exercise – 2

Use the Present Perfect Tense: a) I just…………. (wash) that floor. b) The movie…………. (be) released recently. c) The film …………. just (begin) d) My brother …… (not pay) the telephone bill yet. d. Present Perfect Continuous Tense

The Present perfect Progressive (continuous) tense is a combination of the Present Perfect and the Present Progressive tenses, in form / has / have + been + V1 + Ing) as well as in meaning.

In Present Perfect Continuous tense, the action began sometime in the past but it is still continuing in the present.

Affirmative sentences Has been / Have been + 1st form + Ing

Negative sentences Has not been /Have not been + 1st form + Ing.

Interrogative sentences Has/Have+ Subject + been.?

Look at the following sentences –

You have been playing.

You have not been playing.

Have you been playing?

It should be noted that you are supposed to use for and since before the given time. For is to be used to express the period of time i.e. for five years, for three days, for four hours, etc. Since is to be used to express the point of time i.e. since 2014, since March, since 5’O clock, since Wednesday etc.

DDE, GJUS&T, Hisar 435 | English (Compulsory) BA-101

Special care is to be taken in sentences where time factor is not given and you should use Present Continuous Tense. If the number of times is mentioned, you are required to use Present Perfect Tense.

Exercises

Fill in the blanks with the Present Perfect Continuous tense:

1) My brother is a writer. He……… (write) novels for 5 years.

2) He………… (fly) kites since morning.

3) Harshit………. (not keep) good health for three days.

4) We ………….(live) here since 2001.

5) ………….. she …………….(knit) her sweater for two hours.

5.3.4 Past Tense:

The Past tense is a grammatical tense whose principal function is to place an action or situation in past time. Past tense in English has two semantic features: -

• The speaker conceptualizes the event as having occurred at some specific time in the past.

• The event is presented as wholly located in the past, in a time frame that is separated from the present. James Joyce was born in Dublin 1882. He lived in Ireland until 1904 and spent rest of his life abroad. Here “was born” is interpreted referring to a point of time, while “lived” and “spent” are interpreted as being of longer duration. a. Past Indefinite Tense/ Simple Past:

The Simple past is used to say that an event occurred or that something was the case at a particular time in the past.

Mohan ran a thorn into his foot yesterday.

You lived in my neighbourhood last year.

DDE, GJUS&T, Hisar 436 | English (Compulsory) BA-101

Uses Of Simple Past

1. If we want to say that a situation existed over a period of time in the past, we use the simple past-M.F. Hussain lived abroad in his last year.

2. When we talk about an activity that took place regularly or repeatedly in the past, but which no longer occurs, we use simple past.

-When my grandmother was alive, she fed sparrows early in the morning every day. used to is also used to express such situations. However, unlike ‘would’, ‘used to’ can also describe past states and situations. ‘Used to’ is not common in negative situations.

I’m not quite as healthy as I used to be.

1.When we talk of something that happened in the past, and we mention a situation existed at that time, we use the simple past. All the streets in this part of old Delhi look alike. Study the table: -

Affirmative 2nd form of the verb.

Negative did not + 1st form of the verb.

Interrogative Did – Subject + 1st form….?

For e.g.

He played cricket. Did he play cricket?

He did not play cricket. Did he not play cricket?

Exercise - 1

Complete the sentences using simple past tense: ---

1) She…………. (call) me last night.

2) Columbus …………. (discover) America.

3) Last year it ……………… (not rain) heavily.

DDE, GJUS&T, Hisar 437 | English (Compulsory) BA-101

4) Gopal … (not attend) the office yesterday.

5) My health has improved since I……………… (go) to Nainital.

Exercise – 2

Change the following sentences into negative and Interrogative sentences: -

1) Your father hates the poor.

2) I know his name.

3) He came to see me.

4) The thieves broke into his house.

5) He played tennis. b. Past Continuous Tense:

The Past Continuous Tense is often used to denote an action which continued for some time in the past. The most common use of the past continuous/progressive is to talk about what was already happening at a particular past moment.

When I got up this morning, it was raining.

Affirmative was /were + 1st form + Ing

Negative was not / were not + 1st form + Ing

Interrogative was / were + subject...?

Uses of Past Continuous Tense

1. Past Continuous tense is used in the sentences where we have adverbs like continually, constantly, forever etc.

The patient was continually asking for help.

2. Past continuous tense is used for temporary actions and situations. When we talk about longer, more permanent situations, we usually use the simple past:

I was running downstairs when I slipped and fell.

DDE, GJUS&T, Hisar 438 | English (Compulsory) BA-101

3. Used without a time expression, the past continuous can indicate gradual development. When used with a point in time, it expre sses an action which began before that time and probably continued after it.

-At nine, she was having dinner.

4. When the conjunction ‘while’ is used to connect two or more clauses, we make use of Past Continuous tense in both the clauses. e.g.

-While I was reading, my brother was playing piano.

Exercise

Use the Past Continuous tense:

1) She cut her finger while she……… (cut) potatoes.

2) The children……… (play) when I saw them.

3) While Radha was singing, Suman ……. (dance)

4) He … (not read) his books after the examination. c. Past Perfect Tense

Past perfect tense is used to express an action which has been completed in the past before a particular moment. We use the past perfect to go back to an earlier past time, to talk about things that had already happened at the time we are talking about. For e.g. I explained that I had forgotten my keys.

Affirmative` had + 3rd form

Negative had not + 3rd form

Interrogative had – subject .. ?

He had finished his work before I went out.

He had not finished his work before I went out.

Had he finished his work before I went out?

DDE, GJUS&T, Hisar 439 | English (Compulsory) BA-101

Had he not finished his work before I went out?

Usually this tense is used in simple sentences where words like till, before, already, since etc. come.

Uses of Past Perfect Tense

1.It is commonly used in ‘reported speech’ after past verbs like said, told asked, explained, thought, wondered, etc.

I thought I had sent the cheque a week before.

2. We make use of past perfect tense and past indefinite tense in two clauses of a sentence when there are two actions taking place, one of which happens before the other. E.g. The bell had already rung before I reached the college.

3. Past perfect is also used in conditional sentences for e.g. If you had worked hard, you would have passed.

4. It is also used to express unfulfilled desire of the past. e.g. I wish I had gone there.

The Past Perfect can be used with ‘till’ / ‘until’ and ‘before’ to emphasize the completion or expected completion of an action. He refused to go till he had seen all the details of the case.

Exercise

1.He…. (not revise) his courses till then.

2.He……. (take) his breakfast before he left for the college.

3.I thought that my sister……(come) d. Past Perfect Continuous Tense:

Past Perfect Continuous Tense is used when the action began in the past and it continued and it was still continuing at the appointed time in the past. The Past

DDE, GJUS&T, Hisar 440 | English (Compulsory) BA-101

Perfect Continuous tense is not used with verbs that are not used in continuous form, except with want and sometimes, wish:

The child was delighted with his new toy. He had been wanting one for a long time.

Affirmative Had been + 1st form +Ing

Negative Had not been + 1st form + Ing

Interrogative Had + Subject – been -?

Exercise

1. Sudesh………. (work) in the company for six years before she………… (leave) it.

2. He …………. (sleep) for two hours when the thieves went into his house.

3. She……………. (prepare) tea for five minutes when the guests came.

4. His finger… (not bleed) for half an hour when the doctor checked it.

5.3.5 Future Tense

In English, there are four ways of discussing future time:

a. Simple future / Future Indefinite Tense b. Future Continuous Tense c. Future Perfect Tense d. Future Perfect Continuous Tense a. Simple Future Tense

Future Indefinite Tense is used to express an action which is believed to take place sometime in future. This tense is used to express intention, determination, duty, offer, assurance etc.

Take a look at the following sentences:

You shall go home today.

I will sing today.

DDE, GJUS&T, Hisar 441 | English (Compulsory) BA-101

You shall play tomorrow.

The function will begin at 9 00 am.

Affirmative Will / shall + 1st form

Negative Will not / shall not + 1st form

Interrogative Will / shall – subject -?

For e.g.

I shall go to Delhi. Shall I go to Delhi?

I shall not go to Delhi. Shall I not go to Delhi?

Use of ‘will’ and ‘shall’

Will is used for:

• Intention, Determination, Invitation, Assurance.

• Request or offering help.

• Planned future incident / event.

• Spontaneous decisions.

Shall is used for:

• Duty: You shall defend your motherland.

• Suggestion: Shall we go for a coffee.

• Offer: Shall I bring a glass of water for you?

• Rules: You shall obey the traffic rules.

Exercise-

Use Simple Future tense and convert each of the sentences into negative and interrogative sentences:

1. She……my friend to the park. (take)

DDE, GJUS&T, Hisar 442 | English (Compulsory) BA-101

2. They………………. a house soon. (buy)

3. I ………. the party. (attend)

4. Will you……… the salt? (pass)

5. He …………… drinking soon? (stop) b. Future Continuous Tense

Future Continuous tense is used to express an action that is going on at a given point of time in the future. It is used to express future action that will begin in future and continue after a given period of time. The following table gives the various forms of the simple future continuous tense:

Affirmative Will be / shall be +1st form + Ing

Negative Will not be / shall not be + 1st form + Ing

Interrogative Will / shall - subject – be?

This tense is used to express future events that are planned for e.g.

-I will be staying here till the exams are over.

This tense is not used in conditional clauses. In its place, we use the Present Perfect tense. e.g.

-When we have finished our homework, we shall play.

In Interrogative sentences this tense is used to get information in a polite manner for e.g.

Will you be coming with us to watch the movie

We shall be singing.

We shall not be singing.

Shall we be singing?

Shall we not be singing?

DDE, GJUS&T, Hisar 443 | English (Compulsory) BA-101

DDE, GJUS&T, Hisar 444 | English (Compulsory) BA-101

Exercise-

Fill in the blanks with future continuous tense:

1. Don’t ring up now, Sohna ……. (rest) at this time.

2. Rohan’s sister…… (marry) today.

3. Rahul …. (buy) a car soon.

4. Soham …. all day today for her birthday (shop)

5. After some years, people……. (go) to Moon for holiday. c. Future Perfect Tense”

Future perfect Tense is used to express the completion of an action or event in the future. This tense form is used to talk about a future action that can be completed in a particular time.

Look at the sentence below: The gardener will have watered the plants before the sun rises.

Affirmative Will/shall+have+3rd form verb

Negative Will/shall not+have+3rd form

Interrogative Will /shall +subject+have+3rd form of verb

Exercise-

Fill in the blanks with Future Perfect tense forms of the words given in brackets and also convert the sentences into negative and interrogative forms:

1. Rajesh…. by the time you call him. (sleep)

2. The Police……the thief by now. (catch)

3. I…………. my work by the time you come back home. (finish)

4. I ……………… ten thousand rupees by the year end. (save)

DDE, GJUS&T, Hisar 445 | English (Compulsory) BA-101 d. Future Perfect Continuous Tense:

Future Perfect Continuous tense expresses the continuity of an action up to a specific time in future. This tense form is used to express actions that began in the past and which will be in progress over a period of time that will end in the future. Look at the sentence below: By the end of the year, I shall have been working in the college for 11 years.

The following table gives the various form of the Future Perfect Continuous tense:

Affirmative Will have been / shall have been + 1st form + ing

Negative Will not have been /shall not have been + 1st form + ing

Interrogative Will / shall – subject – have been?

I shall have been doing washing tomorrow.

I shall not have been doing washing tomorrow.

Shall I have been doing washing tomorrow?

Shall I not have been doing washing tomorrow?

Use Future Perfect Continuous tense in the following sentences: -

1. In another two years, they …… (work) here for five years.

2. By the time father retires, I …………… for five years. (retire)

3. We ……. the live match. (watch)

4. When I come back to you at 6 O’ clock, you……………… (rest) for 3 hours.

5.4 Further Main Body of the Text

5.4.0 Voice- Active and Passive

5.4.1 Active Voice and Passive Voice

DDE, GJUS&T, Hisar 446 | English (Compulsory) BA-101

When the sentence is presented though the point of view of the subject; the subject is the agent of the action and the action verb is used to denote the action, it called active voice. They have a subject a verb and a direct object.

For Example: Mr. Sharma clicked the photograph. Here ‘Mr. Sharma’, the subject is the doer of the action. The action is indicated by the action verb ‘clicked ‘The object of the action is ‘photograph’. When the sentence is presented from the point of view of the ‘object’; the subject is not the agent of the action and the action is shown through the passive verb, it is called the Passive voice. The sentence has only an object, and a verb. The subject placed at the end can be omitted.

For Example: The photograph was clicked by Mr. Sharma.

Here object ‘photograph’ is given the main position, and the action is denoted by the passive verb, ’was clicked’. The subject Mr. Sharma can be omitted.

Read the following sentence carefully.

The grandmother gave the sweets to me. (Active Voice) Here,’ sweets’ is the direct object and ‘me’ is indirect object. We can change this sentence into passive form by two ways.

i) The sweets were given to me by the grandmother.

5.4.2 Difference between Active Voice and Passive Voice:

To understand the difference in both voices, we should focus on the subject and the object of a sentence. We know that every sentence has a subject, a verb and an object. The sequence of subject and object as in Active Voice, is reversed when it is expressed in Passive Voice. Read the following example for better understanding this difference –

Active Voice Radha loves Geeta. (Subject) (Verb) Active form

Passive Voice Geeta is loved by Radha.

DDE, GJUS&T, Hisar 447 | English (Compulsory) BA-101

(Subject) (Verb) (Object) (Passive form)

Active Voice: Subject +Verb +object

Passive Voice: Object+ Verb+ Subject

Apart from reversing the sequence of subject and object, the form of the verb of sentence also changes in both Voices. The only form of verb used in Passive Voice is the 3rd form of the verb which is also called Past Participle.

It will be noticed that when the Verb is changed from the Active Voice to Passive voice, the object of the transitive verb in the Active Voice becomes the subject of the Verb in the Passive Voice. Since the object of a verb in the active voice becomes the subject of the passive form, it follows that only Transitive Verbs can be used in the passive voice, because an Intransitive Verb has no object.

5.4.3 General Rules to change the Voice:

Rules for changing Active Voices into Passive Voices are as follows:

1.As mentioned earlier, the structure of sentence will be reversed in Passive Voice. The places of the subject and the object will interchange. While changing from Active Voice to Passive Voice subject will shift to the place of Object and the object will take the place of subject in Passive Voice.

Example: Radha lights the candle. (Active Voice)

The candle is lighted by Radha. (Passive Voice)

2.If object have the pronoun of objective case change it with the help of this rule:

Me = I Us = We

You= you him= He

Her = She it = It

Then = They Whom = who

DDE, GJUS&T, Hisar 448 | English (Compulsory) BA-101

If subject have the pronoun of nomination case change it with the help of this rule, like: -

I = by me we = by us

You = by you He = by him

She = by her it = by it

They= by them who = by whom

The passive voice is formed with the suitable tense of the verb be followed by the past participle. Study this table:

Tense (or Modal+ base) Active Voice Passive Voice

Simple present take am taken

takes is taken/are taken

Present Continuous am taking am being taken

is taking is being taken

are taking are being taken

Present Perfect has taken has been taken

have taken have been taken

Simple Past took was taken/were taken

Past Continuous was taking was being taken

we’re taking were being taken

Past perfect had taken had been taken

Simple Future will take will be taken

shall take shall be taken

Can/may/must etc + base can take can be taken

DDE, GJUS&T, Hisar 449 | English (Compulsory) BA-101

must take must be taken

3. Only Past Participle form or 3rd form of verb (e.g. taken etc.) will always be used as main verb in passive voices for all tenses. No other form of verb will be used as main verb. It can be seen in all the examples given on this page.

4. The word “by” will be used before subject in the passive voice. Example:

The peon opened the gate. (Active Voice)

The gate was opened by the peon. (Passive Voice)

5. The Active Voice is used when the agent (i.e. doer of the action) is to be made prominent; the Passive, when the person or thing acted upon is to be made prominent. The passive is generally preferred when the active form would use an indefinite or vague pronoun or noun (somebody, people, we, they etc.) as subject; that is, when we do not have the clear idea about agent or do not know who the agent is. For example:

Active = I have been invited to the party.

Passive = Someone has invited me to the party.

Active = My pen has been stolen.

Passive = Somebody has stolen my pen.

5.4.4 Transitive and Intransitive Verbs:

We have already learnt that a verb is a word that expresses action, feeling or existence. Many verbs can be both transitive and intransitive. An action verb with a direct object is transitive while an action verb with no direct object is intransitive. Now study the following sentences:

(a) Ram laughs. (b) Rajat plays.

(b) dances. (d) Suraj sleeps.

DDE, GJUS&T, Hisar 450 | English (Compulsory) BA-101

In these sentences no object is used with verb (laughs, dances, plays, sleeps) and the meaning of the sentence is also clear so these words are intransitive verbs. When an object is used with the verb it is called Transitive verb. Let us see these examples:

1. Ram writes a letter.

2. Gita plays tennis.

3. Neha loves her mother.

4. Sachin sings a song.

In these sentences object letter, tennis, mother, and song are respectively used for the verbs writes, plays, loves, and sings. These words are intransitive verbs.

Read the examples:

Transitive Verb Intransitive Verb

1. Open all the windows the car stopped

2. The boys fly their kites. Birds fly

3. The driver stopped the car. The glass broke.

4. He broke the glass. Sit there

We can notice in the following examples that a Transitive verb is one that is used with an object. In the following sentences open, fly, stopped and broke are transitive verbs. Similarly stopped, fly, broke and sit are intransitive verbs. So, we can conclude that a transitive verb is that which is used with an object and an intransitive verb does not have an object.

Change from Active Voice to Passive Voice:

Note the changes from Active Voice to Passive Voice in these sentences.

Active Voice Passive Voice

1. The hunter killed the lion. The lion was killed by the hunter.

DDE, GJUS&T, Hisar 451 | English (Compulsory) BA-101

2. I write a letter. A letter is written by me.

3. I eat an apple. An apple is eaten by me.

4. He bought a car. A car was bought by him.

As studied earlier it can be noticed that the sequence of the subject and the object of the sentence is reversed while converting the sentence from Active voice to passive voice.

Active Voice = Subject + Verb + Object

Passive Voice = Object + Verb + Subject

Let us see some more examples: - When Verb that take both a direct and an indirect object in the Active voice are changed to the Passive, either object may become the subject of the Passive Verb, while the other is retained.

Transitive verb with two objects

Sometimes Verb has two objects in which one object is person and the other is thing. Person is the indirect object and thing is the direct object. Remember, many a times Indirect object comes first and direct object comes at the second position. For example:

1. Mr. Sharma teaches us Hindi.

2. Ram gave a gift to me.

3. Mother told me a story.

Exercises

Change the following sentences so that the verbs will be in the Passive Voice.

1. He loves me.

2. She lights the candle.

3. He is driving a car.

DDE, GJUS&T, Hisar 452 | English (Compulsory) BA-101

4. She is watering the plants.

5. Who is singing a song?

6. I am not running a race.

7. She has not stolen my book.

Change the following sentences so that the verbs will be in the Active Voice:

1. All the milk was drunk by the cat.

2. I was not expected by them.

3. The windows have been cleaned.

4. A letter is written by me.

5. A camera is bought by him.

6. The dish had not been tested by me.

Study the Table:

Tense Active Voice Passive Voice

Present Indefinite 1st form+/s/es is/am/are/+ 3rd form

(Negative) do not/does not+1st form is/am/are+not+3rd form

Present continuous is/am/are+V1+ing is/am/are+being+V3

(Negative) is/am/are+not+1stform+ing is/am/are+not+being+3rd form

Present perfect has/have+3rd form has/have+been+3rd form

(Negative) has/have+not+3rd form has/ have +not been+3rd form

Past Indefinite 2nd form was/were+3rd form

(Negative) did not+1st form was/were+not+3rd form

Past continuous was/were+1st form +ing was/were+being+3rd form

(Negative) was/were+not+1st form + ing was/were+not+being+3rd form

DDE, GJUS&T, Hisar 453 | English (Compulsory) BA-101

Past perfect had+ 3rd form had been + 3rd form

(Negative) had not + 3rd form had not+ been+ 3rd form

Future Indefinite will/shall+1st form will/shall+ be+ 3rd form

(Negative) will/shall not + 1st form will/shall+ not be +3rd form

Future Perfect will/shall+ have + 3rd form will/shall+have+been+3rd form

5.5 Check your Progress -

A. Fill the gaps with the correct tenses.

1. I (learn) …… English for seven years now.

2. But last year I (not / work) …… hard enough for English, that's why my marks (not / be) … really that good then.

3. As I (pass / want) ……. my English exam successfully next year, I (study) harder… this term.

4. During my last summer holidays, my parents (send) …… me on a language course to London.

5. It (be) …. great and I think I (learn) …. a lot.

6. Before I (go) …. to London, I (not / enjoy) …. learning English.

B. Change into passive voice

1.We had lost the key. -

2.They had started a fight. –

3.The sun rises from the east

4.Reema can do skydiving.

5. I ate the strawberry pie

DDE, GJUS&T, Hisar 454 | English (Compulsory) BA-101

5.6 SUMMARY

The concept of English verb tenses is very important in establishing effective communication. Hence, if you want to maintain both ways of communication better, that is, speaking and writing you need to gain mastery over English tenses, because a command of twelve basic tenses of English language will aid you immensely in gaining effective communication skills. The term, tense, has been derived from the Latin word “tempus” meaning time. Since there are many ways in which we express the time of action, we use tenses.

 TYPES OF TENSES

Present- Simple Present -Base verb (+ es/es for third person):

I watch the news every day.

Present Continuous-am/is/are + present participle:

I am watching the news.

Present Perfect-Has/have + past participle:

I have watched the news already.

Present Perfect Continuous-Has/have + been + present participle:

I have been watching the news since I was a teenager.

Past -Simple Past- Verb+ ed or irregular verb:

I watched the news.

Past Continuous - Was/were + present participle:

I was watching the news.

Past Perfect- Had + past participle:

I had watched the news before I went to bed.

DDE, GJUS&T, Hisar 455 | English (Compulsory) BA-101

Past Perfect Continuous- I had been watching the news for 20 minutes before you came home.

Future- Simple Future- Will+ verb

Am/is/are + going to + verb

I will watch the news / I am going to watch the news.

Future Continuous- Will be + present participle

Am/is/are + going to be + present participle:

I will be watching the news at 9pm. / I am going to be watching the news at 9pm.

Future Perfect- Will have + past participle

Am/is/are + going to have + past participle

I will have watched the news before 10pm. / I am going to have watched the news.

Future Perfect Continuous- Will have been + present participle

Am/is/are + going to have been + present participle

 Active and Passive voice: Words come together to form a sentence and these sentences can be formed in more than one way. The way these sentences are made make a lot of difference in writing Active voice: When a subject is directly acting on the object, the sentence is written in Active voice Passive voice: When the object is acted upon by the subject, the sentence is written in Passive voice. In both the above sentences, the meaning remains the same and only the structure is what that changes. Usually, the structure or sequence of the subject, verb, and object expressed in the active voice sentence gets reversed in the passive voice of the same sentence. To understand the difference, just focus on how the subject and object change the structure of the sentences in the table below:

Active Voice Passive Voice

DDE, GJUS&T, Hisar 456 | English (Compulsory) BA-101

I ate the strawberry pie The strawberry pie was eaten by me

I bought a Honda car A Honda car was bought by meet object as the subject)

I bought a Honda car A Honda car was bought by me

The sun rises from the east East is where the sun rises from

Reema can do skydiving. Sky diving can be done by Reema

Note- how the meaning has stayed the same throughout.

5.7 KEYWORDS

Simple Present Tense: it is used to describe an action, event or condition in simple way.

Present Continuous Tense: It is used to denote an action which is going on at the time of speaking.

Present Perfect Tense: It is used in the sentences which began its action in the past but it will be completed in the present.

The Past tense is a grammatical tense whose principal function is to place an action or situation in past time.

Simple Future Tense: it is used to express an action which is believed to take place in future

Subject it refers to the person or thing that is doing the action expressed by the verb.

Object: Object in a sentence as the entity that is acted upon by the subject.

5.8 Self-Assessment Questions (SAQs)

Q1. Explain Present Tense and its types?

Q2. Describe Present Indefinite and Present Continuous tense with suitable examples?

DDE, GJUS&T, Hisar 457 | English (Compulsory) BA-101

Q3. Explain Past Tense and its types?

Q4. How will you differentiate between Past Perfect and Past Perfect Continuous tense?

Q5. Explain Future Tense with its types?

Q6. Write a short note on Active Voice with suitable examples.?

Q7. Write a short note on Passive Voice with suitable examples.?

Q8. Discuss in detail Transitive Verbs?

5.9 Answers to Check Your Progress-

A-Tenses

1. I have been learning English for seven years now.

2.But last year I was not working hard enough for English, that's why my marks were not really that good then.

3.As I want to pass my English exam successfully next year, I am going to study harder this term.

4.During my last summer holidays, my parents sent me on a language course to London.

5. It was great and I think I learned a lot.

6.Before I went to London, I had not enjoyed learning English.

B. voice -

1.The tables had been cleaned by Joe.

2.We had lost the key. - The key had been lost by us.

3, East is where the sun rises from hey had started a fight. –

4. A fight had been started by them.

5. Skydiving can be done by Reema.

DDE, GJUS&T, Hisar 458 | English (Compulsory) BA-101

6. The strawberry pie was eaten by me.

5.10 References/ Suggested Readings

• New Oxford Advanced Learners Dictionary. • English Grammar, Composition & Usage: J.C. Newfield, Revised by N.K. Agarwal. • English Grammar – An outline of General Phonetics; R.K. Bansal; OUP: 1971 • Spoken English for India; R.K. Bansal and J.B. Harrison; Longman. Intermediate English Grammar; Raymond Murphy etc.

DDE, GJUS&T, Hisar 459 | English (Compulsory) BA-101

Subject: English

Course Code- B.A.-101 Author: Dr. Astha Gupta

Lesson- 6 Editor: Dr. Shakuntla Devi

Introduction to Narration and Phrasal Verbs

Structure of Lesson

6.1 Learning Objectives

6.2 Introduction

6.3 Main Body of the Text

6.3.0 Reported Speech

6.3.1 Narration (Direct and Indirect speech)

6.3.2 Direct Speech

6.3.3 Indirect Speech

6.3.4 Change the adverbs of the Direct Speech.

6.3.5 Rules of change of Pronouns

6.4 Rules for Speech

6.4.1 Rules of change of verb or Tense

6.4.2 Rules for Change in Narration of different type of sentences

a. Assertive Sentences

b. Interrogative Sentences

6.4.3 Reporting exclamatory sentences

6.5 Further Main Body of the Text

DDE, GJUS&T, Hisar 460 | English (Compulsory) BA-101

6.5.0 Phrasal-Transitive Verbs

6.5.1 Phrasal verbs

6.5.2 Inseparable Phrasal Verbs (Transitive)

6.5.3 Three-Word Phrasal Verbs (Transitive)

6.6 Phrasal-Intransitive Verbs

6.6.1 Intransitive Phrasal Verbs

6.7 Check Your Progress

6.8 Summary

6.9 Keywords

6.10 Self-Assessment Questions (SAQs)

6.11 Answers to check your progress

6.12 References/ Suggested Readings

6.1 Learning Objectives:

• To make the students to choose words with greater precision

• To acquaint the students with the specialist vocabulary associated with communication and literary area.

• To make the students learn to communicate knowledge and ideas in appropriate way.

• To inculcate among the student’s skill to identify words and/or phrases related to the topic.

• This part will help you to analyze sentences structurally and functionally for the practical usage. -To give valuable knowledge of narration and phrasal verbs to students.

DDE, GJUS&T, Hisar 461 | English (Compulsory) BA-101

6.2 Introduction:

Vocabulary, or word meaning, is one of the keys to comprehension. Student's develop vocabulary indirectly through reading, listening, and speaking. Students’ background knowledge and prior experiences play a large role in vocabulary development. They build connections between known words and unknown words and develop a deeper understanding of their reading. Along with the knowledge of correct grammar, it is important to know its correct practical use. In this part, you will study sentences in larger context for social as well as academic purposes. You will learn to construct and synthesize the sentences. You will also learn to transform the sentences without changing their meaning.

In this unit, vocabulary exercises are provided. These exercises encourage students to figure out the meaning of unfamiliar words and expressions from context. The purpose of reading the prose, “On Saying Please” is to pose one or more questions for students to consider as they read the piece, giving them some aspect, feature, or idea on which to focus their attention. Students will be referred back to these questions after they read and discuss the piece to confirm their understanding. Expansion questions are interpretive and require a critical thinking. These questions lend themselves to various interpretations, and allow students to connect their personal experiences to the literature. The questions in this section deal with issues of values clarification, requiring students to reflect on their personal values as these relate to the unit themes.

6.3 Main Body of the Text

6.3.0 Reported Speech

6.3.1 Narration (Direct and Indirect speech)

When we want to report a conversation to someone who was absent at the time of actual conversation, we can do it by two ways. i) Quoting of the actual speech of the speaker is called Direct Speech.

DDE, GJUS&T, Hisar 462 | English (Compulsory) BA-101 ii) Reporting the speech of the speaker into one’s own words is called Indirect or Reported Speech. For Example: Leena said, I am helping my mother now.”

(Direct Speech) Leena said that she was helping her then. (Indirect Speech) The direct speech is marked with double inverted commas. Here ‘said’ is called principle or reporting verb. There are some changes made while changing the direct speech into indirect speech. Rules of changing the Direct Speech to Indirect /Reported Speech. i) Remove the double inverted commas and use the conjunction ‘that’ after the reporting verb. ii) If the reporting verb is in the past tense then change the whole speech into the past tense. But if the reporting verb is in the present tense then do not change the tense.

For Example: a) She said, “I like shopping.” (Reporting verb ‘said’ is in past tense. Direct speech – simple present tense) She said that she liked shopping. (Direct speech changes to simple past tense) b) she says, “I like shopping.” (Reporting verb ‘says’ is in present tense. Direct speech - simple present tense) She says that she likes shopping.

Note- The changes in the tense of the direct speech in other cases. c. I am going to the mall” said Vishal. (Present continuous tense)

Vishal said that he was going to the mall. (Past continuous tense) d) The teacher said, ‘Radha’ has passed with good grades.”(Present perfect tense)

The teacher told that Radha had passed with good grades. (Past perfect tense)

Art of reporting the words of a speaker is called Narration. There are two main ways of reporting the words of a speaker.

DDE, GJUS&T, Hisar 463 | English (Compulsory) BA-101 a. Direct Speech b. Indirect Speech

6.3.2 Direct Speech- In this form, the actual words of the speaker are put in quotes “”.

Ex: -

Rama said,” I am very busy now.”

In the above example the Speaker i.e

Rama is known as Reporter, said is known as Reporting verb and

“I am very busy now.” is known as Reported Speech.

6.3.3 Indirect Speech

In this form, the actual words of the speaker are transformed and said/written in a simple manner.

Ex: Rama said that he was very busy then. Basic rules to convert a Direct Speech to Indirect Speech. There are five basic things that have to be changed while converting a Direct speech to an Indirect speech. To change the reporting verb according to the reported speech. To remove the inverted comma ’s from the direct speech and replace them with an appropriate conjunction. To change the pronoun of reported speech accordingly.

6.3.4 Change the adverbs of the Direct Speech.

Active - Rajiv said to me,” I shall go to the picture today”

Passive - Rajiv told me that he would go to the picture that day.

In the above example-

Reporting verb said to is changed into told. Inverted Commas are replaced by the conjunction that Reported speech’s pronoun I is changed into He.

DDE, GJUS&T, Hisar 464 | English (Compulsory) BA-101

Reported speech’s verb shall go is changed into would go. As another change today is changed with the word that day.

6.3.5 Rules of change of Pronouns:

NominativePossessive Objective Reflexive

I My Me Myself

We Our Us Ourselves

You Yours You Yourself

He His Him Himself

She Her Her Herself

They Their Them Themselves

Pronouns are changed according to a rule designated as SON where

S stands for Subject

O stands for Object

N stands for No change.

First person changes to subject of Reporting Verb

Second person changes to Object of Reporting Verb

There is no change if it is a Third person.

Rule No 1.

1st Person pronoun of Reported speech is changed according to the Subject of Reporting verb.

Direct: She says, “I am in tenth class.

Indirect: She says that she is in tenth class.

Direct: I say, “I am an honest man.”

DDE, GJUS&T, Hisar 465 | English (Compulsory) BA-101

Indirect: I say that I am an honest man.

Rule No 2.- 2nd Person pronoun of Reported speech is changed according to Object of Reporting verb.

Direct: She says to me, “you have done your work”

Indirect: She tells me that I have done my work.

Direct: She says to her, “You have done your work”

Indirect: She tells her that she has done her work.

Direct: I say to them, “you have done your work.”

Indirect: I tell them that they have done their work.

Rule No 3- 3rd Person Pronoun of Reported speech is not changed.

Direct: He says, “He does not work hard”

Indirect: He says that he does not work hard.

Direct: Everybody says, “They have spoken the truth”

Indirect: Everybody says that they have spoken the truth

6.4 Rules for Speech

6.4.1 Rules of change of verb or Tense

Rule No.1

If reporting verb is given in Present or Future tense then there will be no change in the verb or tense of Reported speech.

Direct: The teacher says, “Gayatri performs on the stage”

Indirect: The teacher says that Gayatri performs on the stage.

Direct: The teacher is saying, “Gayatri performs on the stage”

Indirect: The teacher is saying that Gayatri performs on the stage.

DDE, GJUS&T, Hisar 466 | English (Compulsory) BA-101

Direct: The teacher will say, “Gayatri is performing on the stage”

Indirect: The teacher will say that Gayatri is performing on the stage.

Rule No.2

If reporting verb is given in Past tense then the tense of the verb of Reported Speech will change into corresponding Past tense.

Direct: The teacher said, “I am suffering from fever.”

Indirect: The teacher said that she was suffering from fever.

Direct: She said to me, “I took the breakfast in the morning”.

Indirect: She told me that she had taken the breakfast in the morning.

-- Corresponding Changes to past form in an indirect speech from the verb in Reported speech.

Simple present changes to Simple Past

Present Continuous changes to Past Continuous

Present Perfect changes to Past Perfect

Present Perfect Continuous changes to Past Perfect Continuous

Simple Past changes to Past Perfect

Past Continuous changes to Past Perfect Continuous

In Future Tense will/Shall changes to would

Can changes to Could

May changes to Might

Some of the exceptional cases of Rule 2

Exception 1:

DDE, GJUS&T, Hisar 467 | English (Compulsory) BA-101

If Reporting speech has Universal Truth or Habitual fact then there is no change in the Tense.

Direct: Our teacher said, “The earth is round”

Indirect: Our teacher said that the earth is round. (Universal Truth)

Direct: Rajiv said to me, “He plays with right hand”

Indirect: Rajiv told me that he plays with the right hand. (Habitual fact)

Exception 2:

If reporting speech has Past Historical Fact then there is no change in the Tense.

Direct: Our teacher said, “Asoka left war after the conquest of Kalinga”.

Indirect: Our teacher said that Asoka left war after the conquest of Kalinga.

Exception 3:

If Reporting speech has two actions to be happening at a time when there is no change in the Tense.

Direct: She said “My wife was making lunch when I was studying”

Indirect: She said that her wife was making lunch when she was studying.

Exception 4:

If Reporting speech has some Imagined Condition then there is no change in the Tense.

Direct: She said, “If I were rich, I would help him.”

Indirect: She said that if she were rich, she would help her.

Exception 5:

If Reporting speech contains had 3rd form, to-infinitive and would, could, should, must, might, ought to etc. then there is no change in the Tense.

Direct: She said to me, “You should obey your elders.”

DDE, GJUS&T, Hisar 468 | English (Compulsory) BA-101

Indirect: She told me that I should obey my elders.

Some other small changes that used to take place while changing Direct Speech to Indirect Speech.

Here Changes to There

Now Changes to Then

This Changes to That

These Changes to Those

Today Changes to That day

To-night Changes to That night

Yesterday Changes to the previous day

Last night Changes to the previous night

Last week Changes to the previous week

Tomorrow Changes to the next day

Next Week Changes to the following week

Ago Changes to Before

Thus, Changes to so

Hence Changes to Thence

Hither Changes to Thither

Come Changes to Go

Note- in indirect speech we talk about such incidents that are after the time of reporting and had happened away from the place of reporting therefore the words that show nearness has to be replaced by the words that show distance.

Some exception in these changes-

DDE, GJUS&T, Hisar 469 | English (Compulsory) BA-101

1.Come is changed to go if there is some word given after come that shows nearness.

2. If this, here and now points to such a thing, place or time that is in front of the speaker then no change takes place in Indirect Narration.

6.4.2 Rules for Change in Narration of different type of sentences a. Assertive Sentences

Rule 1-

If there is no object after Reporting verb there it should not be changed.

If there is some object after Reporting verb then say is changed to tell, says to tells and said to told.

According to the context said to can be replaced by replied, informed, stated, added, remarked, asserted, assured, pleaded, reminded, reported or complained etc.

Rule 2

Put conjunction that in place of “ ”.Change the pronouns of the Reported speech as enlisted earlier.

Examples showing some Assertive Sentences Changed into Indirect form.

Direct: She said to me, “I shall sleep now”

Indirect: She told me that she would play then.

Direct: He said, “My brother’s marriage comes off next month.”

Indirect: He said that his brother’s marriage would come off the following month. b. - Interrogative Sentences

Rule 1

An interrogative sentence is meant to ask questions, therefore, Reporting verb said/said to is changed to asked.

Said to can also be changed into enquired or demanded

DDE, GJUS&T, Hisar 470 | English (Compulsory) BA-101

Rule 2

If question is formed with the help of any of the helping verbs like is/are/am, was/were, has/have, do/does, will/would etc then “ __” are to be replaced by if or whether

If the question is formed with the help of words starting with “Wh ‘ like who, whose, what, whom, when etc (also known as W family) or How then to replace “___” no conjunction is used.

Rule 3

In these sentences question form of the sentence is removed and full-stop is put at the end of the sentence.

Helping verb is /are/am, was/were etc should be put after the subject.

If the interrogative sentence is expressing positive feeling then do/does of the Direct speech is removed while converting it into Indirect speech.

If the interrogative sentence is expressing negative feeling then do/does of the Direct speech is changed into did while converting it into Indirect speech.

Did or has/have the interrogative sentence is changed to had.

Rule 4

Pronouns and verbs are changed according to the set rule of Narration. 4. Modals: c. Modal verbs like shall, will, can, may change in reported speech. Let’s follow some examples:

Direct: John said, “I will be there.”

Indirect: John promised that he would be there.

Direct: The boy said, “May I come in?”

Indirect: The boy asked if he could come in. (note: may become could when it implies permission)

DDE, GJUS&T, Hisar 471 | English (Compulsory) BA-101

Direct: “I may not call you.” Said Bobby.

Indirect: Bobby said that she might not call me.

Direct: “I shall practice more.” said Barbara.

Indirect: Barbara said that she would practice more.

Direct: Joseph said, “Shall I buy the car?”

Indirect: Joseph asked if he should buy the car.

Note: shall become should if it implies a question.

6.4.3 Reporting exclamatory sentences:

To change direct exclamatory speeches to the indirect one we need to replace interjection (hurrah, wow, alas, oh, etc.) with joining clause ‘that’ and the exclamatory wh-words (what, how) will be replaced by ‘very’ before the adjective in the reported clause.

Examples:

Direct: Clare said, “Hurrah! Barcelona won the match!”

Indirect: Clare exclaimed with joy that Barcelona had won the match.

Direct: I said, “Alas! My pet died.”

Indirect: I exclaimed with grief that my pet had died. We will use ‘to’ as joining clause before the reported command or request, and the reported verb will be changed according to the moods of the sentence (e.g., ordered, requested, urged, advised, forbade or begged)

Example:

Direct: The man said, “Please, bring me a chair.”

Indirect: The man requested to bring him a chair.

Direct: The officer said, “Fall back!”

DDE, GJUS&T, Hisar 472 | English (Compulsory) BA-101

Indirect: The officer ordered to fall back.

Direct: Mother said, “Listen to your elders.”

Indirect: Mother advised me to listen to my elders.

Direct: Mr. Murphy said, “Do not go near the house.”

Indirect: Mr. Murphy forbade going near the house.

6.5 Further Main Body of the Text

6.5.0 Phrasal-Transitive Verbs:

6.5.1 Phrasal verbs-

Phrasal verbs are formed by adding particles with verbs. This particle which is added is either a preposition or an adverb. But the whole phrasal verb has one defi nite meaning. Change of particle added leads to change in the meaning of the phrasal verb. It is very important to note that phrasal verbs have completely different meanings from the parts of which they are made of. Now some of the important phrasal verbs are given for you to learn. You should pay special attention to their meanings and you are not to be misguided by the meaning of particle added to the verb. You should try to use them English can be used in both formal or polite way and informal, familiar or colloquial manner. Those who learn English in the classroom and not exposed amply to outside the classroom, may not have access to its colloquial variety. A phrasal verb is combination of a standard verb (especially native) like ‘go’ or ‘look’ and a pre position or an adverb which are called particles. This combination forms a new verb with totally different meaning. For example, a. She put a box on the table. b. She put out a candle.

In the sentence (b) put out means ‘stop something from burning’. In the sentence (a) put has its common meaning ‘keep’.

DDE, GJUS&T, Hisar 473 | English (Compulsory) BA-101

Phrasal verbs are important because they are extremely common in colloquial English. If you are not familiar with them, you will find informal English difficult to understand. A good dictionary has the e ntries of the phrasal verbs. If you want to master phrasal verbs and consequently English, develop a habit to look for phrasal verbs. There are three types of phrasal verbs:

1. Phrasal verbs (intransitive) that do not take an object:

blow in: My brother blew in unexpectedly with his family. (visit unexpectedly)

come along: If you want, you can come along. (accompany)

2. Phrasal verbs (transitive) with an Object:

hand back: Will she hand back my book? (return)

pick out: Sunita picked out a lively suit. (choose)

3. Phrasal verbs with two particles: stand up for: Everybody must stand up for the truth. (defend) put up with: Nobody should put up with injustice. (tolerate) English can be used in both formal or polite way and informal, familiar or colloquial manner. Those who learn English in the classroom and not exposed amply to outside the classroom, may not have access to its colloquial variety. A phrasal verb is combination of a standard verb (especially native) like ‘go’ or ‘look’ and a preposition or an adverb which are called particles. This combination forms a new verb with totally different meaning. For example,

You have to do this paint job over.

You have to do over this paint job.

When the object of the following phrasal verbs is a pronoun, the two parts of the phrasal verb must be separated:

You have to do it over.

DDE, GJUS&T, Hisar 474 | English (Compulsory) BA-101

Verb Meaning Example

Blow Up Explode The terrorists tried to blow up the railroad station.

Bring up Mention a topic My mother brought up that little matter of my prison record Again.

Bring up Raise children It isn't easy to bring up children nowadays.

Call Off Cancel They called off this afternoon's meeting

Do Over Repeat a job Do this homework over.

Fill Out Complete a form Fill out this application form and mail it in.

Fill up Fill to Capacity She filled up the grocery cart with free food.

Find out Discover My sister found out that her husband had been

planning something.

Give Away Give something to The filling station was giving away free gas.

someone else for free

Give back Return an object My brother borrowed my car. I think he's not about to give it Back.

Hand in Submit something The students handed in their papers and left the room. (Assignment)

Hang up Put something on She hung up the phone before she hung up her clothes.

hook or receiver

Hold up Delay I hate to hold up the meeting, but I have to go to the

bathroom.

Hold up (2) Rob Three masked gunmen held up the Security Bank this afternoon.

Leave out Omit You left out the part about the police chase down Asylum Avenue.

Look over Examine, Check, the lawyers looked over the papers carefully before

questioning the witness. (They looked them over carefully.)

DDE, GJUS&T, Hisar 475 | English (Compulsory) BA-101

Look up Search in a list You've misspelled this word again. You'd better look it up.

Make up Invent a story or lie She knew she was in trouble, so she made up a story about going to the movies with her friends.

Make out Hear, Understand He was so far away, we really couldn't make out what he was saying.

Pick out Choose There were three men in the line-up. She picked out the guy she thought had stolen her purse.

Pick up Lift something off The crane picked up the entire house. (Watch them pick it up.)

something else

Point out Call attention to As we drove through Paris, Francoise pointed out the major

historical sites.

Put away Save or Store We put away money for our retirement. She put away the cereal

boxes.

Put off Postpone We asked the boss to put off the meeting until tomorrow. (Please put it off for another day.)

Put on Put clothing on the I put on a sweater and a jacket. (I put them on quickly.)

body

Put out Extinguish The firefighters put out the house fire before it could spread. (They

put it out quickly.)

Read over Peruse I read over the homework, but couldn't make any sense of it.

Set up To arrange, begin My wife set up the living room exactly the way she wanted it. She

set it up.

Take down Make a written note These are your instructions. Write them down before you forget.

Take off Remove clothing It was so hot that I had to take off my shirt.

Talk over Discuss We have serious problems here. Let's talk them over like adults.

DDE, GJUS&T, Hisar 476 | English (Compulsory) BA-101

Throw away Discard That's a lot of money! Don't just throw it away.

Try on Put clothing on to see She tried on fifteen dresses before she found one, she liked.

if it fits,

Try out Test I tried out four cars before I could find one that pleased me.

Turn down Lower volume Your radio is driving me crazy! Please turn it down.

Turn down (2) Reject He applied for a promotion twice this year, but he was turned

down both times.

Turn up Raise the volume Grandpa couldn't hear, so he turned up his hearing aid.

Turn off Switch off electricity We turned off the lights before anyone could see us.

Turn off (2) Repulse It was a disgusting movie. It really turned me off.

Turn on Switch on the Turn on the CD player so we can dance.

electricity

Use up Exhaust, use The gang members used up all the money and went out to rob some

completely more banks.

6.5.2 Inseparable Phrasal Verbs (Transitive) - With the following phrasal verbs, the lexical part of the verb (the part of the phrasal verb that carries the "verb-meaning") cannot be separated from the prepositions (or other parts) that accompany it: "Who will look after my estate when I'm gone?"

Verb Meaning Example

Call on Ask to recite in class The teacher called on students in the

back row.

Call on (2) Visit The old minister continued to call on his

sick parishioners.

DDE, GJUS&T, Hisar 477 | English (Compulsory) BA-101

Get over Recover from sickness I got over the flu, but I don't know if I'll

or disappointment ever get over my broken heart

Go over Review The students went over the material before

the exam. They should have gone over it twice.

Go through Use up; consume They country went through most of its coal

reserves in one year. Did he go through all his

money already?

Look after Take care of My mother promised to look after my dog while

I was gone.

Look into Investigate The police will look into the possibilities of

embezzlement.

Run across Find by chance I ran across my old roommate at the college

reunion.

Run into Meet Carlos ran into his English professor in the

hallway.

Take after Resemble My second son seems to take after his mother.

Wait on Serve It seemed strange to see my old boss wait on

tables.

6.5.3 Three-Word Phrasal Verbs (Transitive)- With the following phrasal verbs, you will find three parts: "My brother dropped out of school before he could graduate."

Verb Meaning Example

Break in on Interrupt (a conversation) I was talking to Mom on the phone when

the operator broke in on our call.

DDE, GJUS&T, Hisar 478 | English (Compulsory) BA-101

Catch up with Keep abreast After our month-long trip, it was time to

catch up with the neighbours and the news

around town.

Check up on Examine, Investigate Boys promised to check up on the condition of

the summer house from time to time.

Come up with To contribute After years of giving nothing, the old parishioner

(suggestion, money) was able to come up with a thousand-dollar

donation.

Cut down on Curtail (expenses) We tried to cut down on the money we were

spending on entertainment.

Drop out of Leave school I hope none of my students drop out of school

this semester.

Get along with Have a good relationship I found it very hard to get along with my brother

with when we were young.

Get away with Escape blame Man cheated on the exam and then tried to get

away with it.

Get rid of Eliminate The citizens tried to get rid of their corrupt mayor in the recent election.

Get through with Finish When will you ever get through with that program?

Keep up with Maintain pace with It's hard to keep up with the Joneses when you lose

your job!

Look forward to Anticipate with pleasure I always look forward to the beginning of new.

Look down on Despise It's typical of a jingoistic country that the

DDE, GJUS&T, Hisar 479 | English (Compulsory) BA-101

citizens look down on their geographical neighbours.

Look in on Visit (somebody) We were going to look in on my brother-in-law, but

he wasn't home.

Look out for Be careful, anticipate Good instructors will look out for early signs of failure in their students

Look up to Respect First-graders really look up to their teachers.

Make sure of Verify Make sure of the student's identity before you let him into the classroom.

Put up with Tolerate The teacher had to put up with a great deal of nonsense from the new students.

Run out of Exhaust supply The runners ran out of energy before the end of the

race.

Take care of Be responsible for My oldest sister took care of us younger children after Mom died.

Talk back to Answer impolitely The star player talked back to the coach and was thrown off the team.

Think back on Recall I often think back on my childhood with great

pleasure.

Walk out on Abandon Her husband walked out on her and their three

children.

6.6 Phrasal-Intransitive Verbs:

6.6.1 Intransitive Phrasal Verbs - The following phrasal verbs are not followed by an object: "Once you leave home, you can never really go back again."

DDE, GJUS&T, Hisar 480 | English (Compulsory) BA-101

Verb Meaning Example

Break down Stop That old jeep had a tendency to break down functioning just when i needed it the most.

Catch on Become Songs seem to catch on in California first and popular then spread eastward.

Come back Return to a Father promised that we would never come place back to this horrible place.

Come in Enter They tried to come in through the back door, but it was locked.

Come to Regain He was hit on the head very hard, but after consciousness several minutes, he started to come to again.

Come over To visit The children promised to come over, but they never do.

Drop by Visit without We used to just drop by, but they were never appointment home, so we stopped doing that.

Eat out Dine in a When we visited Paris, we loved eating out in restaurant the sidewalk cafes.

Get by Survive Uncle Heine didn't have much money, but he always seemed to get by without borrowing money from relatives.

Get up Arise Grandmother tried to get up, but the couch was too low, and she couldn't make it on her own.

Go back Return to a It's hard to imagine that we will ever go back to place Lithuania.

Go on Continue He would finish one dickens novel and then

DDE, GJUS&T, Hisar 481 | English (Compulsory) BA-101

just go on to the next.

Go on (2) Happen The cops heard all the noise and stopped to see what was going on.

Grow up Get older Charles grew up to be a lot like his father.

Keep away Remain at a The judge warned the stalker to keep away from distance his victim's home.

Keep on Continue with He tried to keep on singing long after his voice (with gerund) the same was ruined

Pass out Lose He had drunk too much; he passed out on the consciousness, sidewalk outside the bar. faint

Show off Demonstrate Whenever he sat down at the piano, we knew haughtily he was going to show off.

Show up arrive Day after day, Efrain showed up for class twenty minutes late.

Wake up Arouse from I woke up when the rooster crowed sleep

Some Other Verbs are as Follows:

Blow up: explode bring in: Yield as the result of sale. blow out: extinguish break down: Demolish; collapse; fall. bring up: Educate or rear. break out: Appear suddenly. call for: demand break into: Enter by force. call forth: evoke break up: Dissolve. call off: divert; distract call on: visit bring about: Cause.

DDE, GJUS&T, Hisar 482 | English (Compulsory) BA-101 call up: recollect fall out: quarrel carry on: manage fall upon: attack carry out: execute fall through: fail cast off: discard get at: obtain cast out: expel get back: recover come about: occur get down: descend come across: meet with accidentally get on: advance; make come by: acquire progress get on with: live agreeably come of: issue from with come upon: encounter get over: surmount cry out against: complain loudly get through: pass against get up: rise cry up: extol praise give away: distribute cut down: reduce cut off: kill, destroy give in: submit; yield give up: abandon; surrender cut out for: fitted for do away with: abolish go after: pursue go forward: proceed drawback: recede draw up: compile; arrange go on: continue fall back: retreat go through: examine fall back upon: have recourse to hold back: keep back; conceal some help hold on: continue fall in with: concur with holding

DDE, GJUS&T, Hisar 483 | English (Compulsory) BA-101 hold out: endure lay out: invest hold up: support, sustain look after: deposit keep back: conceal look after: take care of keep from: refrain from look known upon: despise keep under: control look for: sear for keep up: maintain turn on: switch on lay by: save for future use turn out: expel lay down: surrender turn up: arrive

6.7 Check Your Progress-

1.Fill in the blanks with appropriate phrasal verbs.

Complete with one of these phrasal verbs: be through, go on, fill in, take off, stay out, speak up.

1. Could you ………. this application form, please?

2. I´ll never talk to you again. We …!

3. If you don´t ………., we can´t hear you.

4. I´m tired because I ………. too late last night.

5. The plane ………. late because of the bad weather.

2. Complete with one of these phrasal verbs: go off, put off, see off, take off, turn off.

1. Let´s go to the airport to ………. them ……….

2. The plane doesn´t ………. till 5 o´clock.

3. He was sleeping soundly when the alarm clock ……….

DDE, GJUS&T, Hisar 484 | English (Compulsory) BA-101

4. The meeting has been ………. till next month.

5. Don´t forget to ………. all the lights when you leave.

3. Complete with: carry on, get on, hold on, put on, try on.

1. Hi! Is Mr. Knight in?

………., I´ll call him.

2. Excuse me, could I ………. this dress, please?

3. How are you ………. at college?

4. Are you still ………. with your tennis lessons?

5. It was a bit chilly, so she ………. her jacket.

4. Complete with: give up, hang up, look up, wake up, wash up.

1. The kind of housework I hate most is ……….

2. If he rings back, just ……….

3. I didn´t know that word, so I ………. it ………. in a dictionary.

4. Don´t ……… You can do it if you try hard.

5. When I ………. in the middle of the night, I had some temperature.

5. Match the phrasal verbs with their corresponding synonyms.

1. put off a. cancel

2. call off b. switch off

3. look up c. postpone

4. go off d. continue

5. carry on e. explode

6. turn off f. check1. I said to him, “Will you go to Delhi?”

2.change appropriate direct speech into indirect speech.

DDE, GJUS&T, Hisar 485 | English (Compulsory) BA-101

1. I said to him, “Will you go to Delhi?

(1) I asked him will he go to Delhi.

(2) I said to him would he go to Delhi.

(3) I asked him if he would go to Delhi.

(4) I said to him would you go to Delhi.

2. He said, ‘I have read this novel.’

(1) He said that he has read this novel.

(2) He said that he had read that novel.

(3) He said that he read that novel.

(4) He said that he had read this novel.

3. Tania said to her friend, ’Can you lend me an umbrella?’

(1) Please give me an umbrella Tania requested her friend.

(2) Will you lend me your umbrella; Tania asked her friend.

(3) Tania requested her friend to lend her an umbrella

(4) Tania asked her friend to give her an umbrella.

4. Sita said, ‘I may go there.’

(1) Sita says that she may go there.

(2) Sita says that she Is going there.

(3) Sita said that she will go there.

(4) Sita said that she might go there.

5. My friend requested me to bring him a sandwich.

(1) He said, ‘My friend, please bring me a sandwich.’

(2) My friend said, ‘Will you bring me a sandwich.’

DDE, GJUS&T, Hisar 486 | English (Compulsory) BA-101

(3) ‘Please bring me a sandwich’, said my friend.

(4) ‘Please bring my friend a sandwich’, said he.

6. He said, ‘If I had the tools, I could mend the car.’

(1) He said that if he had the tools, he could mend the car.

(2) He tells that he had the tools he could mend the car.

(3) He said I could mend the car if I have the tools.

(4) He told he could mend the car.

7. She said to the children, ‘You mustn’t play with fire.

(1) She told the children they were not to play with fire.

(2) She told the children that they mustn’t play with fire.

(3) She told the children not to play with fire.

(4) She told the children they are not to play with fire.

8. Everybody says, ‘How well she sings!’

(1) Everybody comments that she sings well.

(2) Everybody says how well she sings.

(3) Everybody tells that she sang very well.

(4) Everybody says that she sang well.

9. He says that he is very sorry.

(1) He said, ‘He was very sorry’.

(2) He says, ‘I am very sorry’.

(3) He said, ‘He is very sorry’.

(4) He told me. ‘I felt sorry’.

DDE, GJUS&T, Hisar 487 | English (Compulsory) BA-101

10. Daman says, ‘Priya is working in Ahmedabad.’

(1 Daman say that Priya was working in Ahmedabad.

(2 Daman said that Priya was working in Ahmedabad.

(3) Daman said that Priya works in Ahmedabad.

(4) Daman says that Priya is working in Ahmedabad.

11. Anne said. ‘It is time to leave for the meeting.’

(1) Anne said that it was time to leave for the meeting.

(2) Anne told that it is time to leave for the meeting.

(3) Anne said its time to leave for the meeting.

(4) Anne said that it was time for the meeting.

12. The old man said that he would soon be leaving the world.

(1) The old man said, “I shall soon be leaving the world.”

(2) The old man said. “I am soon leaving the world.”

(3) The old man said, “I would soon leave the world.”

(4) The old man said, “I am going to leave the world soon.”

6.8 Summary –

There are three degrees of comparison-positive, comparative and superlative .Paired conjunctions can be used to join two sentences without changing the meaning Depending on the point of view of presentation of the sentence, There are two voices: Active, and Passive There are three tenses in English grammar: Present, Past, and Future. They are further divided into simple, continuous, perfect and perfect continuous. Speech can be reported in two ways: Direct, and Indirect.

Phrasal verbs are important because they are extremely common in colloquial English. If you are not familiar with them, you will find informal English difficult to

DDE, GJUS&T, Hisar 488 | English (Compulsory) BA-101 understand. A good dictionary has the entries of the phrasal verbs. If you want to master phrasal verbs and consequently English develop a habit to look for phrasal verbs. There are number of phrasal verbs which is very useful to make appropriate sentences.

Reported Speech- i) Remove the double inverted commas and use the conjunction ‘that’ after the reporting verb. ii) If the reporting verb is in the past tense then change the whole speech into the past tense. But if the reporting verb is in the present tense then do not change the tense.

For Example: a) She said, “I like shopping.” (Reporting verb ‘said’ is in past tense. Direct speech – simple present tense) She said that she liked shopping. (Direct speech changes to simple past tense) b) she says, “I like shopping.” (Reporting verb ‘says’ is in present tense. Direct speech - simple present tense) She says that she likes shopping.

Note- The changes in the tense of the direct speech in other cases. sentence)

She enquired whether she can come to my house. b) The judge said to the accused, answer the question.”(Imperative sentence)

The judge commanded the accused to answer the question. c) She said, ‘How generous of me to help me!” (Exclamatory sentence)

She exclaimed that it was very generous of him to help her. d)”do not do it again please. ”She told him. (Request)

She begged him not to do that again.

DDE, GJUS&T, Hisar 489 | English (Compulsory) BA-101

10) He is a better basketball player than everyone else in the team. (Change the degree to superlative)

11) Teacher announced, "I have corrected all the answer sheets." (Change to indirect speech)

6.9 Keywords-

 Quoting of the actual speech of the spe aker is called Direct Speech.  Reporting the speech of the speaker into one’s own words is called Indirect or Reported Speech. For Example: Leena said, I am helping my mother now.”  Remove the double inverted commas and use the conjunction ‘that’ after the reporting verb.  If the reporting verb is in the past tense then change the whole speech into the past tense.  . A good dictionary has the entries of the phrasal verbs.  If you want to master phrasal verbs and consequently English, develop a habit to look for phrasal verbs. –  Phrasal verbs are formed by adding particles with verbs. This particle which is added is either a preposition or an adverb.

6.10 Self- Assessment Questions (SAQs):

1. Explain direct and indirect speech?

2. Write down 15 phrasal verbs and explain it with example?

3. Use direct sentences into indirect speech with explanation?

6.11 Answers to check your progress

Exercise 1- 3. speak up

1. fill in 4. stayed out

2. are through 5. takes off / took off

DDE, GJUS&T, Hisar 490 | English (Compulsory) BA-101

Exercise 2- 4. go off – explode

1. see them off 5. carry on – continue

2. take off 6. turn off – switch off

3. went off

4. put off 2. Answers of direct speech into

5. turn off indirect speech 1. (3) ,2. (2) ,3. (3) ,4. (4), 5. (3), 6. Exercise 3- (1), 1. hold on 7. (2) , 8. (1) , 9. (2 ,10. (4), 2. try on 11. (1), 12. (1) 3. getting on

4. carrying on

5. put on . Exercise 4-

1. washing up

2. hang up

3. looked it up

4. give up

5. woke up

Exercise 5-

1. put off – postpone

2. call off – cancel

3. look up – check

6.12 References/ Suggested Readings

DDE, GJUS&T, Hisar 491 | English (Compulsory) BA-101

 New Oxford Advanced Learners Dictionary.  English Grammar, Composition & Usage: J.C. Nesfield, Revised by N.K. Agarwal  English Grammar – An outline of General Phonetics; R.K. Bansal; OUP: 1971  Spoken English for India; R.K. Bansal and J.B. Harrison; Longman.  Intermediate English Grammar; Raymond Murphy etc.

DDE, GJUS&T, Hisar 492 | English (Compulsory) BA-101

Subject: English

Course Code- B.A.-101 Author: Dr. Astha Gupta

Lesson- 7 Editor: Dr. Shakuntla Devi

Introduction to Comprehension -seen and unseen passages

Structure of Lesson

7.1 Learning Objective

7.2 Introduction

7.3 Main Body of the Text

7.4 Unseen Passages

7.5 Seen Passages

7.6 Check Your Progress

7.7 Summary

7.8 Keywords

7.9 Self-Assessment Questions (SAQs)

7.10 Answers to Check Your Progress

7.11 References /Suggested Readings

7.1 Learning Objectives

After studying this unit, you will be able to:

• This unit helps you to understand the concept of comprehension.

DDE, GJUS&T, Hisar 493 | English (Compulsory) BA-101

• It presents the concepts of ideas, facts, agreement, purpose, tone and attitude

• Explain seen and unseen passages

• Give answers based on unseen passages

• Give answers based on seen passages.

7.2 Introduction

Comprehension of an unseen passage means a complete and thorough understanding of the passage. The main object of comprehension is to test one’s ability to grasp the meaning of a given passage properly and also one’s ability to answer, in one’s own words, the questions based on the passage. A variety of questions like short answer type questions, completion of incomplete sentences, filling the blanks with appropriate words and exercises based on vocabulary are set forth for the purpose. Before attempting to answer the questions on a passage, it is necessary to read the passage again and again so that a general idea of the subject of the passage becomes clear. Once the passage is clear, it is easy to answer the answers of the questions.

One should also keep the following points in mind before answering the questions set on a given passage:

1. Read the passage quickly to have some general idea of the subject matter.

2. Read the passage again and underline the important points.

3. Read the questions and try to know what has been asked.

4. Read the passage again and underline the portions where the probable answers may be available.

5. Use, as far as possible, your own words to answer the questions in a precise and brief way.

DDE, GJUS&T, Hisar 494 | English (Compulsory) BA-101

6. Always use complete sentences while answering a question.

7. If you are asked to give the meaning of some words or phrases, try to express your idea, in your own words, as clearly as possible.

8. Don’t give your own opinions or comments about anything unless you are asked to do so.

7.3 Main Body of the Text

7.4 Unseen Passages

Passage 1

Read the following passages carefully and answer the questions set below:

There is a story of a woman who thought She had a right to do what she liked. One day, this gentlelady was walking along a busy road, spinning her walking-stick round and round in her hand, and was trying to look important. A man walking behind her’ objected You ought not to spin your walking-stick round and round like that!” She said. ”I am free to do what I like with my walking-stick,” argued the gentlewoman.

‘Of course, you are,” said the other man, “but you ought to know that your freedom ends where my nose begins.” The story tells us that we can enjoy our rights and our freedom only if they do not interfere with other people’s rights and freedom.

Questions-

1. Why was the gentle woman on the road moving his walking stick round and round?

2. Who objected her?

3. What argument did the lady give?

4. Was the other satisfied with argument?

5. What did she say in reply?

DDE, GJUS&T, Hisar 495 | English (Compulsory) BA-101

6. Complete the following statements with the correct options:

A. The gentle woman was walking along a……………………….

(i) lonely road.

(ii) busy road.

(iii) narrow road.

B. The gentle woman was ……………………….

(i) running along the road.

(ii) disturbing others on the road.

(iii) spinning his walking-stick round and round.

C. The man who protested was a……………………….

(i) teacher.

(ii) passer-by.

(iii) policeman.

7. Write True or False against each of the following statements:

(a) The gentlewoman was spinning the walking-stick round and round in his hand to drive away the dogs.

(b) The gentlewoman was walking along a busy road.

(c) The man walking behind praised her action.

(d) The gentle woman thought that she had a right to do whatever she liked.

(e) We can enjoy our rights and freedom even if it interferes with other people’s rights and freedom.

8. Give synonyms of the following words:

(a) Spinning

DDE, GJUS&T, Hisar 496 | English (Compulsory) BA-101

(b) Interfere

Answers

1. The gentlewoman on the road was moving her walking stick round and round because she wanted to look important.

2. A woman walking behind him objected her.

3. The gentle woman argued that he was free to do what liked with her walking-stick.

4. No, the other woman was not satisfied with his argument.

5. The other man said that he ought to know that her (the gentlewoman’s) freedom ends where her (the passer-by’s) nose begins.

6. A (ii), B. (iii), C. (ii)

7. (a) false, (b) true, (c) false, (d) true, (e) false.

8. (a) Spinning—Moving (b) Interfere—Meddle

Passage 2

On August 15,1947, India was granted independence from British colonial dominance. This was an event of international significance. From this day onwards, the Indian Union assumed a role in world politics that was appropriate enough to modify the thinking of nations. Concepts like non-alignment, tolerance, non-violence and peaceful co-existence were introduced by India into the international vocabulary. ‘Our dreams concern India'', said the first Prime Minister of the country, Pt Jawaharlal Nehru, on the eve of achieving freedom, ''but they also concern the whole world, for the world has become one.'' Mahatma Gandhi chose Nehru to become the first PM of independent India. With his vision of unity of the nation, he became the

DDE, GJUS&T, Hisar 497 | English (Compulsory) BA-101 architect of modern India. It was to be a federal state based on secularism- the commitment to an Indian identity, which was above all races, castes, and religions. It was a vision of extraordinary dimensions. Nehru as well as his daughter and successor, , were well aware that only a firm anchorage in the Indian identity could unite the nation and enable it to make an impact on the world. In these days of mutual interdependence, the western powers and America cannot afford to ignore what is happening in this region. It is possible that the fate of the Asian democracies would one day be decided on the banks of the . If India fails to remain a democracy, this would trigger the fall of the whole of South and Southeast Asia. That is why the Indian Prime Minister has to play a role that goes well beyond duties related to internal affairs only. It is of international political significance.

1) What made Pt. Nehru the architect of modern India?

Ans. due to his vision of the unity of the nation.

2) Which concepts have been introduced by India into the international vocabulary?

Ans. non-alignment, tolerance, non-violence and peaceful coexistence

3) Which word in the passage describes the phrase to live together with mutual tolerance?

Ans. co-existence

4) Which word describes the word 'trigger' used in the passage?

Ans. aggravate (Give choices)

5) Select the word synonymous to Dimensions

(a) support (b) size (c) importance (d) height

Ans.(b)

DDE, GJUS&T, Hisar 498 | English (Compulsory) BA-101

6) Why does the role of the Indian Prime Minister go beyond the duties related to the internal affairs of the country?

Ans. because these are the days of mutual interdependence and the fate of Asian democracies depends upon India.

7) Select the word that is opposite in meaning to dominance used in the passage

(a) government (b) rule (c) subordination (d) importance

Ans. (c)

8) Give a suitable title to the passage.

Ans. Role of the Indian PM (any other title will do)

9) How can India make an impact on the world?

Ans.- a firm anchorage in the Indian identity.

10) How does the passage define secularism?

Ans. the commitment to an Indian identity which was above all races, castes and religions.

Passage 3

Wild Life and Forest Conservation

India plays an important role in wildlife conservation. The love for wildlife is a part of Indian culture. It is a densely populated country and people have different religions with their own faith in God. The people think that different animals are related to different gods, which ensure their conservation. The wildlife means all organisms living in their natural habitat. The concern for wildlife is concern for man himself. Because all the species are interrelated, for example, in an ecosystem if the number of a particular species increases or decreases, whole ecosystem is disturbed or imbalanced. So, in addition to the economic importance, the wildlife balances

DDE, GJUS&T, Hisar 499 | English (Compulsory) BA-101 population and maintains food chain and natural cycle. Therefore, it helps to preserve environment as a self- sustaining system.

The wildlife is conserved by conservation of forests and at present by latest gene technologies. Many programs are held today in the field of agriculture, animal husbandry and fishery. Plant breeders are also able to produce high yi elding and disease resistant varieties. The genes of animal are also stored in the gene libraries. The cloning is also there and there are more chances of cloning in future. Thus, man has a responsibility for conservation of wildlife with help of In-situ and Ex-situ conservation. In-situ conservation is primarily related to the natural biodiversity within the evolutionary dynamic ecosystems of the original habitat or natural environment. It is best, easiest, most advantageous, besides being most feasible method to conserve biological diversity. In-situ conservation includes a comprehensive system of protective areas. These include National Parks Sanctuaries, Nature Reserves, Cultural Landscapes, Biosphere reserves and several others.

Ex-situ conservation is conservation outside their habitats by perpetuating sample populations in genetic resource centers, zoo, botanical gardens, culture collections etc, or in the form of gene pools and storage for fish, germplasm banks for seeds, pollen, semen, ova, cells etc.

On the basis of your reading of the above passage, answer the following questions.

(a) How do the Indian religions and culture help in the protection of wild life?

(b) How are the different species related to one another?

(c) What is meant by in-situ conservation?

(d) What is ex-situ conservation?

(e) Frame two questions on the basis of the underlined part of the passage.

(f) Find the correct options to complete the given sentences:

DDE, GJUS&T, Hisar 500 | English (Compulsory) BA-101

(A) Forests can help in ...

(i) the production of high yielding and disease restraint variety.

(ii) ex-situ conservation.

(iii) in-situ conservation.

(iv) cloning of animals.

(B) Ex-situ conservation means ...

(i) conservation of wildlife in natural environment.

(ii) conservation of wildlife outside its natural environment.

(iii) perpetuation of a large number of animals.

(iv) Biosphere Reserves.

(g) Use the words ‘concern’ and ‘chain’ as verbs.

(h) Find words in the passage which mean the opposite of the following: -

(i) destruction (para 1) (ii) prone (para

Answers

(a) The love of wildlife is a part of Indian culture. Moreover, the people think that different animals are related to different gods. Thus, Indian religions and culture help in the protection of wildlife.

(b) Different species are related to one another in an ecosystem. If the number of a species increases or decreases, the whole system is disturbed.

(c) In-situ conservation means the conservation of the species within the ecosystems of the original habitat. It is the best, easiest and most advantageous. It is also the most feasible method to conserve biological diversity.

DDE, GJUS&T, Hisar 501 | English (Compulsory) BA-101

(d) Ex-situ conservation is the conservation outside the habitats of the animals. It is done by perpetuating sample population in genetic resource centers. Many species have been kept alive solely through these facilities.

(e) (i) Why is it necessary to worry about wildlife?

(ii) Why is every species very important?

(f) (A) (i) in-situ conservation.

(B) (ii) conservation of wildlife outside its natural environment.

(g) Concern: Drug abuse concerns us all.

Chain: The convict was chained.

(h) (I) conservation (ii) resistant.

Passage 4

An important aspect of the growth of modern towns and cities is the increasing population density. Mr. J. P. Orr, the Honourable Chairman of the Mumbai Improvement Trust, pointed out in a recent lecture on the subject, how it affected the health and prosperity of the inhabitants. Life in most thriving towns is intimately connected with the local trades and industries. Unhealthy conditions in factories and workshops, and offensive trades have been prevented in big towns, and this had led to better health. The question of density is more difficult to deal with in India, because, older towns have been built on different principles. People not only want to live healthy but also live in communities that give them greater social convenience, comfort and safety. They care for the health, comfort and beauty of the town, and these conditions of a well-built and well-arranged town are still insisted on by the people. So as long as individuals obey the laws of health, and their habits and customs are free from insanitary effects, mere density of population does not perhaps tend to increase the death rate and harbour diseases. But in the present day, it is apparent that the habits of people

DDE, GJUS&T, Hisar 502 | English (Compulsory) BA-101 have changed greatly and they do not obey the laws of health and cleanliness as well as their forefathers used to do in days when cities expanded and people flourished. It is, therefore, necessary to modify the old plans of city buildings in the light of modern sanitary laws and requirements. In this lecture, Mr. Orr spoke mainly of the density of the city of Mumbai. But his observations are of considerable interest to all those whose population is huddled in narrow quarters, without adequate air and light.

1) The density of population does not increase the death rate and harbour diseases as long as… (Complete the sentence)

Ans. People obey the laws of health and their habits are free from insanitary effects.

2) Why is it necessary to modify the old plans of city buildings?

Ans. Because the habits of people have greatly changed and they do not obey the health laws.

3) Life in most thriving towns is connected with … (Complete the sentence)

Ans. local trade and industry.

4) In addition to health what do the people want in towns?

Ans. social convenience, comforts and safety.

5) What has led to better health in big towns?

Ans. prevention of unhealthy conditions in factories and workshops.

6) What should be kept in mind while modifying the old plans of city buildings?

Ans. Modern sanitary laws and requirements.

DDE, GJUS&T, Hisar 503 | English (Compulsory) BA-101

7) The word 'thriving' in the above passage means a) populated b) modern c) growing d) well planned

Ans. c)

8) Select a synonym for 'to huddle' a) grow b) heaped together c) scatter d) sporadic

Ans. b)

9) Select the right antonym for 'to harbour' a) shelter b) cherish c) banish d) infectious

Ans. b)

10) Which city did Mr. Orr spoke of?

Ans. Mumbai

7.5 Seen Passages

Passage 1.

Read the following paragraph and answer the questions given below:

Among the chief sources of education available to Tagore, was, a quiet garden adjoining his family house. Here he used to spend much of his time, absorbing the peace and beauty of nature. It was through this early contact with nature that he acquired the serenity of mood that distinguished him all his life. It was in this garden that he came to understand the principle of harmony that was at work throughout the Universe. At the same time, he formed the habit of observing and reflecting on things.

1) How did Tagore spend much of his time in the garden?

Ans. absorbing the peace and beauty of nature

2) How did the garden near Tagore's house serve him?

Ans. as a chief source of education

3) What did Tagore come to understand about universe?

DDE, GJUS&T, Hisar 504 | English (Compulsory) BA-101

Ans. The principle of harmony.

4) Give the antonym of .. a) ugliness b) chaos from the passage

Ans. a) beauty b) harmony

5) Give a suitable title

Ans. Tagore A Nature [Answer can vary

Passage 2

Human intelligence is too vast and subtle a phenomenon to be reduced to a trio of digits. It’s even hard to say what we mean by smart. The world is full of brilliant poets who can’t balance a cheque book, and genius physician incapable of driving a manual-shift car. Understanding social cues, creating works of art and spawning inventions are all crucial mental tasks that bear little relationship to how well a person can fill a printed test form. It’s worth remembering too that IQ isn’t quite the same thing as intelligence. As Stephen Jay Gould pointed out in his 1983 book, The Mis measure of Man, the mere fact that we can consistently measure something, in this case, IQ doesn’t mean that it has any significance or correspondence to any intuitive, man-on-the street concept. By way of analogy, if we measured everybody’s height and divided it by his or her weight, we could come up with a heaviness quotient–‘HQ’. After years of research, we might find that Europeans are slimmer than Chinese, get more exercise or are more robust in some vague, undefined way. Without additional information we couldn’t tell. Likewise, a person’s measured IQ may relate only indirectly to a layman’s notion of being smart.

Based on your reading of the above passage, answer the following questions as briefly as

Possible.

DDE, GJUS&T, Hisar 505 | English (Compulsory) BA-101

(a) Why is it difficult to define Smartness?

(b) Give two examples from the passage of work which the author calls ‘Crucial mental tasks.

(c) Give in your words Stephen Jay’s view about IQ.

(d) How far is a common man’s notion of intelligence related to IQ?

(e) Find words from the passage which mean the same as the following:

(i) rarefied (ii) decisive.

Answers-

(a) Smartness is a vague term. Some people may be capable of doing great things but incapable of doing things that many common men can easily do. So, it is very difficult to define smartness.

(b) The two examples are:

(i) creating works of art (ii) making inventions.

(c) Stephen Jay believes that IQ is not the same thing as the common man’s concept of intelligence.

(d) The common man’s notion of intelligence is only vaguely related to IQ. The truth is that these two things are quite different from each other. (e) (i) subtle (ii) crucial.

Passage 3

There is only one natural satellite —the Moon. But there are many man-made or artificial satellites revolving around the Earth. Aryabhatt, IRS-IB and INSAT-2A are some Indian artificial satellites. They are more than 300 km away from the Earth. At this height, the Earth’s gravity and atmosphere do not affect them. Artificial satellites are of many kinds. Of them, weather satellites send information and pictures of clouds, storms and ocean currents. They are used to predict the weather. This information is also used to warn people about storms, floods, forest fires, etc.

DDE, GJUS&T, Hisar 506 | English (Compulsory) BA-101

Communication satellites are used to transmit telephone conversation and TV programmes across the Earth. They are helpful in sending messages through fax machines, using mobile phones and the internet. Science satellites send information about outer space. Astronomers study this data and get to know more about things in space. These satellites are launched into space by rockets. Once beyond the Earth’s gravity and atmosphere, they are sent circling around the Earth by the force of another rocket. There is nothing in space to obstruct the path or speed of these satellites. So they continue to keep going around the Earth, just like the Moon.

1. Answer the following questions briefly

(a) How are artificial satellites launched into space?

(b) Why are artificial satellites kept at a distance of more than 300 km away from the Earth?

(c) Mention any two kinds of artificial satellites.

(d) How is the information sent by communication satellites used?

(e) Find words from the passage which mean the same as the following:

(i) unnatural (ii) forecast.

2. Find words/phrases from the passage which mean the same as the following:

(a) facts/things certainly known (and from which conclusions may be drawn)

(b) set in motion/space

(c) block up/put something in the way of

7.6 Check your progress-

Read the following passage carefully and answer the questions given below.

Passage 1.

Among the chief sources of education available to Tagore, was, a quiet garden adjoining his family house. Here he used to spend much of his time, absorbing the

DDE, GJUS&T, Hisar 507 | English (Compulsory) BA-101 peace and beauty of nature. It was through this early contact with nature that he acquired the serenity of mood that distinguished him all his life. It was in this garden that he came to understand the principle of harmony that was at work throughout the Universe. At the same time, he formed the habit of observing and reflecting on things.

1) How did Tagore spend much of his time in the garden?

2) How did the garden near Tagore's house serve him?

3) What did Tagore come to understand about universe?

4) Give the antonym of

a) ugliness b) chaos from the passage?

5) Give a suitable title?

Passage -2

On August 15,1947, India was granted independence from British colonial dominance. This was an event of international significance. From this day onwards, the Indian Union assumed a role in world politics that was appropriate enough to modify the thinking of nations. Concepts like non-alignment, tolerance, non-violence and peaceful co-existence were introduced by India into the international vocabulary.

''Our dreams concern India'', said the first Prime Minister of the country, Pt Jawaharlal Nehru, on the eve of achieving freedom, ''but they also concern the whole world, for the world has become one.'' Mahatma Gandhi chose Nehru to become the first PM of independent India. With his vision of unity of the nation, he became the architect of modern India. It was to be a federal state based on secularism- the commitment to an Indian identity, which was above all races, castes, and religions.

It was a vision of extraordinary dimensions. Nehru as well as his daughter and successor, Indira Gandhi, were well aware that only a firm anchorage in the Indian identity could unite the nation and enable it to make an impact on the world. In

DDE, GJUS&T, Hisar 508 | English (Compulsory) BA-101 these days of mutual interdependence, the western powers and America cannot afford to ignore what is happening in this region. It is possible that the fate of the Asian democracies would one day be decided on the banks of the Ganges. If India fails to remain a democracy, this would trigger the fall of the whole of South and Southeast Asia. That is why the Indian Prime Minister has to play a role that goes well beyond duties related to internal affairs only. It is of international political significance.

1) What made Pt. Nehru the architect of modern India?

2) Which concepts have been introduced by India into the international vocabulary?

3) Which word in the passage describes the phrase to live together with mutual tolerance?

4) Which word describes the word 'trigger' used in the passage?

5) Select the word synonymous to Dimensions?

(a) support (b) size (c) importance (d) height

6) Why does the role of the Indian Prime Minister go beyond the duties related to the internal affairs of the country?

7) Select the word that is opposite in meaning to dominance used in the passage?

(a) government (b) rule (c) subordination (d) importance

8) Give a suitable title to the passage?

9) How can India make an impact on the world?

10) How does the passage define secularism?

Passage- 3

Read the following passage carefully and answer the questions given below.

An important aspect of the growth of modern towns and cities is the increasing population density. Mr. J. P. Orr, the Honourable Chairman of the Mumbai

DDE, GJUS&T, Hisar 509 | English (Compulsory) BA-101

Improvement Trust, pointed out in a recent lecture on the subject, how it affected the health and prosperity of the inhabitants. Life in most thriving towns is intimately connected with the local trades and industries. Unhealthy conditions in factories and workshops, and offensive trades have been prevented in big towns, and this had led to better health. The question of density is more difficult to deal with in India, because, older towns have been built on different principles. People not only want to live healthy but also live in communities that give them greater social convenience, comfort and safety. They care for the health, comfort and beauty of the town, and these conditions of a well-built and well-arranged town are still insisted on by the people. So as long as individuals obey the laws of health, and their habits and customs are free from insanitary effects, mere density of population does not perhaps tend to increase the death rate and harbour diseases. But in the present day, it is apparent that the habits of people have changed greatly and they do not obey the laws of health and cleanliness as well as their forefathers used to do in days when cities expanded and people flourished. It is, therefore, necessary to modify the old plans of city buildings in the light of modern sanitary laws and requirements.

In this lecture, Mr. Orr spoke mainly of the density of the city of Mumbai. But his observations are of considerable interest to all those whose population is huddled in narrow quarters, without adequate air and light.

1) The density of population does not increase the death rate and harbour diseases as long as… (Complete the sentence).?

2) Why is it necessary to modify the old plans of city buildings?

3) Life in most thriving towns is connected with … (Complete the sentence)?

4) In addition to health what do the people want in towns?

5) What has led to better health in big towns?

6) What should be kept in mind while modifying the old plans of city buildings?

7) The word 'thriving' in the above passage means?

DDE, GJUS&T, Hisar 510 | English (Compulsory) BA-101

a) populated b) modern c) growing d) well planned

8) Select a synonym for 'to huddle'?

a) grow b) heaped together c) scatter d) sporadic

9) Select the right antonym for 'to harbour'?

a) shelter b) cherish c) banish d) infectious

10) Which city did Mr. Orr spoke of?

Passage 4

Read the following passage and answer questions given below it.

At one time in the history of India, most women knew very well how to bring up their infants and they lived a perfectly healthy life, free from diseases. The overall standard of women and children in the country was much better than those of other civilizations of that period. But ever since India was exposed to frequent invasions by foreign nations, life became unsafe and property was unprotected, and people were forced to congregate in towns in such a compact way, that it led to awful insanitation and diseases. The traditional knowledge of domestic and personal health and hygiene was ignored. Women were confined indoors for fear of insults and a train of social and unhealthy dangers followed. It is a problem now, how we can restore the original conditions of healthy and happy life in India. This is a socio-economic problem which needs to be given priority to bring back the original culture and restore welfare of women and children in India.

1) What was the main cause of the poor health conditions of women in India?

2) What question has the writer posed before the readers?

3) How did life become unsafe and property unprotected?

4) Why were the women confined indoors?

5) When were women leading a perfectly healthy life?

DDE, GJUS&T, Hisar 511 | English (Compulsory) BA-101

6) What does the word ' congregate ' mean in the passage?

7) How are foreign invasions responsible for poor health conditions?

8) Write the antonym of ' confine ' from the following

a) forced b) directed c) to keep out d) reject

9) Give a suitable title to the passage.

Passage 5

Read the following passage carefully and answer the questions given below:

Swami Dayanand was the first to use the word 'Swayraja' even before Dada bhai Naoroji and Tilak used it. The British regarded Arya Samaj as a sediti ous organization and during the martial law days (1919) students of DAV College, Lahore, were ordered to go for roll call by the police thrice a day. It is now being increasingly accepted that during the Sepoy Revolt of 1857 Dayanand played a historic role, not as a soldier but by inspiring the freedom fighters. It was (and is even now) taught in school textbooks that the Aryans migrated here from Central Asia. Dayanand was the first to debunk that view and assert that the Aryans were the original residents of Arya Vrat (India); now many historians agree with him. Indians told the British rulers: ''You quit because you are foreigners.'' Their reply was “You too were migrants from other lands. Only you came much before us. So first you quit and then we will see." They invented this theory of Aryan migration as a part of their imperial policy of 'divide and rule'. The aim was to put the Dravidians - according to them original residents of this country against the Aryan invaders. Swami Dayanand said: " Ours is not any independent religion. I am under the and my duty is to preach the Truth.'' Dayanand in his own humble way said: " I am not a know all. If you discover any error in me, you may correct it after reason and argument." "" Anything that cannot be justified by reason and Common sense must be rejected as irreligious - an idea too revolutionary for his age.

(1) During the Sepoy Revolt of 1857, what was the role of Swami Dayanand?

DDE, GJUS&T, Hisar 512 | English (Compulsory) BA-101

(2) What did the Britishers think of Arya Samaj?

(3) What did Swami Dayanand consider it his duty?

(4) Which other leaders used the word Swaraj?

(5) What viewpoint of history books was debunked by Swami Dayanand?

(6) Which idea of Swami Dayanand was considered revolutionary?

(7) What role did Swami Dayanand play during the Sepoy Revolt?

(8) Who were considered as original residents of India?

(9) Give the Vedic name of India.

Passage 6

Read the following passage and answer the questions given below it.

It was in Germany and France that the first successful attempts were made to produce an internal-combustion engine driven by petrol. In England people were strangely timid about horseless vehicles. English inventors were handicapped by a quaint old law that forbade any such vehicle to attain a greater speed than four miles an hour, and compelled each one to be preceded by a man carrying a red flag. This law was not repealed until 1896. The earliest motor cars were looked upon as mere jokes, or as rather dangerous playthings, by everyone except their inventors. Some of them were single-seaters, others would carry two or even three people; but all were noisy, clumsy, queer-looking things. When in 1888, Carl Benz, a German, produced a three wheeled internal-combustion car, a great forward stride had been made. Another German, whose name, Daimler, is often seen on motor cars to this day, was experimenting about the same time, and testing a petrol-driven engine. It is easy to understand how the introduction of the petrol driven engine revolutionized road transport throughout the world. Until then the necessary power to push a vehicle along could not be obtained without the cumbersome tanks, boilers and the furnaces of the stem engine. The internal- combustion engine is light in weight and small in

DDE, GJUS&T, Hisar 513 | English (Compulsory) BA-101 size by comparison, the fuel is burned in it, so that there is no waste, like the dusty cinders of a coal fire.

1) How did most people regard early motor cars?

2) What made the English inventors handicapped?

3) What does 'repeal' mean?

4) Give the antonym of 'clumsy'.

5) How were the earliest motor cars designed?

6) What did Carl Benz design?

7) What revolutionized road transport?

8) How did the internal-combustion engines score over their counterparts?

9) Give a word for ' outdated' from the first paragraph?

10) Write a suitable title for the passage.

7.7 Summary

Comprehension of an unseen passage means a complete and thorough understanding of the passage. The main object of comprehension is to test one’s ability to grasp the meaning of a given passage properly and also one’s ability to answer, in one’s own words, the questions based on the passage.

The word comprehension means understanding. Understanding the passage means the following. In every passage, there are three different flows. On the printed page, what the reader physically sees is a flow of words, phrases and sentences. Underlying these words, phrases and sentences there are rational thought processes which consist of facts, ideas and arguments.

7.8 Keywords

Passage: A paragraph based on the story.

Seen passage: A passage seen by you.

DDE, GJUS&T, Hisar 514 | English (Compulsory) BA-101

Unseen passage: A passage not seen by you

7.9 Self- Assessment Questions (SAQs)

State whether the following statements are true or false.

1. Comprehension of an unseen passage means a complete and thorough understanding of the passage.

2. There is a story of a man who thought he had a right to do what he liked.

3. The story tells us that we can enjoy our rights and freedom anywhere and any type.

4. Differentiate seen and unseen passages.

5. Write a seen passage.

7.10 Answers to Check your progress

1. True 2. True 3. False

Passage-1 Answers

1- Absorbing the peace and beauty of nature

2- As a chief source of education

3- The principle of harmony.

4). a) beauty b) harmony ,5) - Tagore A Nature [Answer can vary

Passage- 2 Answers

1 - Due to his vision of the unity of the nation.

2- non-alignment, tolerance, non-violence and peaceful coexistence

3- co-existence

4- aggravate (Give choices)

5) - (b)

DDE, GJUS&T, Hisar 515 | English (Compulsory) BA-101

6) - because these are the days of mutual interdependence and the fate of Asian democracies depends upon India.

7) - (c)

8) - Role of the Indian PM (any other title will do)

9) - A firm anchorage in the Indian identity.

10- The commitment to an Indian identity which was above all races, castes and religions1)

Passage-3 Answers

1. People obey the laws of health and their habits are free from insanitary effects.

2) - Because the habits of people have greatly changed and they do not obey the health laws.

3) - local trade and industry.

4) - social convenience, comforts and safety.

5) - prevention of unhealthy conditions in factories and workshops.

6 - Modern sanitary laws and requirements.

7)- c)

8)- b)

9)- b)

10) - Mumbai

7.11 References /Suggested Readings

 Books English for Competitive Exams, By DR. R.P. Bhatnagar  Unique Quintessence of General English, Edited by DR. S. Sen & Others and revised by DR. G.S. Mansukh ani.

DDE, GJUS&T, Hisar 516 | English (Compulsory) BA-101

 A Background to the Study of English Literature, D.K. Patnaik, Swastik Publications  Online links: o www.writingcentre.uottawa.ca/hypergrammar/preposit.html o http://www.dl.ket.org/latin2/grammar/ch34-degofadj.htm

DDE, GJUS&T, Hisar 517 | English (Compulsory) BA-101

Subject: English

Course Code- B.A.-101 Author: Dr. Astha Gupta

LESSON- 8 Editor: Dr. Shakuntla Devi

Introduction to Paragraph Writing

Structure of Lesson

8.1 Learning Objectives

8.2 Introduction

8.3 Main Body of the Text

8.3.0 Paragraph and Its Types

8.3.1 Narrative Paragraphs

8.3.2 Descriptive Paragraphs

8.3.3 Explanatory Paragraphs

8.3.4 Compare and Contrast Paragraphs

8.3.5 Defining Paragraphs

8.3.6 Classifying Paragraphs

8.3.7 Persuasive or Argumentative Paragraphs

8.4. Further Main Body of the Text

8.4.0 Structure of Paragraph

8.4.1 Title of Paragraph

8.4.2 Third Point and Support

8.4.3 Thesis statement and A Good Thesis

DDE, GJUS&T, Hisar 518 | English (Compulsory) BA-101

a. A Thesis Statement

b. A Good Thesis

8.4.4 Elements of Paragraph

8.4.5 Writing Topics of Paragraphs

8.5 Check Your Progress

8.6 Summary

8.7 Keywords

8.8 Self- Assessment Questions (SAQs)

8.9 Answer to Check Your Progress

8.10 References /Suggested Readings

8.1 Learning Objectives

After studying this unit, you will be able to:

• Write a paragraph

• Explain all types of paragraph.

8.2 Introduction

It is a group of sentences that introduces, presents and develops one main idea about the topic. And it can be divided into three major parts.

1. The Topic Sentence

 It is normally the first sentence of the paragraph.  It conveys the overall point of the paragraph.  It helps the writer focus on the idea written about.  It helps the reader know about what the paragraph is all about.

DDE, GJUS&T, Hisar 519 | English (Compulsory) BA-101

2. The Supporting Details

 They are sentences used to support the main idea stated in the topic sentence.  They give more information about the main idea through examples.  They say in details what the topic sentence says in general.  They should be clear evidence that what the topic sentence says is trustworthy.  They should be strong convincing points on which the topic sentence can rely upon.

3. The Concluding Sentence

 It is a reflection of the main idea pronounced in the topic sentence.  It sums up what the topic sentence and the supporting details talk about.  It is the closing sentence that reminds the readers of what they have to value.  It is compulsory for the completion of the paragraph unity.  It eventually indicates the end of a paragraph.  It prepares the reader for a smooth transition to the next paragraph if there is one.

8.3 Main Body of the Text

8.3.0 Paragraph and Its Types:

A paragraph is a collection of sentences that deal with one subject. This is a paragraph—all of these sentences talk about what a paragraph is. An effective paragraph consists of a topic sentence, sentences that support this topic (the body of the paragraph), and a conclusion. The topic sentence in this paragraph is the first one, where the word is defined. Everything after that sentence is the body of this paragraph. The conclusion of this paragraph is the last sentence. When you change the topic, you start a new paragraph—I will change to a new paragraph next, to discuss different types of paragraphs and how to write an effective paragraph. A paragraph can contain as many sentences and words as you need—just be sure that you have said everything you need to say before you conclude the paragraph.

DDE, GJUS&T, Hisar 520 | English (Compulsory) BA-101

Did u know? Each paragraph should tell your reade r about one subject, and should leave them with a good idea of whatever you are talking about.

Types of Paragraph:

There are seven or eight different types of paragraph.

8.3.1 Narrative Paragraphs—

Narrative paragraphs are often used to describe what a pe rson does over a period of time. A narrative paragraph tells a story. The events in a narrative paragraph are usually arranged in chronological order. These are the paragraphs that tell you what is going on in a story, and move things along.

These sentences all lead the reader toward the idea that a paragraph is just a way of communicating.

Read this example narrative paragraph, notice how words like ‘later’ are used to connect what happens.

Yesterday evening I got home from work at 6 o’clock. My wife had prepared dinner which we ate immediately. After I had cleaned up the kitchen, we watched TV for about an hour. Then we got ready to go out with some friends. Our friends arrived at about 9 o’clock and we chatted for a while. Later we decided to visit a jazz club and listen to some music. We really enjoyed ourselves and stayed late. We finally left at one o’clock in the morning. For example -

1. You can own Clyde “Sweet Feet” Livingston’s shoes. Just make a bid. The best part is the money you give for the famous sneakers will be tax deductible since it is going to charity.

2. Latvia is a country in north-eastern Europe. It is bordered by Estonia to the north and Lithuania to the south. Russian and Belarus are to the east. In the west Latvia is bordered by the Baltic Sea. The capital of Latvia is Riga which is located on the northern shore. Forty-one percent of the country is covered with forests. Latvia holds over 12,000 small rivers and over 3,000 lakes.

DDE, GJUS&T, Hisar 521 | English (Compulsory) BA-101

3. For Sale Eat Plenty of Onions. Good for digestion, the liver, the stomach, the lungs, the heart, and the brain.

4. To make a diorama of the Texas desert, you will need the following materials: a shoebox, construction paper, sand, clay, plastic animals, and cactus. First cover the inside of the shoebox with construction paper to make the sky. Next place a layer of sand to make the desert floor. Plant cactus in the sand. Finally add plastic animals or animals made from clay to your desert scene.

5. July 20 I must leave today for Camp Green Lake. I’ve never bee n to camp before. This could really be an adventure. I’m kind of looking forward to the experience.

6. O Sinners, Let’s go down

Let’s go down, let’s go down

O Sinners, Let’s go down

Down in the valley to pray

Show me the way

Good Lord, show me the way

Show me the way

Good Lord just show the way

7. Forecast for Camp Green Lake Clear skies. Low 75°. High 105°. Winds 10 to 15 mph.

8. Reward for the person who gives information to help in the recovery of Clyde “Sweet Feet” Livingston’s shoes.

9. A man was sitting with his feet up on a desk. He turned his head when Stanley and the guard entered, but otherwise didn’t move. Even though he was inside, he wore sunglasses and a cowboy hat. he also held a can of soda, and the sight of it made Stanley even more aware of his own thirst.

DDE, GJUS&T, Hisar 522 | English (Compulsory) BA-101

10. Number One Bully Service Derrick Dunne Bully I will help you get the job done. Toilets are my specialty.

8.3.2 Descriptive Paragraphs—

These paragraphs give descriptions of something so that you can form a mental image of what is going on. It let the reader touch, taste, see, hear and smell what you are describing. The reader should feel as if they can see what you are describing clearly. You want to paint a picture as you write the descriptive paragraph.

Here are a few guidelines to help you write a great descriptive paragraph.

 Describe particular smells and tastes in the paragraph. Use the most descriptive words possible to allow the reader to smell or taste what you are describing. For example: “The homemade cookies filled the air with the scent of warm chocolate, and the chocolate morsels filled your mouth with the taste of cocoa.”  Add the senses of touch and hearing to your paragraph wherever possible. Describe certain textures and sounds. For example: “The silk garment felt smooth and fluid over my skin, and it had the sound of a gentle breeze.”  Use similes and metaphors when you write your descriptive paragraph. These literary devices strengthen your paragraph if used properly.  Insert descriptive adjectives to modify your nouns. Don’t just say “blue ocean.” Describe the actual colours you see in the ocean. Use more descriptive words such as aquamarine or indigo to describe the shade.  Try personification to give human characteristics to inanimate objects. For example: “The tree stood proudly with her arms stretching toward the sky.”  Be sure to make your paragraph long enough to give an adequate description. Describe the scene or object in as many ways as you can, but check that your paragraph is coherent.

Here is an example of a descriptive paragraph:

DDE, GJUS&T, Hisar 523 | English (Compulsory) BA-101

I am sixty years old, rather tall and I have blue eyes and short black hair. I wear casual clothes as I teach students in a relaxed atmosphere. I enjoy my job because I get to meet and help so many different people from all over the world. During my spare time, I like playing tennis which I play at least three times a week. I also love listening to classical music and I must admit that I spend a lot of money on buying new CDs! I live in a pretty seaside town on the Italian coast. I enjoy eating great Italian food and laughing with the likable people who live here.

Answer these questions about yourself.

• How old are you?

• What do you look like?

• What kind of clothes do you wear? Why?

• What kind of job do you do? Do you like it?

• What are your favourite hobbies? Why do you like them?

• Where do you live?

• Do you like living there? Why or why not?

Answer (Exercise 1)

• I am forty years old.

• I teach students. It means that I am teacher.

• I wear casual clothes as I teach students in a relaxed atmosphere.

• I do teach. I enjoy my job because I get to meet and help so many different people from all over the world.

• My hobbies are playing tennis, listening classical music and buying new CDs.

• I live in a pretty seaside town on the Italian coast.

DDE, GJUS&T, Hisar 524 | English (Compulsory) BA-101

• I like to live there because I enjoy eating great Italian food and laughing with the likable people who live here.

8.3.3 Explanatory Paragraphs—

This is sometimes divided into “Explaining With Examples “and “Explaining a Process”. Either way, these paragraphs provide an explanation for something, so that you can understand it better. This whole paragraph is an explanatory one!

In order to write a paragraph, first you think about what you want to say. Pretend that you are explaining things to your friends, or to a younger person. Try to explain in simple terms that are easy to follow. Once you have thought about it, start writing down what you would say out loud. That’s all you need to do to write a paragraph.

8.3.4 Compare and Contrast Paragraphs—

These are the paragraphs that give similarities and differences between things. Paragraphs are like conversations. Each conversation is a series of statements, questions, or explanations that pass along information. Each paragraph is also a series of sentences that pass along information. A paragraph is different from a conversation because a paragraph can be edited and changed after you write it down, and a conversation can’t be taken back once you have spoken the words.

8.3.5 Defining Paragraphs—

These paragraphs give you a definition for some term. A definition tells you what a word or term means. This paragraph tells you what a defining paragraph is, so this paragraph is a defining paragraph about defining paragraphs! When you define something, you want to use simple words so that your reader will understand what you are saying.

8.3.6 Classifying Paragraphs-

DDE, GJUS&T, Hisar 525 | English (Compulsory) BA-101

These are paragraphs which divide something into groups or categories. This entire section is a classifying paragraph which tells you the different kinds of paragraph that you can use!

8.3.7 Persuasive or Argumentative Paragraphs-

These are paragraphs that try to convince the reader to agree with something. Writing a good paragraph just takes practice. You will be able to write well if you keep at it! Anyone at all can learn how to write a good paragraph, even if they don’t make perfect grades or speak wonderful English. All you have to do is be willing to practice writing, and you can do it! A hortatory exposition is a special type of argument that is written in specific language. To write hortatory exposition, you use words that focus on the writer instead of on the reader (I, me, mine). You also use more abstract language such as passive voice (“it was done” instead of “they did it”) and present tense instead of the usual past tense (“I am in town” instead of “I was in town”). Hortatory exposition is just an argument which is phrased in a less emotional, more passive voice. The way that you write paragraphs is simply to pretend that you are talking to someone. Instead of telling them whatever you want to say, you write it down instead. Here are some good tips for writing effective paragraphs:

• “Tell Them What You Are Going To Tell Them”—writing is the same as making a speech —first, you want to give the audience an idea of what is coming up. This will be your topic sentence, and should give a pretty good idea of what the paragraph is going to be about. A good topic sentence should be specific instead of general, and should convey some sort of emotion—either an attitude, a belief, or a conviction.

• “Tell Them”—next, you write your supporting sentences—be sure that each one supports the topic sentence—if you think of a sentence that goes off on a tangent or starts a new topic, put it into another paragraph.

DDE, GJUS&T, Hisar 526 | English (Compulsory) BA-101

• “Tell Them What You Just Told Them”—your conclusion sentence should repeat the basic idea of the topic sentence using different words.

You might also keep in mind these additional tips:

• Unity and Coherence—Your paragraph should all be about the same topic, without wandering around discussing many different things. You should also be as coherent as possible, use simple language instead of big words whenever possible, link your sentences with bridges (see next tip), and use logical arguments and facts.

• Bridges—You can link the sentences and paragraphs by using key words which you repeat throughout your writing, by using synonyms and similar words, or by following a logical argument and proceeding step-by-step throughout. Using some sort of order, such as chronological (time) or structural order can help link paragraphs. The reader can guess what is coming next by knowing how time works, or by following along as you describe items in a series.

• Development—Make sure your topic sentence is adequately discussed in the paragraph. While it is possible to have a one -sentence paragraph, you will usually need several sentences to discuss the topic. Use facts, statistics, and details. Cite what other people have said about the topic (remember to use quotes and give credit where due). Give a timeline if possible. Give examples in a story or anecdote. Define terms and explain similarities and differences. Describe causes and consequences.

• Transitions and Signposts—you can use words and phrases to alert your readers and let them know what’s going on in your paragraph. Transition words and sentences help your ideas flow from one paragraph to another, and contain phrases like “in addition,” “another point,” or “afterwards.” Signpost words and sentences “point the way” to let your readers know where your arguments and descriptions are headed—a signpost could be a bold word or phrase, a dot or arrow, or even an indentation. Signposts are another way to “tell them what you are going to tell them” and “tell them what you just told them.”

DDE, GJUS&T, Hisar 527 | English (Compulsory) BA-101

Notes Use a “hook” or interesting fact to make people want to read your paragraphs.

8.4 Further Main Body of the Text

8.4.0 Structure of a Paragraph

Topic Sentence To generate interest Yellow-spotted lizards are a deadly States the main idea threat to all humans at Camp Green Lake.

Details Introduces Main Point #1 They invaded the area after the life- First Point and gives information or giving lake dried up. Their bite is Support examples always fatal. Details Introduces Main Point #2 It was a yellow-spotted lizard that Second Point and gives information or killed Kate Barlow. and Support examples.

Details Introduces Main Point #3 The yellow-spotted lizards threatened Third Point and gives information or Stanley and Zero while they were digging and Support examples for Stanley’s great-grandfather’s suitcase. The reason Stanley and Zero survived is because they had eaten so many onions that the lizards, who do not like onion blood, did not wish to bite them.

Conclusion Restates the topic sent- With this one exception, meeting a yellowness, often leaves the spotted lizard at Camp Green Lake meant reader with something to certain death. think about.

Questions

1. What is the main idea of this paragraph?

2. What is the concluding sentence?

3. What do you notice about the first line of the paragraph?

4. Do all the sentences in the paragraph support the main idea?

5. What is the first main point? second main point? third main point?

DDE, GJUS&T, Hisar 528 | English (Compulsory) BA-101

Activity

Decide which group of sentences would make good paragraphs. Write yes or no before each

Paragraph to show your answer.

1. ______

Stanley Yelnats and his family have a history of bad luck. The first Stanley’s great grandfather made a fortune on the stock market. The family’s bad luck struck him when he moved from New York to California and was robbed by Kissin’ Kate Barlow. Stanley’s father is an inventor. He works very hard and is smart, however, none of his inventions ever work. Stanley is accused of a crime he didn’t commit and is sent to Camp Green Lake as punishment. It seems none of the Yelnats can escape the family curse.

2. ______Camp Green Lake dried up and the people who lived near it moved away over a hundred years ago. Now the lake is desert-like with temperatures around ninety-five degrees. Deserts cover one-fifth of the Earth’s surface. The only place to find shade is between two trees in the Warden’s yard. Rattlesnakes and scorpions hide under rocks and in the holes the campers dig. Deserts do not have many large animals because there is not enough water for them to survive.

3. ______Stanley’s father was an inventor. He was trying to discover a way to recycle old sneakers. Clatonia Joaquin Dorticus invented an apparatus for applying dyes to the sides of the soles and heels of shoes. George de Mestral invented Velcro. While hiking, he had noticed that burrs stuck to his clothing. He used this idea to develop one strip of nylon with loops, and another with hooks. Making an Outline After taking notes for a report the ne xt step is to make an outline from the notes. Compare the outline below to the graphic organizer from Lesson 1.

8.4.1 Title of Paragraph

Outline Part of a Paragraph

DDE, GJUS&T, Hisar 529 | English (Compulsory) BA-101

I. Deadly yellow-spotted lizards. The main topic is used to build the topic sentence. Yellow-spotted lizards are a deadly threat to all humans at Camp Green Lake.

 Moved to Green Lake after lake dried up  Bite always fatal Details First Point and Support. They invaded the area after the life-giving lake dried up. Their bite is always fatal.  Killed Kate Barlow Details Second Point and Support It was a yellow-spotted lizard that killed.

How to Make an Outline

1. Write a title that lets the reader know the subject of the report.

2. Main topics are listed using Roman numerals and periods.

3. Points or facts are listed under each topic using capital letters and periods.

4. Subtopics are extra information that is needed to clarify a point or fact. They are listed under each fact as needed. Subtopics are listed using lower case letters and periods.

Questions

Use the outline above to answer these questions.

1. What is the main topic in the paragraph above?

2. How many points are made in this paragraph?

3. Did each point need additional clarification?

4. What do the Roman numerals represent?

5. What are the capital letters used for?

6. How many points were made in the paragraph?

Activity - Multiple Choice

1. Which piece of information is missing from the outline?

DDE, GJUS&T, Hisar 530 | English (Compulsory) BA-101

I. Juvenile Probation Camps Services

A. Provide educational services

B. Counselling

D. Work experience

E. Recreational programs

8.4.2 Third Point and Support

The yellow-spotted lizards threatened Stanley and Zero while they were digging for Stanley’s great-grandfather’s suitcase. The reason Stanley and Zero survived is because they had eaten so many onions that the lizards, who do not like onion blood, did not wish to bite them.

Conclusion With this one exception, meeting a yellow spotted lizard at Camp Green Lake meant certain death.

1. Which piece of information is missing from the outline?

II. Negative Impacts of Youth Being Tried in Adult Courts

A. Given harsher penalties in adult court

B. Little or no rehabilitative programs

C. Criminal records hard to expunge, erase or strike out

D. Could be denied military service

E. Youth held in adult facilities are more likely to return to a previous pattern of behaviour

a. Youth at a greater risk of being a victim in an adult jail or prison

b. In 2002, 2225 youth serving life without parole

c. Since 1999 the number of youth in adult prisons dropping

d. In Connecticut, North Carolina, and New York 16-year old considered adults

DDE, GJUS&T, Hisar 531 | English (Compulsory) BA-101

2. When researching youth being tried in the adult court system which is the least reliable source of information?

a. a national newspaper

b. a book published by the National Council on Crime and Delinquency

c. a government website

d. a student’s journal about life in prison

3. Which piece of information is missing from the outline?

III. What are juvenile boot camps?

A. Correctional programs for delinquent youth in a military-style environment

B. Emphasize discipline

C. Stress physical conditioning

D. Restricted to non-violent or first-time offenders

a. Some researchers have found that boot camp graduates are more likely to be re - arrested

b. Staff has expressed concern that too many youths lack the maturity and self- control to succeed in a military-style program

c. Research proves that small, community-based programs are more successful and less expensive than correctional institutions

d. Rigorous alternative to longer terms of confinement in juvenile correctional facilities

4. Which piece of information is missing from the outline?

I. Description of the Yellow Spotted Lizard

A. Exactly 11 yellow spots

B. Yellow-green body

DDE, GJUS&T, Hisar 532 | English (Compulsory) BA-101

. C Red eyes

D. Black teeth

E. Milky white tongue

a. Should have been named a “red-eyed” lizard

b. If you’ve ever been close enough to see the yellow spots, you are probably dead

c. Six to ten inches long

d. Likes to live in holes

Answer Key:

Multiple Choice

1. a. Youth at a greater risk of being a victim in an adult jail or prison

2. d. a student’s journal about life in prison

3. d. Rigorous alternative to longer terms of confinement in juvenile correctional facilities

4. c. Six to ten inches long

8.4.3 Thesis statement and A Good Thesis: a. A Thesis Statement: It is the main idea of an essay. It is often a point you want to argue or support in an essay. The thesis statement explains to a reader the main idea of the essay, and the writer’s opinion on that idea. A thesis statement is usually one sentence. It is often placed in the introductory paragraph of an essay. A thesis statement is a claim that could be argued. The essay will contain evidence and opinions that support the argument. A thesis statement should contain a topic (main idea of what you are writing about) contain an opinion about the topic (what your attitude is toward the topic)

Subject + Attitude = Thesis

DDE, GJUS&T, Hisar 533 | English (Compulsory) BA-101

Thesis Don’ts

 Avoid starting your thesis sentence with In my opinion, I believe, I think, etc. Thesis statements are always your take on the topic.  Just stating a fact. A thesis is something you plan to make an argument about.  Attempting two topics at once (even if they seem related). Pick one and stick with it. b. A Good Thesis:

It is Limited

Examples:

Good teachers make Mountain City Elementary a fantastic school. One reason to live in Mountain City is access to many wonderful places to fish. Needs To Be Limited The world is a magnificent place to live. Mountain City is a great place to live.

A Good Thesis is Concise

Examples:

Good Mountain City Elementary needs several changes to its facility to make it a better school. needs to be Concise Some problems with Mountain City Elementary School is that it needs a larger playground, an air-conditioned gym, an auditorium, restrooms connected to each classroom, running water in the classrooms, and a number of other physical changes to the building.

 A Good Thesis is Specific  A Good Thesis is Not a Subject  A Good Thesis is Not a Title  A Good Thesis is Not a Statement of Absolute Fact

Practice 1

Below you will see pairs of sentences. Each pair contains one sentence that would make a better thesis statement. Write T before the sentence in each pair that would

DDE, GJUS&T, Hisar 534 | English (Compulsory) BA-101 make a good thesis statement. Write N before each sentence that needs improvement.

1. ______Sachar illustrates that forming trustworthy friendships is beneficial to the characters in Holes.

2. ______Rex and Alan’s nicknames are X-Ray and Squid.

3. ______When Stanley asks Theodore where he can find water, Theodore grabs Stanley, throws him down, and says his name is Armpit.

Good

Daily writing practice has led to improved writing skills for the students at Mountain City Elementary. Needs To Be Specific Mountain City Elementary is a good school. Good The cost of living in Mountain City is lower than in most other cities in the United States. Should Not be a Subject Cost of Living

Good

Good teaching has led to an increase in TCAP scores at Mountain City Elementary. Should Not be a Title Rising TCAP Scores at Mountain City Elementary Good The climate in Mountain City is ideal for outdoor sports, Should Not be an Absolute Fact The average temperature for Mountain City in winter is 34 and in summer is 75 degrees.

4. ______Brutality is all around Camp Green Lake, and its harmful results are often shown.

5. ______In Holes Camp Green Lake is dry, hard, and hot which symbolize Stanley’s feelings of being trapped and sad while God’s Thumb is green and fertile which symbolize Stanley’s feelings of being free and happy.

6. ______In Holes, the physical environment is a symbol of Stanley’s feelings.

7. ______Onions signify a healing power in the novel.

8. ______Onions - A Food Rich in Sugar and Oil

DDE, GJUS&T, Hisar 535 | English (Compulsory) BA-101

9. ______Over 3,000 species of lizards live throughout the world with the largest number living in warm climates.

10. ______The Yellow Spotted Lizard in Holes represents death.

Practice 2

Read the following report about bullying, then answer the multiple-choice questions that follow.

A bully is a person who is mean or hurtful to others. Bullies often use a combination of threats and shame to annoy others. Bullies may hurt others physically by hitting, kicking or pushing. They may also be mean by calling names, teasing, leaving a kid out of the group on purpose, or scaring others. Some bullies many even threaten people or try to make people do things they don’t want to do.

1. Which sentence is the thesis statement for this paragraph?

a. Characteristics of Bullies

b. A bully is a person who harasses a weaker peer physically, mentally, or socially.

c. Researchers agree that bullying contains three elements: aggressive and negative behaviour, behaviour is carried out repeatedly, one person has power over another.

d. One form of bullying is spreading gossip.

Bullying is a problem that affects many kids. Bullies may cause a person to feel scared, worried, or embarrassed. The stress of being bullied may even make a person feel physically ill. Having a bully around may make a person afraid to go to the restroom, lunchroom, or playground. Worrying about a bully may affect a person’s ability to concentrate on schoolwork.

2. Which sentence is the thesis statement for this paragraph? a. Persistent bullying may have a number of negatives effects on an individual.

DDE, GJUS&T, Hisar 536 | English (Compulsory) BA-101 b. On March 21, 1993 Curtis Taylor committed suicide after being bullied for three years. c. In the 1990s, the United States witnessed many school shootings which were connected to bullying. d. Effects of bullying People become bullies for a number of reasons. Some are looking for attention. They think that bullying is a way to be more popular. By being a bully a person is trying to make himself feel more powerful. Some bullies come from families in which name calling and pushing others around is a normal way to act. These bullies are just copying what they have learned from home.

3. Which sentence is the thesis statement for this paragraph? a. Reasons for Bullying b. People become bullies for all the all the wrong reasons. c. Eighty-five percent of the time another kid witnesses a bully acting out. d. People become bullies to gain attention, to become more popular, to feel power over others, and because they are copying behaviour of others. Dealing with a bully may be difficult, but there are some things you can do. Sometimes you may turn a bully off just by acting brave. If you walk away, a bully will be less likely to give you trouble. You may also tell a bully to stop teasing or scaring someone else. Telling an adult is important. Teachers and parents can help. Go to recess, lunch, etc. with a buddy. Often bullies try to get a person alone before bullying. Finally don’t bully back. Fighting back gives the bully what he is looking for. In incidents bullies get into trouble. If they continue to bully others, eventually they have few friends. The power they wish slips away. Bullies who wish popularity soon realize that they are labelled as troublemakers.

4. Which sentence is the thesis statement for this paragraph? a. Several victims have been suing bullies directly.

DDE, GJUS&T, Hisar 537 | English (Compulsory) BA-101 b. Many schools have programs designed to teach students cooperation. c. Kids who are being bullied can turn the situation around by trying several actions. d. Dealing with Bullies

5. To gather more information about bullying, the most reliable source would be a. an encyclopedia b. a fictional story c. a tabloid newspaper d. an entertaining TV show

8.4.4 Elements of Paragraph

A paragraph should include the following elements:

Topic sentence — The topic sentence should ideally be the first sentence of the paragraph and should introduce the topic that will be discussed in the paragraph.

Support sentences — The supporting sentences should provide additional information that is relevant to the topic and helps support the argument of the topic. Generally, there are between 2 and 4 support sentences per paragraph.

Conclusion — The conclusion needs to be a short, cohesive sentence that concludes the topic of the paragraph.

8.4.5 Writing Topics of Paragraphs

One way that you can practice paragraph writing skills is to introduce paragraph writing topics. These paragraph writing topics can be used for nearly any type of classroom, regardless of grade level or subject matter:

 Write a paragraph about an important person in history. Be sure to address the topic of the paragraph, such as that person’s career, birthplace or prominent contribution. Use supporting sentences to expand on the topic chosen.

DDE, GJUS&T, Hisar 538 | English (Compulsory) BA-101

 Write a paragraph about your bedroom at home. Be descriptive, and use adjectives to describe how the space looks and how you feel when you are in your bedroom.  Write a paragraph about a holiday that you do not celebrate. Use facts within the supporting sentences to explain the holiday and the traditions that are associated with it.  Write a paragraph about an insect. Make sure to use scientific data and observations to create strong support within the paragraph.  Write a paragraph arguing your opinion on a controversial topic. Make sure to use factual information to support your opinion, and conclude with why you feel the way that you do.  Write a descriptive paragraph about your garden. Identify the plants that are in the garden, and use descriptive phrases to make the reader feel as if they are walking through your garden.  Write a paragraph providing instructions on how to code your favourite game. Be sure to offer clear direction, and don’t forget to use transitional phrases to guide the reader from one step to the next.  Write a paragraph about a new invention that you would create. Use descriptive phrases to describe your invention and to support the topic.  Write a persuasive paragraph asking your classmates to help take care of the school better. Offer several reasons within your supporting sentences as to why they should care about the well-being of their school building.  Write a paragraph describing a recent vacation that you took with your family. Be sure to describe the paragraph in detail, and conclude with what you loved most about your trip.  One way that you can help students practice their paragraph writing skills is to introduce paragraph writing topics. These paragraph writing topics can be used for nearly any type of classroom, regardless of grade level or subject matter.

DDE, GJUS&T, Hisar 539 | English (Compulsory) BA-101

 Write a paragraph describing one object, but use all of the senses in your description. Include a variety of adjectives to bring the paragraph to life for the reader.  Write a paragraph providing directions for how to care for your pet. Describe the pet and include the materials needed to properly care for the animal.  Write a persuasive paragraph about why you should earn more money for the chores you complete for your parents. Give solid reasons to support your argumentative topic.  Write a paragraph about a different culture. Introduce the reader to this culture through your supporting sentences.  Write a paragraph describing your favourite view, whether it is the view out your window from home or a fleeting glimpse of a sunset on vacation. Describe the scene in detail.  Write a paragraph in response to a recent news article that you have read. Use the topic sentence to address your feelings on the news piece, and the supporting sentences to back up your opinion.  Write a paragraph about an item that you feel your school needs. Form an argument and support your cause with facts and data.  Write a paragraph about a special childhood toy or memento. Describe why it is important to you and how it makes you feel.  Write a paragraph about a recent science experiment that you conducted in school. Identify what you observed, what you learned and what improvements you might make to the experiment.  Write a paragraph about the career you are considering. Explain why you are choosing that career path, and how you plan to accomplish your goals. Paragraph writing is a valuable skill that your students will use throughout their lifetimes. Regardless of the career that they choose, they will need strong writing skills to get them where they need to go. Both you and your students

DDE, GJUS&T, Hisar 540 | English (Compulsory) BA-101

8.5 Check Your Progress-

Question1.Value of Discipline-

There’s a lot of indiscipline in your school. It is not liked by the school authorities. They ask you, the vice Head Boy, to talk to the junior children about the ‘Value of Discipline’ so as to make them understand the importance of discipline in one’s life. Write a paragraph on the above, mentioned topic.

Question 2:

Need for Moral Education in Schools

There is a fall in values in today’s life. Keeping this cue in mind, write a paragraph on ‘The Need for Moral Education in Schools’, so as to make the youth of today, aware of high moral standards.

Question 3:

Write a paragraph on ‘The Importance of Good Health’.

Question 4:

The level of pollution in big cities is increasing steadily. It is becoming more and more important to bring pollution under control. Write a paragraph on ‘Environmental Pollution’ so as to make the people aware of the factors causing pollution.

Question 5:

Reading skills are essential to succeed in society. It is a source of knowledge and pleasure. Write a paragraph on ‘The significance of Reading’.

Question 6:

Observe the following visual and write a paragraph on the famous personality.

8.6 Summary-

DDE, GJUS&T, Hisar 541 | English (Compulsory) BA-101

• It is a group of sentences that introduces, presents and develops one main idea about the topic.

• A paragraph is a collection of sentences that deal with one subject.

• Descriptive paragraphs let the reader touch, taste, see, hear and smell what you are describing.

• Narrative paragraphs are often used to describe what a person does over a period of time. A narrative paragraph tells a story. The events in a narrative paragraph are usually arranged in chronological order.

8.7 Keywords-

Paragraphs: A group of sentences that deals with a subject

Wiki Answers: It is a site

Descriptive paragraph: Let the reader, touch, taste, see, hear and smell what you are describing.

-Cause: Something that produces cause and effect, a person, thing or event that makes something happen as a result

Chronological: Arrangement of events according to the sequence order of time

Coherence: Logical connection of ideas

Cohesion: Relating to movement from one sentence to another

Comparison: Examining one thing against another to show the points of likeness or difference

Contrast: Comparing two things or people to make the difference clear

Definition: Giving the meaning

Developers: Sentences which support and develop the main idea

Illustration: Example to support a point

DDE, GJUS&T, Hisar 542 | English (Compulsory) BA-101

Modulators: Sentences that help presenting different viewpoints.

8.8 Self-Assessment Questions (SAQs)

Fill in the blanks:

1. Paragraph is a group of ......

2. There is a ...... different types of paragraphs.

3. Passages and paragraphs are ...... thing

4. “Paragraph is a collection of sentences” explain it.

5. What is descriptive paragraphs?

6. Explain the narrative paragraph.

Answers: Self- Assessment

1. sentences 2. Two 3. Same.

8.9 Answer to Check Your Progress-

Answer 1:Importance of discipline -

Discipline is the law of nature. It is in man’s best interest to observe discipline in all walks of life. Discipline is the backbone of character. Without discipline, nothing great can be achieved in life. A man who does not observe discipline in life has to suffer. He becomes lethargic and arrogant. He is disliked by all. A student who does not obey his teachers and does not do his homework does not get success. An employee who does not work sincerely in office may be sacked any time. If sportsmen do not observe discipline cannot hope to win any match.

In the army, discipline is of utmost importance. Discipline leads to harmony whereas indiscipline leads to confusion. No life is worth living without discipline.

Conclusion -A disciplined person is an asset to himself, to his family, to his society, to the nation and to mankind. Discipline works everywhere. It controls the physical

DDE, GJUS&T, Hisar 543 | English (Compulsory) BA-101 movements and our morals. There is no sphere in heaven and the Earth where discipline does not dominate.

Answer:2 Need for Moral Education in Schools

Philosophers have explained that the main purpose of education is to have a sound character. For this reason, moral education is gaining importance in the field of education. Man is a social animal. He learns social habits like helping people, respecting elders and being polite, during school years. When he learns good social habits, he becomes a responsible citizen. Moral education enables a person to choose the right path. It’s not that only teachers in schools can impart moral education to the children, but parents also play a great role by becoming role models for the children. What is moral education? It can be anything which teaches a person moral values and virtue. Values are the set of guidelines that a human being learns to follow.

Nobody is born with moral values in him or her. It is inculcated in him or her by his or her education and experience of life.

Conclusion -Therefore, it becomes essential to inculcate moral values during school years as these are the most formative years in a child’s life. Education moulds the moral values in our characters. By educating children about morals and the young learner values in life we can make them fit for the society as well as the nation.

Answer 3. Health is wealth. Good health is the greatest blessing that a person can have. It is the key to happiness. It is the secret of every happy man. Only a healthy person can enjoy life to its fullest and discharge his duties well. On the other hand, an ailing person is a burden on all. Therefore, we must try our best to maintain sound health. Fresh air, balanced diet, regular exercise, proper rest and stress-free life are pre-requisites for good health. We should drink plenty of water, avoid over eating and should refrain from alcohol.

DDE, GJUS&T, Hisar 544 | English (Compulsory) BA-101

Most of all, we should have regular health check-ups. We must laugh more as laughing is a therapy and the secret of good health. The care of the body regarding food, exercise, rest and protection against disease, are essential for the preservation of sound health.

Conclusion- Life is for living. Without health, life is deprived of not only much of its usefulness, but also its joys and pleasures. Let us remember that life is worth living only by a healthy body, mind and soul.

Answer:4 Environmental Pollution

The problem of environmental pollution is becoming more and more serious day by day. The ecological cycle is being disturbed by the people. Man is exploiting nature to the extent that natural resources are depleting. The main reason for environmental pollution is over population and poverty. The increased demand for food and other resources forces man to cut down forests which leads to an increase in the carbon dioxide ratio in the atmosphere. This results in heating up of the atmosphere. This is called global warming. Due to this, seasons are also undergoing a change.

All this forecasts disaster for human beings and other living beings. In order to bring pollution under control we need to conserve our forests, reduce the use of paper, petrol etc. Earth must be saved at any cost for the sake of survival.

Conclusion- The need of the hour demands a check in environmental pollution. The government should take necessary ste ps in this direction.

Answer:5 Significance of Reading

Reading broadens the vision of an individual. Bacon wrote in his essay. ‘Of Studies’. “Reading making a full man: conference a ready man: and writing an exact man”. In shaping the personality of an individual, the importance of reading cannot be denied. A person reads not only for the sole purpose of gaining knowledge, but also to get pleasure, provoke critical and original thinking and to broaden his vision. Reading

DDE, GJUS&T, Hisar 545 | English (Compulsory) BA-101 makes man a better conversationalist and helps to relieve stress. Newspaper reading has become an important aspect in the life of a modern man.

Conclusion -A typical modern man may get his breakfast or not, but he must get his newspaper. A good reader leads a disciplined life. Reading helps him to become a well-disciplined. It boosts imagination and creativity in an individual. He is able to mix well with others and is never alone as books are his best friends.

Answer 6-The famous personality.

Arvind Kejriwal was born on 16th August, 1968. He is an Indian politician and social activist. He served the Indian Revenue Service as an officer. He was the seventh Chief Minister of Delhi. He is also the current Chief Minister, having won the 2015 Delhi Assembly Elections. He is the leader of the Aam Admi Party (AAP). Kejriwal is a Mechanical Engineering graduate. In 2006, Kejriwal was awarded the Ramon Magsaysay Award for ‘Emergent Leadership’ recognizing his involvement in a movement Parivartan, using the RTI legislation. It was a campaign against corruption. He donated his Magsaysay award money to the Public Cause Research Foundation, a Non-Governmental Organization (NGO). In 2012, he launched the Aam Admi Party and defeated Chief Minister Sheela Dixit. Following the election, he became the Chief Minister of Delhi on 28th December, 2013. He resigned 49 days later, an action he later regretted.

Conclusion- He apologized for quitting midway. He achieved a sweeping majority in the Delhi Legislative.

8.10 References /Suggested Readings-

 Books English for Competitive Exams, By DR. R.P. Bhatnagar  Unique Quintessence of General English, Edited by DR. S. Sen & Others and revised by DR. G.S. Mansukhani.  A Background to the Study of English Literature, D.K. Patnaik, Swastik Publications

DDE, GJUS&T, Hisar 546 | English (Compulsory) BA-101

 Online links: o www.paragraphwriting.com o http://daria-przybyla.suite101.com/seven-types-of-paragraphs-a99946 o http://www.whitesmoke.com/how-to-write-a-paragraph

DDE, GJUS&T, Hisar 547 | English (Compulsory) BA-101

NOTE

DDE, GJUS&T, Hisar 548 | English (Compulsory) BA-101

NOTE

DDE, GJUS&T, Hisar 549 | English (Compulsory) BA-101

NOTE

DDE, GJUS&T, Hisar 550 | English (Compulsory) BA-101

NOTE

DDE, GJUS&T, Hisar 551 | English (Compulsory) BA-101

NOTE

DDE, GJUS&T, Hisar 552 |